You are on page 1of 314

Prime

Mat he ma ti c s Ser
ies

9
Approved by
Government of Nepal, Ministry of Education, Science and
Technology Curriculum Development Centre Sanothimi,
Bhaktapur as and additional learning materials.

Authors
Raj Kumar Mathema
Dirgha Raj Mishra Bhakta Bahadur Bholan
Uma Raj Acharya Yam Bahadur Poudel
Naryan Prasad Shrestha Bindu Kumar Shrestha

Editors
Anil Kumar Jha
Dhurba Narayan Chaudhary
Hari Krishna Shrestha

Language Editor
Mrs. Tara Pradhan

Pragy
a Books
Pragya Books and Distributor Pvt. Ltd.
and
Distributors Pvt. Ltd.
Printing history
First Edition 2074 B.S.
Revised Edition 2077 B.S.

Layout and design


Ramesh Maharjan

© Publisher

All rights reserved. No part of this book, or designs and illustrations here within, may be
reproduced or transmitted in any form by any means without prior written permission.

ISBN : 978-9937-9170-3-2

Printed in Nepal

Published by
aB
Pragy ooks
Pragya Books and Distributors Pvt. Ltd.
and Kathmandu, Nepal
Distributors Pvt. Ltd.
E-mail : pragyabooks100@gmail.com
Preface

Prime Mathematics Series is a distinctly outstanding mathematics series designed in compliance


with Curriculum Development Centre (CDC) to meet international standards. The innovative, lucid
and logical arrangement of the content makes each book in the series coherent. The presentation
of ideas in each volume makes the series not only unique, but also a pioneer in the evolution of
mathematics teaching.
The subject matter is set in an easy and child-friendly structure so that students will discover
learning mathematics a fun thing to do. A lot of research, experimentation and careful gradation have
gone into the making of the series to ensure that the selection and presentation is systematic, innovative
and both horizontally and vertically integrated.
Prime Mathematics Series is based on child-centered teaching and learning methodologies,
so the teachers will find teaching this series equally enjoyable. We are optimistic that this series shall
bridge the existing inconsistencies between the cognitive capacity of children and the course matter.
We owe an immense debt of gratitude to the publishers for their creative, thoughtful and inspirational
support in bringing about the series. Similarly, we would like to acknowledge the tremendous support
of teachers, educationists and well-wishers for their contribution, assistance and encouragement in
making this series a success.
We hope the series will be another milestone in the advancement of teaching and learning
mathematics in Nepal. We solicit feedback and suggestions from teachers, students and guardians alike
so that we can refine and improvise the series in the future editions.
Our team would like to express our special thanks to Mr. Nara Bahadur Gurung, Mr. Ram
Narayan Shah, Mr. Tulsi Kharel, Mr. Mani Ram Khabas, Mr. Umesh Acharya, Mr. J. Phuldel, Mr.
Kamal Raj Tripathee, Mr. Rudra Prasad Pokharel, Mr. Uttam Prasad Panta, Mr. L.N. Upadhyaya,
Mr. Shakti Prasad Acharya, Mr. Upendra Subedi, Mr. Kul Narayan Chaudhary, Mr. Bishonath
Lamichhane, Mr. Harilal Lamichhane, Mr. Govinda Paudel, Mr. Krishna Aryal, Mr. Nim Bhujel,
Mr. Santosh Simkhada, Mr. Pashupati Upadhyaya, Mr. Dipak Adhikari, Mr. Mukti Adhikari,
Mr. Dipendra Upreti, Mr. Dipak Khatiwada, Mr. Narayan Nepal, Mr. Raj Kumar Dahal, Mr. Bhim Raj
Kandel, Mr. Prem Giri,Mr. Iswor Khanal, Mr. Balram Ghimire, Mr. Om Kumar Chhetri, Mr. Ram Hari
Bhandari, Mr. Krishna Kandel, Mr. Madhav Atreya, Mr. Harihar Adhikari, Mr. Chura Gurung, Mr. Shiva
Devkota, Mr. Chandra Dev Tiwari, Jivan K.C., Raghu Kandel and Baikuntha Marhattha and Subash Bidari
for their Painstaking effort in peer reviewing of this book.
Contents
1. Sets
1.1 Sets................................................................................................................. 2
1.2 Venn Diagrams and Set operations ......................................................... 5
- Exercise 1.1 .............................................................................................13
1.3 Application of Sets ....................................................................................15
- Exercise 1.2 ............................................................................................18

2. Arithmetic
2.1 Profit and Loss............................................................................................ 26
- Exercise 2.1 ........................................................................................... 32
2.2 Commission .................................................................................................. 33
- Exercise 2.2........................................................................................... 35
2.3 Discount ....................................................................................................... 36
- Exercise 2.3........................................................................................... 39
2.4 Taxation ....................................................................................................... 40
- Exercise 2.4........................................................................................... 42
2.5 Bonus............................................................................................................. 43
- Exercise 2.5........................................................................................... 45
2.6 Share and Dividend ................................................................................... 46
- Exercise 2.6........................................................................................... 49
2.7 Home Arithmetic ....................................................................................... 50
2.7.1 Electricity Billing............................................................................ 50
- Exercise 2.7.1 ............................................................ 53
2.7.2 Water Billing ................................................................................... 55
- Exercise 2.7.2............................................................ 56
2.7.3 Telephone Billing ............................................................................ 58
- Exercise 2.7.3............................................................ 60
2.7.4 Taxi Fare ...........................................................................................61
- Exercise 2.7.4............................................................ 62

3. Mensuration
3.1 Area of Pathways........................................................................................71
- Exercise 3.1 ........................................................................................... 75
3.2 Surface Area and Volume of Prisms ..................................................... 77
- Exercise 3.2........................................................................................... 80
3.3 Area of Four Walls, Floor and Ceiling .................................................. 83
- Exercise 3.3........................................................................................... 89
3.4 Volume of Walls ......................................................................................... 90
- Exercise 3.4........................................................................................... 95
4. Alebra
4.1 Factorisation............................................................................................. 100
- Exercise 4.1.1 ....................................................................................... 107
- Exercise 4.1.2 ...................................................................................... 109
4.2 Indices ........................................................................................................110
- Exercise 4.2 ..........................................................................................116
4.3 Exponential Equation................................................................................118
- Exercise 4.3 ..........................................................................................121
4.4 Ratio and Proportion ............................................................................... 122
4.4.1 Ratio ................................................................................................ 122
- Exercise 4.4.1 ........................................................... 126
4.4.2 Proportion ...................................................................................... 128
- Exercise 4.4.2 ........................................................... 137
4.5 Simultaneous Equations.......................................................................... 140
4.5.1 Linear Equation ............................................................................. 140
- Exercise 4.5.1 ........................................................... 145
4.5.2 Graphical Method ......................................................................... 146
- Exercise 4.5.2 ........................................................... 148
4.6 Quadratic Equation ................................................................................. 148
4.6.1 Solving a Quadratic Equation by Factorization Method .... 149
- Exercise 4.6.1 ........................................................... 151
4.6.2 Solving Quadratic Equation by Completing the Square ...... 153
- Exercise 4.6.2 ........................................................... 156
4.6.3 Solving Quadratic Equation by using Formula ....................... 157
- Exercise 4.6.3 ........................................................... 160

5. Geometry
5.1 Triangle ...................................................................................................... 168
- Exercise 5.1 ...........................................................................................171
- Exercise 5.2 ......................................................................................... 180
5.2 Parallelogram ............................................................................................ 183
- Exercise 5.3 ......................................................................................... 190
5.3 Mid-Point Theorem.................................................................................. 193
- Exercise 5.4 ......................................................................................... 196
5.4 Similarity ...................................................................................................200
- Exercise 5.5 .........................................................................................205
5.5 Pythagoras Theorem ...............................................................................209
- Exercise 5.6 ......................................................................................... 213
5.6 Construction of Quadrilaterals ............................................................ 215
- Exercise 5.7 ......................................................................................... 219
5.7 Circle...........................................................................................................220
- Exercise 5.8 .........................................................................................227
6. Trigonometry
6.1 Trigonometric Ratios ..............................................................................234
- Exercise 6.1 ..........................................................................................239
6.2 Trigonometric Ratios of Some Standard Angles .............................242
- Exercise 6.2 .........................................................................................246

7. Statistics
7.1 Graphs and Pie Charts ............................................................................252
- Exercise 7.1 ..........................................................................................260
7.2 Measure of Central Tendency ..............................................................262
7.2.1 Mean ................................................................................................262
- Exercise 7.2.1 ...........................................................................265
7.2.2 Median.............................................................................................266
- Exercise 7.2.2 ..........................................................................268
7.2.3 Mode ................................................................................................269
- Exercise 7.2.3 ..........................................................................269
7.2.4 Quartiles ........................................................................................270
- Exercise 7.2.4 ..........................................................................272

8. Probability
8.1 Probability .................................................................................................276
- Exercise 8.1 ..........................................................................................282
8.2 Empirical probability ...............................................................................285
- Exercise 8.2 .........................................................................................286

n Model Question Set ................................................. 291

n Prime questions for more practice ................................. 303


Chapter

1
Sets
Objectives:
At the end of this chapter, the
students will be able to:
 find the set relation using set
operation between the sets.
 represent the set operations in
the Venn diagram.
 solve the word problems related
to cardinality of sets using Venn
diagram.

Teaching Materials:
Chart paper, flass cards, different
colour board markers, charts of related
formula, geometrical instruments
 Historical facts
1. The set theory was developed by a German mathematician Georg Cantor
(1845-1918). The “New Mathematics” which helps in solving problem in
some case, more easily and with more pleasure, makes much use of sets.
Sets provide a useful way of representing groups of things and correlating them.
2. Augustus De Morgan (1806-1871; England) was born blind in one eye in
1806 in Madras, India. His famous laws on set theory states that if A and B are subsets of a
universal set then the complement of union of A and B is the intersection of the complements
of A and B i.e. (AB) = AB and the complement of the intersection of A and B is the
union of complements of A and B i.e. (AB) = AB.

1.1 Sets
A collection or aggregate of well defined objects is called a set. e.g. the set of natural numbers, the set
of vowels of English alphabets. The objects of a set are called its elements or members. If x is an
element of the set A, then we write xA (x belongs to A).

Set Notation
The elements of a set are enclosed with curly brackets { } and a set is denoted by capital letters. A, B,
C, N, V, ……… etc.
For example:
N = {1, 2, 3, …….. }
V = { a, e, i, o, u}
A set can be described mainly by three methods
1. Description method: In this method, the set is described in words with the common properties
of the elements.
For example:
(i) A is a set of digits of Hindu-Arabic number system,
(ii) V is a set of vowels of English alphabets.
2. Listing or Roster Form (tabular form): In this method, we list all the elements.
For example:
(i) A set of digits in Hindu-Arabic number system, A = {0, 1, 2, 3, 4, 5, 6, 7, 8, 9}
(ii) A set of vowels of English alphabets, V = {a, e, i, o, u}
(iii) A set of integers, Z = {…….. -2, -1, 0, 1, 2, ……….}
(iv) A set of whole number, W = {0, 1, 2, 3, …….}

2 | Mathematics - 9 Sets
3. The Rule method or Set builder form: In this form, we specify the set with ‘defining
property’ giving a variable, like A = {x : x has property p}. Which is read as: A is the set of x
such that x has property p.
For example:
(i) N = {x : x is a natural number}
(ii) W = {x : x is a whole number}
(iii) A = {x : x  20, x  N}
(iv) S = {x : (x – 1)( x + 1) = 0}
Note: Specially, we denote the numbers as:
(i) Natural numbers, {1, 2, 3, ………… } = N
(ii) Whole numbers, {0, 1, 2, 3, ……….. } = W
(iii) Integers, {……………. -3, -2, -1, 0, 1, 2, 3, ……….. } = Z
(iv) Rational number = Q
(v) Real number = R

Various Sets
The Empty Set or Null Set or Void Set: A set which contains no elements is called the empty
set and is denoted by  or { }.
For example:
(i) A = {x : x + 1= 0, x  N} = 
(ii) B = {Boys of class IX of Saint Mary’s School} = 
Note:
(i) Since there is one and only one empty set with different descriptions, hence we say ‘the empty set’
or ‘the null set’ or ‘the void set’ instead of ‘an empty set’, ‘a null set’.
(ii) The empty set is denoted by  which is not Greek letter ‘phi’ but Danish letter ‘oe’.
(iii)   {0}, since {0} is a set whose element is 0.
(iv)   {}, since {} is a set whose element is .
Finite Set: A set with finite number of elements is called a finite set.
For example:
(i) A = {1, 2, 3, …………., 100}
(ii) B = {x : x is a district of Nepal}
Infinite Set: A set which is neither a null nor a finite set is called an infinite set.
For example:
(i) N = {1, 2, 3, ……………….}
(ii) A = {Rational number between 1 and 2}

Sets Mathematics - 9 |3
Singleton Set: A set containing only one element is called a singleton set.
For example: {}, {0}, {1}, {x : x is an even number between 1 and 3}.
Universal Set: A set which contains all the sets under consideration as sub-sets is called a
Universal set. In other way, a set which contains all the possible elements under consideration is
called a universal Set. It is denoted by U. In some texts, we also find a universal set denoted by
 (pxi) or X or [ ].
For example: Let's consider two sets A = {1, 2, 3, 4, 5} and B ={1, 3, 5, 7}, for these two sets,
universal set may be {1, 2, 3, 4, 5, 6, 7, 8, 9} or {x : x  N} or {x : x  W} or {x : x  Z} or
{x : x  Q}.
Note: The choice of a universal set is not unique.

Relations between the sets:


Disjoint Sets: Two sets are disjoint if they have no element in common.
For example:
(i) If A = {1, 3, 5, 7, 9} and B = {2, 4, 6, 8}, then A and B are disjoint sets.
(ii) If M = {x : x  N} and N = {x : x  Z, x < 0}, then M and N are disjoint sets.
Overlapping / Joint Sets: If two sets have some common elements, they are called overlapping sets.
For example: If A = {1, 2, 3, 4, 5} and B = {2, 4, 6, 8}, two sets A and B have common elements 2
and 4. So the sets A and B are overlapping sets.
Equal Sets: Two sets are said to be equal, if they contain the same elements.
For example:
(i) If A = { x : x is letter of the word LATENT} and B = { x : x is letter of the word TALENT}
then A =B or AB.
(ii) If P = { x : x is a digit of number 123} and Q = { x : x is a digit of the number 321} then P = Q
or PQ.
Equivalent Sets: Two sets are equivalent, if and only if a one-to-one correspondence exists
between them. In other words A & B are equivalent sets if they have equal number of elements.
For example: If A = {1, 2, 3} and B = {p, q, r}, then A and B are equivalent sets.
Note: If two sets are equal they must be equivalent, however two equivalent sets need not be equal.
Sub Sets: Set A is a subset of B, if every elements of set A is also an element of Set B. It is denoted
as A  B, [read as A is subset of B, or A is contained in B] or B A [read as B contains A]. And B is
called the super set of A.
For example:
(i) If A = {a, b} and B = {a, b, c} then AB.
(ii) If P = {1, 2, 3} and Q = {1, 2, 3}, then PQ.

4 | Mathematics - 9 Sets
Power Set: The set of all subsets of a set A is called the power set of the set A.
For example: If A = {a, b}, then the power set of A = {{ }, {a}, {b}, {a, b}}. If A = {a, b, c}, then the
power set of A = {, {a}, {b}, {c}, {a, b}, {b, c}, {a, c}, {a, b, c}}.
Note:
(i) Every set is a subset of itself i.e. AA.
(ii)  is subset of every set i.e. A.
(iii) Number of subsets of a set A containing n elements is 2n.
For A = {a}, sub-sets of A are:
 and {a}, where number of subsets = 2 = 21
For A = {a, b}, subsets of A are:
, {a}, {b}, {a, b}, where number of subsets = 4 = 22.
For A = {a, b, c}, subsets of A are:
, {a}, {b}, {c}, {a, b}, {b, c}, {a, c}, {a, b, c}.where number of subsets = 8 = 23.
 If number of elements of A is n then number of subsets = 2n.
Proper sub-set: Set A is a proper subset of set B, if every element of set A is an element of set B
and at least an element of B is not an element of A, it is denoted as AB.
If N is a set of natural numbers, W whole numbers, Z integers, Q rational numbers and R real numbers
then N  W  Z  Q  R.
Note:
(i) A set is not a proper subset of itself.
(ii)  is a proper subset of every set.
(iii)  is not a proper subset of itself.
(iv) Number of sub-sets of a set A containing n elements is 2n – 1.

1.2 Venn Diagrams and Set operations


Venn-diagrams: John Venn (1834-1923, England) first illustrated sets graphically in 1870 in a paper
on “Boole’s logical system.”
A Venn-diagram consists of a rectangle representing universal set U and subsets of U by circles or
ovals inside it. The sets of elements representing different operations like Union, intersection,
difference and complements are shown by shading the required regions.
Leonard Euler (1707 – 1783, Switzerland) also used diagrams to represent sets. So, Venn-diagrams
are sometimes called Euler-Venn-diagrams.
U U U
A A B A B

AU A and B are A, B, C are


disjoint sets overlapping sets

Sets Mathematics - 9 |5
U U U
A B
AB B A

A and B are AB BA or AB


overlapping sets

Set Operations
There are four set operations:
a. Union
b. Intersection
c. Difference
d. Complement
a. Union of Two or More Sets
The union of two sets A and B is the set of the elements that belong either to A or to B or to
both A and B.
Mathematically, AB = {x : x  A or x  B}.
For two disjoint sets For three disjoint sets
U U
A A B C
B

AB ABC
For two overlapping sets For three overlapping sets
U U
A B
A B

C
AB ABC

6 | Mathematics - 9 Sets
For AB For A and B overlapping and B and C overlapping
U U
B A B C
A

AB = B ABC
b. Intersection of Sets
Intersection of two sets A and B is the set of the elements which belong to both A and B.
Mathematically, AB = {x : x  A and x  B}.
When A and B are disjoint When A and B are overlapping When AB
U U U
A B A B
B A

AB =  AB AB = A


When A, B, C are overlapping
U U U
A B A A B
B

C C C

ABC (AB)C (AB)(AC)(BC)


c. Difference between of Two Sets
Difference between two sets A and B denoted A-B is the set of elements which belong to A but
don’t belong to B.
Thus, A – B = {x : x  A, x  B}.
When A and B are disjoint When A and B are overlapping
U U
A B A B

A–B=A A–B

Sets Mathematics - 9 |7
U U
A B A B

C C

(AB) – C A – (BC)
d. Complement of a Set
Complement of a Set A is denoted as Ā or A or Ac which is the set of the elements which
belong to the universal set U but do not belong to A.
Thus, Ā or Aor Ac = U – A
U U U
A A B A B

_ ____ ____
A AB AB
U U U
A B A B
A B
A-B ABC
C C ABC
____ _______
A–B ABC
Special Properties of Complements of Sets
=
1. (A) = A or A =A
Proof:
= _
A = {x : x  A }
= {x : x  A }
=
A =A
2. De Morgan’s law
______ _ _
(a) (AB) = A  B
______ _ _
(b) (AB) = A  B

8 | Mathematics - 9 Sets
Proof:
______
(a) (AB) = {x : x  (AB)}
= {x : x  A and x  B}
_ _
= {x : x  A and x  B }
______ _ _
 (AB) = A  B
______
(b) (AB) = {x : x  (AB)}
= {x : x  A or x  B}
_ _
= {x : x  A or x  B }
_ _
= {x : x  ( A  B )}
______ _ _
 (AB) = ( A  B )

Worked Out Examples


Example 1: If U = {0, 1, 2, ………, 9}, A = {1, 2, 3, 4} and B = {1, 3, 5, 7}, find AB and
illustrate in a Venn-diagram.
Solution: Here,
U = {0, 1, 2, ………, 9}
A = {1, 2, 3, 4}
B = {1, 3, 5, 7}
 AB = {1, 2, 3, 4}  {1, 3, 5, 7}= {1, 2, 3, 4, 5, 7}
Represnting the above information in a Venn-diagram,
U
A B
2 1 5
6 4 3 7
89
0
The shaded region denotes AB.

Example 2: If U = {x : x  12, x  N}, A = {x : x is a multiple of 4, 1 < x  12} and


B = {x : x is an even number, 2  x < 11}, find AB and illustrate in a
Venn-diagram.

Sets Mathematics - 9 |9
Solution: Here,
U = {x : x  12, x  N} = {1, 2, 3, ……,12}
A = {x : x is a multiple of 4, 1 < x  12} = {4, 8, 12}
B = {x : x is an even number, 2  x < 11} = {2, 4, 6, 8, 10}
Now, AB = {4, 8, 12}  {2, 4, 6, 8, 10} = {4, 8}
Represnting the above information in a Venn-diagram,

The shaded region denotes AB.

Example 3: If U = {1, 2, 3, 4, 5, 6, 7, 8}, A = {1, 2, 3, 4} and B = {3, 4, 5, 6}, find A – B


and B – A.
Solution: Here,
U = {1, 2, 3, 4, 5, 6, 7, 8}, A = {1, 2, 3, 4}, B = {3, 4, 5, 6}
Now, A – B = {1, 2, 3, 4} - {3, 4, 5, 6}
 A - B = {1, 2}
And B – A = {3, 4, 5, 6} - {1, 2, 3, 4}
 B - A = {5, 6}.

Example 4: If U = {0, 1, 2, 3, ………., 12}, A = {1, 2, 3, 5, 6, 8, 9}, B = {3, 4, 5, 6, 7, 10}


and C = {5, 6, 8, 9, 10, 11}, find (AB) – C and illustrate it in a Venn-
diagram.
Solution: Here,
U = {0, 1, 2, 3, ………., 12}
A = {1, 2, 3, 5, 6, 8, 9}
B = {3, 4, 5, 6, 7, 10}
and C = {5, 6, 8, 9, 10, 11},
then AB = {1, 2, 3, 5, 6, 8, 9}  {3, 4, 5, 6, 7, 10}
= {1, 2, 3, 4, 5, 6, 7, 8, 9, 10}
Now, (AB) – C = {1, 2, 3, 4, 5, 6, 7, 8, 9, 10} - {5, 6, 8, 9, 10, 11}
(AB) – C = {1, 2, 3, 4, 7}

10 | Mathematics - 9 Sets
Represnting the above information in a Venn-diagram,
U
A B

1 3 4
2 5 7
8 6
9 10
11 12
0
C
(AB) – C

The shaded region denotes (AB) – C.


Example 5: If U = {0, 1, 2, 3, 4, 5, 6, 7, 8, 9}, A = {1, 2, 3, 5, 7} and B = {2, 4, 5, 6}, find
the following sets and illustrate them in a Venn-diagram.
(a) A (b) (AB)' (c) (AB) (d) (A – B)
Solution: Here,
U = {0, 1, 2, 3, 4, 5, 6, 7, 8, 9}, A = {1, 2, 3, 5, 7}, B = {2, 4, 5, 6}
a. A = U – A = {0, 1, 2, 3, 4, 5, 6, 7, 8, 9} – {1, 2, 3, 5, 7} = {0, 4, 6, 8, 9}
It is illustrated in a Venn-diagram as

A'
b. AB = {1, 2, 3, 5, 7}  {2, 4, 5, 6}
 AB = {1, 2, 3, 4, 5, 6, 7}
Now,
(AB) = U - (AB)
= {0, 1, 2, 3, 4, 5, 6, 7, 8, 9} – {1, 2, 3, 4, 5, 6, 7}
= {0, 8, 9}
It is illustrated in a Venn-diagram as

Sets Mathematics - 9 |11


c. (AB) = {1, 2, 3, 5, 7}{2, 4, 5, 6}
 AB = {2, 5}
Now, (AB) = U - (AB) = {0, 1, 2, 3, 4, 5, 6, 7, 8, 9} – {2, 5}
(AB) = {0, 1, 3, 4, 6, 7, 8, 9}
Which is illustrated in a Venn-diagram as

d. A – B = {1, 2, 3, 5, 7} - {2, 4, 5, 6}
A - B = {1, 3, 7}
Now, (A - B) = U – (A – B) = {0, 1, 2, 3, 4, 5, 6, 7, 8, 9} – {1, 3, 7}
(A - B) = {0, 2, 4, 5, 6, 8, 9}
Which is illustrated in a Venn-diagram as

Example 6: If A = {1, 2, 3, 4}, B = {1, 3, 5, 7, 9} and U = {0, 1, 2, 3, 4, 5, 6, 7, 8, 9} show


that (AB) = A  B.
Solution: Here,
A = {1, 2, 3, 4}, B = {1, 3, 5, 7, 9} and U = {0, 1, 2, 3, 4, 5, 6, 7, 8, 9}
(AB) = {1, 2, 3, 4}  {1, 3, 5, 7, 9}
= {1, 2, 3, 4, 5, 7, 9}
Now, (AB) = U - (AB)
= {0, 1, 2, 3, 4, 5, 6, 7, 8, 9} - {1, 2, 3, 4, 5, 7, 9}
 (AB)= {0, 6, 8} ... (i)
And A = U – A = {0, 1, 2, 3, 4, 5, 6, 7, 8, 9} - {1, 2, 3, 4}
 A = {0, 5, 6, 7, 8, 9}
B = U – B = {0, 1, 2, 3, 4, 5, 6, 7, 8, 9} - {1, 3, 5, 7, 9}

12 | Mathematics - 9 Sets
 B = {0, 2, 4, 6, 8}
Now, A  B = {0, 5, 6, 7, 8, 9}  {0, 2, 4, 6, 8}
 A  B = {0, 6, 8} ... (ii)
Hence, from (i) and (ii), we get
(AB) = A  B Proved.

Exercise 1.1
1. If U = {1, 2, 3, ……….., 12}, A = {2, 3, 5, 7, 11} and B = {1, 2, 3, 4, 5}, list the following sets
and illustrate in a Venn-diagram.
(a) AB (b) AB
(c) A–B (d) A
2. If U = {0, 1, 2, 3, 4, 5, 6, 7, 8, 9}, A = {2, 3, 5}, B = {1, 4, 7, 8}, and C = {1, 3, 5, 7, 9}, find
(a) A  B (b) BC
(c) (C – B)A (d) (AB)C
3. If U = {a, b, c, d, e, f, g, h, i, j, k, l}, A = {a, b, c, d, e}, B = {b, c, d, e, f, g, h, i} and
C = {b, d, e, g, h, i, j}, find and show the following sets in a Venn-diagram.
(a) (ABC) (b) (AB)C
(c) ABC (d) (AB)  (AC)  (BC)
4. Using set symbols, write down expressions for the shaded portion in the following Venn-diagrams.
(a) (b)
U U
A B A B

(c) (d)
U U
A B A B

Sets Mathematics - 9 |13


5. Using the symbols, write down expressions for the shaded portion in the following Venn-diagrams.
(a) (b)
U U
A B A B

C C

(c) (d)
U U
A B A B

C C

6. Study the given Venn-diagram and list the elements and shade in the Venn-diagram for each of
the following sets.
____ ____
(a) A–B (b) B–A (c) B'A
_____ _____
(d) AB (e) AB (f) U – B'

7. Study the given Venn-diagram and list the elements and shade the region of each of the
following sets.
_______
(a) (ABC ) (b) (AB)C
_____
(c) C – (AB) (d) (AB) – C

8. If A = {a, b, c}, B = {b, c, d, e} and C = {c, d, e, f} are subsets of the universal set
U = {a, b, c, d, e, f, g, h}, verify the following.
(a) A(B – C) = (AB) – (AC) (b) A(BC) = (AB)  (AC)
_ _ _ ____ ____ _
(c) A ( B – C ) = AB – AC (d) A B = B – (AB)
9. A and B are two subsets of a universal set U such that AB  .
____
(a) Draw a Venn-diagram to represent the relation between A and B and shade AB .
_ _
(b) Re-draw the Venn-diagram and shade A  B .
____ _ _
(c) Write down the relation between (AB ) and ( A  B ).
(d) What is this law specially known as?

14 | Mathematics - 9 Sets
10. If Q = {quadrilateral}, T = {Trapezium}, P = {Parallelogram}, U
R = {Rectangle}, S = {Square} and X = { Rhombus}, label the
given sets with appropriate symbols.
11. Given U = {Triangles}, I = {Isosceles triangles},
E = {Equilateral triangles}, R = {Right angled triangles}
(a) Draw a Venn-diagram to show the relationship between these sets.
(b) What are the measures of the angles of a triangle belonging to IR?

1.3 Application of Sets


Cardinality of a set
The number of elements in a set is known as the cardinal number or cardinality of the set. The cardinal
number of a set A is denoted as n(A). Cardinal numbers of the sets AB, AB, A – B, A are denoted
as n(AB), n(AB), n(A – B), n(A) etc. If A = {a, e, i, o, u}, then n(A) = 5.

Cardinality relation of union of two sets


If A and B are two overlapping sets in a universal set U, then n(AB) = n(A) + n(B) - n(AB)
Proof:
If n(A) = a, n(B) = b and n(AB) = c,
then number of elements belonging to A only, no(A) = a – c.
Number of elements belonging to the set B only, no(B) = b – c.
U
From the Venn-diagram, we get
A B
n(AB) = no(A) + no(B) + n(AB)
=a–c+b–c+c=a+b–c
 n(AB) = n(A) + n(B) - n(AB) ... (i)
no(A) = a - c no(B) = b - c
If the two sets A and B are disjoint, then n(AB) = 0
n(AB) = c
 n(AB) = n(A) + n(B) - n(AB)
= n(A) + n(B) – 0
 n(AB) = n(A) + n(B) ... (ii)
In this case n(AB) is maximum.
If AB, then n(AB) = n(A), then
n(AB) = n(A) + n(B) – n(AB)
= n(A) + n(B) – n(A) = n(B)
 n(AB) = n(B) ... (iii)
In this case, n(AB) is minimum.

Sets Mathematics - 9 |15


Some other important relations:
1. n(AB) = n(U) - n(AB)
2. no(A) = n(A – B) = n(AB)
also no(A) = n(A) – n(AB)
no(B) = n(B – A) = n(AB)
also no(B) = n(B) – n(AB)

Worked Out Examples


Example 1: If A = {a, b, c, d, e, f} and B = {b, d, f, h, j}, find n(A), n(B), n(AB) and n(AB).
Solution: Here,
A = {a, b, c, d, e, f}
 n(A) = 6
B = {b, d, f, h, j}
 n(B) = 5
AB = {b, d, f}
n(AB) = 3
And n(AB) = n(A) + n(B) - n(AB)
=6+5–3=8
Example 2: If n(A) = 16, n(AB) = 7 and n(AB) = 22, find n(B).
Solution: Here,
n(A) = 16, n(AB) = 7 and n(AB) = 22, n(B) = ?
We have,
n(AB) = n(A) + n(B) - n(AB)
or, 22 = 16 + n(B) – 7
 n(B) = 13.
Example 3: If n(U) = 50, n(A) = 20, n(B) = 26 and n(AB)
____ = 8, find
(i) n(A – B) (ii) n(AB) (iii) n(AB)
Solution: Here,
n(U) = 50, n(A) = 20, n(B) = 26, n(AB) = 8
(i) n(A – B) = n(A) - n(AB) = 20 – 8
 n(A – B) = 12
(ii) n(AB) = n(A) + n(B) – n(AB)
= 20 + 26 – 8
 n(AB) = 38
____
(iii) n(AB) = n(U) – n(AB)
= 50 – 38 = 12.

16 | Mathematics - 9 Sets
Example 4: In a class of 25 students, 17 like volleyball, 15 like basketball and 10 like
both the games. Illustrate the above information in a Venn-diagram and
find the number of students who do not like any of the games.
Solution: Let V and B be the sets of students who like volleyball and basketball respectively.
Here, total number of students n(U) = 25, n(V) = 17, n(B) = 15 and n(VB) = 10
 n(VB) = n(v) + n(B) - n(VB) U
= 17 + 15 – 10 = 22. V B
____ n() = 15
Now, n(VB) = n(U) – n(VB)
= 25 – 22 = 3 n(V)
Therefore, the number of students who
n(V) = 17 n(V) = 10
don’t like both the games is 3.
Example 5: In a class of 50 students, all the students participate in at least one of
the activities music or dance. If 35 students are taking part in music and
45 in dance, find how many students are taking part in both the
activities. Show the information in a Venn-diagram as well.
Solution: Let A and B be the sets of students who participate in music and dance respectively.
Here, total number of students n(U) = n(AB) = 50, n(A) = 35 and n(B) = 45.
We have, U
A B
n(AB) = n(A) + n(B) - n(AB)
n() = 45
n(AB) = n(A) + n(B) - n(AB)
= 35 + 45 – 50 n(A) = 0
= 80 – 50
n(A) = 35 n(A) = 30
= 30.
Therefore, 30 students are taking part in both the activities.
Example 6: In a group of 60 people, 18 drink tea but not coffee, 35 drink tea. If 4 of
them do not drink any of the drinks, find
(i) how many people drink both tea and coffee?
(ii) how many people drink coffee but not tea?
(iii) illustrate the above information in a Venn-diagram.
Solution: Let A and B be the sets of people who drink tea and coffee respectively, then set of people
who drink tea but not coffee is (A – B) and those who drink coffee but not tea is (B – A).
____
Here, n(U) = 60, n(A) = 35, n(A – B) = 18, n(AB) = 4.
Now,
(i) n(AB) = n(A) - n(A - B)
= 35 – 18 = 17
 17 people drink both tea and coffee.
_____
(ii) n(U) = n(A) + n(B) - n(AB) + n(AB)
60 = 35 + n(B) - 17 + 4

Sets Mathematics - 9 |17


or, n(B) = 60 - 22 = 38
 n(B - A) = n(B) - n(AB)
= 38 - 17 = 21
 21 people drink coffee but not tea.
(iii) Venn-diagram:
U
A B
n() = 18
n() = 21
n(B) = 4

n(A) = 35 n(A) = 17

Example 7: In a survey of people 70% like football, 50% like volleyball, 35% like both
the games and 30 people like neither of such games. Find the total
number of people who took part in the survey by drawing venn diagram.
Also find who like only one game.
Solution: Let, F and V denotes the number of people who like football and volleyball respectively.
n() = 100% = x (say)
70x n() = x
n(F) = 70 x of x = 100
70x 50x
n(F) = 100 n(V) = 100
50x
n(V) = 50% of x = 100
35x
n(FV) = 35% of x = 100 35x 35x 15x
_____ 100 100 100
n(FV) = 20
35x 35x 15x
Here, we have, 100 + 100 + 100 + 30 = x ____
n(FV) = 30
15x
or, 100 = 30
 x = 200
Then, Total no. of people = 200
35x 15x 50×200
Number of people who like only one game = 100 + 100 = 100 = 100

Exercise 1.2
1. (a) If A = {2, 4, 6, 8} and B = {7, 8, 9, 10} are two subsets of a universal set
____
U = { x : x  10, x  w}, find n(A), n(B), n(AB) and n(AB) .
(b) If A = {a, b, c, d} and B = {c, d, e, f, g}, verify that n(AB) = n(A) + n(B) - n(AB).
(c) If A = { x : x is a letter in the word “difference”} and B = {x : x is a letter in the word
“Complement”}, find n(AB).

18 | Mathematics - 9 Sets
2. (a) If n(A) = 10, n(B) = 8 and n(AB) = 3, find n(AB).
(b) For two sets A and B, if n(A) = 35, n(B) = 25 and n(AB) = 48, find n(AB).
(c) A and B are subsets of a universal set U. If n(U) = 100, n(A) = 55, n(B) = 65 and
____
n(AB) = 30, find (i) n(AB) (ii) n(only A) (iii) n(AB) .
(d) M and N are subsets of a universal set U. If n(U) = 95, no(M) = 35, no(N) = 25 and
____
n(MN) = 15, find n(MN).
U
P Q
3. (a) The cardinal numbers of the respective sets are shown in the 28
given Venn-diagram. Find the cardinal number of the
universal set U. 10
25 13

(b) Cardinal numbers of the respective sets are shown in the


U
Venn-diagram given alongside. Find n(AB).
A B
no(B) = 5x
n(AB) = 5
n(AB) = 14
no(A) = 3x

(c) Study the Venn-diagram and find no (A) and no (B),


where n(U) = 200 is provided.

(d) The cardinalities of the respective sets are given in


____
the Venn-diagram. If n(AB) = 18, find the
cardinality of the universal set U.

4. (a) If AB, n(A) = 20 and n(B) = 45, find the value of n(AB) and n(AB).
____
(b) If AB, n(B) = 40 and n(AB) = 15, find n(U).
____
(c) If UBA, n(U) = 60 and n(B) = 15, find the value of n(AB) .
(d) If n(A) = 10 and n(B) = 15, what may be the minimum and maximum value of
(i) n(AB) and (ii) n(AB)?
(e) If n(A) = 9 and n(B) = 6, find the maximum and minimum value of (i) n(AB)
(ii) n((AB).

Sets Mathematics - 9 |19


5. (a) In a survey of 100 persons of a locality, it was found that 63 could speak Tamang, 59
could speak Newari and 32 could speak both the languages.
(i) How many could speak none of the two languages?
(ii) How many could speak only Tamang language?
(iii) Illustrate the above information in a Venn-diagram.
(b) Out of 90 civil servants, 65 were working in the office, 50 were working in the field,
and 35 were working in both the premises.
(i) How many civil servants were absent?
(ii) How many civil servants were working in the field only?
(iii) Represent the above information in a Venn-diagram.
(c) A survey of 200 students revealed that 100 didn't like to play football, 120 did not like
to play basketball and 40 did not like to play any of the games.
(i) How many students liked to play both the games?
(ii) How many students were interested in these games?
(d) In a survey of a group of people, it was found that 62 people didn't like to drink tea, 85
didn't like coffee, 33 liked both and 36 didn’t like both the beverages.
(i) Illustrate the above information in a Venn-diagram.
(ii) How many people were surveyed?
(iii) How many people liked to drink tea?
6. (a) In a survey of 500 tourists who visited Nepal, it was found that 300 visited Pokhara and
340 visited Lukla and every tourist visited at least one place.
(i) How many tourists visited both places?
(ii) How many of them visited only one place?
(iii) Represent the above information in a Venn-diagram.
(b) Students having computers at their homes own at least one of the popular computer
games Alien Invaders and Race Car Derby, 40 have Alien Invaders, 45 have Race Car
Derby and 25 have both the games.
(i) Illustrate the above information in a Venn-diagram.
(ii) How many students have only one game?
(iii) How many students have computers?
(c) A class of 42 students take part in at least one of the activities sports and music. If 30
take part in sports and 24 in music, find how many students take part in both activities.
Also find how many students take part in sports only and music only. Represent the
result in a Venn-diagram.
(d) A group of students play either cricket or football or both. 28 play cricket, 30 play
football, and 18 play both the games,
(i) Illustrate the above information in a Venn-diagram.
(ii) How many students are there in the group?

20 | Mathematics - 9 Sets
7. (a) In a class of 60 students, 15 students liked Maths only, 20 liked English only and 5 did
not like both of the subjects.
(i) Find the number of students who like both.
(ii) Find the number of students who liked at least one subjects.
(iii) Illustrate above information in a Venn-diagram.
(b) Out of 100 students, 80 passed in science, 71 in Mathematics, 10 failed in both subjects
and 7 did not appear in the examination. Find the number of students who passed in
both subjects by representing the above information in a Venn-diagram.
(c) In a survey of 53 people, 20 like Nepali TV Serial, 25 like the Hindi TV serial and 10
did not like both TV Serials. If 3 people did not use to watch the TV, find the number of
people who like both serials by using a Venn-diagram.
(d) In an examination, 35% of students passed in social studies only, 25% passed in health
only. If 10% failed in both subjects,
(i) what percent of students passed in both subjects?
(ii) what percent of students passed in social studies?
(iii) represent the result in a Venn-diagram.
8. (a) In a survey of some customers, it was found that 65% preferred laptop and 85% preferred
desktop computers. If there were not any customer who did not like both computer,
(i) what percent were there who preferred both the computers?
(ii) illustrate the above information in a Venn-diagram.
(b) In a survey among some people of a group. It was found that 25% of them liked
Literature only, 50% of them liked Music only and 10% of them liked none.
(i) What percent of people liked both?
(ii) What percent of people liked music?
(iii) Illustrate these information in a venn-diagram.
(c) In an examination, 70% of the students passed science, 60% passed mathematics and 12
students passed both subjects. If 10% failed in both the subjcts.
(i) find the percent of students who passed both the subjects.
(ii) find the total number of students.
(iii) represent the above information in a Venn-diagram.
(d) In a survey of community who watch television, 40% like Jire Khurshani, 50% like
'Tito Satya' and one fifth people like none of them.
(i) Draw a Venn-diagram of the above information.
(ii) What percentage of them like both serial?
9. (a) In a survey of bus travelers, 60% liked to take Day bus while 75% liked Night bus for
long tour. If 72 travelers liked to take Night bus only,
(i) how many travelers were surveyed?
(ii) what percent liked both busses?
(iii) illustrate the above information in a Venn-diagram.
(b) In a group of students, 50% liked tea, 70% liked coffee, 10% did not like both and 120
like both. By using a Venn-diagram, find the total number of students.

Sets Mathematics - 9 |21


(c) In a survey of a group of tourists it was found that 70% liked to visit Chitwan, 60%
liked to visit Pokhara, 40% liked to visit both the places and 40 tourists did not like to
visit both places. By using a Venn-diagram, find the total number of tourists, who took
part in the survey.
(d) In a survey of internal tourists, 20% of tourists liked Sauraha only, 50% liked Lumbini
only and 10% tourist did not like both the place. If 200 tourists liked Sauraha, find the
total number of tourists in the survey by drawing a Venn-diagram.
10. (a) In a survey of students of New Horizon School the number of students who liked maths
is two thirds of the number of students who liked Nepali, 10% liked maths only, 30%
liked Nepali only. Find the percent of the students who did not like both the subjects by
using a Venn-diagram.
(b) In a survey of a school, the number of students who liked maths is one third of students
who liked science, 2% liked Maths only, 30% liked science only. Find the percent of
student who did not like both by using a Venn-diagram.
(c) In a survey of 20,000 students of class ten in Chitwan district, 18% of the students take
tuition before S.L.C. examination. Out of them, 40% take tuition in English only, 10%
in maths only and 20% in other subjects, find the number of students who take tuition in
English and maths both subjects.
11. (a) In a survey, it was found that the ratio of the people who like modern songs and folk
songs is 8:9 out of which, 50 people liked both songs, 40 liked folk songs only and 80
liked none of the songs.
(i) Show the above information in a Venn-diagram.
(ii) Find the number of people who participated in the survey.
(b) In a survey of 120 people, the number of people who like to rear. Only cows is three
times the number of people who like to rear only buffalo. If 10 people liked to rear both
and 30 people did not like to rear both then,
(i) find the number of people who liked to rear cows.
(ii) illustrate these information in a Venn-diagram.
(c) Out of 50 people, the number of people who like only Dashain Festival is 10 less than
two third of people who like only Tihar Festival. If 10 people like both Festivals and 20
did not like both festival then,
(i) represent the above information in a Venn-diagram.
(ii) find the number of people who like Dashain.
(d) In a survey, one-fifth students like maths only, 30 students didn’t like maths, If 35
students like science and 25 like none of them,
(i) show the above information in a Venn-diagram.
(ii) how many students like maths?
12. (a) In a survey two third people like to eat 'Dhido' only, 12 people did not like 'Dhido', 20%
people like to eat rice and 8 people did not like to eat them.
(i) Show the above information in a Venn-diagram.
(ii) How many people like to eat both the things?

22 | Mathematics - 9 Sets
(b) From the adjoining Venn-diagram, if n() = 60, find U
A B
(i) n(AB) ii) n(A – B)
39 4x y x 24
____
(iii) n(AB)
(c) In a survey of some people, the ratio of people who like milk but not curd and who like
curd but not milk is 6:5. The ratio of people who like both and dislike both is 7:9. If
50% did not like milk and 44 people like only one of them, find the number of people
that participated in the survey by using a Venn-diagram.

Unit Test
Time: 40 minutes F.M.- 24

1. In a class of 120 students, 95 like account, 80 like biology. If there are none who don’t like
both subjects, find
(a) The number of students who like both subjects.
(b) The number of students who like account only.
(c) Show the above information in a venn- diagram.
2. In a survey of 100 people, it was found that 65 like folk song, 55 like modern song and 35
like folk as well as modern song. Then
(a) Draw a Venn-diagram to show the above information.
(b) How many people did not like both the songs?
3. Out of 100 students in an examination of class 9, 70 passed in Mathematics, 60 passed in
Science and 20 failed in both subjects. Find the number of students who passed in both
subjects by using a Venn-diagram.
4. In a survey of 200 students, 30 liked neither to sing nor to dance, 60 liked to song only and
50 liked to dance only. Then
(a) Show the above information in a Venn-diagram.
(b) Find the number of students who can do both the sing as well as dance.
_____
5. If n(A) = 65, n(B) = 50, n(AB) = 30 and n(AB) = 17, find n(U) and and show the result
in a Venn-diagram.
_____
6. If n(U) = 120, n0(A) = 50, n0(B) = 40, n(AB) = 25, find n(AB) and n(AB).

Sets Mathematics - 9 |23


Answers ____________________________________________________________
Exercise 1.1
1. (a) {1, 2, 3, 4, 5, 7, 11} (b) {2, 3, 5} (c) {7, 11} (d) {1, 4, 6, 8, 9, 10, 12}
2. (a) {0, 1, 4, 6, 7, 8, 9} (b) {4, 8} (c) {3, 5} (d) {1, 3, 5, 7}
3. (a) {k, l} (b) {b, d, e, g, h, i} (c) {b, e} (d) {b, c, d, e, g, h, i}
4. (a) B – A (b) (AB) (c) (A) (d) (AB) – (AB)

5. (a) A(BC) ¯¯¯¯¯¯¯¯


(b) (ABC) (c) (AB)(AC)(BC) (d) (AB)(BC)
6. (a) {1, 2, 3, 4, 5, 9, 10} (b) {1, 4, 5, 6, 7, 8, 9, 10} (c) {6, 7, 8} (d) {1, 9, 10}
(e) {1, 2, 3, 6, 7, 8, 9, 10} (f) {2, 3, 4, 5}
7. (a) {k, l} (b) {c, e, g, j} (c) {g, h, i, j} (d) {a, f, k, l}
_____ __ __
9. (c) (AB) = A  B (d) De-Morgan’s law
11. (b) 45, 45°, 90°
Exercise 1.2
1. (a) 4, 4, 1, 4 (c) 12
2. (a) 15 (b) 12 (c) (i) 90 (ii) 25 (iii) 10 (d) 20
3. (a) 50 (b) 94 (c) 58, 78 (d) 72
4. (a) 45, 20 (b) 55 (c) 45 (d) (i) 15, 25 (ii) 0, 10
(e) (i) 15, 9 ( ii) 6.0
5. (a) 10, 31 (b) 10, 15 (c) 20, 160 (d) 144, 82
6. (a) 140, 360 (b) 35, 60 (c) 12, 18, 12 (d) 40
7. (a) (i) 20 (ii) 55 (b) 68 (c) 5
(d) (i) 30% (ii) 65%
8. (a) 50% (b) (i) 15% (ii) 65% (c) (i) 40% (ii) 30 (d) 10%
9 (a) (i) 180 (ii) 40% (b) 400 (c) 400 (d) 500
10. (a) 30% (b) 56% (c) 1080
11. (a) 200 (b) 70 (c) 12 (d) 45
12. (a) 8 (b) (i) 19 (ii) 20 (iii) 16 (c) 76

24 | Mathematics - 9 Sets
Chapter

2
Arithmetic
Objectives:
At the end of this chapter, the
students will be able to:
 collect and solve the problems
related to discount, profit and
loss, commission, taxation,
dividend and bonus.
 calculate the electricity bill,
water bill, telephone bill and taxi
fare.

Teaching Materials:
Bills of grocery with discount, charts
of rule of tax published by government,
electricity bill, water bill, telephone
bill of a household, rate list of taxi
fare published by government.
2.1 Profit and Loss
The terms used in the transaction of goods while buying and selling are:
Cost price (C.P) : The price for which an article is purchased.
Selling price (S.P) : The price for which an article is sold.
If the selling price is more than the cost price, there is profit.
 Profit (P) = S.P – C.P
or, S.P = C.P + P
or, C.P = S.P – P
If the selling price is less than the cost price, there is loss.
 Loss (L) = C.P – S.P
or, S.P = C.P – L
or, C.P = S.P + L
To distinguish between a profit and profit percentage, let’s consider an example:
Raj bought an article for Rs. 50 and sold for Rs. 75. Then he made a profit of Rs. 25. Similarly, Jay
purchased an article for Rs. 200 and sold for Rs. 225. Then, he also made a profit of Rs. 25. Though,
both of them made same amount of profit but their profit percentage is different. Let's see how,
When C.P is Rs. 50, Raj made a profit of Rs. 25.
25
When C.P is Re. 1, Raj made a profit of Rs. 50 .

25 50
When C.P is Rs. 100, Raj made a profit of Rs. × 100
50 2

 Profit percentage of Raj is 50%.


Similarly,
When C.P is Rs. 200, Jay made a profit of Rs. 25.
25
When C.P is Re. 1, Jay made a profit of Rs. 200

25
When C.P is Rs. 100, Jay made a profit of Rs. 200 × 100

 Profit percentage of Jay is 12.5%


Therefore, profit and profit percentage are two different concepts. Similarly loss and loss
percentage are two different things.
Profit
 Profit Percentage = C.P × 100%

26 | Mathematics - 9 Arithmetic
If we simplify it, we get
S.P 100 + P %
C.P = 100
Similarly,
Loss
Loss percentage = C.P × 100%

S.P 100 - L %
and =
C.P 100
From the illustration above it is understood that profit percentage or loss percentage is calculated on
the cost price.
Things to remember
 Overhead expenses like repair, maintenance, transportation and other expenditure are added to the cost price to
get the total cost price. The profit percent or loss percent is calculated on the total cost price.
 To calculate the profit or loss when CP and SP of different number of articles are given, we have to calculate the
CP and SP of equal number of articles.
 While calculating profit or loss in case of break, leakage, lost on the way, damage etc, SP should be made with
number of articles in good condition.

Worked Out Examples


Example 1: If the cost price of 8 articles is equal to selling price of 6 articles, find the
profit percent.
Solution: Here,
Cost price of 8 articles = Selling price of 6 articles = Rs. x (say)
x
 C.P of 1 article = Rs. 8 ,

And
S.P of 6 articles = Rs. x
x
S.P of 1 article = Rs. 6

As, S.P > C.P, there is profit.


Profit (P) = S.P – C.P
x x 4x - 3x x
= 6 – 8 = 24 = Rs. 24

x
P 24
Then, Profit percent = CP × 100 = x × 100
8

Arithmetic Mathematics - 9 |27


8 1
= x × x ×100 = 333 %
24 3
1
Hence, the profit percentage is 333 %.

Example 2: A stationer bought 3000 books at Rs. 125 each. Later, he found that 500
books were damaged and he sold the remaining books at Rs. 175 each.
Find his gain or loss percent.
Solution: Here,
Cost price of 1 book = Rs. 125
C.P of 3000 books = Rs. 125 × 3000 = Rs. 3,75,000
No. of books damaged = 500.
No. of good books = 3000 – 500 = 2500
Selling price of 1 book = Rs. 175
S.P of 2500 books = Rs. 175 × 2500 = Rs. 4,37,500
As S.P > C.P, there is profit.
 Profit (P) = S.P – C.P
= 4,37,500 – 3,75,000= Rs. 62,500
Then,
P 62500 2
Profit percent = C.P × 100 = 375000 × 100 = 163

2
Hence, the profit percentage is 16. 3 %

Example 3: Avay sold a mobile set for Rs. 32,900 and made 6% loss. At what price
should he sell the mobile set to make 6% profit?
Solution: Here,
Case I Case II
Selling price (S.P) = Rs. 32,900 Cost price (CP) = Rs. 35,000
Loss percent = 6 Profit percent (P%) = 6
We have, Selling price (SP) =?
S.P 100 - L % We have,
C.P = 100 S.P 100 + P %
32900 100 - 6 C.P = 100
or, CP = 100
SP 100 + 6
or, 35000 = 100
32900
or, 0.94 = C.P S.P 106
or, 35000 = 100
or, C.P = Rs. 35,000
or, S.P = Rs. 37,100

Hence, Avay should sell the mobile set for Rs. 37,100 to have 6% profit.

28 | Mathematics - 9 Arithmetic
Example 4: Yash bought two music systems for Rs. 52,000. He sold them making 6%
profit on one and 6% loss on the other. If their selling price is same, find
his total gain or loss percent.
Solution: Here,

For the 1st music system For the 2nd music system
Let, Cost price (CP) = Rs. x Cost price (CP) = Rs. (52,000 - x)
Profit percent (P%) = 6. Loss percent (L%) = 6
We know, We know,
SP 100 + P% S.P 100 - L%
CP = 100 C.P = 100
SP 100 + 6 SP 100 - 6
or, x = 100 or, 52000 - x = 100

106x 94(52,000 - x)
or, SP = 100 or, S.P = 100
But, their selling price is same.
106x 94(52,000 - x)
 100 = 100
or, 106x = 4888000 - 94x
or, 200x = 48,88,000
or, x = Rs. 24440
106x 106 × 24440
 S.P of the music system = 100 = 100 = Rs. 25906.40

 Total selling price (SP) = Rs. 51,812.80


Total cost price (CP) = Rs. 52,000
As C.P > S.P, there is loss.
 Loss = C.P - S.P
= Rs. 52,000 – Rs. 51,812.80 = Rs. 187.20
L 187.20
 Loss percent (L%) = C.P × 100 = 52000 × 100 = 0.36%

Hence, the loss percentage is 0.36%.


Example 5: A sold an article to B at a profit of 10%. B sold the same article to C at a
profit of 15%. If C paid Rs. 3162.50 to B, how much did A pay for it?
Solution: Here,
For B (According to question)
Profit percent (P%) = 15
Selling price of B is the cost price of C.
 S.P = Rs. 3162.50.

Arithmetic Mathematics - 9 |29


We know
S.P 100 + P %
C.P = 100
3162.50 115
or, C.P = 100
3162.50
or, C.P = 1.15
or, C.P = Rs. 2750.
Hence, A paid Rs. 2500 for the article.
For A (According to question)
Selling price of A is the cost Price of B
 S.P = Rs. 2750
Profit percent (P%) = 10
Now,
S.P 100 + P %
C.P = 100
2750 100 + 10
or, C.P = 100

2750
or, 1.1 = C.P

 C.P = Rs. 2500.


Example 6: An article when sold at a profit of 15% yields Rs. 380 more than when
2
sold at a loss of 16 %. What was the cost price of the article?
3
Solution: Here,
Case I Case II
Let, Cost price (CP) = Rs. x 2 50
Loss percent (L%) = 163 = 3
Profit percent (P%) = 15
We know, C.P = Rs. x
S.P 100 + P % We know,
C.P = 100 S.P 100 - L %
C.P = 100
S.P 100 + 15
or, x = 100
100 - 50x
115x  3
or, S.P = 100 or, S.P = 100
250x
or, S.P = 300

But, the difference in the S.P is Rs.380.


115x 250x
 100 – 300 = 380

30 | Mathematics - 9 Arithmetic
345x - 250x
or, 300 = 380

or, 95x = 380 × 300


or, x = Rs. 1200
Hence, the cost price of the article is Rs. 1200.
Example 7: Sunny bought 100 pens at Rs. 75 each. He sold first 30 pens at Rs. 80
each, and next 50 pens at Rs. 90 each. At what price should he sell the
remaining pens so as to make 20% profit on the total outlay?
Solution: Here, Cost price (CP) of 1 pen = Rs. 75
C.P of 100 pens = Rs. 75 × 100 = Rs. 7500
Selling price (SP) of 1 pen = Rs. 80
S.P of first 30 pens = Rs. 80 × 30 = Rs. 2400
S.P of 1 pen = Rs. 90
S.P of next 50 pens = Rs. 90 × 50 = Rs. 4500
Number of remaining pens = 100 – (50 + 30) = 20
Let, S.P of 1 pen = Rs. x
Then S.P of remaining 20 pens = Rs. 20x
Total S.P = Rs. (2400 + 4500 + 20x) = Rs. (6900 + 20x)
P % = 20
We have,
S.P 100 + P %
C.P = 100
6900 + 20x 100 + 20
or, 7500 = 100

or, 6900 + 20x = 120 × 75


or, 20x = 2100
or, x = 105
Hence, the remaining 20 pens should be sold at the rate of Rs. 105 per pen so as to
have a profit of 20% in total outlay.
Example 8: A crooked shopkeeper uses false balance. He cheats 20% while
purchasing goods and 20% while selling the goods. What is his profit
percentage by cheating?
Solution: Here, Let, the shopkeeper wants to buy 100 kg goods using a false balance. At the
time of purchasing he cheats 20% means he receives 120 kg goods.
Again, he uses false balance and cheats 20% while selling the goods. So his profit
20
= 20% of 120kg = 100 × 120 kg= 24 kg

His total profit by cheating = (20 + 24) = 44 kg


44
i.e. Profit percentage = 100 × 100 % = 44%

Arithmetic Mathematics - 9 |31


Exercise 2.1
1. (a) Jay bought an item for Rs. 50 and sold making a profit of 10%. Find out the selling
price of the item?
(b) A retailer sold a shirt for Rs. 1000 and he made a profit of 25%. Find the purchased
price of the shirt.
(c) Raj bought a guitar for Rs. 1500. After using for a few days, he sold it and had a loss of
20%. Find the price for which the guitar was sold.
(d) A sold an article for Rs. 810 so that he could have a loss of 10%. Find his cost price.
2. (a) Chunu bought a sound system for Rs. 25,000. He sold it for Rs. 30,000. Find his profit
percentage.
(b) Amar bought a motor-cycle for Rs. 1,50,000. After using for a few months, he sold it
for Rs. 1,25,000. Find his loss percentage.
(c) If the C.P of 10 articles is equal to S.P of 15 articles, find the loss percent.
(d) If the C.P of 12 pens is equal to S.P of 8 pens, find the gain percent.
3. (a) A shopkeeper bought 2000 eggs at Rs. 5 each. On the way, 400 eggs were broken and
he sold the remaining eggs at Rs. 8 each. Find his gain or loss percent.
(b) A fruit seller bought 1500 apples at Rs. 4 per apple. Later on he found that 300 apples
were bad. He sold the remaining apples and gained 20%. Find the price at which each
apple was sold.
(c) Jay sold a T-shirt for Rs. 660 and made 10% profit. At what price should he sell to have
15% profit?
(d) Bobby sold a fan for Rs. 950 and had 5% loss. At what price should he sell the fan to
have 6 % profit?
4 (a) By selling a watch in Rs 150 and 200 loss and profit are happened respectively. If the
loss and the profit are equal, find it's cost price.
(b) By selling an article in Rs 16000 and Rs 19000 some loss and some profit are happened
respectively. If loss is one – fourth of profit, find its cost.
(c) An article is sold for Rs 260 at a gain. Had it been sold for Rs 200 there would have a
loss equal to 50% of the original gain, find the cost price of the article.
5. (a) By selling 75 apples a seller gains the selling price of 15 apples , find his gain
percentage.
(b) Rabin bought a second hand bike for Rs 50,000. He spent Rs 5000 on its repair. If he
sold it for Rs 60000, find his gain or loss percentage.
(c) An article is bought and sold at a profit one – third of selling price. Find the profit
percentage.
(d) A man sells an article at Rs 600 and makes a profit of 20% of selling price. Calculate
his actual profit percentage.
6. (a) A man bought two shirts for Rs. 1200 each. On selling the first shirt, he had 10% profit
and 5% loss on the second shirt. Find his gain or loss percent on whole.

32 | Mathematics - 9 Arithmetic
(b) A man bought a pair of shoes for Rs. 1500. He sold it to Yash at a profit of 15%. Yash
sold it to Karan at a loss of 10%. How much did Karan pay for it?
(c) Jay bought two watches for Rs. 10,000. He sold them to have 5% profit on one and 5%
loss on the other. If their selling price was the same, find his total gain or loss percent.
(d) A sold an article to B at 20% profit. B sold the same article to C at 25% profit. Again, C
sold the same article to D at 10% loss. If D paid Rs. 4500 to C, at what price did A
purchase the article?
7. (a) A shopkeeper sells a pant at 10% profit. If he would have sold it with Rs. 150 more, the
profit would have been 15%. Find the purchased price of the pant.
(b) An article when sold at 10% profit yields Rs. 200 more than when sold at 10% loss.
What was the cost price of the article?
(c) A man bought some oranges at the rate of 20 for Rs. 100. If he sold all of them at a
profit of 20%, then how many oranges did he sell for Rs. 30?
(d) Yash bought a certain number of apples at Rs. 100 per 20 apples and equal number at
Rs. 180 per 30 apples. He mixed them and sold them at Rs. 175 per 25 apples. Find his
gain or loss percent.
8. (a) A shopkeeper bought 150 books and sold 100 of them at a profit of 30% and the rest at
10% loss. If he had sold all the books at once at 25% profit, he would have received
Rs. 300 more. Find the cost price of each book.
(b) A man bought 120 pens for Rs. 7200. He sold 40 pens at Rs. 70 each and 60 pens at
Rs. 80 each. At what price should he sell the remaining pens so as to make a profit of
20% in total investment?
(c) A dealer bought 80 purses at Rs. 70 each. He sold 20 purses at Rs. 80 each and next 40
purses at Rs. 75 each. At what price should he sell the remaining purses so as to have
30% profit on the total outlay?
(d) Manju bought a bale of cloth at Rs. 120 per meter. She sold 40m cloth at Rs.105/m and
the remaining at Rs. 150/m and made 20% profit. How many meters of cloth had she
bought in total?

2.2 Commission
In many business, a third party plays a role between a buyer and seller. The third party is known as
agent. The agent is paid certain sum for his service. The sum paid to the agent is known as
commission. An insurance company pays 4% commission to the agent. The T.V manufacture pays
15% commission to the dealer. It means commission is calculated as a certain percentage of selling
price.
 Commission amount = Commission % of selling price
Net S.P. = S.P. – Commission % of S.P.

Arithmetic Mathematics - 9 |33


Worked Out Examples
Example 1: The monthly salary of a salesman is Rs 5000. The departmental store
gives 2% commission on the total sales. If the total sales of a certain
month is Rs 1,50,000, what is the monthly income of that month of the
salesman?
Solution: Commission rate = 2%
Total sales = Rs 1,50,000
Commission = 2% of Rs. 1,50,0000
2
= 100 × 1,50,000 = Rs 3000

Monthly salary = Rs 5000


Total income of the month = Salary + Commission
= 5000 + 3000 = Rs 8000
Example 2: A land broker sold a plot of land for Rs 95,00,000. What is the
commission received by him from the land owner at the rate of 6%. Also,
find the net sum received by the land owner.
Solution: Selling price of the land = Rs 95,00,000
Commission rate = 6%
Commission = 6% of 95,00,000
= Rs 5,70,000
Net sum received by the land owner = 95,00,000 – 5,70,000
= Rs 89,30,000.
Example 3: A motor cycle dealer sold a motor cycle for Rs 1,50,000. The dealer gave
Rs 7500 as commission to the salesman. Find the rate of commission.
Solution: Selling price = Rs 1,50,000
Commission = Rs 7500
Commission rate = x % (say)
We have,
Commission = x % of 1,50,000
x
or, 7500 = 100 × 1,50,000

 x = 5.
Hence, the commission rate is 5%.
Example 4: A landowner sold a plot of land for Rs 90,00,000. He had an agreement
with land broker to pay the commission as 1% for first Rs 30,00,000.
0.5% for the next Rs 45,00,000 and 0.25% for the remaining sum. What is
the commission paid by the land owner to the broker and also the sum
received by the land-owner?

34 | Mathematics - 9 Arithmetic
Solution: Selling price = Rs 90,00,000
= Rs 30,00,000 + Rs 45,00,000 + Rs 15,00,000
Commission = 1% of 30,00,000 + 0.5% of 45,00,000 + 0.25% of 15,00,000
= 30,000 + 22,500 + 3750
= Rs 56,250
Net sum received by the land owner = Rs. 90,00,000 – Rs. 56,250
= Rs 89,43,750
Hence, the commission received by the land broker is Rs 56250 and the net sum
received by the land owner is Rs 89,43,750.

Exercise 2.2
1. (a) A real estate agent sold a piece of land for Rs 32,00,000. What was the commission
received by him at the rate of 10%?
(b) An insurance agent paid a yearly premium of Rs 14,000 of his customer. What
commission did he receive at the rate of 5%?
(c) A company salesman’s monthly salary is Rs 12,000. He gets 5% commission on the
total sales. If the total sales of a certain month is Rs 4,50,000, find his income of that
month.
(d) A girl working in a departmental store has monthly salary Rs 6000. She gets 2%
commission on total sales. Find her income of that month if the total sales is Rs
2,50,000.
2. (a) An agent receives Rs. 4,50,000 as commission on selling a Japanese car. If the
commission rate is 10%, at what price was the car sold?
(b) A land broker received Rs. 5,12,000 as a commission. At what price did he sell the land
if the commission rate was 8%?
(c) A mechanics was able to sell a customer’s motor-cycle to a motor cycle exchange
company for Rs. 45,000. If the mechanics received Rs. 2700 as commission from
exchange company, find the rate of commission.
(d) An agent sold a Toyota car for Rs 70,00,000. He got Rs. 3,50,000 as commission. Find
the commission rate.
3. (a) Yash’s house was sold for Rs. 65,00,000 by an agent. The agent was paid 5%
commission by the seller Yash. What is the net sum received by Yash?
(b) A land owner’s land was sold by a broker for Rs. 80,00,000. The broker received 9%
commission from the land owner. What is the net sum received by land-owner?
(c) A sales man’s monthly salary is Rs. 8000. He gets 8% commission on the total sales. If
his income of a certain month is Rs. 17,000, what is the total sales of that month?
(d) The monthly salary of a boy working in a company is Rs 6000. He gets extra 6%
commission on the total sales. If he gets Rs 12,000 on a certain month, what is the sales
of that month?

Arithmetic Mathematics - 9 |35


4. (a) A man sold his plot of land for Rs. 50,00,000 with the help of a broker. They had an
agreement of paying commission of 2% on first Rs. 15,00,000, 1% on next
Rs. 15,00,000 and 0.5% on rest of the money. What is the commission received by the
broker?
(b) The company’s rule for commission is as follows.
Monthly sales Commission rate
Re 1 – Rs 10,00,000 1%
Rs 10,00,000 – Rs 30,00,000 1.5%
Above Rs 30,00,000 2.5%
Calculate the commission received by the sales agent from the sales of Rs 50,00,000.

2.3 Discount
Discount
The price on which a seller wishes to sell an article is known as marked price or tag price or quoted
price or list price or labelled price.
The deduction or concession given in the marked price is known as discount.
 Discount (D) = Discount percentage of marked price and discount = marked price – selling price
Therefore, discount is calculated on the marked price.
Discount amount
Rate of discount = Marked price × 100%

But, if there is no discount, then selling price = marked price. So, if there is discount,
S.P = MP – D.

Worked Out Examples


Example 1: An article is marked to sell at Rs. 1300; if 10% discount is allowed, find
the discount amount and the selling price.
Solution: Here,
Marked price (M.P) = Rs. 1300
Discount rate = 10%
Discount amount (D) =?
Selling price (S.P) =?
We have,
10
Discount amount = 10% of M.P = 100 × 1300 = Rs. 130
Selling price, (S.P) = M.P – D
= 1300 - 130
= Rs. 1170.

36 | Mathematics - 9 Arithmetic
Example 2: From the given condition, find the discount amount and selling Price.
Discount 10%  Rs. 25,000/-

Solution: Here,
Marked price of the mobile set (M.P.) = Rs. 25,000
Discount rate = 10%, Discount amount =? and Selling Price =?
Now,
Discount amount = 10% of M.P.
10
= 100 × 25000 = Rs. 2500

Again,
Selling Price (S.P.) = M.P. – Discount
= 25000 – 2500 = Rs. 22,500
Example 3: A motor-bike is marked to sell for Rs. 1,80,000. The buyer got 10%
discount on the occasion of Tihar and later on 5% further discount on
cash payment. Find the price of the motor-bike purchased.
Solution: Here,
Marked price (M.P) = Rs. 1,80,000
S.P after 10% discount = MP – 10% of MP
= 1,80,000 – 10 × 1,80,000
100
= 1,80,000 – 18000 = Rs. 1,62,000
The customer got 5% further discount on cash payment. This discount is given on S.P
after 10% discount.
 Final S.P = 1,62,000 - 5 × 1,62,000
100
= 1,62,000 – 8100
= Rs. 1,53,900
Hence, to purchase the motor-bike, the buyer has to pay Rs. 1,53,900.
Example 4: If a shopkeeper allows a discount of 20% in an article, he loses Rs. 600.
But if he allows a discount of 5%, he gains Rs. 1575. Find the marked
price and the cost price of the article.
Solution: Here,
Case I
Let, Marked price = Rs. x
Discount = 20%, Loss = Rs. 600
 Selling price = M.P – D
= x – 20% of x = Rs. 0.80x
Now, L = C.P – S.P

Arithmetic Mathematics - 9 |37


or, 600 = C.P – 0.80x
or, C.P = Rs. (600 + 0.8x).
C.P = (600 + 0.8x)
= 600 + 0.8 × 14500
= Rs. 12,200
Case - II
Marked price = Rs. x
Cost price = Rs. (600 + 0.8x)
Discount = 5 %
Profit = Rs. 1575
Now, S.P = MP - D
= x – 5% of x = Rs. 0.95x
Again, P = S.P - C.P
or, 1575 = 0.95x – 600 – 0.8x
or, 2175 = 0.15x
or, x = 14,500
MP = Rs. 14,500
Example 5: Anish sold a computer at the marked price, making a profit of 40%. If
10% discount was allowed the profit would have been Rs. 6500. Find the
cost price of the computer.
Solution: Here,
Case – I Case– II
As he sold the computer on marked D = 10%
price, there is no discount. P = Rs. 6500
 Selling price = marked price 7x
MP = Rs. 5
Profit percent = 40
Let, cost price = Rs. x CP = Rs. x
2
We have 7x
SP = MP – D = 5 - 10 × 7x
S.P 100 + P % 100 5
C.P =
50
100
70x -7x
MP 100 + 40 = 50
or, x = 100
63x
= Rs. 50
MP 140 7
or, x = 100
5
7x
or, MP = Rs. 5

Now,
Profit = S.P – C.P

38 | Mathematics - 9 Arithmetic
63x
or, 6500 = 50 - x
500
or, 6500 = 13x
50
or, C.P = Rs. 25000.
Hence, the cost price of the computer is Rs. 25,000.

Exercise 2.3
1. (a) An article is marked to sell at Rs. 1000. If 20% discount is allowed, find the discount
amount and the selling price.
(b) A toy bought for Rs. 600 is marked to sell at Rs. 900. If 15% discount is allowed, what
is the profit?
(c) A pant bought for Rs. 1000 is marked to sell at Rs. 1500. Because of small damage,
40% discount was given. Find the loss percentage.
2. (a) In an article, 20% discount is given. If the customer paid Rs. 640 for the article, find the
marked price.
(b) An electric iron labelled Rs. 1200 was sold for Rs. 900 giving a certain discount, find
the discount rate.
3. (a) The marked price of a pressure cooker is Rs. 1800. Two successive discounts 10% and
5% are given. Find its selling price.
(b) A sound system is marked to sell at Rs. 28000. Two successive discounts 12% and 5%
are allowed. Find its selling price.
4. (a) If a shopkeeper allows a discount of 25%, he loses Rs.500. But if he allows a discount
of 5%, he gains Rs. 1500. Find the marked price and the cost price.
(b) A man sells a D.V.D player at a discount of 8% and makes a profit of Rs. 780. If he
doesn't allow a discount he would gain a profit of Rs. 1500. Find the marked price and
cost price of the D.V.D player.
(c) Raj marked the price of a mobile set to have a gain of Rs. 500. If he allows a discount of
8%, he will have Rs. 140 loss. Find the marked price of the mobile set.
5. (a) If an i-pod is sold at the marked price, there is a profit of 16%. If 15% discount is
allowed, there is a loss of Rs. 350. Find the cost price of the i-pod.
(b) Yash bought an item for Rs. 8000 after a discount of 20% on marked price. If he sells
the item at the marked price, find his profit percent.
(c) Mana paid Rs. 6800 to purchase an article after a discount of 15% on the marked price.
If he sells the article taking Rs. 500 more than the marked price, what is the profit
percent?
6. (a) A shopkeeper bought a shirt for Rs. 2500. He fixed the price of the shirt to make a profit
of 20% after allowing a discount of 20%. Calculate the marked price.

Arithmetic Mathematics - 9 |39


(b) A suit piece after allowing a discount of 10% was sold at 20% profit. Had it been sold
after allowing 15% discount, there would have been Rs. 800 profit. Find the cost price
of the suit piece.
(c) An article is sold at Rs. 2080 with 20% discount. Find the cost price where marked
price is 30% above the cost price.
7. (a) A trader fixes his goods at 20% above the cost price. He allows 15% discount. If the
marked price is Rs. 14400, find his profit percentage or loss percentage.
(b) A T.V set is marked to sell at a profit of 10%. If the shopkeeper sells it at a discount of
2
Rs. 500, then there is a profit of 63 %, find the cost price.

(c) Price of an article is marked 40% above the cost price. If it is sold giving 20% discount,
what will be the profit percentage?

2.4 Taxation
The sum of money paid by people or by organization, company etc. to the government for public
purposes is known as tax. There are many types of taxes, eg. income tax, excise tax, property tax etc.
We are going to deal with income tax.

Income tax
If the income of a person exceeds a specified amount, a tax is imposed on the income above the
specified amount. The specified amount on which tax is not imposed is known as tax allowance. Also,
provident fund, citizen investment fund and insurance premium are tax free amount. The income
above the tax allowance is known as taxable income.
Taxable income = Total income – Tax allowance.
Also, Income tax = Tax rate × Taxable income.
And, Net income = Annual income – Income tax.

Worked Out Examples


Example 1: The annual income of a person’s is Rs. 2,00,000. If his tax allowance is
Rs. 1,40,000, what is the tax that he should pay per year at the rate of 15%?
Solution: Here,
Annual income = Rs. 2,00,000
Tax allowance = Rs. 1,40,000
Taxable income = Rs. 2,00,000 – Rs. 1,40,000 = Rs. 60,000
Income tax = 15% of Rs. 60,000 = Rs. 9000
Hence, his income tax is Rs. 9000 per year.
Example 2: If Rs. 17,000 is left after paying an income tax at the rate of 15%, what is
the income?
Solution: Here,
Tax rate = 15%
Income left after paying the tax = Rs. 17,000

40 | Mathematics - 9 Arithmetic
Let the income be Rs. x
If the tax allowance is not given then the income is considered as the taxable income.
 x - 15% of x = Rs. 17000
or, 85x = Rs. 17,00,000
 x = Rs. 20,000
Hence, the income is Rs. 20,000.
Example 3: The monthly income of a serviceman is Rs. 25,000. His tax allowance is
Rs. 75,000. How much does he pay as income tax per month, if the tax
rate of the 1st two lakh taxable income is 10% and 15% for the remaining
taxable income?
Solution: Here,
Monthly salary = Rs. 25,000
Annual income = Rs. 12 × 25,000 = Rs. 3,00,000.
Tax allowance = Rs. 75,000
Taxable income = Rs. 300,000 - Rs. 75,000
= Rs. 2,25,000 = Rs. 2,00,000 + Rs. 25,000
Tax per annum = 10% of 2,00,000 + 15% of 25,000
= 20000 + 3750 = Rs. 23750.
23750
Tax per month = Rs. 12 = Rs. 1979.17

Example 4: The monthly income of Rajani is Rs. 31,000. Her tax allowance is
Rs. 82,000. After paying income tax, if Rs. 28,100 is her net income per
month, what is the tax rate?
Solution: Here,
Monthly income = Rs. 31,000
Annual income = Rs. 12 × 31,000 = Rs. 3,72,000
Tax allowance = Rs. 82,000
Taxable income = Annual income – Tax allowance
= Rs. 3,72,000 – 82,000 = Rs. 2,90,000.
Net income per month = Rs. 28,100
Net income per year = Rs. 12 × 28,100 = Rs. 3,37,200.
Income tax = Annual income – Annual net income
= 3,72,000 – 3,37,200 = Rs.34,800
Let the tax rate be x %
x % of taxable income = Rs. 34,800
x
or, 100 × 2,90,000 = Rs. 34,800

 x = 12
Hence, the tax rate is 12%

Arithmetic Mathematics - 9 |41


Exercise 2.4
1. (a) Raj’s annual income is Rs. 2,25,000. The tax allowance for him is Rs. 1,60,000. If the
tax rate is 14%, how much does he have to pay as income tax per year?
(b) Ajay’s annual income is Rs. 2,40,000. The tax allowance is Rs. 1,60,000. If the tax rate
is 15%, what is his income tax per month? Also, find his net-income.
(c) Yash’s annual income is Rs. 3,20,000.. His provident fund is Rs. 36,000, citizen
investment fund is Rs. 60,000 and insurance premium is Rs. 14,000, which are tax free
amounts. If the tax rate for the first taxable two lakh is 1% and for the taxable income
above Rs. 2,00,000 is 15%, find the income tax per year.
(d) Kabita’s monthly salary is Rs. 25000, her tax allowance is Rs. 50,000. The tax rate for
the first taxable income Rs. 2,00,000 is 1% and 15% for the taxable income above
Rs. 2,00,000. Find income tax she should pay per month and also her net income.
2. (a) If Rs. 24,000 is left after paying an income tax at the rate of 20%, what is the income?
(b) After paying an income tax at the rate of 25%, income left is Rs. 30,000. Find the
income.
(c) Raj’s annual income is Rs. 4,00,000 in which Rs. 1,00,000 is tax allowance. If he pays,
Rs. 45,000 as income tax per year, what is the tax rate?
(d) Yash’s monthly income is Rs. 30,000. His tax allowance is Rs. 60,000. If he pays
Rs. 3,000 as income tax per month, what is the tax rate?
3. (a) Kabita pays income tax Rs. 32,500 per year at the rate of 13% p.a. If her tax allowance
is Rs. 86,000, find her annual income.
(b) Amar pays income tax Rs. 3125 per month. His tax allowance is Rs. 62,000. If the
income tax rate is 15%, find his monthly income.
(c) The yearly income of Arjun is Rs. 3,24,000 and his tax allowance is Rs. 74,000. After
paying income tax, if his net income per year is Rs. 2,86,500, find the income tax rate.
(d) The monthly income of Prem is Rs. 32,000. His tax allowance is Rs. 84,000. After
paying the income tax if Rs. 29,500 is his net income per month, what is the tax rate?
4. (a) Anmol’s monthly salary is Rs. 15,000. He deposits 10% to provident fund and 15% to C.I.F.
which are tax free amounts. What is the yearly tax on the remaining sum at the rate of 1%?
(b) Arya’s monthly income is Rs. 20,000. She pays a premium of Rs. 20,000 to an
insurance company and has medical allowance Rs. 5000 which are tax free. What
income tax, should she pay p.a. if the tax rate for the first two lakhs taxable income is
1% and 15% for the remaining sum?

42 | Mathematics - 9 Arithmetic
2.5 Bonus
When a business company goes in profit, it distributes certain percentage of the annual profit to its
employees as an incentive. This is other than their salary. This incentive amount is known as Bonus.
Bonus amount = bonus % of yearly net profit
If the number of employees is N, bonus is equally distributed then bonus received by an employee
Bonus amount
= N

Worked Out Examples


Example 1: A company makes a profit of Rs 2,00,00,000 in a certain fiscal year. The
management decided to distribute 6% of profit as bonus equally among
20 employees. Find the bonus amount that each employee will receive.
Solution: Here,
Number of employees = 20
Profit = Rs 2,00,00,000
Bonus = 6% of profit
6
= 100 × 2,00,00,000 = Rs 12,00,000

(12,00,000)
 Bonus amount that each employee receive = 20 = Rs 60,000

Hence, each employee will receive Rs 60,000 as bonus.


Example 2: A company made a profit of Rs. 85,00,00,000. The bonus is distributed
equally among 170 employees and each received Rs. 4,00,000. Find the
bonus rate.
Solution: Here,
Profit = Rs. 85,00,00,000
Number of employees = 170
Bonus received by each employee = Rs. 4,00,000
Thus,
Total bonus distributed = 170 × Rs. 4,00,000 = Rs. 6,80,00,000
Now,
Bonus = x % of 85,00,00,000
x
or, 6,80,00,000 = 100 × 85,00,00,000

 x = 8%
Hence, the bonus rate is 8%.

Arithmetic Mathematics - 9 |43


Example 3: The profit of a company is Rs. 95,50,40,000. The bonus is distributed at
the rate of 9% equally among certain number of employees. If each
employee received Rs. 2,14,884, what is the number of employees?
Solution: Here,
Profit = Rs. 95,50,40,000
Bonus rate = 9%
Bonus received by each = Rs. 2,14,884
Number of employees = x (say)
Total Bonus = Rs. 214884x
Then,
Bonus = 9% of 95,50,40,000
9
or, 214884x = 100 × 95,50,40,000

 x = 400.
Hence, the number of employees is 400.
Example 4: The profit of a company is Rs 82,50,000. The bonus is distributed to the
employees of the company according to their salary as follows.
Monthly salary scale Bonus rate
Rs. 5000 – Rs 10,000 0.125%
Rs. 10,000 – Rs 20,000 0.25%
Rs. 20,000 – Rs 30,000 0.5%
(i) What is the bonus amount received by an employee whose salary is
Rs. 8500?
(ii) What is the bonus amount received by an employee whose salary is
Rs. 25000?
Solution: Here,
Profit = Rs. 82,50,000
(i) Bonus received by an employee whose salary is Rs 8500 = 0.125% of profit
0.125
= 100 × 82,50,000

= Rs. 10,312.50
(ii) Bonus received by an employee whose salary is Rs 25000 = 0.5% of 82,50,000
0.5
= 100 × 82,50,000

= Rs. 41,250

44 | Mathematics - 9 Arithmetic
Exercise 2.5
1. (a) A development bank makes a profit of Rs. 10,00,00,000. The management decided to
give 8% bonus of profit equally among 250 employees. Find the bonus amount that
each employee will receive.
(b) A business company made a profit of Rs. 20,50,00,000. It is decided to give a bonus at
the rate of 9% equally among 450 employees. What is each employee’s bonus amount?
(c) A finance company made a profit of Rs. 30,75,00,000. A bonus of 10% profit is
distributed equally among 150 shareholders. What is the bonus amount that each share
holder will receive?
(d) A bank made a profit of Rs. 90,00,00,000. The management decided to distribute 10% bonus
equally among its 250 employees. What is the bonus amount that each employee will receive?
2. (a) A business company made a profit of Rs. 80,00,00,000. There were 400 employees and
each received Rs 1,60,000 as a bonus. Find the bonus rate.
(b) There is a profit of Rs. 95,00,00,000 of a business firm. The firm distributed bonus
equally among 500 employees and each received Rs. 2,28,000. What is the bonus rate?
(c) A company decided to distribute bonus at the rate of 12% of the net profit
Rs. 94,50,00,000 equally among its employees. If each employee received Rs 2,52,000,
what is the number of employees?
(d) Each employee of a company received Rs. 3,20,125. If the company has distributed
bonus at the rate of 13% of profit Rs. 98,50,00,000, then, what is the number of
employees?
3. (a) A company made a profit of Rs. 2,50,00,000. Bonus is distributed among the employees
according to their salaries.
Monthly salary scale Bonus rate
Rs. 5000 – Rs 8,000 0.125%
Rs. 8,000 – Rs 12,000 0.25%
Rs. 12,000 – Rs 20,000 0.5%
(i) What is the bonus received by an employee whose salary is Rs. 6500?
(ii) What is the bonus received by an employee whose salary is Rs. 15000?
(b) A bank made a profit of Rs. 4,00,00,000. The employees received bonus according to
their salaries as follows.
Monthly salary scale Bonus rate
Rs. 4000 – Rs 8,000 0.25%
Rs. 8,000 – Rs 12,000 0.5%
Rs. 12,000 – Rs 16,000 1%
(i) Find the bonus received by an employee whose salary is between Rs. 8000 –
Rs 12000.
(ii) Find the bonus of an employee whose salary is in between Rs. 12000 –
Rs. 16000.

Arithmetic Mathematics - 9 |45


2.6 Share and Dividend
A firm: An association formed by few persons to carry a business investing small amount of capital is
called a firm.
A company: A firm with large amount of capital and numerous persons as partners is called a
company.
Shares: 1. Whenever more than one persons or firms invest time or money jointly in a profit making
service or business, the persons or firms are share holders and the profit made in divided among
themselves in proportion to their work or time or money invested. The amount received by each is
called dividend. (share over profit)
Example 1: As proprietors of a school, Himesh, Dipesh and pramesh have shares of
Rs 25,00,000, Rs 20,00,000 and Rs 15,00,000 respectively. In a certain
period the school made a net profit of Rs 30,00,000. Which they decided
to divide among themselves. Find each's share in the profit.
Solution: Himesh's share = Rs 25,00,000
Dipesh's share = Rs 20,00,000
Pramesh's share = Rs 15,00,000
 Their share in the ratio 25,00,000:20,00,000:15,00,000 = 5:4:3
Total profit made = Rs 30,00,000
5
Now Himesh's share in the profit = 5+4+3 × Rs 30,00,000
5
= 12 × Rs 30,00,000

= Rs 12,50,000
4
Dipesh's share in the profit = 5+4+3 × Rs 30,00,000

4
= × Rs 30,00,000
12
3
And Pramesh's share in the profit = 5+4+3 × Rs 30,00,000
= Rs 7,50,000
Therefore, Himesh, Dipesh and Pramesh get Rs 12,20,000 Rs 10,00,000 and Rs
7,50,000 respectively.
2. When the capital of the business is beyond the financial capacity of the company/firm, a public
company of many partners are formed and required capital is divided into small units called shares of
nominal value. The nominal value of a share is not fluctuable. Obviously the company promoters
own a big portion of the shares and the remaining shares are divided among the public. A person who
purchases one or more shares is a share holder who is an owner of the company.
Dividend: At the end of a fiscal year, the company declares profit called dividend. The dividend is
expressed in terms of percentage of the nominal value. A share holder receives his/her dividend in
proportion to their investment.

46 | Mathematics - 9 Arithmetic
Market value: Once share holders buy shares or stocks of company/companies, they can keep the
shares always with them or sell them to other person who want to buy the shares. But the company
doesn't pay for the shares and take them back. Selling and buying of shares or transfer of shares can
take place only through a stock broker in stock exchange market.
Selling price of a share is called market value or quoted value, which fluctuates time to time. If the
marked value is equal to the nominal value (face value) it is said to be at par. If the market value is
greater than the nominal value, it is said to be at premium or above par and if the market value is less
than the nominal value, it is said to be at discount or below par. Shares own by a person or group
from one or more than one companies is called a stock or share market.
Example of shares in share market is shown below:
Nepal Stock Exchange
Transaction of 9 Kartik, 2073
Cost of Transaction
Company Name Final Cost Difference
previous day Number
Agriculture 653 645 3856 -8
Development Bank
Api Power Co. Ltd. 590 586 1991 -4
Everest Bank Ltd. 3601 3550 14256 -51
Total Transaction Cost 713,272,506 Transaction Number 573

Example 2: Find the dividend for 50 shares of Rs. 100 each at 8% rate of dividend.
Solution: Here,
Number of shares = 50
Nominal value of a share = Rs. 100.
Rate of dividend = 8%
 Nominal value of 50 shares = 50 x Rs. 100
= Rs. 5000
Now, dividend = 8% of Nominal value
8
= 100 × Rs. 5000

= Rs. 400
Note: Dividend is always taken as percentage of nominal value.
Example 3: The price of Rs. 100 shares is Rs. 150 and the company declares a
dividend of 15%. Find
(i) How many shares can be purchased for Rs. 18000?
(ii) Dividend.
(iii) Rate of interest on the investment.
Solution: Here,
Market value of a share = Rs. 150
Investment = Rs. 18000

Arithmetic Mathematics - 9 |47


Investment Rs. 18000
i. Number of shares = Market Value = Rs. 150 = 120

Nominal value of a share = Rs. 100


Nominal value of shares = 120 × Rs. 100 = Rs. 12000
ii.. Dividend = 15% nominal value
15
= 100 × Rs. 12000

= Rs. 1800
iii. Rate of interest (return on the investment)
Dividend
= Investment × 100%

Rs. 1800
= Rs. 18000 × 100% = 10%

Example 4: A company declares a dividend of 12% on the shares of face value


Rs. 100. A man buys some shares and gets 15% in his investment. Find
out for what price he bought the share.
Solution: Here,
Nominal value of a share = Rs. 100
12
Dividend in a share = 12% of nominal value= 100 × Rs. 100) = Rs. 12

Let the market price of a share = Rs. x.


Then, dividend = 15% of investment
15
Rs. 12 = 100 × x

x = Rs. 80
Example 5: What is the interest percentage on a capital invested in 18% share when
Rs. 10 share cost Rs. 12?
Solution: Here,
Nominal value of a share = Rs. 10
Dividend = 18% of nominal value
18
= 100 × Rs. 10

= Rs. 1.8
Market value of a share = Rs. 12
Dividend
 Interest = Market value × 100%

Rs. 1.8
= Rs. 12 × 100%

= 15%
Note: Dividend distributed at the end of a year is equivalent to the interest on the investment i.e. market value.

48 | Mathematics - 9 Arithmetic
Exercise 2.6
1. (a) A village security committee appointed three persons A, B and C for night time security
of the village from 6 pm to 6 am. It A, B and C watch for 3 hours, 4 hours and 5 hours
respectively each day, and the committee provides Rs 24,000 per month for the security.
Find their salaries.
(b) A, B and C can do a work in 4 days, 5 days and 10 days respectively. Divide Wages of
Rs 33,000 of a certain week among them.
(c) Dinesh, Dipesh and Digdarshan invested Rs 10,00,000, Rs 15,00,000 and Rs 25,00,000
respectively to run a business and made a profit of Rs 12,00,000 in a certain year. Find
how much will the dividend be each's share.
(d) Investments of X, Y and Z are 45%, 25% and 30% respectively in a business form. Find
their dividends from the total profit of Rs 5,00,000.
2. (a) A development bank made a net profit of Rs 40,00,00,000. If the bank distributes 8% of
the profit to its share holders as dividend, find the total dividend.
(b) A company earned Rs 6,00,00,000 net profit in a year. It the company distributed Rs
34,80,000 to its share holders as dividend, what was the rate of dividend?
(c) A share holder of a hydro-electric power company owns certain number of shares of
nominal value Rs 100 each. If the company decided to distribute 20% dividend and the
share holder received Rs 1,00,000 as dividend, find the number of shares belonging to
the share holder.
(d) A share holder of a company owns 2,000 shares the company declares to distribute 12%
dividend for the year and the share holder received Rs 24,000 as dividend what is the
nominal value of a share.
3. (a) If the shares in a company stands at Rs. 118 each, how many shares can be bought for
Rs. 14160?
(b) A man invests Rs. 1,27,500 to purchase shares of a bank at Rs. 425 each. He sells them
when the share value becomes Rs. 450 each. What is his income?
(c) When a Rs. 100 shares costs Rs. 240 in the share market, Arnav invests Rs. 33600 to
buy shares of a company. At the end of the fiscal year, the company pays 13% dividend.
Find his income of that year from shares.
(d) Hari invests Rs. 1,20,000 in buying Rs. 100 shares. If the nominal value of the shares is
Rs. 75000. Find the market value of each share.
4. (a) A man invests Rs. 2880 to buy shares of a company at Rs. 90 each and sells them at
Rs. 120 each. Find his profit percentage.
(b) What is the percentage interest on the capital invested in 18% share, when Rs. 100 share
costs Rs. 120?
(c) Mrs. Gharti buys a Rs. 300 share in a company which pay dividend of 10% which she buys
at such a price that her profit is 20% of her investment. At what price is the share bought?
(d) A man buys some Rs. 100 shares when the market value is Rs. 250 per share for
Rs. 30,000 and makes a profit of 6.4% in his investment. What is the percentage
dividend allowed?

Arithmetic Mathematics - 9 |49


5. (a) A company declares a dividend of 15% and Rs. 100 share which is quoted at Rs. 240.
(i) Find how many shares can be purchased with Rs. 50,400.
(ii) What is the dividend?
(iii) What is the rate of interest on the investment?
(b) A company declares a dividend of 20% on a Rs. 100 share which is quoted at Rs. 150.
(i) What is the total cost of 100 shares?
(ii) What is the annual income of the shares?
(iii) What is the percentage return on the investment?
(c) A jobber buys Rs. 20 shares in a company which pays 12% dividend by investing
Rs. 12800. If his income is Rs. 960, find
(i) The market value of each share
(ii) The number of shares bought.
(iii) The nominal value of the shares.
(d) Imran invests RS. 3,00,000 in 15% Rs. 110 shares at Rs. 120. When the shares rise to
Rs. 130, he sells out enough shares to purchase a mobile set for Rs. 39000. Find
(i) The number of shares he still holds.
(ii) His loss in annual income.

2.7 Home Arithmetic


2.7.1 Electricity Billing
1 unit = 1kilowatt hour = 1000 watt hour. It means, if an electric
appliance of 1000 watt is used for 1 hour, it consumes 1 unit of
electricity. Thus, consumption of electricity is measured in number of
units.
How to calculate the units of electricity consumed? Say, the reading of
meter for the month of Baisakh is 2050 and that of Jestha is 2090
units. Units of electricity consumed in the month of Jestha = Reading
of Jestha (Final reading) - Reading of Baisakh (Initial reading)
= 2090 – 2050 = 40
Thus, units consumed = Final reading - Initial reading. Here, final
reading means units shown at the current month and initial reading
means units shown in previous month.
Charge rate of Electricity Authority is as follows.
(i) Charge for the 1st 20 units at the rate of Rs. 4 per unit is Rs. 80.
(ii) Charge for 21 units - 250 units is Rs. 7.30 per unit. It means
charge for next 230 units is at the rate of Rs. 7.30 per unit.
(iii) Charge for the units above 250 units is Rs. 9.90 per unit.
(iv) 3% rebate up to 7 days from the day of meter reading.
(v) From 8th day to 22nd day from the day of reading, no rebate.

50 | Mathematics - 9 Arithmetic
(vi) From 23rd day to 30th day from the day of meter reading, 5% fine is charged.
(vii) From 31st day to 40th day from the day of meter reading, 10% fine is charged.
(viii) From 41st day to 60th day from the day of meter reading, 25% fine is charged.

Worked Out Examples


Example 1: The reading of the month, Baisakh is 2058 and that of the month Jestha
is 2208. What is the electricity charge for the month of Jestha?
Rules of Electricity Authority are as follows.
Minimum charge (upto 20 units) = Rs. 80
Cost per unit (21 - 250 units) = Rs. 7.30
Solution: Here,
Final reading = 2208 units
Initial reading = 2058 units
Units consumed = 2208 units – 2058 units = 150 units = 20 units + 130 units.
Cost of 1st 20 units = Rs. 80
Cost of next 130 units = Rs. (130 × 7.3) = Rs. 949
Total charge = Rs. (80 + 949) = Rs. 1029
Hence, the charge of electricity for the month of Jestha is Rs. 1029.

Arithmetic Mathematics - 9 |51


Example 2: The charge for the 1st 20 units is Rs. 80. The charge for the units from
21 - 250 units is Rs. 7.50 per unit and the charge for the units above 250
units is Rs. 9.90 per unit. What is the cost of 340 units of electricity?
Solution: Here,
Units consumed = 340 units = (20 + 230 + 90) units.
Cost of 1st 20 units = Rs. 80
Cost of next 230 units = Rs. 230 × 7.50 = Rs. 1725
Cost of next 90 units = Rs. 90 × 9.9 = Rs. 891
Hence, the total cost of 340 units of electricity = Rs. (80 + 1725 + 891) = Rs. 2696
Example 3: The charge of electricity for the first 20 units is Rs. 80 and the charge
from 21 to 250 units is calculated at the rate of Rs. 7.30 per unit. If the
total charge of electricity was Rs. 810, how many units of electricity was
consumed?
Solution: Here,
Cost of 1st 20 units = Rs. 80
Let, the extra units be x
Then, cost of x extra units = Rs. 7.30x
Total cost = 80 + 7.30x
or, 810 = 80 + 7.30x
or, 730 = 7.30x
or, x = 100units.
 Total units consumed = 100 + 20 = 120
Example 4: The reading of Jestha is 4010 and that of Ashad is 4130. The cost of 1st
20 units is Rs. 80. The charge for the units from 21 - 250 units is Rs. 7.30
per unit. The meter reading is done on 30th Ashad. There is rebate of 3%
if payment is made on the 1st 7days from meter reading. How much is the
cost if payment is done on 2nd Shrawan?
Solution: Here,
Final reading = 4130 units
Initial reading = 4010 units
Units consumed = 4130 units – 4010 units = 120 units = 20 units + 100 units.
Cost of 1st 20 units = Rs. 80
Cost of next 120 units = Rs. (100 × 7.30) = Rs. 730
Total cost = Rs. (80 + 730) = Rs. 810
Rebate = 3% of 810 = Rs. 24.30
 Cost after rebate = Rs. (810 - 24.30) = Rs. 785.70
Example 5: A house hold consume 71 units electricity in a math . Using the new
tariff given above of 5A : find the total bill of electricity of the household.
Solution: Here,
Total consumed units = 71
Service charge = Rs 100
Energy charge = 20 × 3 + 10 × 7 +20 ×8.50 + 21 × 10

52 | Mathematics - 9 Arithmetic
= 60 + 70 + 170 + 210
= Rs 510
Total bill = Service charge + energy charge
= 100 + 510
= Rs 610
o'lg6 ;]jf z'Ns -% PlDko/_ OghL{ z'Ns ;]jf z'Ns -!% PlDko/_ OghL{ z'Ns
) – @) #)÷– #÷– %) $
@! – #) %)÷– &÷– &% &
#! – %) &%÷– *.%)÷– !)) *.%)÷–
%! – !%) !)) ÷– !) ÷– !@% !)
!%! – @%) !@% ÷– !! ÷– !%) !!
@%! – $)) !%) ÷– !@ ÷– !&% !@
$)) eGbf dfly !&% ÷– !# ÷– @)) !#
Example 6: The reading of 1st kartik is 9,345 units and reading on 1st mangsir is 9,660
units . As per rule of new tariff of NEA related to 5A, find the to electricity
bill of the household.
Solution: Here,
Consumed units 1st kartik = 9345 units
Consumed units 1st mangsir = 9660 units
Total consumed units = 9660 – 9345 = 315 units.
Total service charge = Rs 150
Energy charge = 20 × 3 + 10 × 7 + 20 × 8.5 + 100 × 10 + 100 × 11 + 65 × 12
= 60 + 70 + 170 + 1000 + 1100 + 780
= Rs 3180
Total charge = 150 + 3180 = Rs 3330

Exercise 2.7.1
1. (a) The reading of the month, Jestha is 4012 and that of the month, Ashad is 4142. The
electricity charge up to 20 units is Rs. 4 per unit and Rs. 7.30 per unit from 21 to 250
units. What is the charge for electricity?
(b) The minimum charge for the 1st 20 units is Rs. 80 and Rs. 7.50 per unit from 21 to 250
units. What is the charge for 180 units of electricity?
(c) The minimum charge for the 1st 20 units is Rs. 4 per unit. What is the charge for 18
units of electricity?
(d) The minimum charge for the 1st 20 units is Rs. 4 per unit. What is the charge for 10
units of electricity?
2. (a) The charge for the 1st 20 units is Rs. 80. The charge for the units from 21 - 250 is
Rs. 7.50 per unit and the charge for the units above 250 units is Rs. 9.90 per unit. What
is the charge for 300 units of electricity?

Arithmetic Mathematics - 9 |53


(b) The charge for the 1st 20 units is Rs. 80. The charge for the units from 21 - 250 is
Rs. 7.50 per unit and the charge for the units above 250 units is Rs. 9.90 per unit. What
is the charge for 350 units of electricity?
(c) The minimum charge for the 1st 20 units is Rs. 80. The charge for the units from 21 -
250 is Rs. 7.50 per unit. If a customer paid Rs. 1055 in a certain month, how many units
of electricity was consumed?
(d) The minimum charge for the 1st 20 units is Rs. 80. The charge for the units from 21 -
250 is Rs. 7.50 per unit. If a customer paid Rs. 1205 in a month, how many units of
electricity was consumed?
3. (a) The reading of Ashad is 4130 and that of Shrawan is 4330 units. The cost of 1st 20 units is
Rs. 80. The charge for the units from 21 - 250 is Rs. 7.50 per unit. The meter reading is done
on 30th Shrawan. There is a rebate of 3% if the payment is made on the 1st 7days from the day
of meter reading. How much does the customer have to pay if the payment is done on 4th
Bhadra?
(b) The reading of Shrawan was 4330 units and the reading of Bhadra was 4580 units. The
cost of 1st 20 units was Rs. 80 and the charge for the units from 21 - 250 units was
Rs. 7.30 per unit. The customer got 3% rebate as he paid on 1st 7 days from the day of
meter reading. How much was the cost for electricity?
(c) The electricity charge upto 20 units is Rs. 3.90 per unit and Rs. 6.50 per unit from 21 to
250 units. How much should the electricity charge be paid for 85 units?
(d) According to the rate of electricity charges the minimum cost upto 20 units is Rs. 78
and cost per unit from 21 units to 250 units is Rs. 6.50. How much does 110 units cost?
4. (a) The minimum charge for the first 20 Units is Rs. 3 per unit and service charge is Rs. 30.
What is the charge for 15 units of electricity?
(b) The reading of the Month, Ashwin 2073 is 3125 units and that of the month, Kartik
2073 is 3200 units. What is the electricity charge for the month of Ashwin 2073
according to the new billing system?
5. The charge for the first 20 units is Rs. 3 per unit. The charge for the units from 21-30 is Rs. 7
per unit, charge for the units 31-50 is Rs. 8.50 per unit, charge for the units 51-150 is Rs. 10
per unit and charge for the units 151-250 is Rs. 11 per unit. What is the cost of 230 units of
electricity if the service charge is Rs. 125?
6. The charge for the first 20 units is Rs. 3 per unit. The charge for the units from 21-30 is Rs. 7
per unit, charge for the units 31-50 is Rs. 8.50 per unit and the charge for the units 51-150 is
Rs. 10 per unit. The total charge of electricity was Rs. 1100, how many units of electricity was
consumed if the service charge was Rs. 100.
7. The reading of Mangshir 2073 is 5370 units. The cost of first 20 units is Rs. 3 per unit, charge
for the unit (21-30) units is Rs. 7 per unit, charge for the unit (31-50) is Rs. 8.50 per unit,
charge for the units (51-150) is Rs. 10 per unit and charge for the unit (151-250) is Rs. 11 per
unit and service charge is Rs. 125. The meter reading is done on Mangshir 30, 2073 is
Rs. 5545. There is rebate of 3% if payment is made on the first 7 days from the day of meter
reading. how much does the customer have to pay if the payment is done on 3rd Poush?
8. The reading of 1st Magh is 10,450 Units and reading on 30th Magh is 10,630 units. NEA charges 5%
fine if payment is made after 23 days from the day of meter reading. How much does the customer
have to pay if the payment is done on 25th Falgun according to new billing system?

54 | Mathematics - 9 Arithmetic
2.7.2 Water Billing
A house may have water facility with a tap with a meter or a tap without a meter. If there is no meter
connected with a tap, the customer has to pay fixed amount fixed by water supply corporation.
Water bill is prepared on the basis of quantity of water used by consumer as shown by the meter. 50%
of bill is charged as sewerage charge and Rs. 5 as miscellaneous charge.

Worked Out Examples


Example 1: Observe the meter readings of water supply of a certain house.
Months Baisakh Jestha
Units 4012 4020
The sewerage charge is 50% of water bill and Rs. 6 is the miscellaneous
charge. There is a rebate of 3% if the payment is made within the 2nd
week of next month. Find the charge that the consumer has to pay for
Jestha, if he pays on 3rd of Ashad. The cost upto 8 units is Rs. 26.
Solution: Here,
For the month of Jestha
Initial reading = 4012 units
Final reading = 4020 units
Units consumed = 4020 units – 4012 units = 8 units
Cost of 8 units = Rs. 26
Total cost with sewerage charge and miscellaneous charge = 26 + 50% of 26 + 6
= Rs. 45
But the payment is made on the 1st week of next month, so there is a rebate of 3%.
Charge after rebate = Rs. (45 - 3% of 45) = Rs. 43.65
Hence, the consumer has to pay Rs. 43.65
Example 2: Observe the meter readings of water supply of a certain house.
Months Bhadra Asoj
Units 4031 4040

Arithmetic Mathematics - 9 |55


The cost upto 9 units is Rs. 70. The sewerage charge is 50% of bill and
Rs. 10 is miscellaneous charge. If the payment of a bill is made after the
2nd week and within the next month, a customer doesn’t get rebate and
he/she is not fined. Find the charge for the month of Asoj, if the
customer pays on 25th Kartik.
Solution: Here, for Asoj
Initial reading = 4031 units
Final reading = 4040 units
Units consumed = Final reading - Initial reading = 4040 units – 4031 units = 9 units.
Cost of 9 units = Rs. 70
Cost with sewerage charge and miscellaneous charge = Rs. (70 + 50% of 70 + 10)
= Rs. 115
th
As payment is made on 25 Kartik, the customer neither gets rebate nor fined. So, the
customer has to pay Rs. 115.
Example 3: Observe the water meter reading of a certain household.
Months Shrawan Bhadra
Units 2012 2022
The charge upto 10 units is Rs. 50. The sewerage charge is 50% of bill
and Rs. 6 is miscellaneous charge. If the payment of a bill is made in the
third month, a customer is fined 10%. What is the charge if the customer
pays on 25th Mangsir?
Solution: Here, for Bhadra
Initial reading = 2012 units
Final reading = 2022 units
Units consumed = Final reading - Initial reading
= 2022 units – 2012 units = 10 units.
Charge with sewerage charge and miscellaneous charge = Rs. (50 + 50% of 50 + 6)
= Rs. 81
Fine = 10% of 81 = Rs. 8.1
Charge after fine = Rs. 81 + Rs. 8.10 = Rs. 89.10
Hence, the customer has to pay Rs. 89.10

Exercise 2.7.2
1. (a) Observe the water meter readings of a certain house.
Months Baisakh Jestha
Units 3000 3008
The cost upto 8 units is Rs. 40. 50% of water bill is charged as sewerage charge and
Rs. 10 as miscellaneous charge. There is a rebate of 3% if the payment is made within
the 2nd week of next month. Calculate the charge for the month of Jestha, if the payment
is made on the 2nd of Ashad.

56 | Mathematics - 9 Arithmetic
(b) Read the water meter readings given below:
Months Jestha Ashad
Units 3008 3017
The cost upto 9 units is Rs. 50. 50% of water bill is charged as sewerage charge and
Rs. 10 as miscellaneous charge. There is a rebate of 3% if the payment is made within
the 2nd week of next month. Calculate the charge for the month of Ashad if the payment
is made on the 3rd Shrawan.
(c) Observe the table given below, which shows water meter readings.
Months Ashad Shrawan
Units 3017 3027
The cost upto 10 units is Rs. 60. 50% of water bill is charged as sewerage charge and
Rs. 15 as miscellaneous charge. There is a rebate of 3% if the payment is made on the
2nd week of next month. Calculate the charge for the month of Shrawan, if the payment
is made on the 1st Bhadra.
(d) Observe the meter readings of water supply of a certain house hold.
Months Shrawan Bhadra
Units 3027 3037
The sewerage charge is 50% of water bill and Rs. 7 is the miscellaneous charge. The
cost upto 10 units is Rs. 60. There is a rebate of 3% if the payment is made on the 2nd
week of next month. Find the charge that the consumer has to pay for the month of
Bhadra, if he pays on the 4th of Ashoj.
2. (a) Observe the water readings.
Months Baisakh Jestha
Units 4012 4020
The cost upto 8 units is Rs. 80. The sewerage charge is 50% of water bill and Rs. 8 is
the miscellaneous charge. If the payment is made after the 2nd week and within the next
month there is neither rebate nor fine. Find the charge that the customer has to pay for
Jestha if the payment is made on the 2nd of Shrawan.
(b) Observe the table given below of water meter readings.
Months Jestha Ashad
Units 4020 4029
The charge upto 9 units is Rs. 90. The sewerage charge is 50% of water bill and Rs. 10
is the miscellaneous charge. If the payment is made after the 2nd week and within the
next month, there is neither rebate nor fine. Find the charge for the month of Ashad, if
the customer pays on the 28th Shrawan.
(c) Look at the meter readings of water supply of a certain house.
Months Ashad Shrawan
Units 4029 4039
The charge upto 10 units is Rs. 100. The sewerage charge is 50% of water bill and
Rs. 10 is the miscellaneous charge. If the payment is made after the 2nd week and within
the next month, there is neither rebate nor fine. Find the charge for the month of
Shrawan, if the customer pays on the 25th Bhadra.

Arithmetic Mathematics - 9 |57


(d) In the month of Bhadra, the water consumed by a family is 10 units. The charge upto 10
units is Rs. 60. The sewerage charge is 50% of water bill and Rs. 6 is the miscellaneous
charge. If the payment of a bill is made after the 2nd week and within the next month a
customer doesn’t get rebate and he/she is not fined. Find the charge for the month of
Bhadra which the customer pays on the 1st of Kartik.
3. (a) The reading of water meter for Shrawan was 2030 units and that of Bhadra was 2042
units. The charge upto 10 units is Rs. 50 and Rs. 5 for each extra unit. If the sewerage
charge is 50% of the bill and miscellaneous charge is Rs. 5, Calculate the charge if the
payment is made on time.
(b) The reading of water meter for Bhadra was 2042 units and that of Asoj was 2050 units.
The charge upto 8 units is Rs. 40. The sewerage charge is 50% of bill amount and Rs. 5
for miscellaneous charge. If the payment was made after the 4th month then 20% fine
was charged. What was the charge for Asoj, if the customer paid on the 4th Falgun?
(c) Observe the reading of water meter of a certain house given below:
Months Asoj Kartik
Units 5012 5025
The charge upto 10 units is Rs. 50 and Rs. 5 for each extra unit. The sewerage charge is
50% of the bill and miscellaneous charge is Rs. 6. If the payment is done after the 5th
month, 50% fine is charged. What is the charge for the month of Kartik if the payment
is made on the 2nd Baisakh of next year?
(d) The reading of a water meter in the month of Baisakh is 2560 units and that in the
month of Jestha is 2570 units. The charge upto to 10 units is Rs. 70. The sewerage
charge is 50% of bill and miscellaneous charge is Rs. 7. The customer paid after the 5th
month so he is charged 50% fine. What was the charge of water for the month of Jestha?

2.7.3 Telephone Billing


For the use of telephone, a person has to pay money. At present, on a land line 2 minutes call is
considered as a call and charges Re. 1. On it, telecom service charge (TSC) is added. After that, on the
total, VAT is added which gives the grand total which the customer has to pay.

58 | Mathematics - 9 Arithmetic
Worked Out Examples
Example 1: Raj made a call of 6 minutes in business hours. The time from 08:00 -
18:00 hrs. is known as business hours. In business hours a call of 2
minutes is considered as 1 call. Find the charge if 10% TSC is imposed
on it and then 15% VAT. The charge for 1 call is Re. 1.
Solution: Here,
Time of call = 6minutes.
1 call is of 2 minutes.
6
No. of calls = 2 = 3

Cost of 3 calls = Rs. 3 × 1 = Rs. 3


TSC = 10% of Rs. 3 = Rs. 0.30
Total = Rs.(3+ 0.30) = Rs. 3.30
VAT = 15% of Rs. 3.30 = Rs. 0.50
Grand total = Rs. (3.30 + 0.50) = Rs. 3.80
Hence, the charge is Rs. 3.80.
Example 2: The rental call for a month is 200 calls. If Kabita made 198 calls, what is
the charge that Kabita has to pay after the imposition of 10% TSC and
13% VAT, where the charge per call is Re. 1?
Solution: Here,
Rental calls mean minimum calls that the customer has to pay.
Rental calls 200 calls mean, the customer has to pay the charge of 200 calls if he
makes calls less than 200.
Charge with TSC = Rs. 200 + 10% of Rs. 200 = Rs. 220
Then, Total charge with VAT = Rs. 220 + 13% of Rs. 220 = Rs. 248.60
Example 3: The time from 18:00 - 22:00 hours and from 06: 00 - 08:00 hours is known
as normal hours. In normal hours, a call of 4 minutes is considered as a
single call. Sangita talked with her daughter from 06:02 - 06:58 p.m. What
is the charge that Sangita has to pay after 10% TSC and 13% VAT, where
charge per call is Re.1?
Solution: Here,
Time of call = 06:58 - 06:02 = 56minutes
56
Number of calls = 4 = 14

Charge for 14 calls = Rs 14 × 1 = Rs. 14


Charge with TSC = Rs. 14 + 10% of Rs. 14 = Rs. 15.40
Charge with VAT = Rs. 15.40 + 13% of Rs. 15.40 = Rs. 17.40
Hence, the total charge is Rs. 17.40.

Arithmetic Mathematics - 9 |59


Example 4: The time from 22:00 - 06:00 hours is known as off hours. In off hours a
call of 8 minutes is known as 1 call. Dilasha made a call to her friend
from 11:00 - 11:56 p.m. what is the charge after 10% TSC and 13% VAT if
the charge per call is Re. 1?
Solution: Here,
Time of call = 11:56 - 11:00 = 56 minutes
56
No. of calls = 8 = 7

Charge for 7 calls = Rs. 7 × 1 = Rs. 7


Charge with TSC = Rs. (7 + 10% of 7) = Rs. 7.70
Charge with VAT = Rs. (7.70 + 13% of 7.70) = Rs. 8.70

Exercise 2.7.3
1. (a) In an office, the total calls of a day is 90 minutes in business hours. The time from
08:00 - 18:00 hours is known as business hours. In business hours a call of 2 minutes is
considered as a single call and charge is Re.1 per call. Find the charge that the office has
to pay after 10% TSC and 13% VAT.
(b) A travel agency made calls of 240 minutes in a certain day in business hours. The time from
08:00 - 18:00 hours is known as business hours and a call of 2 minutes is considered as a
single call. If the charge is Re. 1 per call, find the total charge after 10% TSC and 13% VAT.
(c) The rental call per month is 200 calls. If a man has made 190 calls in a certain month,
find the charge in a month after 10% TSC and 13% VAT where the charge is Re. 1/call.
(d) The rental call per month is 200 calls. If Raj has made 250 calls in a month, find the
charge in a month after 10% TSC and 13% VAT where the charge for rental call as well
as for extra call is Re. 1/call.
2. (a) The time from 18:00 - 22:00 hours and from 06:00 - 08:00 hours is known as normal
hours. In normal hours, a call of 4 minutes is considered as a single call. Mamata talked
with her friend from 07:20 - 08:00 p.m. what is the charge that Mamata has to pay after
10% TSC and 13% VAT where charge per call is Re. 1?
(b) In normal hours, Raj talked with his friend for 136 minutes. What is the charge after
10% TSC and 13% VAT where charge per call is Re. 1?
(c) The charge for certain number of calls with 10% TSC and 13% VAT is Rs. 79.552. Find
the number of calls if the charge per call is Re. 1.
(d) The charge for certain number of calls with 10% TSC and 13% VAT is Rs. 149.16. Find
the number of calls if the charge per call is Re. 1.
3. (a) The time from 22:00 - 06:00 hrs is known as off hours. In off hours, a call of 8 minutes
is considered as 1 call. Jay talked with his friend from 10:02 - 11:14p.m. What is the
charge after 10% TSC and 13% VAT if the charge per call is Re. 1?

60 | Mathematics - 9 Arithmetic
(b) In off hours, Jyoti made a call to her friend from 10:00 - 12:00 midnight. What is the
charge after 10% TSC and 13% VAT if the charge per call is Re. 1?
(c) In off hours, Nayan made a call of 3 hours. What is the charge after 10% TSC and 13%
VAT if the charge per call is Re. 1?
(d) In off hours, Suryaa made a call of 320 minutes. What is the charge after 10% TSC and
13% VAT if the charge per call is Re. 1?

2.7.4 Taxi Fare


Following are the rates of fare in taxi meter implemented by Nepal bureau of standards and
metrology.
Taxi Time Minimum fare Fare of 200m
Day time taxi 6:00Am to 9 : 00 Pm Rs14 Rs7.40
Night time taxi 9 : 00 Pm to 6: 00 Am Rs 21 Rs11.10
Waiting charge
Day time taxi Rs 7.40 per 2 minutes
Night time taxi Rs 11.10 per 2 minutes.

Worked Out Example


Example 1: Ram takes a taxi to travel 5 kilometers during day time. Find the fare he
has to pay with 3 minutes waiting.
Solution: Here,
Time of travel is day time.
Distance travelled = 5 km = 5 × 1000m. = 5000m.
Rate of fare = Rs. 7.40/200m
Waiting time = 3 minutes, which is equivalent to 4 minutes
Rate of wating charge = Rs. 7.40/2 minutes
Now,
Charge = Minimum charge + Fare of 5000 m + Waiting charge
= Rs. 14 + 5000 m × Rs. 7.40/200 m + 4 minutes × Rs. 7.40/2 minutes
= Rs. 14 + Rs. 25 × 7.40 + Rs. 2 × 7.40
= Rs. 213.80
 Total charge (fare) to be paid = Rs. 213.80.
Example 2 : Sailaja travelled a distance of 10km. before 9pm. and 8km. afterwards by
taxi. Find the total fare she has to pay with 5 minutes waiting time while
getting off.
Solution: Here,
Time before 9pm. is considered as day time and time after 9pm. is considered as night time.

Arithmetic Mathematics - 9 |61


Distance travelled during day time = 10km. = 10 × 1000 m.
= 10000 m.
Taxi Fare for day time journey at the rate of Rs. 7.40 per 200m.
= 10000 m × Rs. 7.40/200 m = Rs. 370
Distance travelled in night time = 8km. = 8 × 1000 m. = 8000 m.
Taxi Fare for night time journey at the rate of Rs. 11.10 per 200 m.
= 8000 m × Rs. 11.10/200 m
= Rs. 444
As she started travelling during day time, minimum charge = Rs. 14
Waiting time = 5 minutes, which is equivalent to 6 minutes
Waiting Charge = 6 minutes × Rs. 11.10/ 2 minutes (Night Time)
= Rs. 33.30
 Total Fare = minimum charge + fare of day time journey + fare of night time
journey + waiting charge = Rs. 14 + Rs. 370 + Rs. 444 + Rs. 33.30 = Rs. 861.30
Hence, she has to pay total fare of Rs. 861.30

Exercise 2.7.4
1. Find the total taxi fare between the two places given in table.
S.No. From To Time Distance Waiting time
a. Kalanki Chabahil 5 p.m 7 km. 3 minutes
b. Airport kalimati 2 A.m 10 km. -
c. Babazar Balain chook 11 A.m 4 km. 6 minutes
2. Sangam hired a taxi and travelled 15km. If the minimum rate is Rs. 14 and the fare goes on at
the rate of Rs. 7.20 per 200 meters. Calculate the total fare paid by him.
3. Mr. Ghising paid Rs. 122 for taxi fare. The minimum charge is Rs. 14 and additional charge
RS. 7.20 per 200meters. How many kilometers did he travel by the taxi?
4. Safal travelled the taxi from Singh Durbar to Sano Thimi of distance 7 km. during day time.
Minimum reservation charge is Rs. 14 and the additional charge Rs. 7.20 per 200 meters. There
was imposed waiting charge Rs. 7.20 per 2 minutes. Find what amount of fare Safal paid for
taxi.
5. Dakshyata hired a taxi to travel from Ratnapark to Godawari which is 20 km. away. The taxi
driver asked whether she wants to pay according to meter with usual rates and rules or a lump
sum Rs. 725. Which will she prefer? By how much is it cheaper?
6. Dikshant travelled 5km. before 9pm. and 4km. afterwards by taxi. Find the total fare he has to
pay with 3 minutes waiting time while getting off. (Use the rates given above)

62 | Mathematics - 9 Arithmetic
Unit Test
Time: 40 minutes F.M.- 24
Group-A (3×1=3)
1. What do you mean a unit of electricity?
2. When the selling price of a watch is Rs 440 and gain percentage is 10%, find the cost price.
3. The total sales of a company is Rs. 2,00,000 in a month and the company allows the
commission at the rate of 0.5% to the agent. Find the amount of commission.
Group-B (4×2=8)
4. The marked price of a pair of shoes Rs 4000. What is its selling price after discount of 10%.
5. The total number of shares of a company is 3,00,000. The company gets an annual profit of
Rs 3,50,00,000. The company board has decided to distribute the dividend 21.5% of total
profit. Find the total dividend amount.
6. Water is supplied through 1 inch pipe line in Hotel jungle lodge. If the Hotel consumed 580
units of water in a month, then find the charge of water with 50% additional service charge.
(minimum charge for 1" pipe upto 5600 liters of water is Rs 3960. Additional charge per
1000 liter is Rs71)
7. Hari travelled 25 km distance by a taxi at Rs 939. If the minimum fare of a taxi is Rs 14,
find the rate of taxi fare per km.
Group – C (2×4=8)
8. A secondary teacher's annual salary is Rs 3,78,000. If he/she pays an annual tax of Rs 14700
at the rate of 1% for the first 3,00,000 and 15% for the next Rs. 1,00,000 of taxable income,
what will be the monthly salary after deduction of tax?
9. Rupan chaudhary made a call of 8 min. in business hours. The time from 08:00-18:00hrs. is
known as business hours. In business hours, a call of 2 minutes is considered as 1 call. Find
the charge of 10% TSC is imposed on it and then 15% VAT. The charge for a call is Rs. 1.
Group – D (1×5 = 5)
10. The shopkeeper marks the price of an article 30% above its cost price and gives the
customers a discount of 10% so that he gains Rs. 34 on the article. Find the marked price of
the article.

Arithmetic Mathematics - 9 |63


Answers ____________________________________________________________
Exercise 2.1
1. (a) Rs 55 (b) Rs 800 (c) Rs 1200 (d) Rs 900
1
2. (a) 20% (b) 16.66% (c) 33 % (d) 50%
3
3. (a) profit 28% (b) Rs 6 (c) Rs 690 (d) Rs 1060
4. (a) Rs 175 (b) Rs 16600 (c) Rs 220
1
5. (a) 25% (b) 9 % (c) 50% (d) 25%
11
10000
6. (a) 2.5% (b) Rs 1552.5 (c) 0.25% (d) Rs.
3
300
7. (a) Rs 3000 (b) 1000 (c) 5 (d) %
11
8. (a) Rs 24 (b) Rs Rs 52 (c) Rs 134 (d) 300

Exercise 2.2
1. (a) 3,20,000 (b) 700 (c) 34,500 (d) 11,000
2. (a) 45,00,000 (b) 64,00,000 (c) 6 (d) 5
3. (a) 61,75,000 (b) 72,80,000 (c) 1,12,500 (d) 1,00,000
4. (a) 55,000 (b) 90,000

Exercise 2.3
1. (a) 200,800 (b) 165 (c) 10
2. (a) 800 (b) 25%
3. (a) 1539 (b) 23408
4. (a) 10,000 & 8000 (b) 9000 & 7500 (c) 8000
5. (a) 25000 (b) 25 (c) 25
6. (a) 3750 (b) 6000 (c) Rs 2000
7. (a) 2 (b) 15000 (c) 12%
Exercise 2.4
1. (a) 9,100 (b) 1000, 2,28,000 (c) 3500 (d) 791.67, 2,40,500
2. (a) 30,000 (b) 40,000 (c) 15 (d) 12
3. (a)3,36,000 (b) 26,000 (c) 15 (d) 10
4. (a)1350 (b) 4250
Exercise 2.5
1. (a) 32,000 (b) 41,000 (c) 2,05,000 (d) 3,60,000
2. (a) 8 (b) 12 (c) 450 (d) 400
3. (a) 31,250 & 1,25,000 (b) 2,00,000 & 4,00,000
Exercise 2.6
1. (a) Rs 6,000, Rs 8,000, Rs 10,000 (b) Rs 15,000, Rs 12,000, Rs 6,000
(c) Rs 2,40,000, Rs 3,60,000, Rs 6,00,000 (d) Rs 2,25,000, Rs 1,25,000, Rs 1,50,000
2. (a) Rs 3,20,00,000 (b) 5.8% (c) 5000 (d) Rs 100
3. (a) 120 (b) 7500 (c) 1820 (d) 160
4. (a) 33.33% (b) 15% (c) 150 (d) 16%

64 | Mathematics - 9 Arithmetic
1
5. (a) (i) 180 (ii) 3600 (iii) 7 %
7
1
(b) (i) 15000 (ii) 2000 (iii) 13 %
3
(c) (i) 32 (ii) 400 (iii) 8000
(d) (i) 2200 (ii) 4950
Exercise 2.7.1
1. (a) 883 (b) 1280 (c) 80 (d) 80
2. (a) 2300 (b) 2795 (c) 150 (d)170
3. (a) 1387.10 (b) 1706.23 (c) 500.50 (d) 663
4. (a) 90 (b) 650
5. 2305 6. 120 units 7. Rs 1649 8. Rs 1365
Exercise 2.7.2
1. (a) 67.90 (b) 82.45 (c)101.85 (d) 94.09
2. (a) 12.8 (b) 145 (c) 160 (d) 96
3. (a) 95 (b) 78 (c) 155.25 (d) 168
Exercise 2.7.3
1. (a) 55.44 (b) 49.16 (c) 248.60 (d) 310.75
2. (a) 12.43 (b) 42.26 (c) 64 (d) 120
3. (a) 11.19 (b) 18.65 (c) 27.97 (d) 49.72
Exercise 2.7.4
1. (a) Rs 287.80 (b) Rs 576 (c) Rs 184.20
2. Rs 554
3. 3km
4. Rs 280.40
5. The ump sum Rs 725, Cheaper by Rs 29
6. 443.2

Arithmetic Mathematics - 9 |65


g]kfnsf] gS;f

66 | Mathematics - 9 Arithmetic
Chapter

3
Mensuration
Objectives:
At the end of this chapter, the
students will be able to:
 solve the problem related to
surface area and volume cubical
and cuboids soon.
 calculate the area of outside
path, inside path and cross path in
rectangular ground.
 calculate the cost of carpeting,
colouring, and plastering in a room
according as the cost per unit.

Teaching Materials:
Problem related objects which is
locally available i.e. brick, tiles, etc.
measuring instruments, like tape,
meter scale, locally available price list
of colouring materials, carpet, etc.
chart paper.
Historical fact
The Greek word “Peripheria”- meaning “ periphery” inspired the symbol  (pi) a
constant, now taken as the ratio of circumference to the diameter of a circle.
Circumference (C)
i.e.  =
Diameter (D)
The accuracy by which the value of  was estimated in different ages by
different mathematicians differently. Willian Jones
4
 Egyptians (1650 B.C),  = 34
 
10 10
 Archimedes (240 B.C),  = 371 <  < 370

377
 Claudius Ptolemy (150A.D),  = 120

355
 Tsu Ching – Chih (480A.D),  = 113

62832
 Aryabhatta (530 A.D),  = 20000

 Francois Viete (1578 A.D) found  correct to nine decimal places.


 Ludolph Van Ceulen (1610A.D) of Netherland computed  to thirty-five decimal places. In his
will this number 3.1415926,5358979,3238462,6433832,7050288 was engraved on his tombstone
and this number is known as ‘ Ludolphian’ number.
In recent times, the approximate value of  has been calculated to more than 100,000 decimal places.
 The symbol  was used by English Mathematicians William Oughtred, Isaac Barrow and David
Gregory in 1737.
 The first person to use  as ratio of circumference to the diameter of a circle was an English
mathematician William Jones (1706).
 Johann Heinrich Lambert (1728-1777, Switzerland) first showed that  is irrational.
A mnemonic is often used to remember the value of  up to seven decimal places.

“May I Have A Large Container Of Coffee?”


π 3 1 4 1 5 9 2 6

68 | Mathematics - 9 Mensuration
Review

Name of the Perimeter or


Shapes Area
figures circumference

1 Triangle 1 P = AB + BC + CA
A = 2 base x
height

2 Square A = l2 p = 4l

1
A = 2 d2

3 Rectangle A=l×b P = 2(l + b)

4 Parallelogram A = base × P = AB + BC + CD
height + DA

5 Equilateral 3 p = 3a
Triangle A = 4 a2

Mensuration Mathematics - 9 |69


6 Isosceles a p = a + 2b
Triangle A = 4 4b2 – a2

7 Rhombus 1 p = 4a
A = 2 d1 x d2

8 Trapezium 1 P = sum of all sides.


A = 2 (b1 + b2)xh

9 Quadrilateral 1 P = sum of all sides.


A = 2 d(h1 + h2)

10. Circle A = r2 or C = 2r

A = (d+2/4) C = d

11 Semi-circle 1 C = r + 2r or
A = 2 r2 or
c = (d/2) + d
1
A = 8 d2

70 | Mathematics - 9 Mensuration
3.1 Area of Pathways
Area of Surrounding Paths
1. Area of surrounding path of fixed width running outside a rectangular field.
Roads or pathways sometimes run all around plots of land, gardens, squares, ponds etc and we need to
find the area covered by such pathways.
E F
Let ABCD be a rectangular plot of a land of length l and breadth b. and a
A B
pathway of uniform width w runs surrounding the plot outside. The length
and breadth of the plot including the path are HG = (l + 2w) and b w
FG = (b + 2w) respectively. l
D C
 Area of plot, ABCD = A1 = l × b ... (i) H l + 2w G
Area of the plot including path, A2 = (l + 2w) (b + 2w)
= lb + 2lw + 2bw + 4w2 ... (ii)
 Area of surrounding path (outside) = A2 – A1
= lb + 2lw + 2bw + 4w2 – lb.
= 2w(l + b + 2w) square units.
2. Area of the pathway of fixed width running inside a rectangular field:
A B
Let a pathway of uniform width w runs inside a rectangular plot ABCD of
E F
length l and breadth b, then length and breadth of the plot EFGH excluding l - 2w
the pathway are (l – 2w) and (b – 2w) respectively.
b - 2w w
Area of the pathway = Area of rectangle ABCD – area of rectangle EFGH
H G
= lb – (l – 2w) (b – 2w) D l C
2
= lb – (lb – 2lw – 2bw + 4w )
= lb – lb + 2lw + 2bw - 4w2
= 2w (l + b - 2w) square units.
Note:
(i) Area of the path running outside the square plot of side a is A = 4w(a + w)
(ii) Area of the path running inside the square plot of side a is A = 4w(a – w).
3. Area of path of uniform width running outside a circular plot: w
A
Let OA = r be the radius of the circular plot and OB = R be the radius of the
r
circular plot including path of width w i.e. r + w = R. O
 Area of the circular path = Area of outer circle – Area of inner circle. R
B
= R2 - r2 = (R2 - r2) sq. units.

Mensuration Mathematics - 9 |71


A T U B
Area of Cross Paths
Let two paths of width w run across a rectangular plot ABCD E F R
Q
crossing each other. b w
P S
H G
Area of the path PQRS parallel to length of the plot = lw.
Area of the path TUVW parallel to breath of the plot = bw. D W V C
l
2
Area of the square EFGH, common to both the paths = w × w = w
 Area of cross path = Area of PQRS + Area of TUVW – Area of EFGH.
= lw + bw - w2
= w(l + b – w) sq. units.
If the plot is a square, l = b
 Area of cross path = w(l + l – w) sq. units.
= w(2l - w) sq. units.

Worked Out Examples


Example 1: A rectangular garden of 32m by 22m is surrounded outside by a path 2m
wide. Find the cost of paving the path at Rs. 40 per m2.
Solution: Here,
Length of the rectangular garden (l) = 32m.
22m
Its breadth (b) = 22m
32m
Width of the path (w) = 2m 2m
Now, area of surrounding path (outside) = 2w(l + b + 2w)
= 2 × 2m (32m + 22m + 2 × 2m)
= 232m2
Again, cost of paving the path = Area × Rate
= 232m2 × Rs. 40/m2 = Rs. 9280.
Therefore, the cost of paving the path is Rs. 9280.
Example 2: The length and breadth of a rectangular plot are in the ratio 3:2. A path
of width 1.5m runs around inside the plot. If the path has the area of
111m2, find the length and breadth of the path.
Solution: Here,
Area of the rectangular plot = 111m2
Width of the surrounding path (w) = 1.5m.
Let the length and breadth be 3x and 2x respectively. (Since length and breadth are in
the ratio 3:2)

72 | Mathematics - 9 Mensuration
We have,
Area of surrounding path (inside) = 2w(l + b – 2w)
or, 111m2 = 2 × 1.5m(3x + 2x – 2 × 1.5m)
or, 111m2 = 3m(5x – 3m)
or, 37m = 5x – 3m
or, 5x = 40m
x = 8m.
 Length (l) = 3x = 3 × 8m = 24m and breadth (b) = 2x = 2 × 8m = 16m.
Example 3: The circumference of a circular garden is 220m and a path 3.5m wide
runs around the garden. Find the cost of plastering the path at Rs. 60 per
sq. metre.
Solution: Here,
Circumference of the circular garden (C) = 220m.
If radius = r then
2r = 220m
22
or, 2 × 7 × r = 220m

r = 35m.
Width of the path(w) = 3.5m
Radius of the garden including path (R) =r+w
= 35m + 3.5m = 38.5m
Area of the path = (R2 - r2)
22
= 7 {(38.5m)2 – (35m)2}
22
= 7 {1482.25m2 – 1225m2}
22
= 7 × 257.5 = 808.5m2
The cost of plastering the path = Rs. 60 per sq.m.
Total cost of plastering the path = Area of path × Rate
= 808.5m2 × Rs. 60/m2
= Rs.48510.
Therefore, the cost of plastering the path is Rs. 48510.
Example 4: A rectangular plot of land is 64m long and 50m wide. Two paths, each of
width 2m one parallel to length and other parallel to width of the plot
cross each other inside the plot. Calculate,
(i) the cost of paving the path with bricks of size 16cm × 14 cm at Rs. 6
per brick.
(ii) The cost of turfing the remaining space at Rs. 4 per sq. m.

Mensuration Mathematics - 9 |73


Solution: Here, A T U B
Length of the rectangular plot (l) = 64m.
Breath of the plot (b) = 50m
P Q
Width of the cross path (w) = 2m. 50m 2m
S R
 Area of the cross path = w (l + b – w)
2m
= 2m(64m + 50m - 2m)
D W V C
= 2m × 112m 64m
2
A = 224m
 A = 2240000cm2
Area of a brick (a) = 16cm × 14cm = 224cm2
Area of paths A 2240000cm2
No of bricks needed = Area of a brick = a = 224cm2 = 10000.

(i) Total cost of bricks at Rs. 6 per brick = No. of bricks × Rate
= 10000 × Rs. 6 = Rs. 60000
(ii) Area of the plot including paths =l×b
= 64m × 50m = 3200m2
 Area of the remaining space = Area of plot - Area of path
= 3200m2 – 224m2 = 2976m2
 The cost of turfing the remaining space at Rs. 4 per sq.m. = 2976m2 × Rs. 4/m2
= Rs. 11904.
Therefore, the cost of turfing the remaining space is Rs. 11904 and the cost of
paving the paths is Rs. 60,000.
Example 5: Two pathways run across a field 80m × 40m crossing each other at a
right angle. The path parallel to the length is 2m wide and parallel to the
breadth is 3m wide. Find the area of the path inside the field. And
calculate the cost of the gravelling the path at Rs. 50 per sq.m.
Solution: Here, two crossing paths are of different widths. Length of the field (l) = 80m and
width of the path along length w1 = 2m. T U
A B
Breadth of the field (b) = 40m.
Width of the path along breadth w2 = 3m
Now area of cross path P Q
40m 2m
S R
= lw1 + bw2 - w1w2
= 80m × 2m + 40m×3m - 2m × 3m 3m
2 2 2
= 160m + 120m - 6m D W V C
80m
A = 274m2
 Area of the cross path is 274m2.
Now cost of gravelling sq.m (C) = Rs. 50
 Total cost (T.C) = A × C = 274 × Rs. 50 = Rs. 13700

74 | Mathematics - 9 Mensuration
Exercise 3.1
1. Calculate the area of the shaded regions of the following figures where width of the shaded
parts are uniform.
(a) (b)

18m
4cm 2m
32cm

(c) (d)

24m
4m 3m
16m

2. Find the area of the shaded parts in each of the following figures.
(a) (b)

0.7m 1.4cm

(c) (d)
4m 4cm

4m 16m 3cm 36cm

20m 42cm

Mensuration Mathematics - 9 |75


3. Find the area of the shaded portion with uniform width given in the figure below.
(a) (b)

30 m

60 m
6m
40 m

50 m 100 m

4. (a) If the rectangular ground has the length 80m, breadth 60m, uniform width of the path of
3m is running around outside of the ground, find the area of the path.
(b) A boarder 8cm wide runs around a towel 120 cm long and 80cm wide. Find the area of
the boarder of the towel.
(c) A path of uniform width of 1.5m runs all around a swimming pool 50m long and 25m
wide. Find the area of the path.
(d) Two paths each of width 4m run across the middle of a rectangular field 24m long and
16m wide. Calculate the area of the paths.
5. (a) A rectangular plot of land is 88m long and has an area 4840m2. If it has a path of
uniform width 2m within it, find the area of the path.
(b) A circular ground has a track of uniform width 2.1m. all around it. If the area of the
track is 568.26m2, find the area of the ground excluding the track.
(c) The cost of constructing a path running inside a rectangular field at Rs. 52 per square
metre is Rs. 7,280. Find the area of the path.
(d) 5000 bricks of size 20cm × 12cm each are required to pave the path running outside a
square field. Calculate the area of the path.
6. (a) A rectangular garden is surrounded by a path of uniform width 3m. If breadth of the
garden is 40m and area of the path is 588m2, find the length of the garden.
(b) A path of uniform width 4m runs outside a square flower bed and has an area of 448m2.
Find the area of the flower bed.
(c) A room measuring 6m × 4m is carpeted leaving space of 25cm all around. Find the area
of the carpet.
(d) The area of a square pond is 400 m2. A path of uniform width surrounds the pond and
its area is 176m2. Find the width of the path.
7. (a) Two cross paths run across a rectangular field of length 90m and breadth 60m. If the
area of the cross paths is 584m2, find the width of the path.
(b) Two cross paths each 3m wide run across a rectangular garden. If the area of the cross
path is 243m2 and the length of the garden is 48m, find the breadth of the garden.
(c) A circular plot of diameter 28m is surrounded by a path of uniform width. If the area of
the path is 346.5m2, find the width of the path.
(d) If the sum of the radii of two concentric circles is 28cm and difference is 12cm, find the
area of the annular part enclosed by the two circles.
8. (a) A rectangular field is 96m in length and 42m in breadth. A path of uniform width 3m runs
immediately inside its boundary. Find the cost of gravelling the path at Rs. 32 per square metre.

76 | Mathematics - 9 Mensuration
(b) A uniform path of width 2m surrounding a pool 20m × 16m is to be paved with marbles
of size 10cm × 8cm each. If a marble costs Rs. 5, find the cost of paving the path.
(c) A square lawn is surrounded by a path of uniform width 3m. The cost of paving the path
with marble at Rs. 200 per m2 is Rs. 36000. Find the area of the lawn and cost of
watering the lawn at Rs. 3 per m2.
(d) A square park 2025m2 in area is surrounded by a road 3m broad. Find the cost of paving
the road with concrete slabs 16cm × 12cm each at Rs. 10 per piece.
9. (a) A circular garden of diameter 56m has a path of uniform width 3.5m running
immediately outside it. Find the cost of plastering the path at Rs. 80 per m2.
(b) Fencing a circular field at Rs. 25 per meter costs Rs. 6600. A path 1.4 m wide runs
around the field. Find the cost of gravelling the path at Rs. 40 per sq. metre.
(c) Length and breadth of a rectangular field are in the ratio 2:1. The cost of gravelling two
cross paths of equal width across the middle of the field at Rs. 25 per sq.m is Rs. 11250
and if the cost of plastering the common part of the two cross paths at Rs. 40 is Rs. 360,
find the cost of turfing the empty space at Rs. 8 per m2.
(d) A park 36m × 20m is to be surrounded by a path of uniform width 2m, paving with
marble slabs 18cm × 12cm each at the rate of Rs. 20 per piece. If 10% of the space of
the path is occupied by cement, find the cost of paving the path.

3.2 Surface Area and Volume of Prisms


A prism is a solid having polygonal cross-section (base) and rectangular lateral surfaces.

Triangular based prism Square based prism Hexagonal based prism

Other prisms:

Mensuration Mathematics - 9 |77


Drawing three Dimensional View of a Solid
There are two basic views.
a. Isometric View: In this view none of the face (Top-bottom, front- back, side faces) are shown
rectangular.

(Use 30° - 60° set square)

Object line (dark)

Measurement line
3cm

Extension line

b. Oblique view: In this view, front face is shown rectangular.

(Use 45° - 45° set square)

Useful Relations
E H
Area of a cross section (base) = a (depends upon shape of
the polygonal base). In the given rectangular based prism,
Area of the cross section ABCD (a) = l × b ... (i) D
A F G
Note:
i. Opposite faces which are congruent determine the l h
cross section. Here ABCD and EFGH are bases (cross
sections). B b C
ii. Distance between these cross-sections is the length/height of the prism. Here CG = h is height.

78 | Mathematics - 9 Mensuration
iii. Rectangular faces between the cross sections or bases are lateral surfaces.
Lateral surface area of the prism = Area of (ABFE + BCGF + CDHG + ADHE)
= AB × AE + BC × CG + CD × CG + AD × AE
= l × h + b× h + l × h + b × h
= (2l + 2b)h
= 2(l + b)h
 L.S.A = P × h ... (ii) where p is the perimeter of the cross section.
 Total surface area (T.S.A) = L.S.A + 2 area of base
or, T.S.A = L.S.A + 2A ... (iii)
And volume of the prism = AB × BC × CG
=l×b×h
 V=A×h ... (iv)
Where A is the area of base.

Worked Out Examples


Example 1: Find the lateral surface area, total surface area and volume of the given
prism.
Solution: Here, D
A
the base is a right angled triangle with B = 90°,
BC = 7cm, AC = 25cm and height h = 30cm
 AB = AC2 - BC2 = (25cm)2 - (7cm)2 = 24cm E
B
1 F
 Base area/area of cross section (A) = 2 .BC. AB
C

1
= 2 × 7cm × 24cm = 84cm2.

Perimeter of the base (P) = AB + BC + CA


= 24cm + 7cm + 25cm = 56cm
Lateral surface area of the prism =P×h
= 56cm × 30cm
 L.S.A = 1680 cm2.
Total surface are area of the prism = L.S.A + 2A
= 1680cm2 + 2× 84cm2 = 1848cm2
And volume of the prism (V) =A×h
= 84cm2 × 30cm
V = 2520cm3.

Mensuration Mathematics - 9 |79


Example 2: Find the total surface area and volume of the prism given in the figure.
Solution: Here,
ABCDEF is the cross-section. Dividing the cross
A
section in rectangles ABMF and EMCD, area of F
cross-section = area of rect.(ABMF + EMCD)
= AB × BM + MC × CD
5cm 4cm
= 12cm × 4cm + 5cm × 4cm E D
 A = 68cm2.
M
Perimeter of the cross-section (p) = AB + BC + CD + B 9cm C
DE + EF + FA
= 12cm + 9cm + 4cm + 5cm + 8cm + 4cm
= 42cm.
Lateral surface area of the prism (L.S.A) =P×h
= 42cm × 6cm [ h = 6cm]
= 252cm2
T.S.A = L.S.A + 2.A
= 252cm2 + 2 × 68cm2 = 388cm2
And volume (V) =A×h
= 68cm2 × 6cm
V = 408cm3
Example 3: If the volume of a cube is 125cm3, find its total surface area.
Solution: Here,
Volume of cube = 125cm3.
If length of its side be l then,
l3 = 125cm3
 l = 5cm.
Now, total surface area of the cube (T.S.A) = 6l2 = 6 × (5cm)2 = 150cm2
Therefore, T.S.A of the cube is 150cm2.

Exercise 3.2
1. Find the total surface area and volume of the following solids.
(a) (b)

18cm

80 | Mathematics - 9 Mensuration
(c) (d)

6cm 12cm

2. (a) The volume of a cube is 216cm3. Find its total surface area.
(b) If the total surface area of a cube is 384cm2, find its volume.
(c) The volume of a cubical block is 350cm3. If the area of its base is 70cm2, find its height.
(d) The total surface area of a rectangular parallelopipe with square cross-section is
6048cm2. If the area of the cross-section is 1296cm2, find its length.
3. Find the total surface area and volume of the given solids:
(a) 4cm (b) 3cm
1cm

5cm 3cm
1cm

9cm

2cm 2cm
(c) (d)

4cm

7cm

4. Calculate the volume of the given solids:


(a) 3cm (b)

4cm 4cm

Mensuration Mathematics - 9 |81


(c) (d)
6cm

3cm

3cm 6cm

6cm

5. (a) The adjoining figure shows a victory


stand. Find the volume and surface area of 50cm
the stand if the bottom of the stand is
50cm
open.
50cm

(b) The figure given alongside is a solid


metal with rectangular hole in it. Find the
volume of the metal.
5cm

13cm

(c) The figure given alongside is a window 20cm


frame. The wood used has cross-section
7.5cm × 8cm. Find the volume of the
wood used.

2m

(d) A swimming pool is 16m broad and 50m long. If its depth is increasing regularly from
1m to 2m, how much water will it contain in litres when it is fully filled?
6. A cubical wooden solid having side 16cm is cut down into 8 equal pieces of cube. Find the
length of side of a new cube.
7. Calculate the volume and total surface area of your class room.
8. Investigate the volume of the stage of program halls of your school.

82 | Mathematics - 9 Mensuration
3.3 Area of Four Walls, Floor and Ceiling
Given figure can be considered as the model of a room. Having H
G
length l, breadth b and height h where ABCD is the floor, EFGH
is the ceiling and ABFE, BCGF, CDHG, DAEH are four walls.
h
A comfortably constructed room is generally cuboidal in shape E
F
where,
D C
Area of floor = area of ceiling = l×b. ... (i)
b
Area of front wall = area of back wall = l×h.
A B
Area of right side wall = area of left side wall = b×h. l

Area of 4 walls = 2lh + 2bh.


Ceiling
= 2h(l + b). ... (ii) Walls
Area of 4 walls, floor and ceiling
= 2lb + 2lh + 2bh.
= 2(lb + bh + bl). ... (iii) Floor

A room also consists windows and doors. Area of 4 walls also


include doors and windows thus, on calculating area of 4 walls we must exclude the areas of doors,
windows, ventilators etc.

Diagonal of a Room H
G

In a cuboidal room, line joining opposite corners of the room is h


unknown as diagonal of the room. In the figure, DF is a diagonal.
E F
All the four diagonals are equal. D C

Here, DAB = 90° b

 BD = AB2 + AD2 A l B
2 2
= l +b
Again in right DBF, where DBF = 90°

 DF = DB2 + BF2
2
= ( l2 + b2) + h2

= l 2 + b 2 + h2

 Diagonal of a room = l2 + b2 + h2 .

Mensuration Mathematics - 9 |83


For Carpeting the Floor of a Room
Area of the carpet = area of the floor = A
If the cost of the carpet per square unit is C,
Then, total cost for the carpet = area × rate = A × C
Note:
Total cost
i. cost/sq.unit = Area i. e. Total cost = A × C/m2

Total cost C
ii. cost/cu.unit = Volume i.e. Total cost = V × m3

Total cost C
iii. cost/piece = Number i.e. Total cost = N × piece.

Total cost C
iv. = Length i.e. Total cost = l ×
cost/unit length m

Worked Out Examples


Example 1: Calculate area of 4 walls, area of 4 walls and floor and area of 4 walls
with floor and ceiling of the room 8m long, 6m wide and 4m high.
Solution: Here, Length of the room (l) = 8m.
Breadth (b) = 6m.
Height (h) = 4m.
We have,
Area of 4 walls = 2h(l + b)
= 2 × 4m (8m + 6m)
= 8m × 14m = 112m2
Therefore, area of 4 walls is 112m2.
Area of 4 walls and floor = 2h (l + b) + lb
= 2 × 4m (8m + 6m) + 8m × 6m
= 8m × 14m + 48m2
= 112m2 + 48m2 = 160m2
 Area of 4 walls and floor is 160m2.
And area of 4 walls, floor and ceiling = 2 (lb + bh + hl)
= 2 (8m × 6m + 6m × 4m + 4m × 8m)
= 2 (48m2 + 24m2 + 32m2)
= 2 × 104m2 = 208m2
 Area of 4 walls, floor and ceiling is 208m2.

84 | Mathematics - 9 Mensuration
Example 2: Perimeter of the floor of a room is 18m and its height 4m. Find the area
of 4 walls of the room.
Solution: Here, Perimeter of the floor (P) = 18cm
Height (h) = 4m
We have,
Area of 4 walls = 2h (l + b)
= 2(l + b) h
=P×h
= 18m × 4m = 72m2
 Area of 4 walls of the room is 72m2.
Example 3: A room with length 8m, breadth 6m and height 4m has a door 2m × 1m
and two windows 1m × 2m each. Find the area of 4 walls excluding door
and windows.
Solution: Here, For the room
Length (l) = 8m.
Breadth (b) = 6m.
Height (h) = 4m.
 Area of 4 walls (A) = 2h (l + b)
= 2 × 4m (8m + 6m)
= 8m × 14m
= 112m2
Area of a door (A1) = 2m × 1m. = 2m2
Area of 2 windows (A2) = 2(2m × 1m) = 4m2.
 Area of 4 walls excluding door and windows = A – A1 – A2
= 112m2 – 2m2 – 4m2 = 106m2
Therefore, area of 4 walls excluding door and windows is 106m2.
Example 4: A room is 10m long and 8m wide. If the area of 4 walls is 180m2, find the
height of the room.
Solution: Here, Length of the room (l) = 10m.
Breadth (b) = 8m.
Let height be h
Area of 4 walls of the room = 180m2
or, 2h(l + b) = 180m2
or, 2h(10m + 8m) = 180m2
180m2
or, h = 36m

 h = 5m.
Therefore, height of the room is 5m.

Mensuration Mathematics - 9 |85


Example 5: A room is 3.5m high. If the area of its 4 walls is 126m2, find the perimeter
of the floor.
Solution: Here, Height of the room (h) = 3.5m.
Area of 4 walls (A) = 126m2.
Perimeter of the room (P) = ?
We have,
Area of 4 walls = P × h
or, 126m2 = P × 3.5m
 P = 36m.
Therefore, perimeter of the floor is 36m.
Example 6: The area of the 4 walls of a square room is 84m2. If the height of the
room is 3.5 m, find the area of its floor.
Solution: Here, Height of the room (h) = 3.5m.
Let length of the square room be l
Area of 4 walls of the square room = 84m2.
 2h(l + b) = 84m2
or, 2 × 3.5m (l + l) = 84m2
84m2
or, 2l = 7m

 l = 6m.
Now,
Area of floor of the room = l2 = (6m)2 = 36m2.
Example 7: A room is 8m long and 6m wide. How long carpet is required to cover the
floor of the room if the width of the carpet is 2 m?
Solution: Here, Let length of the carpet be l
Its width (b) = 2m.
Length of the room (L) = 8m
Breadth of the room (B) = 6m
Since, Area of carpet = area of room.
l×b=L× B
or, l × 2m = 8m × 6m
or, l = 24m.
Therefore, 24m long carpet is required to cover the floor.
Example 8: A room is 10m long, 6m wide and 4m high. Find the cost of carpeting the
floor at the rate of Rs. 50 per square metre and painting the four walls at
the rate of Rs. 25 per square metre.
Solution: Here, Length of the room (l) = 10m
Breadth (b) = 6m

86 | Mathematics - 9 Mensuration
Height (h) = 4m
Area of floor =l×b
= 10m × 6m = 60m2
Cost of carpeting the floor at Rs. 50 per m2 (C1) = Area × Rate
= 60m2 × Rs. 50/ m2
= Rs. 3000
Area of 4 walls = 2h(l + b)
= 2 × 4m(10m + 6m)
= 8m × 16m = 128m2
Cost of painting 4 walls at Rs. 75 per m2 (C2) = Area of 4 walls × Rate
= 128m2 × Rs. 25/m2
= Rs. 3200.
Total cost = C1 + C2
= Rs. 3000 + Rs. 3200
= Rs. 6200.
Therefore, the total cost of carpeting the floor and painting the walls is Rs. 6200.
Example 9: A room is 12m long, 8m broad and 4m high. It has a door 2m × 1m and 2
windows each 2m × 1.5m. Calculate the total cost of papering on the
walls and the ceiling at the rate of Rs. 50 per square metre.
Solution: Here, Length of the room (l) = 12m.
Breadth (b) = 8m
Height (h) = 4m
Size of a door = 2m × 1m
Size of a window = 2m × 1.5m
Total area occupied by the door and windows
a = 2m × 1m + 2(2m × 1.5m)
= 2m2 + 6m2 = 8m2
Area of 4 walls and ceilings = 2h(l + b) + l × b
= 2 × 4m(12m + 8m) + 12m × 8m
= 8m × 20m2+ 96m2
= 160m2 + 96m2
= 256m2
Area of 4 walls and ceiling excluding door and windows = 256m2 – 8m2
= 248m2
2
Now cost of papering on the walls and ceiling at Rs. 50 per m .
= Area × Rate
= 248m2 × Rs. 50/m2
= Rs. 12400.
Therefore, the cost of papering the walls and ceiling is Rs. 12400.

Mensuration Mathematics - 9 |87


Example 10: The cost of painting 4 walls of a room 4m high at Rs. 15 per square
metre is Rs. 2400. If breadth of the room is two third of its length, what
will be the cost of carpeting its floor at Rs. 200 per square metre?
Solution: Here, Height of the room (h) = 4m.
Rate of painting the walls = Rs. 15/m2.
Total cost of painting the walls = Rs. 2400.
Total cost
Area of 4 walls = Rate
2400
= Rs15/m2 = 160m2
If length of the room is l, then
2
Breadth (b) = 3 l

2h(l + b) = 160m2
2
or, 2 × 4m (l + l) = 160m2
3
160 × 3
or, 5l = 8 m

 l = 12m.
2 2 × 12
Breadth of the room (b) = 3 l = 3 m = 8m.
Now, area of floor = l × b
= 12m × 8m = 96m2
 Total cost of carpeting the floor at Rs. 200 per square metre =A×C
= 96m2 × Rs. 200/m2
= Rs. 19200.
Therefore, the cost of carpeting the floor of the room is Rs. 19200.
Example 11: The cost of plastering the walls of a square room at Rs. 10 per m2 is Rs.
3240 and the cost of carpeting its floor at Rs. 25 per m2 is Rs. 8100. Find
the height of the room.
Solution: Here, Cost of carpeting the room at Rs. 25 per m2 is Rs. 8100
T.C
Area of floor = C
Rs 8100
or, l2 = Rs 25

 l = 18m.
Again, cost of plastering the walls at Rs. 10 per m2 is Rs. 3240.
T.C
The area of 4 walls = C
Rs 3240
2h (l + l) = Rs 10/m2
or, 2 × h(18m + 18m) = 324m2
 h = 4.5m
Therefore, the height of the room is 4.5m.

88 | Mathematics - 9 Mensuration
Exercise 3.3
1. (a) Calculate the area of 4 walls of a hall of length 18m, breadth 12m and height 5m.
(b) Find the area of four walls and ceiling of a room 10 m long, 6m wide and 4m high.
(c) Find the area of 4 walls, floor and ceiling of a room 8m long, 6m width and 3.5m high.
(d) Find the area of 4 walls of a room 9m long, 6m wide and 4m high leaving 8m2 for doors
and windows.
2. (a) If the area of 4 walls of a room 6m wide and 3.5m high is 105m2, find the length of the
room.
(b) If the perimeter of a room 4m high is 44m, what is the area of 4 walls of the room?
(c) The area of 4 walls of a room is 56m2.Find the perimeter of the room if the height of the
room is 2 m.
(d) The area of 4 walls of a square room is 216m2. If the room is 4.5m high, find the length
of the room.
3. (a) A room is twice as long as it is broad and it is 3.5m high. If the area of its 4 walls is
105m2, find the area of its floor.
(b) The length and breadth of a chamber of a clock tower are 8m and 6m. If the diagonal of the
chamber is 26m, find the height of the chamber and hence find the area of its 4 walls.
(c) A room is 12m long, 10m broad and 4.5m high. It contains a door of size 1m × 3m and
two windows of size 2m × 1.5m each. Find the area of the four walls of the room
excluding the door and windows.
(d) Area of the ceiling of a room 4m high is 48m2. If the area of 4 walls of the room is
112m2, calculate the length and breadth of the room.
4. (a) If a room 8m long and 5m wide is laid with carpet 2m wide, find the total length of the
carpet required.
(b) A room 8m long and 4m broad is covered with carpet of a certain width. If the total
length of the carpet used is 12.8m, find the width of the carpet.
(c) How many pieces of paper each measuring 1.5m by 0.4m will be required to cover the four
walls of a hall 15m long, 9m wide and 5m high leaving 18m2 for doors and windows?
(d) How many pieces of paper each of area 1.25m2 are required to cover the 4 walls and
ceiling of a room 9m long, 7m wide and 3.5m high, if 7.5m2 of the wall is covered by
doors and windows?
5. (a) Find the cost of plastering 4 walls and ceiling of a room 9m long, 6.5m broad and 4m
high at a rate of Rs. 20 per square metre.
(b) Find the cost of laying a room 7m long and 6.5 m broad with parquet at Rs. 250 per
square metre.
(c) Find the cost of distempering the walls of a room 10m long, 8m wide and 5m high at
Rs.15 per square metre.
(d) The cost of plastering the walls of a room at Rs. 21.50 per m2 is Rs. 3870. Find the cost
of painting the walls at Rs. 16 per square metre.

Mensuration Mathematics - 9 |89


6. (a) The cost of painting the 4 walls of a room at the rate of Rs. 7 per m2 is Rs. 546. If the
room is 6m broad and 3m high, what is its length?
(b) Total cost of plastering 4 walls and ceiling of a room 7m long and 7m wide with
gypsum at Rs. 125 per square metre is Rs. 20125. Find the height of the room.
(c) If the perimeter of a square room is 54m and height 4m, find the total cost of carpeting
the floor at Rs. 48 per m2 and plastering its four walls at Rs. 12 per m2.
5
(d) Doors and windows of a room covers 59 % of the space of the walls. The cost of
distempering the remaining parts at Rs. 12 per m2 is Rs. 1428. If the sum of the length
and the breadth of the room is 14m, find the height of the room.
7. (a) The cost of plastering 4 walls of a room whose length is twice its breadth and thrice its
height at Rs. 7 per m2 is Rs. 1008. What will be the cost of carpeting the floor of the
room at Rs. 200 per m2?
(b) The height of a square room is one third of its length. If the cost of plastering its four walls
and ceiling at Rs. 9.5 per m2 is Rs. 3192, find the cost of carpeting its floor at Rs. 125 per m2.
(c) Length, breadth and height of a room are in the ratio 6:3:2. If the cost of carpeting the
floor at Rs. 120 per square metre is Rs. 13500, find the cost of plastering the walls and
ceiling at Rs. 30 per square metre.
(d) The cost of plastering four walls of a room 4m high and whose breadth is one third of
its length at Rs. 12 per m2 is Rs. 1536. What will be the cost of parqueting the floor at
the rate of Rs. 200 per square metre?
8. (a) The cost of carpeting the floor of a room at Rs. 75 per sq.m. is Rs. 11,250. If the room
contains 660 cu.m. of air, find the height of the room.
(b) The length of a room is twice its breath and it contains 396m3 of air. If the cost of
plastering its floor at Rs. 15 per sq.m. is Rs. 1080, find the cost of plastering its 4 walls
at Rs. 12 per sq.m.

3.4 Volume of Walls


We know,
Volume of a cuboid (V) = l × b × h
or, V= A × h
i.e. V = base area × height
Volume of a cube (V) = l3
Walls are to be built in different prismic and cylindrical shapes.
1. A single wall has as its dimension
Length × thickness × height
If length = l, thickness = t, and height = h, then
Volume (V) = l.t.h

90 | Mathematics - 9 Mensuration
Walls also consist separate doors and windows as thick as the wall and of desired length and
height. For example, if width (length) and height of a door be l and h, then volume occupied
by the door.
V = l  t  h where t is thickness of wall.
2. Surrounding wall (inside)
D C

h
b

l
b
A l B

Let a wall of height h and thickness 't' is to be built surrounding inside a rectangular plot of
length l and breadth b. The base of the surrounding wall is just like a surrounding path of area.
A = 2t(l + b – 2t)
 Volume of the surrounding wall (inside),
V=A×h
 V = 2t(l + b – 2t)h
3. Surrounding wall (outside)
Let a wall of thickness t and height h is to be built surrounding outside a rectangular plot of
length l and breadth b, then the base of the wall is just like a surrounding path (outside) of area.
A = 2t(l + b + 2t)
 Volume of the surrounding wall (outside)
V=A×h
 V = 2t(l + b + 2t)h
4. Cross wall

t b

Let wall of height h and thickness 't' is to be built across a rectangular plot of length l and
breadth b.

Mensuration Mathematics - 9 |91


The base of the wall is just like a cross path of area.
A = t(l + b – t)
Volume of cross wall
V=A×h
V = t(l + b – t)h
Number of bricks and their cost required to build a wall
Let the volume of the wall = V
the Volume of a brick = v then
Volume of wall V
Number of bricks required (N) = Volume of a brick = v

If C is the cost of a brick, then total cost of bricks = Number of bricks × Rate
T.C = NC
Note: Units of all the dimensions of a wall and the bricks should be same.
Volume of material contained in a box
1. If external dimensions are l × b × h, external volume Vext = l × b × h. If thickness of the
material is t then internal dimensions are (l – 2t)(b – 2t)(h – 2t)
 Internal volume = Vint = (l – 2t)(b – 2t)(h – 2t)
 The volume of the material in the box = Vext - Vint.
2. If internal dimension l × b × h and thickness t are given,
Internal volume (Vint) = l × b × h
External volume (Vext) = (l + 2t)(b + 2t)(h + 2t)
 The volume of the material in the box = Vext - Vint.

Worked Out Examples


Example 1: Find the volume of a wall 25m long, 50cm wide and 4m high.
Solution: Here,
Length of the wall (l) = 25m
Thickness of the wall (b) = 50cm = 0.5m
Height (h) = 4m.

92 | Mathematics - 9 Mensuration
Now,
We have, volume of the wall,
Volume of the wall (V) =l×b×h
= 25m × 0.5m × 4m = 50m3
Therefore, the volume of the wall is 50m3.
Example 2: There are 1200 bricks each of size 25cm × 10cm × 5cm in a wall. Find the
volume of the wall.
Solution: Here,
Size of a brick = 25cm × 10cm × 5cm
 Volume of a brick (v) = 0.25m × 0.10m × 0.05m.
v = 0.00125m3.
Number of bricks (N) = 1200
Volume of the wall (V) =N×v
= 1200 × 0.00125m3 = 1.5m3
Therefore, the volume of the wall (V) = 1.5m3
Example 3: A wall is 25m long, 4m high and 30cm thick. If it contains two windows
2m × 1.5m each and a door 3m × 2m, find the cost of bricks of size
15cm ×10cm ×5cm at Rs. 4 per piece.
Solution: Here,
Length of wall (l) = 25m,
Height of the wall (h) = 4m,
Thickness of the wall (t) = 30cm = 0.30m
 Volume of wall = l × h × t = 25m × 4m × 0.30m = 30m3.
Volume of the wall occupied by doors and windows = 2(2m×1.5m×0.3m)+3m×2m×0.3m
= 1.8m3 + 1.8m3 = 3.6m3
Volume of the wall excluding the doors and windows (V)
= Volume of wall – volume of doors and windows.
= 30m3 – 3.6m3
= 26.4m3
Volume of a brick (v) = 15cm × 10cm × 5cm
= 0.15m × 0.10m × 0.05m
= 0.00075m3.
V
Now, number of bricks needed (N) = v

26.4m3
or, N = 0.00075m3 = 35200

Therefore, cost of bricks at Rs. 4 per piece = N × rate = 35200 × Rs. 4 = Rs. 140800

Mensuration Mathematics - 9 |93


Example 4: Find the number of bricks each of size 14cm × 8cm × 5cm to build a wall
3m high and 30cm thick around the inside of a rectangular plot 15m ×
8m.
Solution: Here,
Length of the plot (l) = 15m
Breadth (b) = 8m
Height of the wall (h) = 3m
Thickness of the wall (t) = 30cm = 0.3m
 Volume of the surrounding wall (inside) the rectangular plot
(V) = 2t(l + b – 2t) × h
= 2 × 0.3m(15m + 8m – 2 × 0.3m) × 3m.
= 0.6m × 22.4m × 3cm = 40 .32m3.
Size of a brick = 14cm × 8cm × 5cm
 Volume of a brick (v) = 0.14m × 0.08m × 0.05m = 0.00056m3.
V 40.32m3
Number of bricks (N) = = = 72000.
v 0.00056m3
Therefore, 72000 bricks are required to build the wall.
Example 5: The external dimensions of a closed wooden box are 60cm × 50cm ×
30cm and the thickness of the wood in 2cm. Find the volume of the
wood used to make the box.
Solution: Here,
External length (l) = 60cm, breadth (b) = 50cm and height 30cm.
External volume (Vext) =l×b×h
= 60cm × 50cm × 30cm
= 90000cn3
Thickness of the wood (t) = 2cm
Internal length (l1) = l – 2t
= 60cm – 2 × 2cm = 56cm
Internal breath (b1) = b – 2t
= 50cm – 2 × 2cm = 46cm
Internal height (h1) = h – 2t
= 30cm – 2 × 2cm = 26cm
Internal volume (Vint) = l 1 × b 1 × h1
= 56cm × 46cm × 26cm = 66976cm3
Now, volume of wood = Vext - Vint
= 90000cm3 - 66976cm3
= 23024cm3

94 | Mathematics - 9 Mensuration
Example 6: The internal dimensions of a wooden box are 36cm × 26cm × 16cm. Find
the external volume of the box, if the volume of wood used in 9024cm3.
Solution: Here,
Internal length (l) = 396cm, internal breadth (b) = 26cm and
Internal height (h) = 16cm
 Internal volume (Vint) =l×b×h
= 36cm × 26cm × 16cm
= 14976cm3
Volume of wood (V) = 9024cm3.
External volume of the box (Vext) = Vint + V
= 14976cm3 + 9024cm3
= 24000cm3
Therefore, the external volume of the box is 24000cm3.

Exercise 3.4
1. (a) Find the volume of a wall 10m long, 4m high and 25cm thick.
(b) Find the volume of a brick of size 25cm by 10cm by 5cm in cubic meter.
(c) A wall is 18m long and 3m high. If its volume is 10.8m3, find its thickness.
(d) A wall 3.5m high, 25cm thick. If its volume is 10.5m3, how long is the wall?
2. (a) A wall 20m long, 4m high and 25cm thick has two windows 2m × 1.5m each. Find the
volume of the wall excluding the windows.
(b) 1500 bricks 25cm by 10cm by 5cm each are required to build a wall. Find the volume of
the wall.
(c) Find the volume of each brick if 62500 bricks were used to construct a wall of size
18m × 4m × 1.5m.
(d) Find the number of bricks required to construct a wall 30m × 4m × 0.5m, if the size of
each brick is 25cm × 10cm × 5cm.
3. (a) A brick of volume 1250cm3 costs Rs. 3.50. A man spends Rs. 52,500 to build a wall.
Find the volume of the wall.
(b) A brick costs Rs. 3.50 and Tuyoocha Sahu had to spend Rs. 87,500 to build a wall of
volume 33.75m3. Find the volume of each brick.
(c) A wall 11m × 25cm × 3.5m is built by using bricks each of size 22cm × 10cm × 5cm. If
a brick costs Rs. 2.50, find the total expenditure.
(d) A wall 25m × 20cm × 3m built with 1250cm3 same sized bricks cost Rs. 40800. Find
the cost of each brick.

Mensuration Mathematics - 9 |95


4. (a) Find the volume of a wall 30cm thick to build a room 10m long, 8m wide and 3.5m
high leaving 1.06m3 of the volume for the doors and windows.
(b) A wall 40cm thick and 3.5m high is to be built all around, inside of a rectangular garden
40m × 30.8m with an open gate 3m wide. If 10% of the wall is occupied by the cement,
how many bricks of size 21cm × 13.7cm × 5cm are required to build the wall? Also, if a
brick costs Rs. 3.5, calculate the total cost of bricks.
(c) A rectangular plot 36m × 24.3m is divided into equal quadrants by building walls 30cm
thick and 3m high across the plot. If 10% of the wall is occupied by cement, how many
bricks of size 24cm × 12cm × 6cm are used in the wall?
(d) A cuboidal reservoir 32m × 24m × 4m is to be separated into 4 equal chambers by
constructing cross-walls 40cm thick up to its height. Find the volume of the cross wall.
5. (a) A lid less metal box has external dimension 25cm × 20cm × 12cm. If the thickness of
the material is uniformly 1cm, find the volume of the metal used in the box.
(b) A closed cubical metal box of side 25cm and 0.5cm thick is to be constructed.
(i) Find the volume of the metal required to construct the box.
(ii) The cost of the metal required, if 1cm3 of metal cost is Rs. 0.50.
(c) A cubical tin box has external dimensions 60cm × 50cm × 40cm. The thickness of the
tin is 0.2cm. Find the weight of the box, if 1cm3 of tin weighs 10gm.

Project Work
6. Take the measurements of the dimensions of your class room.
(a) Find the area of 4 walls of the room.
(b) Find the area and total cost of plastering at Rs. 200 per sq.m. floor of the room.
(c) Find the total cost of painting on the ceiling of the class room at the rate of Rs. 250 per
square meter.
(d) Find the area of 4 walls excluding the window and the door.
(e) Calculate the total cost of papering on its 4 walls excluding door and window at the rate
of Rs. 30 per square meter.
7. Measure the length and breadth of the basket ball court of your School. Calculate the number
of tiles of size 2ft. x 1ft. needed to pave on it at the rate of Rs. 315 per piece.

96 | Mathematics - 9 Mensuration
Unit Test
Time: 40 minutes F.M.- 24
Group-A (3×1=3)
1. What is the area of a square when the diagonal (d) is given?
2. Find the area of four walls of the room having length 10m, breath 8m and height 5m.
3. If the area of cross-section of a prism is A cm2 and height is h cm, find its volume.
Group- B (4 × 2 = 8)
4. The perimeter of a square garden is 100m. Find its area.
5. Find the perimeter of a semi-circle having diameter 14cm.
6. A room has length 9ft and breadth 6ft. What is the total cost of carpeting on its floor at the
rate of Rs. 150 per sq. ft?
7. The wall has to be made of length 11m, breadth 1m and height 5m. How many bricks of
dimensions 22 cm×10cm×5 cm are needed.
Group- C (2×4=8)
8. Find the total surface area of given prism.
9 cm
4 cm

7 cm

16 cm

9. A room of length 7m & breadth 5m has a door of side 2m×1.5m & two windows of side
1m×1.5m each. Total cost of colouring on it's four walls at the rate of Rs 7.5 per m2 is Rs
495. Find the height of the room.
Group- D (1× 5=5)
10. The volume of the room having length double of its breadth is 396m3. Total cost of
colouring its celling at the rate of Rs 30 per m2 is Rs 2160 what is the cost of colouring its
four walls at the rate of Rs60 per m2?

Mensuration Mathematics - 9 |97


Answers ____________________________________________________________
Exercise 3.1
1. (a) 352m2 (b) 144m2 (c) 192m2 (d) 252m2
2. (a) 29.26m2 (b) 43.12m2 (c) 128m2 (d) 258m2
2 2
3. (a) 1350m (b) 924m
4. (a) 876m2 (b) 2944cm2 (c) 234m2 (d) 144m2
2 2 2
5. (a) 556m (b) 5544m (c) 140m (d) 120m2
2 2
6. (a) 52m (b) 576m (c) 19.25m (d) 2m
7. (a) 4m (b) 36m (c) 3.5m (d) 1056cm2
8. (a) Rs. 25344 (b) Rs. 1,00,000 (c) Rs. 432 (d) Rs. 3,00,000
9. (a) Rs. 52360 (b) Rs. 15030.4 (c) Rs. 38016 (d) Rs. 2,00,000

Exercise 3.2
1. (a) 660cm2, 600cm3 (b) 600cm2, 864cm3 (c) 247.18cm2, 187.08cm3 (d) 864cm2, 1728cm3
2 3
2. (a) 216cm (b) 512cm (c) 5cm (d) 24cm
3. (a) 462cm2, 540cm3 (b) 60cm2, 72cm3 (c) 322cm2, 246cm3 (d) 452cm2, 504cm3
4. (a) 102cm3 (b) 144cm3 (c) 156cm3 (d) 324cm3
5. (a) 152000cm3, 16800cm2 (b) 556cm3 (c) 69600cm3 (d) 12,00,000 litres.
6. 8 cm

Exercise 3.3
1. (a) 300m2 (b) 188m2 (c) 194m2 (d) 112m2
2. (a) 9m (b) 176m (c) 14m (d) 12m
3. (a) 50m2 (b) 24m (c) 189m2 (d) 8m, 6m
4. (a) 20m (b) 2.5m (c) 370 (d) 134
5. (a) Rs. 2480 (b) Rs. 11375 (c) Rs. 2700 (d) Rs. 2880
6. (a) 7m (b) 4m (c) Rs. 11340 (d) 4.5m
7. (a) Rs. 14400 (b) Rs. 1800 (c) Rs. 10125 (d) Rs. 9600
8. (a) 4.4m (b) Rs. 2376
Exercise 3.4
1. (a) 10m3 (b) 0.00125m3 (c) 20cm (d)12m
2. (a) 18.5m3 (b) 1.875m3 (c) 1728cm3 (d) 48000
3. (a) 18.75m3 (b) 1350cm3 (c) Rs. 21875 (d) Rs. 3.4
4. (a) 38m3 (b) 1,20,000, Rs. 42,000 (c) 28125 (d) 222.4m3
3 3
5. (a) 1446cm (b) 1951cm , Rs. 975.5 (c) 29.36 kg
6. Show your teacher
7. Show your teacher.

98 | Mathematics - 9 Mensuration
Chapter

4
Algebra
Objectives:
At the end of this chapter, the
students will be able to:
 factorize the different algebraic
expression.
 simplify the problem related to
laws of indices.
 solve the exponential equation.
 simplify the surds and rationalize
them.
 solve the problems related to
ratio and proportion.

Teaching Materials:
Chart of algebraic formula, chart of
laws of indices and surds, flash cards,
different color board markers, etc.
4.1 Factorisation
Introduction
Let us take two algebraic expressions 2x and (x – 3y). When these two expressions are multiplied
together, then the product of 2x and (x – 3y) is 2x × (x – 3y) = 2x2 – 6xy.
2x2 – 6xy is a single algebraic expression which is the product of the two expressions 2x and x – 3y.
So, 2x and x – 3y are called the factors of the expression 2x2 – 6xy.
Thus, the factorization is the process by which the given algebraic expression can be expressed as the
product of two or more algebraic expression or terms.
When we factorise an algebraic expression, we write the given expression as the product of its factors.
So, for finding the factors of the given expression, we apply the selected method of factorization for
the particular type of expression. Therefore, it is very important and useful to know about the types of
expression which are to be factorised.
1. (i) Factorisation of the expression by taking common only:
To factorise such as expression, the common factor is taken out and each term of the given
expression should be divided by the common factor to get another factor.
For example: 3x2y – 6xy2 is an expression containing two terms. The both terms have the
common factor is 3xy. Then, 3x2y – 6xy2 = 3xy (x – 2y)
(ii) Factorisation of the expression by making groups before taking common:
To factorise such an expression, the terms of the given expression are to be arranged for
making a suitable groups such that each group has a common factor.
For example: x2 + xy + zx + yz is an expression. In this expression, the first two terms are
taken in a group. Then by taking common in each group, the common factor is (x + y).
Now, x2 + xy + zx + yz
= x(x + y) + z(x + y)
= (x + y) (x + z)
(x + y) and (x + z) are the factors of the given expression x2 + xy + zx + yz.

Worked Out Examples


Example 1: Factorise: x2y + xy2 + 3x + 3y.
Solution: x2y + xy2 + 3x + 3y
= xy(x + y) + 3(x + y)
= (x + y)(xy + 3)
Example 2: Factorise:
(a) 2a (x + y) – b (x + y)
(b) 6x (a – b) + 3xy (b – a)

100 | Mathematics – 9 Algebra


Solution: (a) 2a (x + y) – b (x + y)
let x + y = p, then the given expression is
2ap – bp = p(2a – b)
= (x + y) (2a – b)
 2a(x + y) – b(x + y) = (x + y) (2a – b)
(b) 6x (a – b) + 3xy (b – a)
= 2 × 3x (a – b) + 3xy {– (a–b)}
= 2 × 3x (a – b) – 3xy (a – b)
= 3x (a – b) (2 – y)
Example 3: Factorise:
(a) x2 – xy + y2 – xy (b) x2 – x(2m – n) – 2mn
Solution: (a) x2 – xy + y2 – xy (b) x2 – x(2m – n) – 2mn
= x2 – xy – xy +y2 = x2 – 2mx + nx – 2mn
= x(x – y) – y(x – y) = x(x – 2m) + n(x – 2m)
= (x – y)(x – y) = (x – 2m)(x + n)
2. Factorisation of the expression of the form a – b2: 2
a
2 2
To factorise an expression a – b , we should follow the following steps.
a2 sq. unit a
a. Let's take a square chart paper of length a units, where the area of it is a2 sq.
units.
b. Cut a square piece of length b units from the one corner of the given square
paper as shown in the following given figure. The area of such square piece is b2 sq. units.
Then the area of the remaining part of the square paper is (a2 – b2) sq. units.
a a

a2 a2 - b2 Remaining part
a b a
b b2

c. Cut the remaining part whose area is (a2 – b2) sq. units along a
the diagonal as shown in the figure alongside. a-b
a b
d. Make a rectangle by rearranging two parts after cutting, where
the length and the breadth of the rectangle are (a + b) units and b
(a – b) units respectively.
a-b
b a

(a-b) (a-b)

a b

Algebra Mathematics – 9 |101


Now, area of the rectangle = length × breadth
= (a + b) × (a – b)
= a2 – ab + ab – b2
= a2 – b 2
 Area of the rectangle = (a2 – b2) sq. units

a2 – b2 = (a + b)(a – b).

Here, the expression a2 – b2 is the difference of the square of two terms a and b. From the above
illustration, it is clear that (a + b) and (a – b) are the factors of a2 – b2. Thus, we can use the formula
a2 – b2 = (a + b)(a –b) to factorize the algebraic expression of the form a2 – b2.
Important formulae:
 (a + b)2 = a2 + 2ab + b2 or, (a – b)2 + 4ab
 (a – b)2 = a2 – 2ab + b2 or, (a + b)2 – 4ab
 a2 – b2 = (a + b)(a – b)
 a2 + b2 = (a + b)2 – 2ab or, (a – b)2 + 2ab

Worked Out Examples


Example 1: Factorize:
2 2 2 2
(a) 16a – 49b (b) (2x + y) – (x – 2y)
Solution: (a) 16a2 – 49b2
= (4a)2 – (7b)2
= (4a + 7b)(4a – 7b)
(b) (2x + y)2 – (x – 2y)2
= {(2x + y) + (x – 2y)}{(2x + y) – (x – 2y)}
= (2x + y + x – 2y)(2x + y – x + 2y)
= (3x – y)(x + 3y)
Example 2: Factorize:
(a) 3a2b – 27b3 (b) m2 – 4n2 – m3 + 2m2n
Solution: (a) 3a2b – 27b3 = 3b(a – 9b2)
2

= 3b{(a)2 – (3b)2}
= 3b(a + 3b)(a – 3b)
(b) m2 – 4n2 – m3 + 2m2n = m2 – (2n)2 – m2(m – 2n)
= (m + 2n) (m – 2n) – m2 (m – 2n)
= (m – 2n) (m + 2n – m2)

102 | Mathematics – 9 Algebra


Example 3: Factorize: 4x2 – y2 + 2y – 1.
Solution: Here, 4x2 – y2 + 2y – 1
= (2x)2 – (y2 – 2y + 1)
= (2x)2 – (y – 1)2 [ (a – b)2 = a2 – 2ab + b2]
= (2x + y – 1){2x – (y – 1)}
= (2x + y – 1)(2x – y + 1)

3. Factorization of the expression of the form of a3  b3:


We know that,
(a + b)3 = (a + b)1 (a + b)2
= (a + b) (a2 + 2ab + b2)
= a3 + 2a2b + ab2 + a2b + 2ab2 + b3
= a3 + b3 + 3a2b + 3ab2
= a3 + b3 + 3ab(a + b)
 a 3 + b3 = (a + b)3 – 3ab(a + b)
= (a + b) {(a + b)2 – 3ab}
= (a + b)(a2 + 2ab + b2 – 3ab)
 a3 + b3 = (a + b)(a2 – ab + b2)
So, (a + b) and (a2 – ab + b2) are the factors of a3 + b3.
Similarly,
a3 – b3 = (a – b)(a2 + ab + b2)
So, (a – b) and (a2 + ab + b2) are the factors of a3 – b3.
Thus, to factorize the expression of the form a3 + b3 and a3 – b3, we should use the following
formulae.
a3 + b3 = (a + b)(a2 – ab + b2)
a3 – b 3 = (a – b)(a2 + ab + b2).

Worked Out Examples


Example 1: Factorize:
(a) a6 – 1 (b) 8ax3 – ay3
Solution: (a) a6 – 1 = (a2)3 – (1)3
= (a2 – 1){(a2)2 + a2.1 + 12}
= (a2 –1){(a2 + 1)2 – 2a2 + a2}

Algebra Mathematics – 9 |103


= (a + 1)(a – 1){(a2 + 1)2 – a2)}
= (a + 1)(a – 1)(a2 + a + 1)(a2 – a + 1)
(b) 8ax3 – ay3 = a(8x3 – y3)
= a {(2x)3 – (y)3}
= a (2x – y) {(2x)2 + 2x.y + y2}
= a (2x – y) (4x2 + 2xy + y2)
Example 2: Factorize:
(a) x6– y6 (b) (2x + y)3 – (2x – y)3
Solution: Here,
(a) x6– y6 = (x3)2 – (y3)2
= (x3 + y3) (x3 – y3)
= (x + y) (x2 – xy + y2) (x – y) (x2 +xy + y2)
= (x + y) (x – y) (x2 – xy + y2) (x2 +xy + y2)
(b) (2x + y)3 – (2x – y)3
= {(2x + y) – (2x – y)}{ (2x + y)2 + (2x + y)(2x – y) + (2x – y)2}
= (2x + y – 2x + y){(2x)2 + 2.2x.y + y2 + (2x)2 – y2 + (2x)2 – 2.2x.y + y2}
= (2y)(4x2 + 4xy + y2 +4x2 – y2 + 4x2 – 4xy + y2)
= 2y(12x2 + y2)
Example 3: Factorize: x3 – 2x2 + 8 – 4x
Solution: Here, x3 – 2x2 + 8 – 4x
= (x)3 + (2)3 – 2x2 – 4x
= (x + 2)(x2 – x.2 + 4) – 2x(x + 2)
= (x + 2)(x2 –2x + 4 – 2x)
= (x + 2)(x2 – 4x + 4)
= (x + 2)(x – 2)2
= (x + 2)(x – 2)(x – 2).

4. Factorization of the expression of the form ax2 + bx  c:


To factorize the trinomial expressions of the form ax2 + bx  c, we need to find the two numbers
whose product is ac and the sum or difference is b. Then the given trinomial expression is expanded to
four terms by breaking the middle term and factorization process is performed by grouping.
Let the two numbers be p and q. Then, pq = ac and p q = b. The expression ax2 + bx + c is written as
ax2 + (p  q)x + c.

104 | Mathematics – 9 Algebra


Worked Out Examples
Example 1: Factorize:
(a) x2 + 7x + 12 (b) 4a2 – 16a – 9
Solution: (a) x2 + 7x + 12
Comparing this expression with ax2 + bx + c, then, a = 1, b = 7 and c = 12
Now, a × c = 1 × 12 = 12
The pairs of the possible product are, 1× 12, 2 × 6, 3 × 4,
Among these pairs, the sum of pairs 3 and 4 is 7.
3+4=7
Therefore, the two required numbers are 3 and 4.
Now,
x2 + 7x + 12
= x2 + (3 + 4)x + 12
= x2 + 3x + 4x + 12
= x(x + 3) + 4(x + 3)
= (x + 3)(x + 4)
(b) 4a2 – 16a – 9
Comparing this expression with ax2 + bx + c, then, a = 4, b = –16 and c = –9
Now, a × c = 4 × –9 = –36
The pairs of the possible product are, 1 × – 36, 2 × –18, 3 × –12, 6 × –6, 4 × –9
or, –1 × 36, –2 × 18, –3 × 12, –6 × 6, –4 × 9
Among these pairs, the required pairs whose sum is –16 are 2 and –18 since
2 + (–18) = 2 – 18 = –16
Now, 4a2 – 16a – 9
= 4a2 – (18 – 2)a – 9
= 4a2 – 18a + 2a – 9
= 2a(2a – 9) + 1(2a – 9)
= (2a – 9)(2a + 1)
Example 2: Factorize:
x2 x a2 b2
(a) 12 2 + – 20 (b) 10 2 – 1 – 2 2
y y b a
x2 x
Solution: (a) 12y2 + y – 20

x2 x
= 12y2 + (16 – 15) y – 20

x2 x x
= 12y2 + 16 y – 15 y – 20

Algebra Mathematics – 9 |105


x x x
= 4 y 3 y + 4 – 53 y + 4
   
x x
= 3 y + 4  4 y – 5 
  
2 2
a b
(b) 10b2 – 1 – 2 a2

a2 a b b2
= 10b2 – (5 – 4)b . a – 2a2

a2 a b a b b2
= 10b2 – 5b . a + 4b . a – 2a2

a a b b a b
= 5 b 2b – a  + 2 a 2b – a 
   
a b a b
= 5b + 2 a  2b – a 
  
Example 3: Factorize:
(a) 8x6 – 7x3 – 1
(b) 2(2x + x2)2 – 2(2x + x2) – 12
Solution: (a) 8x6 – 7x3 – 1
= 8x6 – 8x3 + x3 – 1
= 8x3(x3 – 1) + 1(x3 – 1)
= (x3 – 1)(8x3 + 1)
= {(x)3 – (1)3}{(2x)3 + (1)3}
= (x – 1)(x2 + x.1 + 12)(2x + 1){(2x)2 – 2x.1 + (1)2)
= (x – 1)(2x + 1)(x2 + x + 1)(4x2 – 2x + 1)
(b) 2(2x + x2)2 – 2(2x + x2) – 12
Put 2x + x2 = y,
then, 2y2 – 2y – 12
= 2y2 – 6y + 4y – 12
= 2y(y – 3) + 4(y – 3)
= (y – 3)(2y + 4)
Putting the value of y, we get,(x2 + 2x – 3)[2(x2 +2x) + 4]
= (x2 + 2x – 3)(2x2 + 4x + 4)
= (x2 + 3x – x – 3). 2(x2 + 2x + 2)
= {x(x + 3) – 1(x + 3)}.2(x2 + 2x + 2)
= 2 (x + 3) (x – 1) (x2 + 2x + 2)

106 | Mathematics – 9 Algebra


Exercise 4.1.1
1. Factorize the following.
(a) 3x – 4xy (b) 2x(a – b) + 4y(a – b)
(c) 4ax – 6a2x + 8ay (d) 14p2q + 6pq2 – 12pq
(e) 4a2 – b2 (f) 3x3 – 48x
1 16
(g) 9a2 – 4 (h 9x2 – 25y2

(i) x4 – y 4 (j) 27x3 – 12x


(k) 3x2 – 147 (l) 2a2 – 242
(m) a3 + 8b3 (n) x3 – 27y3
4 2 5
(o) m n + mn (p) 16 p4 – 2p
1
(q) 64m6 + n6 (r) x3 + x3

1
(s) x6 + x6 (t) (y + 1)2 – 9

(u) (x – 3)2 – 4(x – 3) (v) 16 – (2a – b)2


(w) (2x–3)2–(3x – 2)2
2. Factorize the following.
(a) 3x2 + 8x + 4 (b) x2 + 7x + 12
(c) a2 + 5a + 4 (d) a2 + 5a – 6
(e) 4a2 – 16a – 9 (f) 3x2 – 7x – 6
4 2
(g) 2a – 5a – 12 (h) 2x6 – x3 – 3
7x2 3y2 2x2 3x
(i) y 2 –4–
x2 (j) y2 – y – 5
10p2 3q2 3a2 10b2
(k) q2 – 1 – p2 (l) b2 – 11 + a2
(m) 9 (a + b)2 + (a + b) – 8 (n) 3 (x + y)2 – 10 (x + y) – 8
1 2 1
(o) 4 (x + y)2 – 5 (x + y)z – 6z2 (p) 7a –  + 5a –  –2
 a  a
(q) –a2 – 12a – 27 (r) –a2 + 9a – 14
3. Factorize:
(a) x2 – y2 – ax + ay (b) a2 – b2 – ax – bx
(c) a2 + ab + ac + bc (d) a2 + 6ab – ca – 6bc
2 2
(e) x – 2 (x+ y) – y (f) b2 – ab + a2 – ab
(g) ab2 – b(a – c) – c (h) x3 – x2 – x + 1
(i) x2y + 2xy – 3x – 6 (j) y3 + 1 + 2y2 + 2y
3 2
(k) x + 2x + 8 + 4x (l) 8 – 36x – 2x2 + 9x3

Algebra Mathematics – 9 |107


(m) a3 – a + b 3 – b (n) x3 – 8y3 + 2xa – 4ya
3 3 2 2
(o) 8a + b + 12a b + 6ab (p) x3 – x – y3 + y
(q) 4x2 – y2 + 2y – 1 (r) 4x2 – 9y2 + 4x + 1
(s) 9a2 – 4b2 + 6a + 1 (t) 1 + 4p + 4p2 – 36p4

Factorization of the expression of the form a4 + a2b2 + b4


To factorize the expression of the form a4 + a2b2 + b4 by using the similar method of factorizing the
expression of the form a2 – b2.
Now, a4 + a2b2 + b4
= a4 + b4 + a2b2
= (a2)2 + (b2)2 + a2b2
= (a2 + b2)2 – 2a2b2 + a2b2 [a2 + b2 = (a + b)2 – 2ab]
= (a2 + b2)2 – a2b2
= (a2 + b2)2 – (ab)2
= (a2 + b2 + ab) (a2 + b2 – ab) [a2 – b2 = (a + b)(a – b)]
= (a2 + ab + b2) (a2 – ab + b2)

Worked Out Examples


Example 1: Factorize: x4 + 6x2y2 + 25y4.
Solution: Here, x4 + 6x2y2 + 25y4
= (x2)2 + (5y2)2 + 6x2y2
= (x2 + 5y2)2 – 2.x2.5y2 + 6x2y2
= (x2 + 5y2)2 – 10x2y2 + 6x2y2
= (x2 + 5y2)2 – 4x2y2
= (x2 + 5y2)2 – (2xy)2
= (x2 + 5y2 + 2xy)(x2 + 5y2 – 2xy)
= (x2 + 2xy + 5y2) (x2 – 2xy + 5y2)
Example 2: Resolve into factors: m4 – 6m2n2 + n4.
Solution: Here, m4 – 6m2n2 + n4
= m4 + n4 – 6m2n2
= (m2)2 + (n2)2 – 6m2n2
= (m2 – n2)2 + 2m2n2 – 6m2n2 [a2 + b2 = (a – b)2 + 2ab]
= (m2 – n2)2 – 4m2n2
= (m2 – n2)2 – (2mn)2
= (m2 – n2 + 2mn) (m2 – n2 – 2mn)
= (m2 + 2mn – n2) (m2 – 2mn – n2)

108 | Mathematics – 9 Algebra


Example 3: Factorize: 4x4 + y4.
Solution: Here, 4x4 + y4
= (2x2)2 + (y2)2
= (2x2 + y2)2 – 2.2x2.y2
= (2x2 + y2)2 – 4x2y2
= (2x2 + y2)2 – (2xy)2
= (2x2 + y2 + 2xy)(2x2 + y2 – 2xy)
= (2x2 + 2xy + y2)(2x2 – 2xy + y2)
Example 4: Factorize: a4 – 6a2 – 7 – 8x – x2.
Solution: Here, a4 – 6a2 – 7 – 8x – x2
= (a2)2 – 2.a2.3 + 32 – 16 – 8x – x2
= (a2 – 3)2 – (16 + 8x + x2)
= (a2 – 3)2 – (x2 + 2.x.4 + 42)
= (a2 – 3)2 – (x + 4)2
= [(a2 – 3) + (x + 4)] [(a2 – 3) – (x + 4)]
= (a2 – 3 + x + 4)(a2 – 3 – x – 4)
= (a2 + x + 1)(a2 – x – 7)
Example 5: Factorize: x2 – y2 – 6x + 14y –40
Solution: Here, x2 – y2 – 6x + 14y – 40
= (x)2 – 2.x.3 + (3)2 – y2 + 14y – 40 – 9
= (x – 3)2 – (y2 – 14y + 49)
= (x – 3)2 – (y – 7)2
= (x – 3 + y – 7)(x – 3 – y + 7)
= (x + y – 10)(x – y + 4)

Exercise 4.1.2
1. Factorize the following.
(a) x 4 +4y4 (b) x4 + 4 (c) a4 + 64b4
1
(d) 324p4 + q4 (e) (7a)4 + 4b4 (f) x4 + 4x4

1 1
(g) 4x4 + 81y4 (h) 4x4 + 625 y4

2. Factorize following.
(a) x4 + x2y2 + y4 (b) x4 + x2 + 1 (c) x4 – 7x2 + 1
(d) 4p4 + 3p2 + 1 (e) m4 – 3m2 + 1 (f) 9a4 – 4a2 + 4
4 2 2 4 4 2 2 4
(g) m – 14m n + n (h) 4a + 35a b + 121 b (i) x4 – 17x2 + 256

Algebra Mathematics – 9 |109


(j) x4 – 11x2 + 1 (k) x4 – 6x2 + 1 (l) a4 – 14a2 b2 + b4
2 2
x y p4 p2
(m) y 2 + 1 + x2 (n) 25x4 – 9x2y2 + 16y4 (o) q 4–7 2+1
q
x4 4y4 1 x4 11x2
(p) + –7 (q) x4 – 7 + (r) – +1
4y4 x4 x4 y4 y2
1 1 x4 y4
(s) 4x2 – 27 + 4x2 (t) x2 – 3 + x2 (u) y 4+1+ 4
x
3. Factorize the following:
(a) x2 – 10x + 24 + 6y – 9y2 (b) p2 – 6p – 40 + 14q – q2
2 2
(c) a – 6a + 8 – b + 2b (d) x4 + 9 – 7x2 + 2xy – y2
(e) a2 – 12a – 28 + 16b – b2 (f) x2 + 3y2 – z2 + 2yz – 4xy
(g) x2 – 90xy + 2000y2 – 550yz – 3025z2
(h) a2 – 2ab + b2 – c2 – 2cd – d2

4.2 Indices
Introduction
Any algebraic term has three parts. The three parts are coefficient, base and power. Let’s take an
algebraic term 3a2. In this term 3a2, 3 is the coefficient of a2, a is the base and 2 is the power of a. The
power of the base is called the index. It is also called the exponent. So, in a term – 4x5. 5 is called the
index of x. Indices is the plural form of index.

Laws of Indices
There are certain rules which are used to solve the problems of indices. These rules are called the laws
of indices.
(i) Product Law of Indices
Let xm and xn are the two algebraic terms, where m and n are assumed to be positive integers,
then xm × xn = xm + n.
Proof:
We know that,
x2 = x × x
x3 = x × x × x
x4 = x × x × x × x
xm = x × x × x × x × x ... ... ... ... to ‘m’ factors
xn = x × x × x × x × x ... ... ... to ‘n’ factors
 xm × xn = (x × x × x × x × x ... ... ... to ‘m’ factors) × (x × x × x × x × x ... ... ... to ‘n’ factors)
= x × x × x × x × x ... ... ... to (m + n) factors
= xm + n

110 | Mathematics – 9 Algebra


Thus, xm × xn = xm + n, where ‘m’ and ‘n’ are positive integers.
Similarly, If a, b, c, d are the positive integers, then
xa × xb × xc × xd = xa + b × xc × xd
= xa+b+c × xd
= xa + b + c + d
So, in general,
xa × xb × xc × xd × xe × ... ... ... = xa + b + c + d + e + ... ... ...
Thus, the index of the product of the two or more algebraic terms with the same base is the sum
of the indices of each term.
For examples:
32 × 3 5 = 3 2 + 5 = 37
(4x)2 × (4x)3 × (4x)5 = (4x)2 + 3 + 5 = (4x)10
(ii) Quotient Law of Indices
Let xm and xn are the two algebraic terms with the same base, where m and n are assumed to be
positive integers, then
xm ÷ xn = xm – n, where m > n.
Proof:
We have,
xm – n × xn = xm – n + n [by using the product law of indices]
m–n n m
or, x ×x =x
xm
or, xm – n = xn

xm
or, xn = xm–n

or, xm ÷ xn = xm – n, where m > n.


But, if m < n, then
1
xm ÷ xn = x n – m = xm–n

Thus, the index of the quotient of the two algebraic terms with the same base is the difference
of the indices of each term.
For examples:
74 ÷ 73 = 74 – 3 = 7
(3a)5 ÷ (3a)2 = (3a)5 – 2 = (3a)3
1 1
(4m)3 ÷ (4m)7 = (4m)7 – 3 = (4m)4

Algebra Mathematics – 9 |111


(iii) Power Law of Indices
n
If xm is an algebraic term with the base x and index m, then (xm) = xmn, where m and n both are
the positive integers and n is the index of xm.
Proof:
We know,
n
(xm) = xm × xm × xm × ... ... ... to n factors.
= xm + m + m + ... ... ... n terms
= xmn
n
(xm) = xmn
Thus, the power of an index of any algebraic term is the product to two indices.
q
It can also be extended as [{(x )n}p]
m
= xmnpq,

m p
x n  = (x )p = x np
m p mp

 y  (yn) y

For examples:
(32)3 = 32 × 3 = 36
(4a)3 × 5 = (4a)15
3 5
 22  = (22)5 = 22 × 5 = 210
3 5 3×5 15

 3  (3 ) 3 3

(iv) Law of Negative Index


1
Let x–m is an algebraic term, where m is a negative integer, then x–m = xm

Proof:
We have, x–m = xm – 2m
= xm ÷ x2m [Quotient law of index]
xm xm
= x2m = xm + m

xm 1
= xm × xm = xm [Product law of indices]

1
 x–m = xm

1
Similarly, xm =
x–m

112 | Mathematics – 9 Algebra


Thus, an algebraic term with negative index is the reciprocal of the term with the positive index.
1 1 1
For example: x–3 = x3 , 4–3x = 43x and 3–2x = 32x

(v) Law of Zero Index


Let ao be an algebraic term with the base a and the index 0, then ao = 1.
Proof:
x
We have, ao = a2 – 2 OR x
=1
= a2 × a–2 and
x x1 1–1 0
1
= a2 × a2 [Law of negative index.] x = x1 = x = x
x0 = 1
=1
 ao = 1
Thus, any algebraic term having the index 0 is always equal to 1. But 0o  1.
For examples, 2o = 1, (100)o = 1, (3x)o = 1 and so on.
(vi) Root Law of Index
m
m
Let x n be an algebraic term with base x and index n , where m and n both are positive
m
n
integers, then, x n = xm
Proof:
We have,

(x ) = x
m n m m m
n n
× x n × x n × ... ... ... to n factors
m m m
+ n + n
= xn ... ... ... n terms
m
×n
= xn = xm
m
n
 xn = xm
m
Thus, x n is equal to nth root of xm.
1
2
For examples, x2 = x , but it is written as only x .
1 3 1
3 4
a3 = a, x4 = (x3)4 = x3

Algebra Mathematics – 9 |113


Now, we may summarize the above laws of indices as the following.
(i) xm × xn = x m + n (ii) (xy)m = xmym
m m
1 (iv)  x  = xm
(iii) xm÷xn = xm–n if m > n and n–m if n > m.
x y y
q (vi) 1 1
(xm)n = xmn , [{(xm)n} ] = xmnpq
p
(v) x–m = xm , x–m = xm
p
(vii) x q = q xP (viii) xo = 1

Worked Out Examples


Example 1: Find the products in the exponential form:
2 3 4 5 x x x 3 –4 7
(a) 3 × 3 × 3 × 3 × 3 (b) 2 × 4 × 8 (c) (5x) × (5x) × (5x)
Solution: Here,
(a) 32 × 33 × 34 × 35 × 3 = 32 + 3 + 4 + 5 + 1 = 315
x x
(b) 2x × 4x × 8x = 2x ×(22) ×(23) = 2x × 22x × 23x = 2x + 2x + 3x = 26x
(c) (5x)3 × (5x)–4 × (5x)7 = (5x)3 + (–4) + 7 = (5x)10 – 4 = (5x)6
Example 2: Find the quotient in their exponential form:
(a) 74 ÷ 72 (b) (25)2 ÷ (125)2
Solution: Here,
(a) 7 4 ÷ 72 = 74 – 2 = 72
1
(b) (25)2 ÷ (125)2 = (52)2 ÷ (53)2= 54 ÷ 56 = 54 – 6 = 5–2 = 52

5
Example 3: Express 2xy 243x–10y–15 with the positive indices without radical sign.
5
Solution: Here, 2xy 243x–10y–15
1 1 1
= 2xy(243)5 (x–10)5 (y–15)5
1
= 2xy(35)5 x–2 y–3
1 1
= 2xy × 3 × x2 × y3

6xy 6
= x2 y 3 = x y 2

Example 4: Evaluate:

2 3 3 4 1
(a) (125)3 (b) 729–1 (c) (144)4 ×
144

114 | Mathematics – 9 Algebra


Solution: Here,
2 2
(a) (125)3 = (53)3 = 52 = 25
3 1
{(729)– }
1 1 1
3 3 – 1
(b) 729–1 = (729–1)2 = 2
= (36) 6 = 3–1 = 3

3 4 3 1
1  1 4
(c) (144)4 × 144 = (144) ×  144
4

3 1 3 1
)4 = (144)4 × (144)–4
–1
= (144)4 × ((144)
3 1 2
–4
= (144)4 = (144)4
1
= [(12)2]2 = 12

Example 5: Simplify:

146 × 155 –2 7 3 9 –5
(a) (b) a3b 3 c6 ÷ a2b–1c 4
356 × 65
Solution: Here,
–2 7 3 9 –5
146 × 155
(a) (b) a3 b 3 c6 ÷ a2 b–1 c 4
356 × 65
–2 7 1 –5 1
=
(2 × 7)6 × (3 × 5)5 = (a b c ) ÷ (a
3 3 6 2
9
2 b–1 c 4 3)
(5 × 7)6 × (2 × 3)5 3 –1 7 3 –1 –5
26 × 76 × 35 × 55 2
= a b c ÷a b c 3 12 2 3 12
= 56 × 76 × 25 × 35
26 × 2–5 × 76 × 7–6 × 35 × 3–5 = a2
3 3
–2
b3
–1
– (–13) c127 – (–512)
= 56 × 5–5 –1 1 7 5
+3 + 12
2 × 7 × 35–5
6–5 6–6 = aob 3 c12
= 56–5 12

2 × 7 × 3o
1 o = ao boc12
= 51 = 1×1×c
2×1×1 2 = c
= 5 =5

xm m + n xnn + p  xp p – m
Example 6: Simplify:  n  × p × –m .
x  x  x 
xm m + n xnn + p  xp p – m
Solution: Here,  xn  × xp × x–m
     
m+n n+p
= (x m–n
) × (x )n–p
× (x p+m
)p–m
m2 – n2 n 2 – p2 p2 – m2
=x ×x ×x
m2 – n2 + n2 – p2 + p2 – m2
=x
= xo
=1

Algebra Mathematics – 9 |115


1 2
Example 7: If x = 23 + 23 , find the value of x3 – 6x – 6.
Solution: Here, x3 – 6x – 6
3
= (2 + 2 ) – 6 (2 + 2 ) – 6
1 2 1 2
3 3 3 3

= (2 ) + (2 ) + 3. 2 .2 (2 + 2 ) – 6(2 + 2 ) – 6
1 3 2 3 1 2 1 2 1 2
3 3 3 3 2 3 2 3

[ (a + b)3 = a3 + b3 + 3ab(a + b)]

( ) – 6 (2 + 2 ) – 6
= 2 + 22+ 3.(2) 3 + 3 22 + 23
1 2 1 2 1
2
2
3

= 2 + 4 + 3.(2) (2 + 2 ) – 6 (2 + 2 ) – 6
1+2
3
1
2
2
3
1
2
2
3

= 6 + 3.2(2 + 2 ) – 6(2 + 2 ) – 6
1 2 1 2
2 3 2 3

= 6 – 6 + 6 (2 + 2 ) – 6(2 + 2 )
1 2 1 2
2 3 2 3

=0+0
=0

Exercise 4.2
1. Find the products in their exponential forms.
(a) 43 × 4–2 × 45 (b) 3a × 9a × 27a × 81–a
(c) (7x)2 × (7x)–3 × (7x)5 (d) (a + b)3 × (a + b)9 × (a + b)–7
2. Find the quotients in their exponential forms.
(a) 137 ÷ 134 (b) 83 ÷ 25
(c) 163 ÷ 82 (d) (4a2)3 ÷ (8a)5
(e) (3p + q)7 ÷ (3p + q)2 (f) (4a3)2 ÷ (2a)7

3. Express the following with the positive indices without radical sign.
3 3 –6 3 5
(a) xy (b) 27a6 b–3 (c) x x3 b–12
4
(d) a2 a–2 (e) (81x–4 y4)3 (f) a3 a–4
4. Evaluate:
4

(a)
3
8 2
(b)
2
(64)3 (c) (4 ) 3
–4
–3

2
3 –3
(d)

(16) 4 (e) 64
27

116 | Mathematics – 9 Algebra


5. Evaluate:
3 1 3
(a) (64)–1 (b) (c) 8x–3
4 –1
81
–3
(d) 1–2 ÷ 1–3 (e)
1
3 3 64
6. Evaluate:
o 2
(a) ( 5) 6
(b)
3
729 –1
(c)  a –3
64
5 3 4
9 32 3 1
(d)
25
×
243
(e) 64 (f) (36)4 × 36
7. Simplify:
2
3 3x x –3
(a) a–2b × ab3 (b) m3 × mx – 1 (c) (8x3 ÷ 27y–3)
–1 2 –3 –5
(d)
3
8x–3y6 ÷
3
27z9 (e)
3 6 –2 4
ab c ÷
4 –4 4 2
a bc (f) x4 y–27 ÷ x –2y 3 –2
x y   x y 
8. Simplify:
4 1 3
53 × 5–3 4 83 272 32–1 5–m × 252m–2
(a)   (b)
 1
 93 × 162 ÷ 81–1 (c) 53m–2 × 15–1
 252 
124 × 154 (50)4 × (30)–4 × (20)–5 (35)4 × (24)5
(d) 183 × 454 (e) (40)3 × (60)–3 × (80)–2 (f) (28)5 × (30)4
9. Simplify:
(a) ax–y × ay–z × az–x (b) (xa+b)a–b × (xb+c)b–c × (xa+c)a–c
q–r r–p p–q x5b – 6c
(c) (ap) × (aq) × (ar) (d) xb + 2c
. x3b – 8c
x2m + 3n × x3m + 6n xm + n × xm – n × xp–3m
(e) xm + 2n × x4m – 4n (f) xp–m
10. Simplify:
2 2 2
1 1 (xa+ b) × (xb+ c) × (xc+ a)
(a) 1 – xa – b + 1 – xb – a (b) 4
(xa xb xc)
x y z
ayz . azx . axy 2x + 3 – 2x + 2
(c) (d)
a  a  a  2x + 2
xa – b + c. xb – c + a. xc – a + b
(e) (am – n)p × (an – p)m × (ap – m)n (f) xa + b + c

Algebra Mathematics – 9 |117


11. Simplify:
b a c b a c b b+c xc c + a xaa + b
(a) xc × xa × xb (b) x c  × xa × xb
x  x  x  x     
m m+n an n – p  ap p – m b b+c–a xc c + a – b xaa + b – c
(c) a n  ×  –p × –m (d) x c  × a × b
a  a  a  x  x  x 
x x2 + xy + y2 ay y
2
+ yz + z2 az z
2
+ zx + x2
(e) ay × az × ax
a     
12. Simplify:
1 1 1
b c a n+p m–n xp + m n – p xm + np – m
(a) xcbc × xaca × xbab (b)  xp – m ×  p – n × n–p
 
x    
x x x  x  x 
x ay y az z ax ab xa bc xb ca xc
(c) a z ×
a x ×
ay (d) xb × xc × xa
m n–m
x + 1m x – 1n a2 – 12 a – 1
 y  y  b   b
(e) (f) n m–n
y + 1m y – 1n b2 – 12 b + 1
 x  x  a  a
1 1 1
(g) 1 + ax–y + az–y + 1 + ay–z + ax–z + 1 + az–x + ay–x
13. (a) If a = 3, b = –1 and c = 0, find the value of (ac)b + (ba)c .
xm m – n xnn – p  xp p – m
(b) If m2 + n2 + p2 = mn + np + pm, prove that:  xn  × xp × xm =1
     
a b c
x a2 – ab + b2 x b2 – bc + c2 x c2 – ca + a2
(c) If a3 + b3 + c3 = 1, show that: x–b × x–c × x–a = x2
     
1 1
– 1
(d) If x = a3 – a 3 , prove that: x3 + 3x = a – a .
1 1

(e) If x = 33 + 3 3, prove that: 3x3 – 9x – 10 = 0
2 2

(f) If x2 + 2 = 33 + 3 3 , prove that: 3x3 + 9x = 8.

4.3 Exponential Equation


Let’s take an equation 2x = 8. In this equation, the variable x is an unknown which is the power of the
base 2. So x is also called the index or exponent of 2. Such an equation in which the variable is
contained in exponent of the base is called the exponential equation. The more examples of the
exponential equations are 5x + 1 + 5x = 6, 2. 3x + 1 = 18, 2x + 4 × 3x + 4 = 144 etc.

118 | Mathematics – 9 Algebra


To solve an exponential equation, we should follow the following steps.
1. We should simplify the equation and reduce the single term on the both sides of the equation.
2. We should simplify the equation by using the laws of indices.
3. We should make the same base on the both sides of the equation.
4. We should compare the index or base and simplify for the required solution.
5. In solving such equations, we should be familiar with the following axioms:
If ax = ab, then x = b
If ax = 1, then ax = ao x = 0

Worked Out Examples


Example 1: Solve:
(a) 2x = 16 (b) 27x = 3x + 4
Solution: Here,
(a) 2x = 16
or, 2x = 24
x=4
(b) 27x = 3x + 4
or, (33)x = 3x + 4
or, 33x = 3x + 4
 3x = x + 4
or, 3x – x = 4
or, 2x = 4
4
or, x=2

 x=2
Example 2: Solve: 3 × 81x = 9x + 4
Solution: Here, 3 × 81x = 9x + 4
or, 3 × (34)x = (32)x + 4
or, 3 × 34x = 32x + 8
or, 31 + 4x = 32x + 8
 1 + 4x = 2x + 8
or, 4x – 2x = 8 – 1
or, 2x = 7
7 1
 x = 2 = 32

Algebra Mathematics – 9 |119


Example 3: Solve: 2x + 2 + 2x = 5
Solution: Here, 2x + 2 + 2x = 5
or, 2x × 22 + 2x = 5
or, 2x (22 + 1) = 5
or, 2x (4 + 1) = 5
or, 2x × 5 = 5
5
or, 2x = 5

or, 2x = 1
or, 2x = 20
 x=0
x+3 x+2
Example 4: Solve: 3 ×2 = 18
x+3 x+2
Solution: Here, 3 ×2 = 18
or, 3x × 33 × 2x × 22 = 18
or, 3x × 2x × 27 × 4 = 18
18
or, (3 × 2)x = 27 × 4

1
or, 6x = 3 × 2

1
or, 6x = 6

or, 6x = 6–1
 x = –1
Example 5: If a = by and b = a2, prove that x – 2y = 0.
x

Solution: Here, ax = by
1 1
or, (a x)x = (b y)x
y And, b = a2
a=b x
... (i) or, a2 = b
From equations (i) and (ii), we get 1 1

y 1 or, (a2)2 = (b)2


x 2
b =b 1

y 1 a = b2 ... (ii)
 x =2

or, x = 2y
 x – 2y = 0 proved.

120 | Mathematics – 9 Algebra


Exercise 4.3
1. Solve:
1
(a) 3x = 27 (b) 2x + 2 = 8 (c) 4x = 128
x
(d) ay – 1 = 1 (e) 52x + 3 = 1 (f) 272 = 36

2. Solve:
(a) 92x = 33 – 2 x (b) 8x = 27 – 4x (c) 64x = 4 x + 6
x+5 x +1 5x + 3 2x – 9
2x
(d) 52x + 1 = (25) (e) 3 2 =9 2 (f) 11 4 = 121 2

3. Solve:
(a) 3 × 27x = 9x – 1 (b) 2 × 8x = 2x + 5 (c) 2 × 83 = 2 x – 4
x–1
(d) 5 × (125) = 52x +3 (e) 4 × 64x – 1 = 1 (f) 36 × 62x–1 = 1
4. Solve:
1
(a) 2x + 1 + 2x = 3 (b) 3x + 1 + 3x = 108 (c) 32x – 3 + 32x = 93

1 2x + 3
(d) 2x + 1 – 2 x – 8 = 0 (e) 3x + 2 + 3x + 1 = 13 (f) 2x + 2 + 2 = 1

5. Solve:
(a) 4x + 2 × 5x + 3 = 100 (b) 3x – 2 × 2x + 1 = 8 (c) 2x + 3 × 3x + 4 = 18
7
(d) 72x + 1 × 52x – 1 = 5 (e) 22x – 3 × 52x – 1 = 25 (f) 23x – 5×bx – 2 = 2x – 2×b1 – x

(g) 33x –4 × a5–2x = 3x+2 × a11 –4x = 0 (h) m5–2x  n2x–1 = m11–5xn5x –7

6. Solve.
1 1 1 1
(a) 2x + 2x = 22 (b) 4x + 4x = 44 (c) 22x+3 + 1= 3×2x+2

1
(d) 5 × 4x+1 – 16x= 64 (e) 3x + 3–x = 99

7. (a) If xm = yn and y = x2, show that m – 2n = 0.


(b) If a = bc, b = ca and c = ab, prove that abc = 1.
1 1
(c) If am = b5 and ab = 1, prove that m + 5 = 0.
(d) If ax ay = (ax)y, show that x(y – 2) + y(x – 2) = 0.
(e) If a = 4x, b = 4y and aybx = 16, show that xy = 1.

Algebra Mathematics – 9 |121


4.4 Ratio and Proportion
4.4.1 Ratio
Let’s consider the ages of two boys. If the age of one boy is 10 years and the age of another boy is 14
10 5
years, the age of the first boy is 14 or 7 times the age of the second boy. In fact we say that the ages
of two boys are in the ratio 10 to 14 or 5 to 7.
The ratio of two quantities is a comparison of their magnitudes where the quantities are of same kind.
x
If x and y are two quantities of the same kind, we say that their ratio is y or x : y (read as x is to y). (It
means the ratio is a fraction of two quantities of the same kind by comparing that two quantities. So,
we can write the ratio of two quantities of the same kind conveniently in the form of a fraction.
It should be noted that the quantities must be of the same kind. So, we cannot compare between Rs. 6
and 10cm. Because Rs. 6 and 10cm. are the different quantities. But we can compare 40cm. with 3m.
after converting m. into cm. and their ratio is 40cm.: 300cm. or 2:15. From this example, we should
also be careful to express two quantities in the same unit before putting their ratio in the form of a
fraction. We should also remember that every ratio is simply a number. It has no unit. Thus, the ratio
15 3
of 15cm. to 20cm. is 20 or 4 only.

A ratio is said to be how many times a quantity is greater or smaller than another quantity of the same
kind. The ratio of a to b is usually written as a:b. The quantities a and b are called the terms of the
ratio. The first term of the ratio is called the antecedent and the second term the consequent. So, in the
ratio 4:7, 4 is the antecedent and 7 the consequent.
Compounded Ratio
a c ac
Let’s suppose a:b and c:d are two ratios. Then a:b × c:d = b × d = bd = ac : bd is called the
compounded ratio of these two given ratios.
Similarly, let’s suppose a:b, c:d and e:f are three ratios. Then, a:b × c:d × e:f i.e. ace: bdf is the
compounded ratio of these given ratios. Thus, the product of two or more ratio is called the
compounded ratio. For example, 4:5 and 7:9 are two ratios. The compounded ratio of 4:5 and
4 7 28
7:9 = 4:5 × 7:9 = 5 × 9 = 45 = 28:45.

Duplicate and Sub–Duplicate Ratios


Let’s suppose a:b is a ratio. The square of the ratio a:b is (a:b)2 i.e. a2:b2 is the compounded ratio of
a:b and a:b. Then a2:b2 is called the duplicate ratio of a:b and a:b is called the sub–duplicate ratio of
a2:b2. For example, let’s take a ratio 2:3.
The duplicate ratio of 2:3 = 22:32 = 4:9

The sub–duplicate ratio of 4:9 = 4 : 9 = 2:3

122 | Mathematics – 9 Algebra


Triplicate and Sub–Triplicate Ratios
Let’s suppose a:b is a ratio. The cube of the ratio a:b is (a:b)3 i.e. a3:b3. The ratio a3:b3 is the
compounded ratio of a:b, a:b and a:b. Thus, the ratio a3:b3 is called the triplicate ratio of a:b. The cube
1 1 1
3
root of a3:b3 = a3:b3 = (a3:b3)3 = (a3)3 : (b3)3 = a:b
Then, a: b is called the sub–triplicate ratio of a3:b3.
For example, let’s take a ratio 4:5. Then, the triplicate ratio of 4:5 = 43:53 = 64:125.

3 3 3
The sub–triplicate ratio of 64:125 = 64:125 = 64 : 125 = 4:5
Inverse Ratio
Let’s take a ratio a:b, where a is the antecedent and b is the consequent. When the antecedent and
consequent of the ratio a:b are interchanged, the ratio becomes b:a. Then the ratio b:a is called the
inverse ratio of a:b. For example, 2:3 be a ratio. Then the inverse of ratio 2:3 is 3:2.

Worked Out Examples


Example 1: Find the ratio of the following:
(a) 6 min to 3 hours (b) 3 kg to 750 gm
Solution: Here,
(a) 6 min to 3 hours
The ratio of 6min to 3 hours = 6 min : 3hours
= 6 min : 3×60 min [... 1 hour = 60 min]
= 1:30
(b) 3 kg to 750gm.
The ratio of 3kg to 750gm = 3kg : 750gm
= 3 × 1000gm : 750gm [...1kg = 1000gm.]
= 3 × 4 : 3 = 4:1
Example 2: If x:y = 3:4 and y:z = 5:7, find x:z and x:y:z.
Solution: Here, x:y = 3:4 and y:z = 5:7
Now, x:y × y:z = 3:4 × 5:7
x y 3 5
or, y ×z=4 ×7
x 15
or, z = 28
 x:z = 15:28.
Again, in the ratio x:y = 3:4, y is 4 and in the ratio y:z = 5:7, y is 5. The L.C.M of 4
and 5 is 20.

Algebra Mathematics – 9 |123


Now, x:y = 3:4 = 3×5:4×5 = 15:20
y:z = 5:7 = 5×4:7:×4 = 20:28
 x:y: z = 15:20:28
Example 3: If x:y = 3:4, find the value of 3x+4y:5x – 2y.
x 3
Solution: Here, x:y = 3:4 i.e. y = 4

3x + 4y
3x + 4y y
Now, 5x – 2y = 5x – 2y [Dividing the numerator and denominator by y.]
y
3x 4y 3 9 9 + 16
y + y 3×4+4 4+4 4 25
= 5x 2y = 3 = 15 = 15 – 8 = 7
y – y 5×4–2 4 –2 4

Example 4: If 5x + 2y:7x + 3y = 9:13, find the value of x:y.


Solution: Here,
5x + 2y:7x + 3y = 9:13
5x + 2y 9
or, =
7x + 3y 13
or, 13(5x + 2y) = 9(7x + 3y) [by the cross – multiplication ]
or, 65x + 26y = 63x + 27y
or, 65 – 63x = 27y – 26y
or, 2x = y
x 1
or, y=2
 x:y = 1:2
Example 5: The sum of two numbers which are in the ratio of 3:5 is 56, find the
numbers.
Solution: Here,
The ratio of two numbers is 3:5.
Let x be a common factor of two numbers.
Then, the two numbers are 3x and 5x.
According to question,
3x + 5x = 56
or, 8x = 56
56
or, x = 8 = 7
Now, 3x = 3 × 7 = 21
5x = 5 × 7 = 35
Hence, the two numbers are 21 and 35.

124 | Mathematics – 9 Algebra


Example 6: Divide Rs. 252 into two parts so that their ratio is 5:9.
Solution: Here,
The ratio of two parts is 5:9
Let x be a common factor of two parts. Then, the two parts are 5x and 9x.
Now, 5x + 9x = Rs. 252
or, 14x = Rs. 252
252
or, x = Rs. 14

x = Rs. 18
Therefore, the first part = Rs. 5x = Rs. 5 × 18 = Rs. 90
The second part = Rs. 9x = Rs. 9 × 18 = Rs. 162
Hence, the two parts of Rs. 252 which are in the ratio 5:9 are Rs. 90 and Rs. 162.
Example 7: What number must be subtracted from each term of a ratio 27:35 to
make it equal to 7:11?
Solution: Let the required number be x.
27 – x 7
Then, 35 – x = 11

or, 11(27 – x) = 7(35 – x)


or, 297 – 11x = 245 – 7x
or, –11x + 7x = 245 – 297
or, – 4x = – 52
52
or, x= 4

or, x = 13
Hence, the required number is 13.
Example 8: The ratio of two numbers is 5:7. If 15 is added to each number, the ratio
becomes 3:4. Find the numbers.
Solution: Here, The ratio of two numbers is 5:7.
Let x be a common factor of two numbers.
Then, two numbers are 5x and 7x.
According to question,
5x +15 3
7x + 15 = 4
or, 4(5x + 15) = 3(7x + 15)
or, 20x + 60 = 21x + 45
or, 20x – 21x = 45 – 60
or, – x = –15
 x = 15
The first number = 5x = 5 × 15 = 75

Algebra Mathematics – 9 |125


The second number = 7x = 7 × 15 = 105
Hence, the required two numbers are 75 and 105.
Example 9: The ratio of the present ages of a daughter and her mother is 5:12. If the
ratio of their ages after 6 years will be 1:2, find their present ages.
Solution:: Here, The ratio of the present ages of a daughter and her mother is 5:12.
Let x be a common factor of their ages.
Then, the present ages of the daughter and her mother are 5x years and 12x years
respectively.
After 6 years, their ages will be (5x + 6) years and (12x + 6) years respectively.
According to question,
(5x + 6) : (12x + 6) = 1:2
5x + 6 1
or, 12x + 6 = 2
or, 2(5x + 6) = 1(12x + 6)
or, 10x + 12 = 12x + 6
or, 10x – 12x = 6 – 12
or, – 2x = – 6
6
or, x=2

 x=3
 The present age of the daughter = 5x = 5 × 3 = 15 years.
The present age of the mother = 12x = 12 × 3 = 36 years.

Exercise 4.4.1
1. Find the ratios of the following:
(a) 9min to 8hours (b) 70cm to 1.4m (c) Rs. 4.50 to 75 paisa
(d) 7km to 500m. (e) 2.5kg to 950gm. (f) 12days to 3weeks.
2. Find the compounded ratio of the following ratios: –
(a) 4:5 and 10:18 (b) 7:9 and 27:21 (c) 2:3, 4:7 and 1:5
3. Find the duplicate ratio of the following ratios: –
(a) 2:5 (b) 2x:7y (c) 12:7 (d) 14a:9b
4. Find the sub – duplicate ratio of the following ratios:
(a) 9:16 (b) 49:36 (c) 169a2:144b2 (d) 4:9
5. Find the triplicate ratio of the following ratios:
(a) 4:7 (b) 2a:b (c) 5:2 (d) 6x:11y
6. Find the sub – triplicate ratio of the following ratios:
(a) 8:27 (b) 125x3:1 (c) 216a3:64b3 (d) 729x6 : 512y9

126 | Mathematics – 9 Algebra


7. Find the inverse ratio of the following ratios:
(a) 12:19 (b) 16x2:21y2 (c) (2x + 3):(9y – 5) (d) 18a:7b
8. (a) If x:y = 4:9 and y:z = 12:11, find x:z and x:y:z.
(b) If a:b = 2:5 and b:c = 3:4, find a:c and a:b:c.
(c) If p:q = 8:11 and q:r = 33:16, find p:r and p:q:r.
9. (a) If a:b = 3:4, find the value of 4a + 3b:12a – b.
(b) If x:y = 6:7, find the value of 4x – y:5x + y.
(c) If p:q = 2:3, find the value of 5p – 2q:2p + q.
2b
(d) If a:b = 2:3, find the value of a – 3

3y
(e) If x:y = 3:4, what will be the value of x – 4 ?

10. (a) If 7a – 11b = 0, what will be the ratio of a:b?


(b) If 6a – 8b = 0, find the value of a:b.
(c) If 3m – 5n:3m + 5n = 1:4, find the value of m:n.
(d) If 5a + b:5a – b = 3:1, find the value of a:b
(e) If 3x – 5y:3x + 5y = 2:5, find the value of x:y.
11. Find the value of x:y in the following each cases.
(a) 2x – 3y:2x + 3y = – 1:3 (b) 2x – 3y = 2y – x
(c) 10x – 8y = 0 (d) 3x – y:2x + y = 1:2
(e) 7x – 4y:3x + y = 5:13 (f) x + 3y:3x + 2y = 5:7

(g) 5x + 3y:4x + 5y = 2:3 (h) x + y : 2x + y = 12: 17


2 3  3 2
12. (a) If a:b:c = 2:5:7, find the value of 2a:3b + c.
(b) If a:b:c = 3:4:7, find the value of (3a + 2b – c):(6a – 2c).
(c) If x:y:z = 4:5:6, find the value of (2x – 3y):(x + 2y – 3z).
13. (a) The sum of two numbers which are in the ratio 5:9 is 98. Find the numbers.
(b) Two numbers are in the ratio of 5:7 and their sum is 168. Find the numbers.
(c) The difference of two numbers whose ratio is 3:5 is 50. Find the numbers.
(d) Two numbers are in the ratio 4:7 and their difference is 57. Find the numbers.
(e) If the angles of a triangle are in the ratio 2:3:4, find the angles of the triangle.
14. (a) Divide Rs. 68 into two parts so that their ratio is 8:9.
(b) Divide 72 into two parts such that their ratio is 5:7.
(c) Divide Rs. 21 into two parts such that their ratio is 3:4.
(d) When Rs. 105 is divided into two parts in the ratio of 2:5, how much money will be in
the first part?
(e) 99 students are divided in two rooms in the ratio of 5:6. How many students will there
be in the second room?
15. (a) What number must be subtracted from each term of a ratio 4:9 so that it may become
equal to 9:4?

Algebra Mathematics – 9 |127


(b) What number must be subtracted from each term of a ratio 5:7 so that it may become
equal to 6:5?
(c) What number must be added to each term of the ratio 4:7 to make it equal to 2:3?
(d) What number must be added to each term of the ratio 29:36 to make it equal to 3:4?
16. (a) The ratio of two numbers is 3:5. If 8 is added to each number, the ratio becomes 7:9.
Find the numbers.
(b) The ratio of two numbers is 2:3. If 17 is added to each number, the ratio becomes 3:4.
Find the numbers.
(c) The ratio of two numbers is 25:37. If 14 is subtracted from each, the ratio of the
remainder is 3:5. Find the numbers.
(d) Two numbers are in the ratio of 4:5. If 7 is subtracted from each, the ratio of the
remainder is 7:9. Find the numbers.
17. (a) The ratio of the present ages of two men is 3:4. If the ratio of their ages after 4 years
will be 7:9, find their present ages.
(b) The ratio of the present ages of two persons A and B is 3:4. If the ratio of their ages
before 7 years was 2:3, what are their present ages be?
(c) The ratio of the present ages of a son and his father is 2:7. After 5 years, the ratio of
their ages will be 3:8. Find their present ages.
18. (a) The ratio of milk and water in a container is 7:4 by volume. If it contains 55 litres of the
mixtures, find the quantities of milk and water in it.
(b) Two numbers are in the ratio of 5:9. If the antecedent is 30, find the consequent.
(c) If the ratio of two complementary angles is 2:7, find the angles.
(d) On a map with a scale of 1cm:1km, what actual distance on the ground is represented on
the map by 8cm?

4.4.2 Proportion
Let a, b, c and d be four quantities. The ratio of a and b is a:b and the ratio of c and d is c:d. If
a:b = c:d, then these two equal ratios are called the proportional and the four quantities a, b, c and d
are said to be in proportion. For example, 4, 5, 16 and 20 are four quantities. The ratio of 4 and 5 is
4:5 and the ratio of 16 and 20 is 16:20. The ratio 16:20 is equivalent to the ratio 4:5. Therefore,
4:5 = 16:20. So, these two ratios are proportional and the four quantities 4, 5, 16 and 20 are in
proportion.
If a:b and c:d are proportional, it can be written as a:b:: c:d and read as ‘a is to b as equal c is to d’. In
the equal ratios a:b::c:d the terms a and d are called extremes and the terms b and c are called means.
a c
The ratios a:b::c:d also can be written as b = d . Then, by the cross–multiplication, it can be written as
ad = bc. ad is the product of extremes and bc is the product of means. Thus, the product of extremes is
equal to the product of means in every proportion.

128 | Mathematics – 9 Algebra


Continued Proportion
Let’s consider three numbers 5,10 and 20. The ratio of 1st and 2nd numbers is 5:10 i.e. 1:2. The ratio of
2nd and 3rd numbers is 10:20 i.e. 1:2. So, 5:10 = 10:20, then we say that 5,10 and 20 are continued
proportion. 5:10 = 10:20 is called the continued proportional. If a:b = b:c, then ac = b2. i.e. b = ac .
Thus, three quantities are said to be in continued proportion if the mean proportional is equal to the
square root of the product of extremes.
Properties of Proportion
If a, b, c and d are in proportion, then the following properties of proportion exist.
(i) Invertendo: If a : b = c : d, then b : a = d : c
Proof: Here,
a:b=c:d
a c
or, b=d
a c
Now, 1  b = 1  d [... Taking receproval of them]

b d
or, 1×a =1×c

b d
or, a =c i.e. b : a = d : c

(ii) Alternendo: If a : b = c : d, then a : c = b : d.


Proof: Here, a : b = c : d
a c
or, b=d
a b c b b
or, b×c=d×c [Multiplying both sides by c ]

a b
or, c =d i.e. a : c = b : d

(iii) Componendo: If a : b = c:d, then (a + b) : b = (c + d) : d


Proof: Here, a : b = c : d
a c
or, b=d
a c
or, b +1=d+1 [Adding 1 on both sides]

a+b c+d
or, b = d
i.e. (a + b) : b = (c + d) : d

Algebra Mathematics – 9 |129


(iv) Dividendo: If a : b = c : d, then (a – b) : b = (c – d) : d.
Proof: Here, a : b = c : d
a c
or, b=d
a c
or, b –1=d –1 [ Subtracting 1 from both sides]

a–b c–d
or, =
b d
i.e. (a – b) : b = (c – d) : d
(v) Componendo and dividendo: If a : b = c : d, then (a + b) : (a – b) = (c + d) : (c – d).
Proof: Here,
a : b = c : d,
By componendo, we have
a+b c+d
b = d ... (i)

By alternendo in equation (i), we have


a+b b
c+d =d ... (ii)

Again, by dividendo in a:b = c:d, we have


a–b c–d
b = d ... (iii)

By alternendo in equation (iii), we have


a–b b
= ... (iv)
c–d d
From equation (ii) and equation (iv), we have
a+b a–b
c+d =c–d
By alternendo, we have
a+b c+d
a–b =c–d
(a + b) : (a – b) = (c + d) : (c – d)
(vi) Addendo: If a : b = c : d, then a : b = c : d = (a + c) : (b + d).
Proof: Here, a : b = c : d
a c
or, b =d

130 | Mathematics – 9 Algebra


By alternendo, we have
a b
=
c d
By componendo, we have
a+c b+d
c = d
Again, by alternendo, we have
a+c c
b+d =d
a c a+c
 b =d = b+d

i.e. a : b = c : d = (a + c) : (b + d)
Similarly,
a c e a + c + e + .....................
b = d = f = ................ = b + d + f + .....................

Worked Out Examples


Example 1: If 1, a, 5 and 15 are proportional, find the value of a.
Solution: Here,
1, a, 5 and 15 are proportional
1 5
a = 15
or, 5a = 15
15
or, a= 5

 a=3
Example 2: If 5, x and 20 are in continued proportion, find the value of x.
Solution: Here,
5, x and 20 are in continued proportion.
5 x
 x = 20
or, x2 = 100
or, x = 100
 x = 10

Algebra Mathematics – 9 |131


Example 3: What constant number should be added to each of the terms 3, 5, 7 and
10 so that they are in proportion?
Solution: Let the constant number to be added be x.
Then, by the question,
3+x 7+x
5 + x = 10 + x
or, (3 + x)(10 + x) = (7 + x) (5 + x)
or, 30 + 3x + 10x + x2 = 35 + 7x + 5x + x2
or, 13x + x2 – 12x – x2 = 35 – 30
 x=5
Hence, the required number to be added is 5.
x 2 6x + y
Example 4: If = , find the value of .
y 3 3x + 2y
Solution: Here,
x 2
y = 3
x y
or, = [ by altrnendo]
2 3
x y
or, 2 = 3 = k(suppose)
 x = 2k and y = 3k
6x + y 6 × 2k + 3k 12k + 3k 15k 5
Now, 3x + 2y = 3 × 2k + 2 × 3k = 6k + 6k = 12k = 4

a b a a–b
Example 5: If = prove that =
b c a+b a–c
a b
Solution: Here, b = c

b c
or, a=b by invertendo

a+b b+c
or, a = b by componendo

a b
a+b=b+c ... (i) by invertendo

a b
Again, a + b = b + c

a a+b
or, b=b+c by alternendo

a–b a+b–b–c
or, b = b+c by dividendo

a–b a–c
or, b =b+c

132 | Mathematics – 9 Algebra


a–b b
or, a–c=b+c ... (ii)

From (i) and (ii), we get


a a–b
 a + b = a – c Proved
2
a b a + b a2 + b2
Example 6: If = , prove that  = 2
b c b + c b + c2
a b
Solution: Here b = c

a+b b
 = by addendo
b+c c
a + b 2 b2
 b + c = c2 ... (i) [... Squaring on both sides]
 
a b
Again b = c

a2 b 2
 b 2 = c2
a2 + b2 b2 + c2
 = by componendo
b2 c2
a2 + b 2 b 2
 b2 + c2 = c2 ... (ii) by alternendo
2 2 2
 a + b = a 2 + b2 from (i) and (ii)
b + c b + c
Proved.
a b (a – b)2 (b – c)2
Example 7: If = , prove that =
b c a c
a b
Solution: Here, b = c

b c
or, a=b by invertendo

a –b b – c
or, a = b by dividendo

(a – b)2 a2 . .
or, (b – c)2 = b2 [ . squaring on both sides]
(a – b)2 a2 . . 2
or, (b – c)2 = a c [ . b = ac]
(a – b)2 a
or, (b – c)2 = c
(a – b)2 (b – c)2
 a = c Proved.

Algebra Mathematics – 9 |133


a b ab cd
Example 8: If = , prove that 2 =
b c a – b2 c2 – d2
a c
Solution: Here, b = d

a2 c2
or, b2 = d2
a2 – b2 c2 – d2
or, b2 = d 2 by dividendo
2 2
b d
or, a2 – b 2 = c2 – d 2 by invertendo

b2 a d2 c ... a = c
or, a – b b c – d2 × d
2× =
2 2
 b d
ab cd
 a2 – b2 = c2 – d2 Proved
2 2
ac a +c
Example 9: If a:b = c:d, prove that = .
bd b2 + d2
Solution: Here,
a:b = c:d
a c
Let b = d = k

a = bk and c = dk
ac bk.dk bdk2
Now, L.H.S = bd = bd = bd = k2

a2 + c 2
R.H.S =
b2 + d 2
(bk)2 + (dk)2
= b2 + d 2
b2k2 + d2k2 k2(b2 + d2) 2
= b 2 + d 2 = b2 + d 2 = k .
L.H.S = R.H.S proved.
a2 + c2 + e2 ce
Example 10: If a:b = c:d = e:f, prove that = .
b2 + d2 + f2 df
Solution: Here,
a:b = c:d = e:f
a c e
or, b = d = f = k(suppose)
 a = bk, c = dk and e = fk
a2 + c2 + e2
Now, L.H.S = b2 + d2 + f2

(bk)2 + (dk)2 + (fk)2 b2k2 + d2k2 + f2k2 k2(b2 + d2 + f2)


= b2 + d 2 + f 2 = b2 + d 2 + f 2 = b2 + d2 + f2 = k2.

134 | Mathematics – 9 Algebra


ce dk.fk dfk2
R.H.S = df = df = df = k2

 L.H.S = R.H.S proved

Example 11: If a, b, c are in continued proportion, prove that:


a3 + b3: b3 + c3 = a(a – b):c(b – c).
Solution: Here,
a, b, c are in continued proportion.
a b
 b = c = k (say)

 b = ck
a = bk = ck.k = ck2
a3 + b3 (ck2)3 + (ck)3 c3k6 + c3k3 c3k3(k3 + 1)
Now, L.H.S = b3 + c3 = (ck)3 + c3 = c3k3 + c3 = c3(k3 + 1) = k3

a(a – b) ck2(ck2 – ck) c2k3(k – 1)


R.H.S = c(b – c) = c(ck – c) = c2(k – 1) = k3

 L.H.S = R.H.S proved.


Example 12: If a, b & c are in continued proportion, show that (a2+b2) (b2+c2) = b2(a+c)2
a b
Solution: Since a, b and c are in continued proportion, b = c

 b2 = ac
L.H.S. = (a2 + b2) (b2 + c2)
= (a2 + ac)(ac + c2)
= a(a + c).c(a + c)
= ac(a + c)2
= b2(a + c)2 = RHS
Proved.
Example 13: If a, b, c and d are in continued proportion, show that (a+d) (b+c) –(a+c)
(b+d) = (b–c)2
a b c
Solution: Since a, b, c and d are in continued proportion, b = c = d

 b2 = ac, c2 = bd and ad = bc
L.H.S. = (a+d) (b+c) –(a+c) (b+d)
= (ab+bd+ac +cd)–(ab+bc+ad+cd)
= ab + bd +ac +cd –ab –bc –ad –cd
= c2 +b2 –bc –bc [... bc = ad]
= b2–2bc+c2
= (b–c)2 = R.H.S. Proved.

Algebra Mathematics – 9 |135


a3 – b3 – c3 a
Example 14: If a, b, c and d are in continued proportion, prove that = .
b3 – c3 – d3 d
Solution: Here,
a, b, c and d are in continued proportion,
a b c
 b = c = d = k (say)

 c = dk
b = ck = dk.k = dk2
a = bk = dk2.k = dk3
a3 – b3 – c3 (dk3)3 – (dk2)3 – (dk)3 d3k9 – d3k6 – d3k3
Now, L.H.S = b3 – c3 – d3 = (dk2)3 – (dk)3 – d3 = d3k6 – d3k3 – d3

d3k3(k6 – k3 – 1)
= d3(k6 – k3 – 1) = k3

a dk3
R.H.S = d = d = k3.

L.H.S = R.H.S proved.


x3 + y3 + z3 xyz
Example 15: If x:a = y:b = z:c, prove that = .
a3 + b3 + c3 abc
Solution: Here,
x y z
a = b = c = k (suppose)
 x = ak, y = bk and z = ck
x3 + y 3 + z3
Now, L.H.S = a3 + b3 + c3

(ak)3 + (bk)3 + (ck)3 a3k3 + b3k3 + c3k3 k3(a3 + b3 + c3)


= a3 + b3 + c3 = a3 + b 3 + c3 = a3 + b3 + c3 = k3

xyz ak.bk.ck abck3


R.H.S. = abc = abc = abc = k3

L.H.S = R.H.S proved


x y z
Example 16: If = = , prove that xy – yz + zt = (x – y + z) (y – z + t)
y z t
Solution: Here,
x y z
y = z = t = k (suppose)
z = tk
y = zk = tk.k = tk2
x = yk = tk2.k = tk3

Now,
L.H.S = xy – yz + zt

136 | Mathematics – 9 Algebra


= tk3.tk2 – tk2.tk + tk.t
= t2k5 – t2k3 + t2k
= tk2 k – tk k + t k
= t k (k2 – k + 1)
R.H.S = (x – y + z)(y – z + t)
= (tk3 – tk2 + tk)(tk2 – tk + t)
= tk(k2 – k + 1).t(k2 – k + 1)
= t2k(k2 – k + 1)2
= t k (k2 – k + 1)
 L.H.S = R.H.S proved.

Exercise 4.4.2
1. (a) If 3, 4, 6 and a are proportional, find the value of a.
(b) If 4, x, 10 and 15 are proportional, find the value of x.
(c) If 4, x and 9 are in continued proportion, find the value of x.
(d) If 3, x and 27 are in continued proportion, find the positive value of x.
(e) If 3, 12, a and 192 are in proportion, find the value of a.
2. (a) What is the value of x if 3, 4, x and 28 are in proportion.
(b) Find the fourth proportion of 3, 4, 9.
(c) What is the value of x when x, 6 and 4 are in continued proportion?
(d) Find the third proportion of 14, 10, 25.
3. (a) Find the value of x in the following proportions:
(i) 4:12: :x:6 (ii) 4:x: :8:12 (iii) 5:10: :20:x
(b) What constant number should be added to each of the terms 8, 13, 10 and 16 so that
they are in proportion?
(c) What number should be added to each of the terms 2,4,7 so that the sums will be in
continued proportion?
(d) What number should be subtracted to each of the terms 4, 5, 7 and 9 so that the
differences will be in proportion?
(e) What number must be deducted from each of the terms 6, 10, 18 so that the difference
will be in continued proportion?
3a – 2b 5x + 2y
4. (a) If a:b = 4:3, find the value of 3a + 2b . (b) If x:y = 5:6, find the value of 5x – 2y .

3a + 4b 2a – 3b
(c) If a:b = 2:5, find the value of 4a + b . (d) If a:b = 3:4, find the value of 2a + 3b .

Algebra Mathematics – 9 |137


a c
5. If b = d by using properties of propertion, prove that:
a c a–b c–d 4a + 7b 4a – 7b
(a) a+b=c+d (b) a+b =c+d (c) 4c + 7d = 4c – 7d
c2 a2 + c2 2 2
a + b = a2 + b2
2
a2 + c2 c
(d) d2 = b2 + d 2 (e) (f) b2 + d2 = d
c + d c + d
2
2
 a – c  = a2 ab + cd a2 + c2
(g) (h) ab – cd = a2 – c2
b – d b
ma + nc a2 + c 2 a(c – a)2 b(a2 + c2)
(i) mb + nd = b2 + d 2 (j) c(b – d)2 = d(b2 + d2)
a c
6. If b = d , prove that:
a–b c–d a2 – b 2 b 2
(a) a+b=c+d (b) c2 – d 2 = d 2
ac a2 – c2 a2 + b2 (a + b)2
(c) bd = b2 + d2 (d) c2 + d2 = (c + d)2
a2 – c2 c 3a + 4c a2 + c2
(e) b2 – d2 = d (f) 3b +4d = b2 + d2
ad + bc 2bd a + ab + b a2 – ab – b2
2 2
(g) a2 + c2 = ab + cd (h) c2 + cd + d2 = c2 – cd – d2
7. If a:b = c:d = e:f, prove that:
2
(a) (a2 + c2 + e2) (b2 + d2 + f2) = (ab + cd + ef)2 (b) a + 2c – 3e = c(a + e)
b + 2d – 3f d(b + f)
ac + ce + ae a2 a2 + c2 – e2 ab + cd – ef
(c) bd + df + bf = b2 (d) ab + cd – ef = b2 + d2 – f2
pa + qc + re 3 ace a3 + c3 + e3 ace
(e) pb + qd + rf = bdf (f) b3 + d3 + f3 = bdf
a+c+e a (a – 2c + e)3 a3 + c3 + e3
(g) b+d+f b = (h) (b – 2d + f)3 = b3 + d3 + f3
a b
8. If b = c by using properties, prove that:
a a–b (a + b)2 (b + c)2 (a – b)2 (b – c)2
(a) =
a+b a–c (b) b 2 = c 2 (c) a = c
3 3 2 2
a + b a(a – b) a –b a
(d) b3 + c3 = c(b – c) (e) b 2 – c2 = c (f) a – 2b + c = (b– c)2:c
a2 + ab b2 + bc (a + b)2 a2 + b2 a2 + ab + b2 a
(g) b 2 = c2 (h) (b + c)2 = b2 + c2 (i) b2 + bc + c2 = c
9. If a, b and c are in continued proportion, prove that:
a2 – ab + b2 a2 + b2
(a) abc (a + b + c)3 = (ab + bc + ca)3 (b) b2 – bc + c2 = b2 + c2
2
(a + b + c) a+b+c
(c) a2 + b 2 + c2 = a – b + c (d) (a + b + c)(a – b + c) = a2 + b2 + c2
b2c2 c2a2 a2b2 3 3 3 1
(e) a + b + c =a +b +c (f) a3 + b3 + c3 = abc (a3b3 + b3c3 + c3a3)

138 | Mathematics – 9 Algebra


p3 + q3 + r3 1 1 1
10. (a) If p, q and r are in continued proportion, show that: p2q2r2 = p3 + q3 + r3.
1 1 1
(b) If x : y : : y : z, prove that x2y2z2x3 + y3 + z3  = x3 + y3 + z3.
 
1
(c) If a : b = b : c, prove that abc (ab + bc + ca) = (a + b + c)3.
3

(d) If x : y : : y : z, prove that y2 + yz + z2 : x2 + xy + y2 = z : x.


11. (a) If a, b, c and d are in continued proportion, prove that: (a3 + b3 + abc) : (b3 + c3 + bcd) = a:d.
(b) If a, b, c and d are in continued proportion, prove that: a + b : c + d = a2 + b2 + c2: b2 + c2 + d2.
(c) If a, b, c and d are in continued proportion, prove that: (a + d)(b + c) – (a + c)(b + d) = (b – c)2.
(d) If a, b, c and d are in continued proportion, prove that: a2 – b2 : b2 – c2 : :b2 – c2 :c2 – d2.
p q r
12. (a) If q = r = s , prove that: (p + q + r)(q + r + s) = pq + qr + rs .
p q r
(b) If q = r = s , prove that: (p2 + q2 + r2)(q2 + r2 + s2) = (pq + qr + rs)2.
a b c
(c) If b = c = d , prove that: (b – c)2 + (c– a)2 + (d – b)2 = (a – d)2.
a b c
(d) If b = c = d , prove that: (a – c)2 + (b – d)2 = (a – d)2 – (b – c)2.
p q r
13. (a) If q = r = s , prove that: pq – qr + rs = (p – q + r)(q – r + s) .
(b) If a, b, c, d, e are in continued proportion, prove that: a : e = a4 : b4.
x y z x3 y3 z3 (x – y – z)3
(c) If a = b = c , prove that: a2 – b2 – c2 = (a – b – c)2 .
(d) If a, b, c, and d are in continued proportion, prove that:
(a2 + b2 + c2) (b2 + c2 + d2) = (ab + bc + cd)2.
x y z
(e) If = = , find the value of x + y + z.
b–c c–a a–b

4.5 Simultaneous Equations


4.5.1 Linear Equation Y
y = 3x - 2
Let's consider an equation 2x + 3y = 5 in x and y. The variables
(4,10)
quantities x and y of the equation 2x + 3y = 5 have the power 1. Such
equation is called the first degree equation. The first degree equation (3,7)
is known as the linear equation.
Every linear equation can be expressed in the form of y = mx + c, (1,1)
X’ X
where m is the slope of the straight line represented by the equation O
(0,-2)
and c is the y–intercept made by the line in the y–axis.
(-1,-5)

Y’

Algebra Mathematics – 9 |139


Graph of Linear Equation
Let's consider an equation y = 3x – 2. In the equation, the value of y depends on the value of x.
Substituting the different values of x, the corresponding values of y can be calculated. When we put
the values of x as –1, 0, 1, 3, 4 in the equation y = 3x – 2 successively, then the corresponding values
of y are obtained which are –5, –2, 1, 7, 10 respectively.
We present these values in a table given below.
x –1 0 1 3 4
y –5 –2 1 7 10
From the above table the order pairs (–1, –5), (0, –2), (1, 1), (3, 7) and (4, 10) are obtained. We plot
these order pairs on a graph paper. By joining these points, a straight line is obtained which is shown
in the above diagram. From the graph we can see that the plotted points of y = 3x – 2 lie on the
straight line. So it is concluded that the graph of any linear equation represents a straight line.
Simultaneous Equations
Let's take an equation x + y = 7. The equation x + y = 7 also can be written as y = 7 – x. The equation
has as many pairs of solutions as substituting the values of x. A few pairs of solutions of the equation
are shown on the following given table.
x 0 2 –1 5 –3 8
y 7 5 6 2 10 –1
From the above table, (0, 7), (2, 5), (–1, 6), (5, 2), (–3, 10), (8, –1) are a few pairs of solutions that
satisfy the equation x + y = 7.
Let's take another equation x – y = 3. This equation has also as many pairs of solutions as substituting
the values of x. A few pairs of solutions of this equation are (0, –3), (1, –2), (–1, –4), (3, 0), (–3, –6),
(5, 2), (6, 3). These pairs of solutions satisfy the equation x – y = 3. But, a pair solution i.e. (5, 2)
satisfies both the equations. Such pair of equations which are satisfied by only one pair of solution is
called the simultaneous equations. Thus, simultaneous equations are a pair of equations. Both the
equations have one unique solution which satisfies on the both equations at the same time.
We can solve the simultaneous equations in two variables by the different methods. But, according to
the curriculum of class IX, we have discussed three methods for solving the simultaneous equations.
The three methods are substitution method, elimination method and graphical method.
(i) Substitution Method
Let’s consider the equations
x + 3y = 7 and 3x – y = 11
Here,
x + 3y = 7 ... (i)
3x – y = 11 ... (ii)
From equation (i)
x = 7 – 3y ... (iii)

140 | Mathematics – 9 Algebra


Substituting the value of x in equation (ii) from equation (iii), we have the equation in one variable
which is
3(7 – 3y) – y = 11
or, 21 – 9y – y = 11
or, –10y = 11 – 21
or, –10y = –10
y = 1
Now, again substituting the value of y in equation (iii), we get
x=7–3×1=7–3=4
 Solution of the equations (i) and (ii) is x = 4 and y = 1.
Check: Substituting the values of x = 4 and y = 1 in equation (i) and (ii), we have
4+3×1=7 and 3 × 4 – 1 = 11
or, 7 = 7 or, 11 = 11
Which are true.
From the solution of above example, it is concluded that the following steps are applied to
solve the simultaneous equations by the substitution method.
 Express the value of x in terms of y or the value of y in terms of x by taking any one
equation from the given two equations.
 Substitute the value of x(or y) in the next equation and make the equation in the single
variable.
 Solve the equation and find the value of x(or y).
 Again, substitute the value of x(or y) in any one equation and solve to find the value of
y(or x).
(ii) Elimination Method
The method of eliminating one of the two variables in a pair of simultaneous equation is to
make the same coefficient of the variables to be eliminated. This can be done by multiplying
the equations by such quantities so that the coefficient of x or y should be the same in two
resulting equations. Then we add or subtract the resulting two equations to eliminate one of the
two variables, we get an equation in a single variable. Then, we solve the equation with single
variable and find the value of the variable. After getting the value of the variable, it is
substituted in one of the equation to get the value of the remaining variable. Let’s learn the
process from the following example.
Consider the two equations
5x + 3y = 5 and 3x – 4y = 32
Here, 5x + 3y = 5 ... (i)
3x – 4y = 32 ... (ii)
Here, we are eliminating the variable y. So, to make the same coefficient of y. Multiplying
equations (i) by 4 and equation (ii) by 3. Then adding the resulting equations.

Algebra Mathematics – 9 |141


20x + 12y = 20
9x – 12y = 96
29x = 116
116
or, x = 29

x=4
Now, substituting the value of x in equation (i), we get
5 × 4 + 3y = 5
or, 20 + 3y = 5
or, 3y = 5 – 20
15
or, y = – 3

y = – 5
Hence, x = 4 and y = –5.
From the solution of the above example, it is concluded that the following steps are applied to
solve the simultaneous equations by the elimination method.
 Make the same coefficients of the variables x or y in the both equations by multiplying
both sides of the equations with the suitable number.
 Eliminate one of the variables x or y by adding or subtracting the resulting equations
according to these equal quantities that have opposite or same sign.
 Solve the equation with the single variable and find the value of the variable.
 Substitute the value of the variable in any one of the equations and find the value of the
remaining variable.

Worked Out Examples


Example 1: Solve by the substitution method: 2x – 3y = 4 and 3x + y = –5
Solution: Here,
2x – 3y = 4 ... (i)
3x + y = –5 ... (ii)
From equation (i),
2x = 4 + 3y
4 + 3y
or, x= 2 ... (iii)

Substituting the value of x in equation (ii) from equation (iii), we get


4 + 3y
3
 2  +y=–5
12 + 9y
or, 2 +y=–5

142 | Mathematics – 9 Algebra


12 + 9y + 2y
or, 2 =–5

or, 12 + 11y = –10


or, 11y = –10 – 12
22
or, y = – 11

 y = –2
Now, substituting the value of y in equation (iii), we get
4 + 3(–2) 4 – 6 2
x = 2 = 2 = – 2 = –1

Hence, x = –1 and y = –2.


Example 2: Solve the following equations by elimination method: 7x– 4y = –9 and
3x–y = 4
Solution: Here,
7x – 4y = – 9 ... (i)
3x – y = 4 ... (ii)
Multiplying equation (ii) by 4 and subtracting equation (i),
12x - 4y = 16
7x - 4y = -9
- +
5x = 16
25
or, x = 5

x=5
Now, multiplying equation (i) by 3 and equation (ii) by 7 and substituting, we get
21x - 12y = -27
21x - 7y = -28
- +
-5y = -55
 y = 11
Hence, x = 5 and y = 11
Note: It is better to use elimination method in the first step and substitution method in the next steps which
we cal mixed method.
x 2 x 3
Example 3: Solve the equations: – = 1 and + =3
3 y 4 y
Solution: Here,
x 2
3 –y =1 ... (i)

x 3
4 +y =3 ... (ii)

Algebra Mathematics – 9 |143


1 1
Multiplying equation (i) by 4 and equation (ii) by 3 and subtracting the resulting equations.

x 1 1
12 - 2y = 4
x 1
+ 1
- 12 - y =-
- 1 - 1 = 1 -1
2y y 4
–1 – 2 1 – 4
or, 2y = 4
3 3
or, – 2y = – 4

or, 6y = 12
y=2
Now, substituting the value of y in equation (i), we get
x 2
3 –2 =1
x
or, 3 –1=1
x
or, 3 =2
x=6
Hence, x = 6 and y = 2.

Exercise 4.5.1
1. Solve the following equations by substitution method:
(a) 4x + 3y = 5 (b) 4x + 5y = 4 (c) y = 2x – 3
7x – y = 15 5x – 3y = 79 x+y=3
(d) 3x + y = 5 (e) 5x + 7y = 1 (f) 11x – 7y = 43
2x + 5y = –1 x + 4y = –5 2x – 3y = 13
2. Solve the following equations by substitution method:
x y 23 8 9
(a) –3x + 2y = –2 (b) 2 +3 = 6 (c) x –y =1
x y 17 10 6
5x + 3y = 5 4 +2 = 4 x +y =7
14 3 3 2 5 3 1
(d) x+y +x–y =5 (e) x +y =1 (f) x – y = 115
21 1 4 3 17 2 7 1
x+y –x–y =2 x +y = 6 x + y = 215

144 | Mathematics – 9 Algebra


3. Solve the following equations by elimination method:
(a) 2x + y = –1 (b) 14x + 9y = 156 (c) x+y=6
5x – 4y = –9 7x + 2y = 58 2x – y = 3
(d) x + 2y = 9 (e) 2x – y = –3 (f) 2x + y = 11
x – 1 = 2y 3x + 2y = 20 x + 3y = 18
4. Solve the following equations by elimination method:
15 1 1 3 2 4
(a) 3x + 4y = 2 (b) x – y = 42 (c) x – y = 15
9 2 2 3 9
y=x–3 x +y =4 x + y = 10
5 2 x y 17 14 40 3
(d) x+y +x–y =3 (e) 3 + 4 = 12 (f) 2x + 3y + 2x – 3y = 2
20 3 x y 3 4 5 3
x+y –x–y =1 5 –3 =–5 2x + 3y + 2x – 3y = 8
5. Solve the following equations:
2 5 11
(a) 4x – 5y = 7 (b) x+y=5 (c) 3x + 4y = 12
5 1 27
x + 2y = 5 5x – 3y = 17 4x + 2y = 20
4 3 3 2 3
(d) 3x + 4y = 4 (e) x+y –x–y =1 (f) 2x – y – 2x + y = 5
12 5 4 5 7
2x – 14 = 3y x+y +x–y =4 2x + y + 2x – y = 215

4.5.2 Graphical Method


In this method, we find a few ordered pairs of solutions of each linear equation of two variables in two
separate tables. The pair of solution of each equation from the separate tables are plotted on the same
graph paper and by joining the plotted points two separate straight lines are obtained. The two straight
lines intersect each other at a point, then the co–ordinates of the intersecting point is the required
solution of the given simultaneous equations. Let's learn the process from the following examples.

Worked Out Examples


Example 1: Solve graphically: x + 2y = 8 and 2x – y = 6.
Y
Solution: Here,
x + 2y = 8 ... (i) (0,4)
2x – y = 6 ... (ii) (4,2)
(8,0)
From equation (i), X’
(3,0)
X
(0,0) O
x + 2y = 8 (2,-2) (10,-1)
x 8 4 0
 x = 8 – 2y y 0 2 4 (0,-6)
From equation (ii),
Y’

Algebra Mathematics – 9 |145


2x – y = 6
or, 2x = y + 6 x 3 4 2
y 0 2 –2
y+6
 x= 2

We plot the points which are obtained from the above two separate tables and joining
them. We get two separate lines which intersect each other at a point (4, 2) in the
above graph.
Hence, the required solution of the given two simultaneous equations is x = 4 and
y = 2.
Example 2: Solve graphically: x + 3y = 7 and 3x – y = 11.
Solution: Here,
x + 3y = 7 ––––––––––––– (i) Y
3x – y = 11 –––––––––––– (ii)
(6,7)
From equation (i),
x + 3y = 7 Table – 1 (1,2)
(-2,3) (4,1)
or, 3y = 7 – x x 1 4 –2 X’ X
O (0,0)
7–x y 2 1 3
 y= 3
(2,-5)
From equation (ii), Table – 2
3x – y = 11 x 2 4 6 Y’
or, 3x – 11 = y y –5 1 7
 y = 3x – 11

Now, we plot the ordered pairs (1, 2), (4, 1) and (–2, 3) from the table–1 on the graph paper and by
joining them to construct a straight line. Similarly, again we plot the ordered pairs (2, –5), (4, 1) and
(6, 7) from the table– 2 on the same graph and construct another straight line by joining them.
In the above graph, two straight lines which are constructed from the equations (i) and (ii) are
intersected each other at a point (4, 1). It means both straight lines are passing through the point (4, 1).
Therefore, x = 4 and y = 1 are satisfied in the given both equations. Form the above two tables, a pair
numbers (4, 1) is only a common point of the two straight lines.
Hence, the required solution of the given two simultaneous equations is x = 4 and y = 1.
From the solution of the above examples, it is concluded that the following steps are applied to solve
the simultaneous equations of two variables by graphical method.
 Prepare the table and tabulate by putting the values of x and y in the both equations.
 Plot the pair numbers on the graph paper and construct the two straight lines by joining
the pair numbers on the graph.
 Find the point on the graph which is the common point where two straight lines
intersect each other.
 The co–ordinates of the common point is the required solution of the given two
simultaneous equations.

146 | Mathematics – 9 Algebra


Exercise 4.5.2
1. Copy and complete the table of values for x and y. Plot the ordered pairs separately from each
table. Find the common point, which is the solution of the equations from the graphs.
(a) 2x –3 = y x+y=3
x 2 5 x 1 4
y –1 y 1
(b) 2x + y = 11 x + 3y = 18
x 3 x –3
y 9 3 y 6 5
(c) 3x – 5y = 10 x – 2y = 4
x 0 –5 x 4
y 1 y –3 1
(d) x+y=7 x–y=3
x 4 0 x 0
y 0 y 1 3
2. Solve the following simultaneous equations graphically:
(a) 2x + y = 3 (b) x–y+3=0 (c) 2x – 3y = 1
2x – y = 1 3x – y = 1 3x + y = –5
(d) 2x + y = 9 (e) 2x – y = 4 (f) x – 2y = –1
2x – y = 11 x–y=3 2x – y = –4

4.6 Quadratic Equation


Introduction
Let’s consider an equation 4x + 2 = x – 1. This equation has only one variable. i.e x. After solving this
equation, the value of x is –1. It means this equation is satisfied by the value of x = –1. So, the
equation contains only one variable and the highest power of that variable is 1. Such type of the
equation is called a first degree equation of one variable. The first degree equation of one variable is
also called a linear equation in one variable.
But, let’s consider another equation x2 + 5x – 6 = 0. This equation has also only one variable but the
highest power of the variable is 2. When we solve this equation, we get the values of x are 1 and –6. It
means the equation x2 + 5x – 6 = 0 is satisfied by both values of x = 1 and x = –6. So, it is called a
second degree equation of one variable. The second degree equation of one variable is called a
quadratic equation. The solution of any quadratic equation has two values. These two values of
the variable are called the roots of the quadratic equation.
Thus, a quadratic equation is a second degree polynomial equation in one variable. The standard form
of the quadratic equation is ax2 + bx + c = 0, where, a, b, cR and a ≠ 0.

Algebra Mathematics – 9 |147


Types of Quadratic Equation
There are two types of quadratic equation which are (i) Pure quadratic equation (ii) Adfected
quadratic equation.
(i) Pure quadratic equation: A quadratic equation which is in the form of ax2 + c = 0 is called a
pure quadratic equation. It means a quadratic equation which does not contain the term with the
variable containing power 1 is known as a pure quadratic equation. For examples: x2 – 16 = 0,
3y2 – 18 = 0, 5x2 – 4 = 0 etc.
(ii) Adfected quadratic equation: A quadratic equation which is in the form of ax2 + bx + c = 0 is
called an adfected quadratic equation. It means the adfected quadratic equation is a quadratic
equation containing term with the variable having power 1 also. For example: 3x2 + 5x + 2 = 0,
x2 + 2x – 15 = 0 etc.
Solution of a Quadratic Equation
We know that a quadratic equation is a second degree equation. So, we obtain two values of the
variable from the given quadratic equation. It means a quadratic equation has two roots. We can solve
a quadratic equation by the various methods. But in this chapter, we discuss three different methods
for solving the quadratic equation.

4.6.1 Solving a Quadratic Equation by Factorization Method


In this method, the right hand side of the given equation is made zero by transposing the terms to the
left hand side. Then the second degree polynomial in the form of ax2 + c or ax2 + bx + c is factorized
and expressed as the product of two linear factors. After that, each linear factor is separately solved to
get the value of the variable which is in the given equation. The values of the variable are the required
solution of the given equation. Let's learn this process in the following examples.

Worked Out Examples


Example 1: Solve: x2 = 36.
Solution: Here,
x2 = 36
or, x2 – 36 = 0
or, (x)2 – (6)2 = 0
or, (x + 6) (x – 6) = 0
[a2 – b2 = (a + b)(a – b)]
Either, x + 6 = 0 i.e. x = –6
OR, x – 6 = 0 i.e. x = 6
 x = ±6
Example 2: Solve: 5x2 + 8x = 21.
Solution: Here,
5x2 + 8x = 21
or, 5x2 + 8x – 21 = 0

148 | Mathematics – 9 Algebra


or, 5x2 + (15 – 7)x – 21 = 0
or, 5x2 + 1 5x – 7x – 21 = 0
or, 5x(x + 3) – 7(x +3) = 0
or, (x + 3) (5x – 7) = 0
Either, x + 3 = 0 i.e x = –3
7
OR, 5x – 7 = 0 i.e x = 5

7
 x = –3 and 5 .

4 4 1
Example 3: Solve: – = .
x–3 x+3 3
Solution: Here,
4 4 1
x–3–x+3 =3
4(x + 3) – 4(x – 3) 1
or, ( x – 3)(x + 3) = 3
4x + 12 – 4x + 12 1
or, x2 – 9 =3

24 1
or, x2 –9 = 3
or, x2 – 9 = 72
or, x2 – 81 = 0
or, (x)2 – (9)2 = 0
or, (x – 9)(x + 9) = 0
Either, x + 9 = 0 i.e = x = –9
OR, x – 9 = 0 i.e = x = 9
 x = ±9
4x – 21 3x – 11
Example 4: Solve: = .
x–6 x–1
Solution: Here,
4x – 21 3x – 11
x–6 = x–1
or, (4x – 21)(x – 1) = (3x – 11)(x – 6) [ By cross – multiplication ]
2 2
or, 4x – 4x – 21x + 21 = 3x – 18x – 11x + 66
or, 4x2 – 25x + 21 = 3x2 – 29x + 66
or, 4x2 – 3x2 – 25x + 29x + 21 – 66 = 0
or, x2 + 4x – 45 = 0
or, x2 + 9x – 5x – 45 = 0
or, x(x + 9) – 5(x + 9) = 0

Algebra Mathematics – 9 |149


or, (x + 9)(x – 5) = 0
Either, x + 9 = 0 i.e x = –9
OR, x – 5 = 0 i.e x = 5
 x = 5 and –9.
x+1 x+2 2x + 13
Example 5: Solve: + = .
x–1 x–2 x+1
Solution: Here,
x + 1 x + 2 2x + 13
x–1+ x–2 = x+1
(x + 1)(x – 2) + (x + 2)(x – 1) 2x + 13
or, (x – 1)(x – 2) = x+1

x2 – 2x + x – 2 + x2 – x + 2x – 2 2x + 13
or, x2 – 2x – x + 2 = x+1

2x2 – 4 2x + 13
or, x2 – 3x + 2 = x + 1
or, (2x2 – 4) (x + 1) = (x2 – 3x + 2) (2x + 13)
or, 2x3 + 2x2 – 4x – 4 = 2x3 + 13x2 – 6x2 – 39x + 4x + 26
or, 2x3 + 2x2 – 4x – 4 = 2x3 + 7x2 – 35x + 26
or, 2x3 – 2x3 + 2x2 – 7x2 – 4x + 35x – 4 – 26 = 0
or, –5x2 + 31x – 30 = 0
or, –(5x2 – 31x + 30) = 0
or, 5x2 – 25x – 6x + 30 = 0
or, 5x (x – 5) – 6(x – 5) = 0
or, (x – 5) (5x – 6) = 0
Either, x – 5 = 0 i.e x = 5
6 1
OR, 5x – 6 = 0 i.e x = 5 = 15

1
 x = 5 and 15 .

Exercise 4.6.1
1. Solve:
(a) (x – 3)(x + 2) = 0 (b) (y – 7)(2y – 9) = 0 (c) (3x – 1)(x – 5) = 0
(d) (2t – 1)(t – 2) = 0 (e) (5 – 6x)(5x + 6) = 0 (f) (9x + 2)(3 – 4x) = 0
2. Solve:
(a) x2 – 25 = 0 (b) 4y2 – 9 = 0 (c) 5t2 = 125
x 9
(d) x2 – 7 = 29 (e) y2 = 5 (f) 4 =x

150 | Mathematics – 9 Algebra


5x 20 3x 27 x 2 x 8
(g) 4 = x (h) 4 – x =0 (i) 2+x=8+x
3. Solve:
(a) 2x(3 – x) = 0 (b) y(y – 5) = 0 (c) x2 – 7x = 0
(d) 3x2 – 18x = 0 (e) t2 – 36t = 0 (f) 5m2 + 10m = 0
2 2
x x t t 3x2 x
(g) 4 = 12 h) 3=8 (i) 2 =6
x2 x 2y2 y 2 7
(j) 50 = 2 (k) 3 =9 (l) x = x2
4. Solve:
(a) (x + 5)(x – 2) = 3(x + 18) (b) (x + 2)(x – 2) = 3x2 – 8
(c) (t – 3)(t + 2) = 10 – t (d) 2(y2 – 8) + y(4 – y) = 4(y + 12)
(e) 3(x + 3)(x – 3) = x2 + 5 (f) 2(p + 3)(p – 2) = 2p + 4
5. Solve:
(a) x2 – 11x + 30 = 0 (b) y2 – 15y + 36 = 0 (c) x2 – 5x – 6 = 0
(d) t2 + 2t – 24 = 0 (e) x2 – 9x = 36 (f) 6p2 + 13p = 5
2 2
(g) 3x – 5x – 2 = 0 (h) 30x = 8x + 27 (i) x2 + 3x – 10 = 0
6. Solve:
(a) 2x2 – 3x + 1 = 0 (b) 9t2 + 6t + 1 = 0
2
(c) 3p – 10p = 8 (d) 2x2 + x – 6 = 0
(e) (x – 3)(x + 5) = 48 (f) (2x – 3)(x – 5) = (x – 3)2
(g) –3y2 + 5y + 8 = 0 (h) (x + 4)(2x – 3) = 6
(i) x2 – 2ax + a2 – b2 = 0 (j) 4(x – 2)2 – 5(x – 2) – 6 = 0
(k) 11(t + 1)(t + 2) = 38(t + 1) + 9t.
7. Solve:
x+1 x+7 7 65y
(a) x + 3 = 2x + 8 (b) 8y + y = 7

2x2 + 10 50 + x2 5 5 2
(c) =7– (d) – =
15 25 x–2 x+2 3
3 3 4p2 + 5 2p2 – 5 7p2 – 25
(e) 1+x +1–x =8 (f) 10 – 15 = 20
x+2 x–2 1 x+3 x–3 1
(g) 2 – x + 2 + x = 44 (h) x – 3 + x + 3 = 22
x+1 x–1 2
(i) x – 1 – x + 1 = 23
8. Solve:
3x – 7 x + 1 x + 2 2x – 3
(a) 2x – 5 = x – 1 (b) x + 3 = 3x – 7

Algebra Mathematics – 9 |151


3y – 8 y – 2 2 3t
(c) 5y – 2 = y + 5 (d) t + 1 = 5t – 1
t 1–t 1 2 3 2
(e) t – 1 – t = 22 (f) x–3 +x–1 =x
1 3 2 x+2 x–2 5
(g) x–1 =2– x –x (h) x–2 –x+2 =6
x x + 1 13
(i) x+1 + x = 6
9. Solve:
1 1 1 x + 4 x – 4 10
(a) x+2 – x–2 –1–x =0 (b) x–4 +x+4 = 3
2t – 3 5 – 3t 5 2x 3x – 1 5x – 11
(c) – = (d) + =
3t – 5 2t – 3 2 x–1 x+2 x–2
x + 3 x – 3 2x – 3 y – 2 11 – 3y 4y + 13
(e) x+2 +x–2 = x–1 (f) y–3 – y–4 = y+1
x+2 x – 2 2x + 6 2x – 1 2x + 1 2
(g) x–2 + x+2 = x–3 (h) 2x + 1 – 2x – 1 = –23
x – 1 x + 3 2(x + 2)
(i) x+1 +x–3 = x–2

4.6.2 Solving Quadratic Equation by Completing the Square


We can use this method to solve a quadratic equation when the left hand expression of the given
equation is not easily factorized.
In this method, we transpose the constant term of the equation to the right hand side and the left hand
expression is made a perfect square by adding a suitable term on the both sides. We can make x2 + 2ax
or x2 – 2ax a perfect square by adding a2, i.e. the square of half of the coefficient of x. let’s learn this
method in the following examples.

Worked Out Examples


2
Example 1: Solve: 4x + 20x + 25 = 0.
Solution: Here, 4x2 + 20x + 25 = 0
or, 4x2 + 20x = – 25
or, 4(x2 + 5x) = –25
25
or, x2 + 5x = – 4

5 5 2 25 5 2
or, (x)2 + 2.x.2 + 2 = – 4 + 2
   
or, x + 52 = – 25 + 25
 2 4 4

152 | Mathematics – 9 Algebra


or, x + 52 = 0
 2
5
or, x+2 =0

5
 x=–2

Example 2: Solve: 2x2 + 7x + 4 = 0.


Solution: Here,
2x2 + 7x + 4 = 0
or, 2x2 + 7x = – 4
7
or, 2x2 + 2 x = – 4
 
7 7 2 4 7 2
or, (x)2 + 2.x.4 + 4 = – 2 + 4
   
2
or, x + 7 = – 4 + 49
 4 2 16
2
or, x + 7 = –32 + 49
 4 16
2
or, x + 7 = 17
 4 16
7 17
or, x+4 =± 4

7 17
or, x=–4± 4

– 7 ± 17
 x= 4

Example 3: Solve: 3x2 – 5x – 2 = 0.


Solution: Here,
3x2 – 5x – 2 = 0
or, 3x2 – 5x = 2
5
or, 3x2 – 3x = 2
 
5 5 2 2 5 2
or, (x)2 – 2.x.6 + 6 = 3 + 6
   
or, x – 52 = 2 + 52
 6 3 6
or, x – 52 = 2 + 25
 6 3 36
or, x – 52 = 24 + 25
 6 36

Algebra Mathematics – 9 |153


2
or, x – 5 = 49
 6 36
2 2
or, x – 5 = ± 7
 6  6 
5 7
 x– 6 =±6

Taking + ve, Taking – ve,


5 7 5 7
x– 6 =6 x– 6 =–6

7 5 7 5
Or, x = 6 + 6 or, x = – 6 + 6

12 –2
Or, x = 6 or, x = 6

1
x=2  x=–
3
1
x = 2 and – 3

x+3 1–x 1
Example 4: Solve by completing square: – x =4 .
x–2 4
Solution: Here,
x+3 1–x 1
x – 2 – x = 44
(x + 3).x – (1 – x)(x – 2) 17
or, x(x – 2) = 4

x2 + 3x – x + 2 + x2 – 2x 17
or, x2 – 2x = 4

2x2 + 3x – 3x + 2 17
or, x2 – 2x = 4

or, 4(2x2 + 2) = 17(x2 – 2x)


or, 8x2 + 8 = 17x2 – 34x
or, 17x2 – 34x – 8x2 – 8 = 0
or, 9x2 – 34x = 8
17 17 2 17 2
or, (3x)2 – 2 × 3x × 3 +  3  = 8 +  3 
   
or, 3x – 172 = 8 + 289
 3 9

or, 3x – 172 = 72 + 289


 3 9

or, 3x – 172 = 361


 3 9

154 | Mathematics – 9 Algebra


or, 3x – 172 = ±192
 3  3
17 19
 3x – 3 = ± 3

Taking + ve sign, Taking – ve sign,


17 19 17 19
3x – 3 = 3 3x – 3 = – 3

19 17 19 17
or, 3x = + or, 3x = – +
3 3 3 3
36 2
or, 3x = 3 or, 3x = – 3

2
or, 3x = 12 x = – 9

x = 4
2
x = 4 and – 9

Exercise 4.6.2
1. Solve by completing the square:
(a) 9x2 – 6x + 1 = 0 (b) x2+ 8x + 16 = 0 (c) x2 – 7x – 98 = 0
2 2
(d) 3x + 2x – 5 = 0 (e) 6t + 13t = 5 (f) 5y = 12 – 3y2
(g) 25s2 + 16 = 40s (h) 4x – 4x2 = –7 (i) 2x2 – 5 = x

2. Solve the following equations by completing the square:


(a) 3x2 + 5x – 2 = 0 (b) 2y2 + 7y + 4 = 0 (c) 3x2 – 2x – 21 = 0
(d) 2t2 – 7t + 5 = 0 (e) 2z2 – 6z + 3 = 0 (f) 5x2 – 8x – 21 = 0

3. Solve by completing the square:


3 3 x 2x – 1 x+1 x+7
(a) 1+x + 1–x =8 (b) 4 – x+1 =1 (c) x + 3 = 2x + 8
3t – 2 3t – 8 2y – 3 3y – 7 4 5 3
(d) 2t – 3 = t + 4 (e) y+2 = y+3 (f) z–1–z+2 =z
x+2 1 1 x–4 6 5 2s – 5 25 2s
(g) – = (h) + = (i) – =
6 x+2 6 4 x+1 4 s–3 3 s–4
2x – 1 2x + 1 2 3 2 2 2x – 9 2x – 7 7
(j) 2x + 1 – 2x – 1 = – 23 (k) x–1+x–3=x (l) 2x – 7 – 2x – 9 = 12

Algebra Mathematics – 9 |155


4.6.3 Solving Quadratic Equation by using Formula
Every quadratic equation can be written in the form ax2 + bx + c = 0. The expression ax2 + bx + c has
no simple factors. So, we solve the equation ax2 + bx + c = 0 by completing the square method.
Now,
ax2 + bx + c = 0
or, ax2 + bx = –c [Subtracting ‘c’ both sides]
2
ax b c
or, a +ax=–a [Dividing both sides by ‘a’ to make the coefficient of x2 unity]

b c
or, x2 + a x = – a

b b 2 c b 2
or, (x)2 + 2.x. 2a + 2a = –a + 2a
   
2
or, x + b  2 = –4ac +2 b
 2a 4a
2
or, x + b  2 = b – 4ac
 2a 4a2
2 2
x + b  = ± b2 – 4ac 
or,
 2a   4a2 

b b2 – 4ac
or, x + 2a = ± 2a

b b2 – 4ac –b ± b2 – 4ac
 x = – 2a ± 2a = 2a
Taking +ve sign, Taking – ve sign,

–b + b2 – 4ac –b – b2 – 4ac
x= 2a x= 2a

–b + b2 – 4ac –b – b2 – 4ac
Thus, 2a and 2a are two roots of x.

–b  b2 – 4ac
Hence, x = 2a is general formula and we can apply it to solve any given quadratic
equation.
Note:
–b  b2 – 4ac
(i) In the formula x = , the term b2– 4ac is called discriminant factor.
2a
(ii) If the discriminant factor b2 – 4ac = 0, the equation has single root.
(iii) If b2 – 4ac > 0 i.e. +ve, the equation has two real roots.
(iv) If b2 – 4ac < 0 i.e. –ve, the equation has no real solution i.e. the equation can not be solved.

156 | Mathematics – 9 Algebra


Work Out Examples
Example 1: Solve by using formula: 4x2 – 7 = 0.
Solution: Here,
4x2 – 7 = 0
or, 4x2 + 0.x – 7 = 0
Comparing this equation with ax2 + bx + c = 0, we get a = 4, b = 0 and c = –7
By using the formula,
–b ± b2 – 4ac
x = 2a
–0 ± 02 – 4 × 4 × 7
= 2×4
4 7

8
7
x = ± 2

Example 2: Solve by using formula: x2 – 9x + 14 = 0.


Solution: Here,
x2 – 9x + 14 = 0
Comparing this equation with ax2 + bx + c = 0, we get a = 1, b = –9 and c = 14
 by using formula,
– b  b2 – 4ac
x =
2a
–(–9)  (–9)2 – 4×1×14
= 2×1
9  81 – 56
=
2
9  25 9  5
= 2 = 2

Taking +ve sign, Taking –ve sign,


9+5 9–5
x= 2 x= 2

14 4
= 2 =7 =2=2

 x = 2 and 7.

Algebra Mathematics – 9 |157


x+2 x–2 4
Example 3: Solve by using formula: – =4 .
x–2 x+2 5
x+2 x–2 4
Solution: Here, x – 2 – x + 2 = 45

(x + 2)2 – (x – 2)2 24
or, (x – 2)(x + 2) = 5
(x2 + 2.x.2 + 22) – (x2 – 2.x.2 + 22) 24
or, (x)2 – (2)2 = 5

x2 + 4x + 4 – x2 + 4x – 4 24
or, x2 – 4 = 5

8x 24
or, x2 – 4 = 5
or, (x2 – 4) × 24 = 8x × 5
or, 24x2 – 96 = 40x
or, 8(3x2 – 12) = 40x
or, 3x2 – 12 = 5x [Dividing both sides by 8]
2
or, 3x – 5x – 12 = 0
Comparing this equation with ax2 + bx + c = 0, we get a = 3, b = –5 and c =–12.
 by using formula
– b  b2 – 4ac
x = 2a
(–5)  (–5)2 – 4 × 3 × –12
= 2×3
5  25 + 144
=
6
5  169
= 6
5  13
= 6
Taking +ve sign, Taking –ve sign,
5 + 13 5 – 13
x= 6 x= 6
18 8
= 6 =–6

4
=3 =–3

4
 x = 3 and – 3 .

158 | Mathematics – 9 Algebra


Exercise 4.6.3
1. Use the formula to solve the following equations:
(a) 81x2 = 16 (b) 49y2 – 100 = 0 (c) 2t2 – 9 = 0
(d) 9y2 – 0.01 = 0 (e) 18x2 – 27x = 0 (f) s2 – 36 = 0
2. use the formula to solve the following equations:
(a) x2 + 8x + 12 = 0 (b) 4x – 4x2 = –7 (c) 4x2 – 3x = 6
(d) 2y2 + 3 – 6y = 0 (e) t2 – 5t –36 = 0 (f) z2 + 20 – 9z = 0
(g) x2 + 7ax + a2 = 0 (h) 2x2 – 3x = 3 (i) 3s2 + 11s = –10
(j) (x – 3)(2x + 3) = 5 (k) (4x – 3)(8x + 5) = 13 (l) (2x – 3)2 = 4
3. Solve by using the formula:
(a) 6x2 – 5x = 0 (b) x2 + 2x – 24 = 0 (c) x(x – 5) = 24
x2 + 4 x2 + 5 t–4 6 5
(d) 5a2 + ax = 41x2 (e) 5 = 6 (f) 4 +t+1=4
x x + 1 25 x 2 7 5x + 1 3x + 1
(g) x + 1 + x = 12 (h) 3+x=3 (i) 7x + 5 = 7x + 1
x+3 x x + 3 3x – 7 x+2 4–x 7
(j) x +x+3 =1 (k) x+ 2 = 2x – 3 (l) x – 1 – 2x = 3
2t – 1 t + 1 4 x+4 x–4 2 y+6 y+1 1
(m) t + 1 + 2t + 1 = 3 (n) x–4–x+4=3 (o) y + 7 – y + 2 = 3y + 1

4. Solve:
(a) px2 – qx + r = 0 (b) lx2 + mx – n = 0
(c) ax2 – bx – c = 0 (d) dx2 + ex + f = 0

Algebra Mathematics – 9 |159


Unit Test
Time: 40 minutes F.M.- 24
Group- A (1 × 1 = 1)
1. If ax + bx + c = 0, what is the value of x?
Group- B (5 × 1 = 10)
2 2
2. Factorize: m – n – mx – nx
3x+4 – 3x+3
3. Simplify: 3x+3
4. If x:y = 3:4, find the value of (x +2y): (4x + y).
a c a2 + c2 ac
5. If b = d, prove that: b2 + d2 = bd

6. Use formula to solve: 2x2 – 3x – 2 = 0


Group- C (2 × 4 = 8)
1 1
– 1
7. If x = a3 – a 3, prove that x3 + 3x = a – a.

8. Solve by completing equation: 2x2 – 7x + 3 = 0


Group- D (1 × 5 = 5)
9. Solve graphically: x – y = 1 and x + y = 5.

160 | Mathematics – 9 Algebra


Answers ____________________________________________________________
Exercise 4.1.1
1. (a) x (3, – 4y) (b) 2 (a – b) (x + 2y) (c) 2a (2x – 3a + 4y)
(d) 2pq (7p + 3q – 6) (e) (2a + b) (2a – b) (f) 3x (x + 4) (x – 4)

(g) (3a + 12) (3a – 12) (h) (3x + 5y4 ) (3x – 5y4 ) (i) (x + y) (x – y) (x2 + y2)

(j) 3x (3x + 2) (3x – 2) (k) 3 (x + 7) (x – 7) (l) 2 (a + 11) (a – 11)


(m) (a + 2b) (a2 – 2ab + 4b2) (n) (x – 3y) (n2 + 3xy + 9y2) (o) mn2 (m + n) (m2 – mn + n2)

(p) 2p (2p – 1) (4p2 + 2p + 1) (q) (4m2 + n2) (16m4 – 4m2n2 + n4) (r) (n + 1n) (n –1 + n1 )
2
2

(s) (x + x1 ) (x –1 + x1 )
2
2
4
4 (t) (y + 4) (y – 2) (u) (x – 3) (x – y)
2
(v) (4 + 2a – b) (4 – 2a + b) (w) 5 (1 – x )
2. (a) (x + 2) (3x + 2) (b) (x + 3) (x + 4) (c) (a + 4) (a – 1)
(d) (a + 6) (a – 1) (e) (2a – 9) (a – 1) (f) (x – 3) (3x + 2)
(g) (a + 2) (a – 2) (2a2 + 3) (h) (x + 1) (x2 – x + 1) (2x3 – 3)

(1 + yx) (1 – yx) (7xy – 3) (2xy – 5) (yx + 1) (2pq + 1) (5 – 2qp )


2 2 2
(i) 2 (j) (k) 2 2

(l) (1 –
a )(b
– 5)
2 2
2b 3a
2 2 (m) (a + b+ 1) (9a + ab – 8) (n) (x + y – 4) (3x + 3y + 2)

(o) (x + y – 2z) (4x + 4y + 3z) (p) (a + 1 – 1a) (7a – 2 – 7a) (q) – (a + 3) (a + 9)

(r) (2 – a) (a – 7)
3. (a) (x – y) (x + y – a) (b) (a + b) (a – b – x) (c) (a + b) (a + c)
(d) (a + 6b) (a – c) (e) (x+y) (x–y–z) (f) (a – b)2
2
(g) (b –1) (ab + c) (h) (x –1) (x + 1) (i) (x + 2) (xy – 3)
(j) (y + 1) (y2 + y + 1) (k) (2 + x) (4 + x2) (l) (2 – 9x) (2 + x) (2 – x)
(m) (a + b) (a2 – ab + b2 – 1) (n) (x – 2y) (x2+ 2xy + 4y2 + 2a) (o) (2a + b)3
(p) ( x– y) (x2 + xy + y2 – 1) (q) (2x + y – 1) (2x – y + 1) (r) (2x + 3y + 1) (2x – 3y + 1)
(s) (3a + 2b + 1) (3a – 2b + 1) (t) (1 + 2p + 6p2) (1 + 2p – 6p2)

Exercise 4.1.2
1. (a) (x2 + 2xy + 2y2) (x2 – 2xy + 2y2) (b) (x2 + 2x + 2) (x2 – 2x + 2) (c) (a2 + 4ab + 8b2) (a2 – 4ab + 8b2)
2 2 2 2 2 2 2 2
(d) (18p + 6pq + q ) (18p – 6pq + q ) (e) (49a + 14ab + 2b ) (49a – 14ab + 2b )

(f)( 2 1
x +1+ 2
2x )( ) (g) (2x + 3y2x + 9y1 ) (2x – 3y2x + 9y1 )
2 1
x –1+ 2
2x
2
2
2
2

(h) (2x +
5y 25y ) ( 5y 25y )
2x 21 2x 1 2
+ 2x – + 2 2

2 2 2 2
2. (a) (x + xy + y ) (x – xy + y ) (b) (x2 + x + 1) (x2 – x + 1) (c) (x2 + 3x + 1) (x2 – 3x + 1)
2 2 2 2
(d) (2p + p + 1) (2p – p + 1) (e) (m + m – 1) (m – m – 1) (f) (3a2 + 4a + 2) (3a2 – 4a + 2)
2 2 2 2
(g) (m + 4mn + n ) (m – 4mn + n ) (h) (2a + 3ab + 11b ) (2a – 3ab + 11b ) (i) (x2 + 7x2 + 16) (x2 – 7x2 + 16)
2 2 2 2

2 2
(j) (x + 3x – 1) (x – 3x – 1) (k) (x2 + 2x – 1) (x2 – 2x – 1) (l) (a2 + 4ab + b2) (a2 – 4ab + b2)

(yx + 1 + yx) (yx – 1 + yx) (pq + 3pq + 1)(pq – 3pq + 1)


2 2
(m) (n) (5x2 + 7xy + 4y2) (5x2 – 7xy + 4y2) (o) 2 2

(p) (
x ) (2y x ) (x + 3 + x1 ) (x – 3 + x1 ) (r) ( + – 1) ( – – 1)
2 2 2 2 2 2
x 2y x 2y 2 2 x 3x x 3x
+3+ 2 –3+
2 2 2 (q) 2 2 2 2
2y y y y y

Algebra Mathematics – 9 |161


(2x + 5 – 2x1 ) (2x – 5 – 2x1 ) (x + 1 – 1x) (x – 1 – 1x) (yx + 1 + yx ) (yx – 1 + yx )
2 2 2 2
(s) (t) (u) 2 2 2 2

3. (a) (x + 3y – 6) (x – 3y – 4) (b) (p + q + 4) (p – q – 10) (c) (a + b – 4) (a – b – 2)


(d) (x2 + x – y – 3) (x2 – x + y – 3) (e) (a + b – 14) (a – b + 2) (f) (x – y – 2) x – 3y + 2)
(g) (x – 4y + 55z) (x – 50y – 55z) (h) (a – b + c + d) (a – b – c – d)

Exercise 4.2
1. (a) 46 (b) 32a (c) (7x)4 (d) (a + b)5
3 4 6 a
2. (a) 13 (b) 2 (c) 2 (d) 9
2
1
(e) (3p + q)5 (f)
23 a
8
x 3a2 x5
3. (a) 2 (b) (c) 12
y b
b5
1 1
27y3
(d) a2 (e) (f) a2
x3
4. (a) 4 (b) 16 (c) 4
1 9
(d) (e)
8 16
1 2
5. (a) (b) 3 (c)
4 x
1
(d) (e) 4
3
1
6. (a) 125 (b) (c) 16
3
2
(d) (e) 2 (f) 6
5
3
2
b 9
7. (a) (b) m (c)
2 4x2y2
3
a
5
2y2 a3c6 y12
(d) (e) (f)
3xz3 5 x37
3
b
64 3 32
8. (a) 1 (b) (c) (d)
81 5 729
5 6
(e) (f)
96 7
2 2
9. (a) 1 (b) x2(a –c )
(c) 1 (d) xb
(e) x11n (f) 1
10. (a) 1 (b) 1 (c) 1 (d) 1
(e) 1 (f) 1
11. (a) 1 (b) 1 (c) 1 (d) 1
(e) 1
(y – z) (z – x) (y – x)
12. (a) 1 (b) 1 (c) a xyz (d) 1
m+n m+n
(e) (yx) (f) (ba) (g) 1

13. (a) 2

162 | Mathematics – 9 Algebra


Exercise 4.3
7
1. (a) 3 (b) 1 (c) – (d) 1
2
3
(e) – (f) 4
2
1 1
2. (a) (b) 1 (c) 3 (d)
2 2
(e) 3 (f) 13
3. (a) –1 (b) 2 (c) 14 (d) 5
2 1
(e) (f) –
3 2
4. (a) 0 (b) 3 (c) 1 (d) 3
(e) –2 (f) –3
5. (a) –1 (b) 2 (c) –2 (d) 0
3 3
(e) (f) (g) 3 (h) 2
2 2
6. (a) 1 (b) 1 (c) –2, –3 (d) 1, 2 (e) 2

Exercise 4.4.1
1. (a) 3:160 (b) 1:2 (c) 6:1 (d) 14:1
(e) 50:19 (f) 4:7
2. (a) 4:9 (b) 1:1 (c) 8:105
3. (a) 4:25 (b) 4x2:49y2 (c) 144:49 (d) 196a2:81b2
4. (a) 3:4 (b) 7:6 (c) 13a:12b (d) 2:3
5. (a) 64:343 (b) 8a3:b3 (c) 125:8 (d) 216x3:1331y3
6. (a) 2:3 (b) 5x:1 (c) 6a:4b (d) 9x2:8y3
2 2
7. (a) 19:12 (b) 21y :16x (c) (9y – 5):(2x + 3) (d) 7b:18a
8. (a) 16:33, 16:36:33 (b) 3:10, 6:15:20 (c) 3:2, 24:33:16
9. (a) 3:4 (b) 17:37 (c) 4:7 (d) 0 (e) 0
10. (a) 11:7 (b) 4:3 (c) 25:9 (d) 2:5 (e) 35:9
11. (a) 3:4 (b) 5:3 (c) 4:5 (d) 3:4
(e) 3:4 (f) 11:8 (g) 1:7 (h) 2:3
12. (a) 2:11 (b) 5:2 (c) 7:4
13. (a) 35, 63 (b) 70, 98 (c) 75, 125
(d) 76, 133 (e) 40°, 60°, 80°
14. (a) Rs. 32, Rs. 36 (b) Rs. 30, Rs. 42 (c) Rs. 9, Rs. 12
(d) Rs. 30 (e) 54
15. (a) 13 (b) 17 (c) 2 (d) –8
16. (a) 6, 10 (b) 34, 51 (c) 50, 74 (d) 56, 70
17. (a) 24 years, 32 years (b) 21 years, 28 years (c) 10 years, 35 years
18. (a) 35 lit, 20 lit (b) 54 (c) 20°, 70° (d) 8 km

Exercise 4.4.2
1. (a) 8 (b) 6 (c) 6 (d) 9 (e) 48
2. (a) 21 (b) 12 (c) 9 (d) 35
3. (a) (i) 2 (ii) 6 (iii) 40 (b) 2 (c) 2 (d) 1 (e) 2
4. (a) 1:3 (b) 37:13 (c) 2 (d) –1:3
13. (e) 0

Algebra Mathematics – 9 |163


Exercise 4.5.1
1. (a) x = 2, y = –1 (b) x = 11, y = –8 (c) x = 2, y = 1 (d) x = 2, y = –1
7 13
(e) x = , y = – (f) x = 2, y = –3
9 9
16 5
2. (a) x = ,y= (b) x = 3, y = 7 (c) x = 2, y = 3 (d) x = 4, y = 3
19 19
3 2
(e) x = – , y = (f) x = 3, y = 5
8 9
3. (a) x = –1, y = 1 (b) x = 6, y = 8 (c) x = 3, y = 3 (d) x = 5, y = 2
(e) x = 2, y = 7 (f) x = 3, y = 5
4. (a) x = 2, y = –1 (b) x = 3, y = 2 (c) x = 5, y = 6 (d) x = 3, y = 2
(e) x = 2, y = 3 (f) x = 33, y = –18
5. (a) x = 3, y = 1 (b) x = 4, y = 1 (c) x = 1, y = 5 (d) x = 4, y = –2
11 6
(e) x = 8 , y = –5 (f) x = 2, y = 1
17 17

Exercise 4.5.2
1. (a) x = 2, y = 1 (b) x = 3, y = 5 (c) x = 0, y = –2 (d) x = 5, y = 2
2. (a) x = 1, y = 1 (b) x = 2, y = 5 (c) x = –1, y = –2 (d) x = 5, y = –1
(e) x = 1, y = –2 (f) x = 3, y = 2

Exercise 4.6.1
9 1 1
1. (a) 3, –2 (b) 7, (c) ,5 (d) , 2
2 3 2
5 6 2 3
(e) ,– (f) – ,
6 5 9 4
3
2. (a) ± 5 (b) ± (c) ± 5 (d) ± 6
2
(e) ± 5 (f) ± 6 (g) ± 4 (h) ±6 (i) ± 4
3. (a) 0, 3 (b) 0, 5 (c) 0, 7 (d) 0, 6 (e) 0, 36
1 2 1
(f) 0, –2 (g) 0, (h) 0, 2 (i) 0, (j) 0, 25
3 3 9
1 1
(k) 0, (l) 0, 3
6 2
4. (a) ± 8 (b) ± 2 (c) ± 4 (d) ± 8
(e) ± 4 (f) ± 2 2
5. (a) 5, 6 (b) 3, 12 (c) –1, 6 (d) 4, –6
1 1 1 3 9
(e) –3, 12 (f) –2 , (g) – , 2 (h) , (i) –5, 2
2 3 3 2 4
1 1 2 1
6. (a) ,1 (b) – (c) – , 4 (d) –2, 1
2 3 3 2
2 1
(e) –9, 7 (f) 1, 6 (g) –1, 2 (h) –4 ,2
3 2
1 8
(i) a ± b (j) 1 , 4 (k) – ,2
4 11
7
7. (a) ± 13 (b) ± (c) ± 5 (d) ± 34
3
1 1 1
(e) ± (f) ± 5 (g) ± 1 (h) ± 9 (i) – , 1
2 5 4

164 | Mathematics – 9 Algebra


1 1
8. (a) 3, 4 (b) –1, 5 (c) 4, 5 (d) ,2 (e) –1, 2
2 3
1 1 2
(f) –1, 2 (g) , 1 (h) – , 10 (i) –3, 2
2 3 5
3 1
9. (a) 0, 4 (b) ± 8 (c) 1, 1 (d) 1 , 4 (e) 0, 4
4 2
23 1 1
(f) 5, (g) 0, 1 (h) – , 1 (i) 0, 5
7 3 4

Exercise 4.6.2
1 2
1. (a) (b) –4 (c) –7, 14 (d) –1 , 1
3 3
1 1 1 4 1±2 2 1± 41
(e) –2 , (f) –3, 1 (g) (h) (i)
2 3 3 5 2 4
1 –7 ± 17 1 1
2. (a) –2, (b) (c) –2 , 3 (d) 1, 2
3 4 3 2
3± 3 2
(e) (f) –1 , 3
2 5
1 2
3. (a) ± (b) 0, 11 (c) ± 13 (d) 1, 10
2 3
1
(e) –1, 5 (f) – , 3 (g) –4, 1 (h) 3, 5
2
77 ± 71 1 1 1
(i) (j) – , 1 (k) –1, 2 (l) , 14
25 4 2 4
Exercise 4.6.3
4 3 3 1
1. (a) ± (b) ± 1 (c) ± (d) ±
9 7 2 30
1
(e) 0, 1 (f) ± 6
2
1±2 2 3± 105 3± 3
2. (a) –2, –6 (b) (c) (d)
2 8 2
(–7 ± 3 5 )a 3 ± 33
(e) –4, 9 (f) 4, 5 (g) (h)
2 4
2 1 7 1 1
(i) –1 , –2 (j) –2, 3 (k) – , –1 (l) , 2
3 2 8 2 2
5 (1 ± 821)a
3. (a) 0, (b) –6, 4 (c) –3, 8 (d)
6 82
(e) ± 1 (f) 3, 5 (g) –4, 3 (h) 1, 6
2 3 4
(i) – , 1 (j) (– 1 ± –3 ) (k) –1, 5 (l) – , 3
7 2 5
3± 37
(m) n) 4(3 ± 10 ) (o) 3
7
2 2
q ± q – 4pr –m± m + 4ln b± b2 + 4ac –e± e2 – 4df
4. (a) (b) (c) (d)
2p 2l 2a 2d

Algebra Mathematics – 9 |165


166 | Mathematics – 9 Algebra
Chapter

5
Geometry
Objectives:
At the end of this chapter, the
students will be able to:
 constant the triangles and
quadrilaterals having different
measurement.
 show the properties of
triangle theoretical as well as
experimentally.
 show the properties of
parallelograms theatrical.
 show the similar relation between
triangles and other polygon.
 construct the triangles and
quadrilateral with given
information.
 identify the different parts of
circle and show the properties
of circles theoretical as well as
experimentally.
 solve the logical problems of
circle.
Teaching Materials:
Geometrical instruments, pencils,
compass, setsquare scale, etc.,
board marker, scissors, thread, solid
objects, different programs related in
computer application, chart paper, etc.
5.1 Triangle A
We have already learnt about the triangle in previous classes. Let’s
make simple revision. In the triangle alongside c b

 Is it a closed figure?
B a C
 How many line segments does it have? What are they?
 How many angles does it have?
 How many intersecting points it has, which are they?
A closed figure formed by three straight lines is called a triangle. A triangle is denoted by a Greek
letter . These straight lines are sides and the points of intersection are called vertices of the triangle.
In the above figure A, B and C are three vertices and AB, BC and CA are the three sides of the
triangle. A, B and C are the three angles of the ABC. The length of the sides of triangle
opposite to each vertex is also represented by small letter of the corresponding vertex, as BC = a,
AC = b and AB = c.

Types of Triangle
According to the measurement of their sides and their angles, the triangles are classified into different
types.
(a) According to sides A

(i) Scalene triangle


If all the sides of a triangle are unequal, then it is called a
B C
scalene triangle.
In the adjoining figure, AB  BC  AC, also A  B  C.
A
Thus, ABC is a scalene triangle.
(ii) Isosceles triangle
If any two sides of a triangle are equal, then it is called an isosceles triangle.
In the adjoining figure,
B C
AB = AC and then the angles opposite to them are also equal i.e. B = C.
Thus, ABC is an isosceles triangle.
(iii) Equilateral triangle
A
If all the sides of a triangle are equal, then it is called an
equilateral triangle. In the adjoining figure, AB = BC = AC. And
each angle of an equilateral triangle is always 60o.
i.e. A = B = C.
B C

168 | Mathematics - 9 Geometry


(b) According to angles
(i) Right angled triangle A
If one angle of a triangle is a right angle, then it is called a right
angled triangle. In the adjoining figure, B is a right angle. So,
it is a right angled triangle.
In a right angled triangle, the remaining angles are acute angles
and the sum of two remaining angles is 90o. The side opposite to
right angle is called the hypotenuse, which is longest side than B C
other two sides of the right angled triangle.
A
(ii) Acute angled triangle
If all the angles of a triangle are acute angle, then it is called an
acute angled triangle. In the adjoining figure, A, B and C
are all acute angles. (each angle less than 90o)
B C
(iii) Obtuse angled triangle A

If one angle of a triangle is more than 90°, then it is called an


obtuse angled triangle.
In the adjoining figure, B is obtuse angle (more than 90o) and B C
the sum of other two angles of obtuse angled triangle is acute
angles. i.e. (A + C) < 90o

Properties of Triangles
A triangle holds some important properties which have already been discussed in previous classes.
Here, some of the important properties are verified experimentally as well as theoretically.

Theorem - 1
The sum of the angles of any triangle is equal to two right angles (180o). Prove
Given: ABC, ACB and BAC are D A E

three interior angles of ABC.


To prove: ABC + ACB + BAC = 180o
Construction: Through the vertex A, draw a line DAE parallel to BC. B C
Proof
S.N Statements S.N Reasons
1. ABC = BAD 1. Being alternate angles, since DE//BC
2. ACB = CAE 2. Being alternate angles, since DE//BC
3. DAE = 180o 3. Being straight angle.
4. BAD + BAC + CAE = DAE 4. Whole part axiom
5. BAD + BAC + CAE = 180o 5. From statements (3) and (4)
6. ABC + BAC + ACB = 180o 6. From statements (1), (2) and (5)
Proved

Geometry Mathematics - 9 |169


Theorem - 2
The exterior angle of a triangle is equal to the sum of the two opposite interior angles. Prove
Given: ABC is a triangle in which the side BC is produced to N where ACN is an exterior
angle of ABC.
To prove: ACN = ABC + BAC
Construction: Through C, draw CD//BA.
Proof
S.N Statements S.N Reasons
1. ABC = DCN 1. Being corresponding angles, since
BA//CD
2. BAC = ACD 2. Being alternate angles, since BA//CD
3. ABC + BAC = DCN + ACD 3. From statements (1) and (2)
4. ACD + DCN = ACN 4. Whole part axiom
5. ABC + BAC = ACN 5. From statements (3) and (4)
Proved
Alternate Method

Proof:
S.N Statements S.N Reasons
1. ABC + BAC + ACB = 180 1. Sum of all interior angles of a triangle
is 180.
2. ACB + CAN = 180 2. Being supplementary angles.
3. ABC + BAC + ACB = ACB + 3. From statements (1) and (2)
ACN
4. ABC + BAC = CAN 4. Cancelling ACB from both sides in
St. (3)
Proved.

Worked Out Examples


Example 1: In the given figure, AB//CD, BCD = 38o and
CAE = 104o. Find the measure of ACB.
E A B
Solution: Here,
ABC = BCD = 38o [Alternate angles are
equal as AB//CD.]
ACB = CAE - ABC [Exterior angle relation
with its opposite interior angles]
ACB = 104o – 38o = 66o C D

170 | Mathematics - 9 Geometry


P
Example 2: In the given figure PRQ = 2xo, QPR = 4xo
4xo
and PQR = 3xo. Find the QPR and PQS.
Solution: Here,
In PQR, PRQ + PQR + QPR = 180o 3xo 2xo
S Q R
[Sum of interior angles of a triangle]
o
or, 2x + 3x + 4x = 180
or, 9x = 180o
180o
 x = 9 = 20o

 QPR = 4 × 20o = 80o


Now, PQS + PQR = 180o [Linear pair]
or, PQS = 180o - 3 × 20o
= 180° - 60° = 120o
A
Example 3: In the figure alongside, OB and OC are angle
bisectors of ABC and ACB respectively. If 68o
o
BAC = 68 , find the size of BOC.
Solution: Here,
O
In  ABC,
BAC + ABC + ACB = 180o x y
x y
[Sum of interior angles of a triangle.] B C
or, 68o + 2x + 2y = 180o
or, 2x + 2y = 180o - 68o
or, 2(x + y) = 112o
 x + y = 56o
Again, In  OBC,
OBC + OCB + BOC = 180o [Sum of interior angles of a triangle]
or, x + y + BOC = 180o
or, BOC = 180o - 56o [ x + y = 56o]
 BOC = 124o.

Exercise 5.1
1. Find the value of x, y and z from the given figures.
(a) A (b) P (c) D
96o 5x B 102o
o
112 E
3x 36o C
B C 3x 136o x
Q R S A

Geometry Mathematics - 9 |171


(d) A (e) A (f) A E
x 72
o
o
82
D
104o
x
x y x y z 44o
54
o
4x B C B C D
B C D

(g) A (h) A (i)


A
x 2x yx
o
36

5x y 2x z 42o
D B C 3x y B D C
B D C

2. Find the value of x, y and z in degree.


(a) A P B (b) P
42o A R B

Q x x S

36o C T D
C R D Q

(c) (d) T
A B
o P A Q
44 x

D E
o x 128o y z
18 R B C S
C

3. (a) In the adjoining figure, AD and BD are angle bisectors A


of BAC and ABC respectively. If ACB = 74o, find
the ADB.

B C
P

(b) In the given figure, PS and RS are angle bisectors of o

QPR and PRQ respectively. If PSR = 108o, find S 108

the size of PQR. Q R

172 | Mathematics - 9 Geometry


A P B
(c) In the given figure, AB//CD, 2QPR = BPQ and
2PRQ = DRQ. Find the size of PQR.
Q
C R D
E
A 108o

(d) In the figure alongside, if CAE = 108o, find the size of


2y C
BCD and ADC. y x
D
2x
B P
A
4. (a) In the given figure, AC and BC are angle bisectors of
C
BAP and ABR respectively to meet at C. Prove that:
1
ACB = 2 (180o - Q). Q B R

A P B
(b) In the adjoining figure, APQ = RPQ and
CRQ = PRQ. Prove that: ABC = 2 (90o - PQR)
Q R

C
A

(c) In the given figure, bisectors of BAC and ABC meet


1 D
at D. Prove that: ADB = 2 (180o + C)

B C

A
(d) In the given figure, ABE = CBE, ACB = ACE,
E
and 2ABF = ACB. Prove that: BEC = BAC - F
ABF.
B C D

Theorem - 3
The sum of any two sides of a triangle is greater than the third sides. Verify experimentally.
Experimental verification
Experiment: Draw three triangles ABC with different shapes and sizes.
A A A

B C B C B C
(i) (ii) (iii)
To Verify: AB+AC>BC, AB+BC>AC, AC+BC>AB

Geometry Mathematics - 9 |173


Verification: Measure the length of sides AB, BC, and CA is each figure and tabulate.
Fig AB BC CA AB + BC AB + CA BC + CA Result
(i) AB+AC>BC, AB+BC>AC, AC+BC>AB
(ii) AB+AC>BC, AB+BC>AC, AC+BC>AB
(iii) AB+AC>BC, AB+BC>AC, AC+BC>AB
Conclusion: The above experiment shows that the sum of any two sides of a triangle is greater
than third side.

Theorem - 4
The angle opposite to the longer side is greater than the angle opposite to the shorter side of any
triangle. Verify experimentally.
Experimental verification
Experiment: Draw three triangles ABC of different shapes and size in which AB is the longest
and AC is the shortest side.
A A
C

B C C B B A
(i) (ii) (iii)
To Verify: ACB > ABC
Verification: Measure the sides AB and AC. Similarly measure the angles opposite to AB and AC
that is ACB and ABC respectively in each figure and tabulate.
AB longest AC shortest ACB angle opposite ABC angle opposite
Fig Result
side side to longest side to shortest side
(i) ACB > ABC
(ii) ACB > ABC
(iii) ACB > ABC
Conclusion: The above experiment shows that the angle opposite to the longer side is greater
than the angle opposite to the shorter side of any triangle.

Converse of theorem - 4
The side opposite to biggest angle is longer than the side opposite to smallest angle of any triangle.
Verify experimentally.
Experimental verification
Experiment: Draw three triangles ABC of different shapes and sizes in which, C is the biggest
and B is the smallest.

174 | Mathematics - 9 Geometry


(i) (ii) (iii)

To Verify: AB > AC
Verification: Measure the biggest C and smallest B. Similarly measure the sides AB and AC
opposite to C and B respectively in each figure and tabulate.
ACB ABC AB (side opposite to AC (side opposite to
Fig Result
(biggest angle) (smallest angle) biggest angle) smallest angle)
(i) AB > AC
(ii) AB > AC
(iii) AB > AC
Conclusion: The above experiment shows that the side opposite to biggest angle is longer than
the side opposite to smallest angle of any triangle.

Theorem - 5
Of all straight line segments drawn to a given straight line from a given point outside it, the
perpendicular is the least. Verify experimentally.
Experimental verification
Experiment: Draw three different line segments PA, PB and PC to XY from the point P and also
draw PM  XY in each figure.
P P P

X A B M C Y XA M B C Y XA B M C Y

(i) (ii) (iii)


To verify: PM < AP, PM < BP, PM < CP
Verification: Measure the length of the line segment PA, PB, PC and PM in each figure and
tabulate.

Length of the line segment


Fig. Result
PA PB PC PM
(i) Perpendicular PM is shorter than other.
(ii) Perpendicular PM is shorter than other.
(iii) Perpendicular PM is shorter than other.
Conclusion: The above experiment shows that of all straight line segments drawn to a given
straight line form a given point outside it, the perpendicular is the least.

Geometry Mathematics - 9 |175


Converse of theorem - 5
Of all the straight line segments drawn to a given straight line from a given point outside of it, the
shortest one is perpendicular to the given line. Verify experimentally.
Experimental verification
Experiment: Draw three different line segment PA, PB and PC to XY from the point P and also
draw shortest line segment PM.
P P P

X A B M C Y XA M B C Y XA B M C Y
(i) (ii) (iii)
To verify: PM  XY
Verification: Measure the length of each line segments and angle made by them to XY and
tabulate.
Measurement of length of line segments and angle made by them.
Fig. PA PAY PB PBY PC PCX PM PMX PMY Result
Angle made by
PM to XY is 90o
Angle made by
PM to XY is 90o
Angle made by
PM to XY is 90o
Conclusion: The above experiment shows that of all the straight line segments drawn to a given
straight line from a given point outside of it, the shortest one is perpendicular to the
given line.

Theorem - 6
Base angles of an isosceles triangle are equal. Prove.
Or
If any two sides of a triangle are equal, the angles opposite to them are equal. Prove.
Given:  ABC is an isosceles triangle in which AB = AC.
To prove: ABC = ACB
Construction: From the vertex A, draw AD  BC.
Proof
S.N Statements S.N Reasons
1. In ABD and ACD 1.
(i) ADB = ADC (R) (i) Being AD  BC
(ii) AB = AC (H) (ii) Given (AB = AC)
(iii) AD = AD (S) (iii) Being common side
 ABD  ACD By R.H.S

176 | Mathematics - 9 Geometry


2. 2. Bing corresponding angles of
ABD = ACD
congruent triangles.
3.  ABC = ACB 3. From (2)
Proved.

Converse of theorem - 6
If two angles of a triangle are equal, the sides opposite to them are also equal. Prove.
Given: In ABC, ABC = ACB
To prove: AB = AC
Construction: From the vertex A, draw AM  BC
Proof
S.N Statements S.N Reasons
1. In ABM and ACM 1.
(i) ABM = ACM (A) (i) Given ABC = ACB
(ii) AMB = AMC (A) (ii) Being AM  BC (by construction)
(iii) AM = AM (S) (iii) Being common side
 ABC  ACB By A.A.S
2. 2. Corresponding sides of the congruent
 AB = AC
triangles
Proved.

Theorem - 7
The bisector of the vertical angle of an isosceles triangle is perpendicular
bisector of the base. Prove.
Given: In ABC, (i) AB = AC (ii) BAD = CAD
To prove: (i) BD = CD (ii) AD  BC
Proof
S.N Statements S.N Reasons
1. In ABD and ACD 1.
(i) AB = AC (S) (i) Given
(ii) BAD = CAD (A) (ii) Given
(iii) AD = AD (S) (iii) Common side of both triangles.
 ABD  ACD By S.A.S axiom
2.(i) 2.(i) Corresponding sides of the congruent
 BD = CD
triangles
(ii) (ii) Corresponding angles of the congruent
ADB = ADC
triangles
(iii) (iii) Adjacent angles of the linear pair being
AD  BC
equal i.e. ADB = ADC
Proved.

Geometry Mathematics - 9 |177


Theorem - 8
The line segment that joins the vertex and mid-point of the base of an isosceles triangle is
perpendicular to the base and bisects the vertical angle. Prove.
Given: (i) ABC is and isosceles triangle in which AB =AC.
(ii) M is a mid-point of BC or, BM = CM
To prove: (i) BAM = CAM (ii) AM  BC (AMB = AMC)
Proof
S.N Statements S.N Reasons
1. In ABM and ACM 1.
(i) AB = AC (S) (i) Sides of an isosceles triangle.
(ii) AM = AM (S) (ii) Being common side
(iii) BD = CD (S) (iii) M is mid-point of BC i.e. BM = CM
(Given)
 ABM  ACM By S.S.S
2.(i)  BAM = CAM 2.(i) Being corresponding angles of congruent
triangles.
(ii) AMB = AMC (ii) Being corresponding angles of
congruent triangles.
(iii) AM  BC (iii) Adjacent angles of the linear pair being
equal
i.e. AMB = AMC.
Proved.
A

Worked Out Examples x

Example 1: From the given figure, find the value of x.


32o
Solution: Here, B C D
o
CAD = ADC = 32 [Base angles of the isosceles triangle ADC.]
ACB = CAD + ADC [Exterior angle is equal to the sum of two opposite interior
angles.]
= 32o + 32o
= 64o
ABC = ACB = 64o [ AB = AC in  ABC]
BAC = x = 180o – (ABC + ACB) [ Sum of interior angles of a triangle]
 x = 180o – (64o + 64o) = 180o – 128o = 52o

Example 2: In the given figure, QR = PR = PS. If TPS = 8 4o, find the size of RPS.
Solution: Here, T
P
Let PQR = xo 84o
 PQR = QPR = x [ QR = PR]
PRS = PQR + QPR
= x + x = 2x
Q R S
178 | Mathematics - 9 Geometry
Again,
PRS = PSR = 2x [ PR = PS]
Now, PQS + PSQ = SPT Exterior angle is equal to the sum of two
or, x + 2x = 84o opposite interior angles.
or, 3x = 84o
 x = 28o
Now, RPS = 180o – (PRS + PSR) [ Sum of interior angles of a triangle]
= 180o – 4x
= 180o – 4 × 28o
= 180o – 112o
= 68o
Example 3: In the adjoining figure, PQR is an isosceles P
triangle in which PQ = PR and QT = RS. Prove that
PST is also an isosceles triangle.
Solution: Here,
Given: In PQR,
PQ = PR, S and T are two points on QR such that QT = SR. Q S T R
To prove: PST is an isosceles triangle.
Proof
S.N Statements S.N Reasons
1.(i) QT = SR 1.(i) Given
(ii) QT - ST = SR - ST (ii) Subtracting common ST on both sides
(iii) QS = RT (iii) Remaining facts from statement (ii)
2. In PQS and PRT 2.
(i) PQ = PR (S) (i) Given
(ii) PQS = PRT (A) (ii) Base angles of an isosceles triangle.
(iii) QS = RT (S) (iii) From statement 1 (iii)
PQS  PRT By S.A.S axiom
Corresponding sides of the congruent
(iv) PS = PT (iv)
triangles.
3. PST is an isosceles triangle 3. Being PS = PT
Proved.
A
Example 4: In the adjoining figure, ABC is an isosceles triangle in
which AB = AC. BO and CO are angle bisectors of
ABC and ACB respectively. Prove that: AO is an
angle bisector of BAC. O

Solution:
Given: (i) ABC is an isosceles triangle in which AB = AC. B C

(ii) BO and CO are angle bisectors of ABC and ACB respectively.


(iii) A and O are joined.

Geometry Mathematics - 9 |179


To prove: OA is an angle bisector of BAC i.e. OAB = OAC.
Proof
S.N Statements S.N Reasons
1.(i) ABC = ACB 1.(i) Base angle of an isosceles triangle.
OB and OC are angle bisectors of ABC
(ii) OBC = OCB (ii)
and ACB respectively.
(iii) OB = OC (iii) Base angles are equal of OBC.
2. In OAB and OAC 2.
(i) AB = AC (S) (i) Given
(ii) OB = OC (S) (ii) From statement 1 (iii)
(iii) AO = AO (S) (iii) Common side.
OAB  OAC By S.S.S
Corresponding angles of congruent
(iv) OAB = OAC (iv)
triangles.
(v) OA is an angle bisector of BAC (v) From statement 2 (iv)
Proved.

Exercise 5.2
Group 'A'
1. Find the value of x, y and a from the following figures.
(a) A (b) P (c) A
x x
44 y
o

x 34o
B D C Q S R B D C

(d) A (e) A (f) A


o
72 2a a
D o
D 56 E
2x
x y a x
B C B D C B C

(g) (h) (i)


A A
E 87o
x
D 50o
y G
o
42 x 158o B C D
B F C H

180 | Mathematics - 9 Geometry


2. Find the values of x, a and y from the following figures. A
(a) E (b) AC (c)
A x a 24o x
102o
xy
D
2a y
y 34o
B C D B D E C
E B C

(d) A (e) P (f) A

45° y xy
o o
X y Y 35 75 R O
S Q
2x x T 112o
x
B C B C

E
A
3. (a) In the given figure, AD = BD and AC = CD. If BAD =
36o, find the size of CAE.

B D C
A

(b) In the adjoining figure, ABC is a right angled triangle. If


D
AD = CD and BD = BC, find ACB.

B C

A D

(c) In the figure alongside, ABCD is a square in which BD is


a diagonal. If AED = 52o, find APB. P E

B C

(d) In the given figure, ABCD is rhombus where DE  BE. If A D

CAD = 70o, find CDE.

B C E

A
Group 'B'
1. In the given figure, ABC is an isosceles triangle in which
X Y
AB = AC. If CX  AB and BY  AC, prove that CX = BY and
AX = AY.
B C

Geometry Mathematics - 9 |181


P
2. In the adjoining figure, RMPQ, QNPR and QN = RM. Prove
that PQ = PR and PM = PN. M N

Q R

3. In the given figure, AC = BD and AD = BC. Prove that AP = BP D


A
and PCD = PDC.

B A
4. In the isosceles triangle ABC, AC = BC. If BP and AP are angle C
bisector of ABC and BAC respectively, prove that CP is also
an angle bisector of ACB. P

B C
A

5. In the given figure, BD = CD = AD. Prove that BAC = 90o.

B D C

6. In isosceles ABC, AB = AC. CD and BE are angle bisectors of


ACB and ABC respectively. Prove that CD = BE. D E

B C
A

7. In the adjoining figure, M is mid-point of BC. If MD  AB, ME


D E
 AC and MD = ME, prove that AB = AC.
B M C

A
8. In the given figure, ABC and ADE are equilateral triangles. If
AC = AE, prove that AX = AY. B
X Y
D

C E

9. In the given figure, ABC and ADE are equilateral triangles. A


Prove that BE = CD. D
B

C E

182 | Mathematics - 9 Geometry


10. In the adjoining figure, A is the mid-point of DE, BAD = CAE and D A E
ADC = AEB. Prove that BCD = CBE.

B C
11. If the bisector of the vertical angle of a triangle bisects the base,
prove that the triangle is an isosceles triangle. A

12. In the adjoining figure, AB = AC, EF//AD and EF = CD. Prove E


that EG = DG.
C
B F G
D
5.2 Parallelogram A

Quadrilateral
A quadrilateral is a closed figure bounded by four line segments in a D
B
plane. In the adjoining figure, ABCD is a quadrilateral where AB, BC,
CD and AD are sides of the quadrilateral.
C
Trapezium P S

A quadrilateral having a pair of opposite sides parallel to each other is


called trapezium. The pair of parallel sides are called bases of the
trapezium. In the adjoining figure, PQRS is a trapezium in which
PS//QR. PS and QR are bases and PQ and SR are legs of trapezium. Q R

Parallelogram
Parallelogram is a quadrilateral having opposite sides are parallel. In the adjoining figure, DEFG is a
parallelogram in which DE//GF and DG//EF.

Properties of parallelogram D G
 Opposite sides of the parallelogram are equal.
 Opposite angles of the parallelogram are equal.
 Diagonals of the parallelogram bisect each other. E F
 The triangles formed by diagonals are equal in area.

Rectangle
A D
Rectangle is a parallelogram having each angle right angle. Or it is a
quadrilateral where all angles are right angles. In the adjoining figure,
ABCD is a rectangle.

B C

Geometry Mathematics - 9 |183


Properties of rectangle
 Opposite sides of a rectangle are equal.
 All angles of a rectangle are right angles.
 Diagonals of the rectangle are equal.
 Diagonals of the rectangle bisect each other.
 The triangles formed by diagonals are equal in area.

Rhombus A D
A rhombus is a parallelogram having all the sides equal. In other words,
a quadrilateral where all sides are equal to each other is called a
rhombus. In the figure alongside, ABCD is a rhombus.
Properties of a rhombus
 All the sides of a rhombus are equal. B C
 Opposite angles of a rhombus are equal,
 Diagonals of a rhombus bisect each other at right angle.
 Diagonals of a rhombus bisect its vertical angles.
 The triangles formed by diagonals are congruent.

Square A D

A square is a rectangle having adjacent sides equal. Or square is a


rhombus having each angle right angle. In the adjoining figure, ABCD
is a square.
B C
Properties of a square
 All sides of a square are equal.
 All angles of a square are right angles.
 Diagonals of a square are equal.
 Diagonals of a square bisect each other at right angle.
 Diagonals of a square bisect its vertical angles.
 The triangles formed by diagonals are congruent.

Theorem - 9
The straight lines joining the end points of two equal and parallel straight lines segments towards
the same sides are also equal and parallel. Prove
Given: AB = CD, AB // CD and the ends
A B
points A, C and B, D are joined.
To prove: AC = BD and AC // BD.
Construction: Join B and C.
C D

184 | Mathematics - 9 Geometry


Proof
S.N Statements S.N Reasons
1. In ABC and BCD
(i) AB = CD (S) (i) Given
(ii) ABC = BCD (A) (ii) Being alternate angles as AB//CD
(iii) BC = BC (S) (iii) Common side
 ABC  BCD By S.A.S axiom
 AC = BD Corresponding sides of congruent
2.(i) 2.(i)
triangles.
(ii) ACB = DBC (ii)
Corresponding angles of congruent
triangles.
AC//BD Alternate angles ACB and DBC being
(iii) (iii)
equal.
Proved.

Theorem - 10
The line segments joining the ends of two equal and parallel line segments towards the opposite
sides bisect each other. Prove. A C
Given: (i) AB = CD and AB//CD.
(ii) The end points A, D and B, C are joined O
which intersect each other at O.
B D
To prove: AD and BC bisect each other at O i.e. AO
= DO and BO = CO.
Proof
S.N Statements S.N Reasons
1. In AOB and  COD. 1.
(i) ABO = DCO (A) (i) Being alternate angle as AB//CD
(ii) AB = CD (S) (ii) Given
(iii) BAO = CDO (A) (iii) Being alternate angles as AB//CD.
 AOB  COD By A.S.A.
Corresponding sides of congruent
2. AO = DO and BO = CO 2.
triangles.
Proved.

Theorem - 11
Opposite sides of a parallelogram are equal. Prove it.
Given: PQRS is a parallelogram i.e. PS//QR, PQ//SR.
To prove: PQ = SR
QR = PS
Construction: Join P and R.

Geometry Mathematics - 9 |185


Proof
S.N Statements S.N Reasons
1. In PQR and PSR 1.
(i) QPR = PRS (A) (i) Being alternate angles as PQ//SR
(ii) PR = PR (S) (ii) Common side.
(iii) PRQ = SPR (A) (iii) Being alternate angles as PS//QR
 PQR  PSR By A.S.A.
Corresponding sides of the congruent
2. PQ = SR and QR = PS 2.
triangles.
Proved.

Converse of the theorem - 11


If the opposite sides of the quadrilateral are equal, the quadrilateral is a parallelogram. Prove.
Given: ABCD is a quadrilateral in which, AB = DC, AD = BC. A D
To prove: ABCD is a parallelogram (i.e. AB//DC and BC//AD)
Construction: Join A and C.

Proof B C
S.N Statements S.N Reasons
1. In ABC and ADC 1.
(i) AB = DC (S) (i) Given
(ii) BC = AD (S) (ii) given
(iii) AC = AC (S) (iii) Common side.
ABC  ADC By S.S.S.
Corresponding angles of the congruent
2. (i) BAC = ACD 2. (i)
triangles.
Alternate angles being equal as BAC
(ii) AB // DC (ii)
= ACD.
BC = AD Since, AB = DC
(iii) (iii)
BC // AD AB //DC
3.  ABCD is a parallelogram 3. Opposite sides being parallel.
Proved.

Theorem - 12
The opposite angles of a parallelogram are equal. Prove.
Given: PQRS is a parallelogram where PQ // SR, PS // QR
To prove: P = R and Q = S.

186 | Mathematics - 9 Geometry


Proof
S.N Statements S.N Reasons
1.(i) P + Q = 180o 1.(i) Sum of co-interior angles as PS//QR
(ii) P + S = 180o (ii) Sum of co-interior angles as PQ//SR
(iii) P + Q = P + S (iii) From statement 1 (i) and (ii).
(iv) Q = S (iv) From statement 1 (iii).
2. Similarly, P = R 2. Same as above statements and reasons.
Proved.
Converse of theorem - 12
If the opposite angles of the quadrilateral are equal, the quadrilateral is a parallelogram. Prove.
A D
Given: ABCD is a quadrilateral where A = C and B = D.
To prove: ABCD is a parallelogram (AB//DC, AD//BC)

Proof B C
S.N Statements S.N Reasons
1.(i) A + B + C + D = 360o 1.(i) Sum of the angles of a quadrilateral.
Being B = D and C = A
(ii) A + D + A + D = 360o (ii)
(given).
o
(iii) 2A + 2D = 360 (iii) From statement 1 (ii).
(iv) A + D = 180o (iv) Dividing both sides by 2.
Being A + D = 180o or sum of co-
(v) AB//DC (v)
interior angles is 180o.
o
2.(i) Similarly, A + B = 180 2.(i) Same as above statements and reasons.
Being A + B = 180o or, sum of co-
(ii) AD//BC (ii)
interior angles is 180o
3. ABCD is a parallelogram 3. Opposite sides being parallel.
Proved.
Theorem - 13
Diagonals of a parallelogram bisect each other. Prove.
P S
Theoretical proof
Given: PQRS is a parallelogram in which diagonals PR and
QS intersect at O. O
To prove: PO = RO and QO = SO.
Proof Q R
S.N Statements S.N Reasons
1. In POQ and ROS 1.
(i) PQ = RS (S) (i) Opposite sides of the parallelogram.
(ii) QPO = SRO (A) (ii) Being alternate angles as PQ//SR.
(iii) POQ = ROS (A) (iii) Vertically opposite angles are equal.
POQ  ROS By S.A.A.
2. PO = RO and QO = SO 2. Corresponding sides of the congruent
triangles.
Proved.

Geometry Mathematics - 9 |187


Converse of theorem - 13
If the diagonals of the quadrilateral bisect each other, the quadrilateral is a parallelogram. Prove.
A D
Given: ABCD is a quadrilateral in which diagonals AC and
BD are bisect each other at O. (i.e. AO = CO and BO
= DO)
O
To prove: ABCD is a parallelogram i.e. AB//DC, AD//BC.
B C
Proof
S.N Statements S.N Reasons
1. In AOB and COD 1.
(i) AO = CO (S) (i) Given
(ii) AOB = COD (A) (ii) Vertically opposite angles.
(iii) BO = DO (S) (iii) Given
AOB  OCD By S.A.S. axiom
Corresponding angles of the congruent
2. (i) OAB = OCD 2. (i)
triangles.
Being alternate angles, since
(ii) AB//DC (ii)
OAB=OCD.
Corresponding sides of the congruent
(iii) AB = DC (iii)
triangles.
3. AD = BC, AD//BC 3. Being AB = DC and AB//DC.
Opposite sides being equal and
4. ABCD is a parallelogram 4.
parallel.
Proved.

Worked Out Examples


A D
Example 1: In the given figure, ABCD is a
parallelogram. DCE is isosceles
triangle in which DC = DE. If CDE =
o
28 , find the size of BAD and ADE.
Solution: Here, B C E
o
DCE + DEC + CDE = 180 [Sum of angles of a triangle]
o o
or, 2DCE + 28 = 180 [DCE = DEC]
o o o
180 - 28 152
or, DCE = = 2 
2  
 DCE = 76o
or DCE = DEC = 76o
or DCE = 180° - DEC
or ADE = 180o – 76o [DEC = 76o]
o
= 104

188 | Mathematics - 9 Geometry


Now,
ADC = DCE = 76o [Alternate angles]
o
or, BAD + ADC = 180 [Co-interior angles]
o o
or, BAD = 180 – 76
= 104o
A D

Example 2: In the adjoining figure, ABCD is a rhombus.


Diagonals AC and BD intersect at O. AC is O
o
produced to E where CD = CE. If CDE = 26 ,
find OAD and ODE. B C
Solution: Here,
CDE = CED = 26o [Base angles of an isosceles triangle] E

sOCD = CDE + CED


= 26o + 26o Exterior angle is equal to the sum of two
o opposite interior angles.
= 52
 OAD = OCD = 52o [Being AD = CD]
And DOE = 90o [Diagonals of the rhombus bisect at right angle.]
ODE = 90o – OED [Remaining angle of right angled triangle.]
o o
= 90 – 26
= 64o
Example 3: In the adjoining figure, AB = CD,
AB//CD, 2BE = AE, 2CF = DF. Prove
that BECF is a parallelogram.
A E B
Solution:
Given: (i) AB = CD, AB//CD, 2BE = AE and 2CF = DF
To prove: BECF is a parallelogram C F D
Proof
S.N Statements S.N Reasons
1.(i) AB = CD 1.(i) Given
(ii) AE + BE = CF + DF (ii) Whole part axiom
(iii) 2BE + BE = CF + 2CF (iii) Given [AE = 2BE and DF = 2CF]
(iv) 3BE = 3CF (iv) Whole part axiom from statement 1
(iii).
(v) BE = CF (v) Division axiom from statement 1(iv).
(vi) BE//CF (vi) Being AB//CD
(vii) EC = BF, EC//BF (vii) BE = CF and BE//CF.
2.  BECF is a parallelogram 2. Opposite sides being equal and
parallel.
Proved.

Geometry Mathematics - 9 |189


Exercise 5.3
Group 'A'
1. Find the value of x & y from the following figures.
(a) A D (b) A D E (c) A D
y y y

x 74o x x
B B
108 o C C B C E
E

(d) (e) (f)


A D E A D A D
y y 30o o
81

x x
x y
B C B C E B C

(g) A D (h) A
(i) A D
x 72o
D 52
o
G E R y
E yo P x
124 x
y B E 32o C
B C F
B C

(j) A E D (k) A B (l) P S T


y x
F x Q 128
o
34o
y
32o P Q R
x
B C D C
A D

2. (a) In the adjoining figure, ABCD is a square. If DAE = 34o, find AEF.
E

B F C
A E D
(b) In the given figure, ABCD is a rectangle. E is any point
on AD such that AB = AE = DE. Find BEC.

B C
A D
(c) In the given figure, ABCD is a parallelogram in which
AF is an angle bisector of BAD. If EFC = 58o, find
B E C
ABE and AEC.
F
190 | Mathematics - 9 Geometry
A D
(d) In the given figure, ABCD is a rhombus and CDE is
an isosceles triangle in which CD = DE. If CDE = 42o,
find the size of BDC.
B C E

A D
3. (a) In the adjoining figure, ABCD is a square and AEC is
an equilateral triangle. Find the size of BCE.
B C
E
A
(b) In the figure alongside, ABC is an isosceles triangle in
which AB = AC and DBCF is a parallelogram. If ECF D E F
= 44o, find AED.
B C

A D
(c) In the adjoining figure, ABCD is a square and PBC is
an equilateral triangle. Find the size of APD. P

B C

A D
(d) In the adjoining figure, ABCD and PQRS are two
squares. If PBQ = 28o, find the size of BRS. P

C S
Group 'B' B Q

1. Prove the following statements. R


(a) If a parallelogram has equal diagonals, then it is a rectangle.
(b) If the diagonals of a parallelogram are at a right angle, the parallelogram is a rhombus.
(c) If the diagonals of a rhombus are equal, the rhombus is a square.
(d) If one angle of a rhombus is a right angle, the rhombus is a square.
(e) If one angle of a parallelogram is a right angle, the parallelogram is a rectangle. A

D
2. In the given sABC and DEF, AB = DE, AB//DE, BC = EF,
BC//EF. Prove that AC = DF and AC//DF. B C

E F
P
A D
3. In the figure alongside, ABCD is a parallelogram. The diagonal
AC is produced to either side to the points P and Q such that
AP = CQ. Prove that BP = DQ and BP // DQ.
B C
Q
Geometry Mathematics - 9 |191
P S
4. In the adjoining figure, PQRS is a parallelogram. A and B are A
two points on the diagonal PR such that PA = RB. Prove that
AQBS is a parallelogram. B
Q R

A M D
5. In the figure alongside, ABCD is a parallelogram. M and N are
midpoints of AD and BC respectively. MN and BD intersect at O
O. Prove that OM = ON and BO = DO. B N C

A M D
6. In the given figure, ABCD is a parallelogram. BM and DN are
angle bisectors of ABC and ADC respectively. Prove that
BM = DN, BM//DN.
B N C

A B
7. In the figure alongside, ABCD is a square. E and F are any E
points on BC and DC respectively, such that DE = AF. Prove P
that DPAF.
D F C

8. In the figure alongside, ABCD is a square in which P, Q, R and S A S D


are any points on AB, BC, CD and AD respectively. If BP = CQ = R
DR = AS, prove that PQRS is also a square.
P

B Q C

9. In the given figure, ABCD is a parallelogram in which AC is a A D


P
diagonal. BP and DQ are perpendiculars to AC. D, P, and B, Q
are joined. Prove that PBQD is a parallelogram.
Q
B C
A D
10. In the adjoining figure, AM and CN are perpendiculars to N
diagonal BD of the quadrilateral. If AO = CO and BM = DN, M
O

prove that ABCD is a parallelogram.


B C

192 | Mathematics - 9 Geometry


5.3 Mid-Point Theorem
Theorem – 14 (A)
A line segment drawn through the mid-point of one side of a triangle and parallel to another side
bisects the third side. A
Given: In ABC,
M is the mid-point of BC i.e. BM = CM F N
BA//MN
To prove: AN = CN
B M C
Construction: Through N, Draw NF | | BC.
Proof
S.N Statements S.N Reasons
1.(i) BMNF is a parallelogram 1.(i) Opposite sides are parallel
(ii) BM = FN (ii) Opposite sides of the parallelogram
(iii) BM = MC (iii) Given
(iv) FN = MC (iv) From (ii) and (iii) of 1.
2. In AFN and NMC 2.
FAN = MNC (A) Being corresponding angles as
(i) (i)
AB//NM.
(ii) ANF = NCM (A) (ii) Being corresponding angles as FN//BC
(iii) FN = MC (S) (iii) From statement 1. (iv)
AFN  NMC By A.A.S.
AN = CN Corresponding sides of the congruent
(iv) (iv)
triangles.
Proved.

Theorem - 14 (B)
A line segments joining the mid-points of any two sides of a triangle is parallel to the third side and
A
it is equal to half of the length of the third side.
Given: In ABC, M and N are mid-points of sides
AB and AC respectively. M and N are joined. M N D
1
To prove: MN // BC, MN = 2 BC
B C
Construction: Draw a line CD parallel to BA from C and
produce MN to meet CD at D.

Geometry Mathematics - 9 |193


Proof
S.N Statements S.N Reasons
1. In AMN and CDN 1.
(i) MAN = DCN (A) (i) Being alternate angles as, BA // CD
(ii) AN = CN (S) (ii) Given
(iii) ANM = CND (A) (iii) Vertically opposite angles.
AMN  CDN By A.S.A.
Being corresponding sides of the
2.(i) AM = CD 2.(i)
congruent triangles.
(ii) AM = BM (ii) Given
(iii) BM = CD (iii) From statements 2. (i) and (ii)
(iv) BM //CD (iv) By construction
(v) MD = BC and MD//BC (v) Being, BM = CD and BM//CD.
3.(i) BC // MN 3.(i) Same line segment (MD//BC)
1 MN = DN (corresponding sides of the
(ii) MN = 2 MD (ii)
congruent triangles.
1 MD = BC {from statements 2. (v) and
(iii)  MN = 2 BC (iii)
3. (iii)}
Proved.

Worked Out Examples A

Example 1: In the adjoining figure, AD = BD and AE = CE. If AX D E


= 7cm and ABC = 52o, find ADE and length of XY. Y
Solution: Here,
B X C
(i) AD = BD and AE = CE, [Given]
DE//BC [ AD = BD and AE = CE]
o
(ii) ABC = ADE = 52 [Corresponding angles, as DE //BC.]
Again, In ABX
AD = BD [Given]
DY // BX [ DE // BC]
AY = XY
1
Thus, XY = 2 AX

1
 XY = 2 × 7cm = 3.5cm
A 3.2cm D
Example 2: In the figure alongside, ABCD is a trapezium in
which AD//BC. If AM = BM, MN//BC, AD = 3.2cm
M N
and BC = 5.8cm, find the length of MP and NP. P

B 5.8cm C

194 | Mathematics - 9 Geometry


Solution: Here,
1. In ABC,
AM = BM, MP//BC [Given]
 AP = CP [ AM = BM and MP//BC]
1
MP = 2 BC [AM = BM and AP = CP]

1
= 2 × 5.8cm = 2.9cm

2. In ACD,
i. DN = CN [AP = CP and AD//PN]
1
ii. PN = AD [AP = CP and DN = CN]
2
1
= 2 × 3.2cm = 1.6cm

Example 3: In the adjoining figure, P, Q, R and S are mid-points of AB, BC, CD and
AD respectively. Prove that PQRS is a parallelogram.
A
Solution:
Given: P, Q, R and S are mid-points of AB, BC, CD and AD
respectively, which are joined in order. P S
To prove: PQRS is a parallelogram C
Construction: Join A and C. Q R
Proof B D

S.N Statements S.N Reasons


1. In ABC 1.
i. BP = AP, BQ = CQ i. Given
1 Being BP = AP and BQ = CQ [from
ii. PQ //AC, PQ = 2 AC ii.
(i)]
2. In ADC, 2.
i. DS = AS, DR = CR i. Given
1 Being DS = AS, DR = CR [from (i) of
ii. SR//AC, SR = 2 AC ii.
2]
3.i. PQ = SR, PQ//SR 3.i. From (ii) of 1 and (ii) of 2.
ii. PS = QR, PS//QR ii. PQ = SR, PQ//SR [from (i) of 3]
Opposite sides being equal and
4. PQRS is a parallelogram 4.
parallel.
Proved.

Geometry Mathematics - 9 |195


Example 4: In the adjoining figure, AD is an angle bisector of BAC. BEAD and
1
BF = CF. Prove that EF//AC and EF = (AC – AB).
2
Solution:
Given: In ABC, AD is an angle bisector of BAC. i.e. BAD = CAD.
A
BE  AD; BF = CF; E and F are joined.
1
To prove: EF//AC and EF = 2 (AC – AB)
G
E
Construction: Produce BE to meet AC at G.
Proof B D F C

S.N Statements S.N Reasons


1. In ABE and AGE 1.
(i) BAE = GAE (A) (i) AD being an angle bisector of BAC.
(ii) AE = AE (S) (ii) Common side
(iii) AEB = AEG (A) (iii) Being right angles
(iv)  ABE  AGE (iv) By A.S.A.
BE = GE Corresponding sides of the congruent
(v) (v)
triangles.
2. In BCG 2.
(i) BE = GE (i) From statements 1. (iv)
(ii) BF = CF (ii) Given
1 From statements 2. (i) and (ii)
(iii)  EF // GC and EF = 2 GC (iii)
3.(i) EF//AC 3.(i) Same line segment
1 1
(ii) EF = 2 (AC – AG) (ii) CG = 2 (AC – AG)
1 AB = AG (corresponding sides of
(iii) EF = 2 (AC – AB) (iii)
congruent triangles.
Proved.

Exercise 5.4
Group 'A'
1. Find the value of x & y in the following figures.
(a) A (b) P
o
81 x

Q S 55o Y
Py x Xy

50o 50o
B C Q R

196 | Mathematics - 9 Geometry


(c) B M
(d) A
A
x 42o y x
26o
E o
E 48 F
N y
D

80o 124o
C B C D

2. Find the value of x & y in the following figures.


(a) P (b) A

3.4cm

M T D 2.2cm 2.6cm E
x Q
5.2cm
S y
Q N R B x P y C

(c) A 3.4cm D (d)

y
P Q
x R

B 7.2cm C

3. (a) In the given figure, AD = BD, AE = CE.


Show that AP = PQ.

D G
(b) In the given figure, BD = AD and BF = FG. F
Prove that BE = CE.
B E C

A D
(c) In the given trapezium ABCD, if
AD//BC, AE = CE and DN = CN, prove M N
E
that AM = BM.
B C

Geometry Mathematics - 9 |197


A D

(d) In the adjoining figure, AD//PQ//BC. If P R Q


AP = BP, prove that DQ = CQ.
B C
Group 'B' A H D
1. In the figure alongside, ABCD is a quadrilateral. E, F, G and H
are the mid-points of AB, BC, CD and AD respectively. Prove E G
that EFGH is a parallelogram.
B F
C
A P B

2. In the adjoining figure, P, Q, R and S are the mid-points of AB, S


Q
BC, CD and AD respectively. Prove that PQRS is a
parallelogram.
C R D
H
D

A G C F
3. In the given figure, A, B, C and D are the mid-points of EH,
EF, FG and HG respectively. Prove that AD = BC and AB = B
DC.
E

P
4. In the given figure, PQRS is a quadrilateral in which A, C, B D
and D are the mid-points of PQ, QR, RS and PS respectively. If A
S
AB and CD intersect at O, prove that AO = BO and CO = DO.
Q O
B
C
R

5. In the figure alongside, PQRS is a parallelogram in which M P M Q


and N are the mid points of PQ and SR respectively. PN and B
MR intersect the diagonal QS at A and B respectively. Prove A
that:
(a) PNRM is a parallelogram S N R

(b) AS = AB = BQ. A D

R
6. In the adjoining figure, ABCD is a trapezium where AD // BC. If P Q
AD // PQ and DQ = CQ, prove that:
(a) QR // BC B C

(b) 2PQ = AD + BC.

198 | Mathematics - 9 Geometry


P S

7. In the given figure, PQRS is a trapezium in which PS // QR. If


M N
MN is a median, prove that:
(a) MN // QR
Q R
1
(b) MN = 2 (PS + QR).
D A

1 M N
8. In the given figure, AD//MN//BC and BM = 2AB.

Prove that AD = BC. B C


P

9. In the adjoining figure, PS is an angle bisector of QPR. PS A


M
1
and QA interest at M at right angle. If MN = 2 (PR – PQ), Q R
S N
prove that QN = RN.

A M D
10. In the figure alongside, ABCD is a parallelogram. Median MN
P
and diagonal BD intersect at P. Prove that:
B N C
(a) MBND is a parallelogram
1
(b) PN = 2 CD.

A P B
11. In the given figure, ABCD is a quadrilateral in which P, Q, R
and S are the mid-points of respective parts. Prove that PQRS Q S
is a parallelogram.
D R C
D
12. In the given figure, M is a mid-point of BC. CA is produced to N
D. If BN is perpendicular to AN where AN is an angle bisector A
1
of BAD, prove that MN = 2 (AB + AC).
B M C

13. E is the mid-point of side AD of a parallelogram ABCD. F is A E D


any point of diagonal BD such that 4DF = BD. EF if produced
to meet CD at G. Prove that CG = DG. C
B M
N

Geometry Mathematics - 9 |199


14. In the given figure, ABCD is a parallelogram in which E is A
mid-point of AD and AM//EC. AM and DC and produced to D
meet at N. Prove that: P

(a) 2AB = DN F
B C
(b) 2CE = AN. Q
E

15. In the figure alongside, P is mid-point of AB and DF.


Similarly, Q is mid-point of BC and EF. Prove that AC = DE.

A D
16. In the adjoining figure, ABCD is a trapezium in which
AD//BC. M and N are mid points of diagonals BD and AC
M N
respectively. Prove that:
(a) MN//AD
B C
1 A
(b) MN = 2 (BC – AD).
E
17. In the figure alongside, D is mid-point of BC. If 2CG = AC and F
BA//DE, prove that 3DF = FG. C
B D

G
5.4 Similarity
Look at the figures alongside, these figures are similar
figures. The figures having the same shape but same or (i)
different sizes are called similar figures; as shown in
figures (i), (ii) and (iii).
(iii)
(ii)

Similarly, geometric figures having the same shape but A D


different sizes are similar figures. In the figure
alongside, the triangles, the squares and the circles are
B C
similar to each other separately because these figures E F
(i)
are of same shape but different sizes. A picture and its
photocopy obtained by enlarging or reducing in size are
r1 r2
the similar pictures. X Y

(ii) (iii)

Similar Triangles D
A
In the adjoining figures (i) and (ii), where A = D = 60°, 60°
60°

B = E = 40°, C = F = 80° in figure (i) and both 40° 80°


B C 40° 80°
triangles are equilateral triangles in figure (ii), these pair E F
of two triangles have same shape but different sizes. (i)

200 | Mathematics - 9 Geometry


They are similar triangles. Symbolically, we denote, L R
ABC DEF and LMN  RST.
M N
S T
(ii)

Geometrically, two or more triangles are said to be similar triangle, under the following conditions.
(i) When all angles of one triangle are respectively equal to the A D
corresponding angles of another triangle, the triangles are
said to be similar triangles. In the adjoining triangles,

E F
B C
A = D, B = E and C = F.
 ABC DEF
Note: When the triangles are similar, the corresponding sides are proportional.
P
(ii) When the corresponding sides of two triangles are X


proportional, the triangles are also similar.
PQ QR PR
i.e. XY = YZ = XZ  PQR XYZ. Q R Y Z
Note: When the triangles are similar, the corresponding angles are equal.
(iii) When any two corresponding sides are proportional and the angles included by them are equal,
the triangles are similar. L
R
LM LN
 i.e. RS = RT and MLN = SRT

 LMN RST M N
S T
L
R
OR


LM MN
RS = ST and LMN = RST

S T
M N
 LMN RST
L
OR R


MN LN
ST = RT and LNM = RTS 
S T
M N
 LMN RST
Note:
(i) Two congruent triangles are always similar but two similar triangles are not necessarily to be
congruent.
(ii) According to the present curriculum, these conditions are not necessary to prove theoretically or
practically (Experimentally).

Geometry Mathematics - 9 |201


Similar Polygons
The properties and conditions for similar polygons are as follows. A D
P S
(i) If the angles of one polygon are respectively
equal to the angles of the other polygon, the 
polygons are similar. Q R
B C
For examples: (i)

In figures (i), A = P, B = Q, C = R


P T

D H
and D = S, then, polygon ABCD  polygon
PQRS. G
S E
Q
Similarly, in figure (ii), P =D, Q = E,
F
R = F, S = G and T = H, then R (ii)
pentagon PQRST  pentagon DEFGH.
If the polygons are similar, then their corresponding sides are proportional
AB BC CD AD
i.e. PQ = QR = RS = PS [from (i)]

PQ QR RS ST PT
and DE = EF = FG = GH = DH [from (ii)]

(ii) If the sides of one polygon are proportional to the corresponding sides of the other polygon, the
polygons are similar. In the adjoining figure,
A D
P S
AB BC CD AD
PQ = QR = RS = PS 
Q R
Then, polygon ABCD  polygon PQRS B C

(iii) The corresponding diagonals of the similar (i)


A D
polygons are proportional to their P S
corresponding sides. i.e. polygon ABCD 
polygon PQRS

Q R
AB AC BD B C
Then, PQ = PR = QS = …………

P T
D H
(iv) Similar polygons can be divided into the same
number of similar triangles. Q S
 E G

Then, PQT  DEH, QRS  EFG and F


R
TQS  HEG

202 | Mathematics - 9 Geometry


A F
(v) The areas of similar polygons are proportional to P U
squares of their corresponding sides.
B E
 Q T
For example,
If polygon ABCDEF  polygon PQRSTU
R S
C D
area of polygon ABCDEF AB2 BC2
Then, area of polygon PQRSTU = PQ2 = QR2 = ……

Worked Out Examples


A B
Example 1: In the adjoining figure, AB//CD, AD and BC
intersect at P. If AB = 4.5cm, BC = 4cm and
`PC= 1cm, find the size of CD.
P
Solution:
1. In ABD and CDP
C D
ABP = DCP (A) [Being alternate angles as AB//CD.]
BAP = CDP (A) [Being alternate angles as AB//CD.]
APB = CPD (A) [Remaining angles.]
 BAP  CDP [By A.A.A]
AB BP
2. (i) CD = CP [Corresponding sides of the similar triangles.]

4.5cm BC - CP
or, CD = CP
4.5cm (4 - 1)
or, CD = 1
or, 3CD = 4.5cm
 CD = 1.5cm
A
Example 2: In the given figure, if ABC = CAD, prove that:
(i) ABC  ACD
2
(ii) AC = BC . CD
Solution: B D C
Given: In ABC, AD meets BC at D such that ABC = CAD.
To prove: (i) ABC  ACD
(ii) AC2 = BC . CD

Geometry Mathematics - 9 |203


Proof
S.N Statements S.N Reasons
1. In ABC and ACD 1.
(i) ABC = CAD (A) (i) Given
(ii) ACB = ACD (A) (ii) Common angle
(iii) BAC = ADC (A) (iii) Remaining angles of triangles
 ABC  DAC By A.A.A
2. (i) AC BC 2. (i) Corresponding side of the similar
CD = AC triangles.
(ii) AC2 = BC . CD (ii) From statement 2. (i)
Proved
A D
Example 3: In the given figure, ABCD is a
parallelogram in which E is the mid-point
of BC. Diagonal AC and DE intersect at M. M
Prove that 2CM = AM
Solution: B E C

Given (i) ABCD is a parallelogram


(ii) E is mid-point of BC i.e BE = CE.
(iii) AC and DE are intersecting at M.
To prove: 2CM = AM
Proof
S.N Statements S.N Reasons
1. In ADM and CEM 1.
(i) ADM = CEM (A) (i) Being alternate angles as AD//EC
(ii) MAD = MCE (A) (ii) Being alternate angles as AD//EC
(iii) AMD = CME (A) (iii) Remaining angles of triangles
 ADM  CEM By A.A.A
AD AM Corresponding sides of the similar
2.(i) 2.(i)
CE = CM triangles.
2CE AM
(ii) (ii) AD = BC = 2CE
CE = CM
(iii) 2CM = AM (iii) From statement 2. (iii)
Proved
D
Example 4: In the given figures, polygon ABCD  S
polygon PQRS. If AB = 3cm, PQ = 2cm, PR = C
R
4cm, CD = 2.25cm, SR = 1.5cm, AD = 7.5cm,
find the size of PS.
A B P Q
Solution:
1. ABC  PQR Similar polygons can be divided into same
number of similar triangles.
AB AC
2. PQ = PR [ABC  PQR]

204 | Mathematics - 9 Geometry


3cm AC
or, 2cm = 4cm [Given values]

AC = 6cm.
Similar polygons can be divided into same
3. ACD  PRS
number of similar triangles.
AC AD
PR = PS [ACD  PRS]

6cm 7.5cm
or, 4cm = PS [Given values and AC = 6cm]

30cm
or, PS = 6cm

PS = 5cm.

Exercise 5.5
Group 'A'
1 Find the values of x and y from the following figures.
(a) A (b) P

D y
7.5cm
S T
2cm x
1.5cm
Q 3cm R
B y E 1cm C

(c) D (d) P

y
M

M N
5cm x 6cm
x

E 7.5cm F Q y N 3cm R

2. Find the values of x and y with statements and reasons in the following figures.
(a) A (b) P

S
y x

B x D C
8cm Q y R

Geometry Mathematics - 9 |205


(c) D (d)
A

y x y
x

E 6cm G F B 4cm D 12cm C


9cm
A

3. (a) In the given figure, DE//BC. If BC = 6cm, AE = 2.25cm, y 2.5cm


DE = 4cm and BD = 3cm, find the values of x and y. D E
4cm
3cm x

B 6cm C
A
(b) In the adjoining figure, BAD = ACB. If BC =
9cm and BD = 4cm, find the length of AB.

B D C

A
(c) In the adjoining figure, AB//FC and 2EF = DE. If
CF = 2.5cm, find AD.
D E F

B C

(d) In the given figure, AB//PQ//DC. If AB = 4xcm, A


CD = 3cm, AP = 5cm and CP = 3cm, find PQ. D
P

B Q C

A B

4. (a) In the given figure, AB//CD. If 2DP = AP, AB = 14cm


P
and BP = 8cm, find CP and CD.
C D

(b) In the given figure, BAC = BDE, find the values of E


CD and DE, if AC = 15cm, AB = 10cm, BD = 5cm and
AE = 2cm.
B D C

206 | Mathematics - 9 Geometry


A
(c) In the given figure, ABC is a right angled triangle where
B = 90°, BDAC. If AC = 16cm, CD = 9cm, find the
length of BC.
D

C B

(d) In the adjoining figure, ABCD is a parallelogram. If E A D


3EF = CF and BC = 12cm, find the length of DE.
F

B C

S
D
5. In the given figure, polygons ABCD and PQRS are P
A
similar. If BC = 6cm, CD = 5cm, BD = 8cm, AB = 4cm
AD = 5cm and QR = 9cm. find PS, QS and RS.
B C Q R
Group 'B'
P S
1. In the adjoining figure, PS//QR and PQS = QRS
,prove that:
(a) QS2 = PS.QR
(b) PQ.QR = QS.RS Q R
(c) PQ.QS = PS.RS
2. In the given figure, ABC = CAD, prove that: A
2
(a) AC = BC.AD
(b) AB.AC = BC.BD
BD AB
(iii) CD = AC B D C
P
3. In the adjoining figure, PQR is a right angled triangle,
in which Q = 90° and STPR. Prove that: T
(a) PR.PT = PS.PQ
PR PS S
(b) QR = ST
R Q
PT PQ
(c) = P T Q
ST QR
4. In the figure alongside, PQRS is a parallelogram. If A
2PT = QT, prove that 3AP = AR.
S R
Geometry Mathematics - 9 |207
P
A Q R D
5. In the adjoining figure, ABCD is a parallelogram. Prove
that AD.PQ = PB.QR

B C
6. In the given figure, ABCD is a parallelogram. If 3CF = A D
1
BC, prove that EF = 4 AF.
E

B C F
7. In the adjoining figure, ABC is a right angled triangle
A
in which B = 90° and BDAC. Prove that
(a) ABC  ABD and AB2 = AC.AD
(b) ABC  BCD and BC2 = AC.CD
D
(c) ABD  BCD and AB.CD = BC.BD

C B

8. In the given figure, QPR = PSQ = 90°. Prove that:


(a) PS2 = QS.RS P
PQ PR
(b) QS = PS
PQ PR
(c) PS = RS Q S R

A D

9. In the adjoining figure, AB//PQ//DC. Prove that P


1 1 1
PQ = AB + DC
B Q C

10. In the given figures, BDE = ACB and ABD = A


CBE. Prove that:
D
(a) ABC  BDE
(b) AC.BD = BC.DE E

B C

208 | Mathematics - 9 Geometry


5.5 Pythagoras Theorem A P

In the figures alongside, ABC and PQR are right angled h h
p b
triangles where B and R are right angles. The sides AC
and PQ are opposite to the right angles and are called 
hypotenuses. C b B Q p R
(i) (ii)
In figure (i), the sides AB and BC are the perpendiculars (p) and bases (b) respectively but in figure
(ii) the side PR and QR are the bases (b) and perpendiculars (p) respectively. The side opposite to the
reference angle is called perpendicular and remaining side is base.
Pythagoras was a Greek Mathematician who lived around 500 BC and discovered the significant facts
about the right angled triangles known as Pythagoras Theorem.

Theorem - 15
In a right angled triangle, square of the hypotenuse is equal to the sum of the squares of remaining
two sides. Verify experimentally.
OR
Area of square formed on hypotenuse of the right angled triangle is equal to the sum of the squares
formed on two remaining sides. Verify experimentally.
Experimental Verification
Experiment: Draw three right angled triangles ABC right angled at B in different sizes and shapes
with the help of pencil compasses and ruler.
A

C
B

A
A B B C
(i) (ii) (iii)
To verify: AB2 + BC2 = AC2
Verification: Measure the lengths of AB, BC and AC in each figure and tabulate.
Fig AB AB2 BC BC2 AC AC2 AB2 + BC2 Results
i AB2 + BC2 = AC2
Ii AB2 + BC2 = AC2
iii AB2 + BC2 = AC2
Conclusion: The above experiment shows that in a right angled triangle, square of the hypotenuse
is equal to the sum of squares of two remaining sides.

Geometry Mathematics - 9 |209


B
Theoretical proof
Given: ABC is a right angled triangle in which B = 90°
To prove: AC2 = AB2 + BC2
Construction: Draw BD  AC
Proof A D C
S.N Statements S.N Reasons
1. In ABC and ABD 1.
(i) ABC = ADB (A) (i) Both are right angles (90°)
(ii) BAC = BAD (A) (ii) Common angle
(iii) ACB = ABD (A) (iii) Remaining angles of right angled
triangle.
 ABC  ADB By A.A.A fact.
AB AC Corresponding sides of the similar
(iv) (iv)
AD = AB triangles.
(v) AB2 = AC.AD (v) From statements 1. (iv)
2. (i) Similarly in ABC  BDC 2. (i) Same as above facts and reasons.
BC AC Corresponding sides of the similar
(ii) (ii)
CD = BC triangles.
(iii) BC2 = AC.CD (iii) Form statement 2. (ii)
3.(i) AB2 + BC2 = AC.AD + AC.CD 3. (i) Form statements 1. (v) and 2. (iii)
(ii) AB2 + BC2 = AC(AD + CD) (ii) From statement 3 (i)
(iii) AB2 + BC2 = AC.AC (iii) Whole part axiom.
AC2 = AB2 + BC2
Proved

Pythagorean Triplets (Triples or Triads)


These are the triple of positive integers a, b and c that can represent legs and hypotenuse of a right
angled triangle such that a2+b2=c2. For example 3, 4, 5; 6, 8,10; 5, 12, 13 etc.
Pythagoras himself devised the given formula to calculate the Pythagorean triples
2n+1, 2n(n + 1), 2n(n + 1) + 1 where n = 1,2,3, ……………
a b c Triples
n a2 + b2 = c2
2n + 1 2n(n + 1) 2n(n + 1) + 1 a, b, c
1 2×1+1=3 2 × 1 (1 + 1) = 4 2 × 1(1 + 1) + 1 = 5 3, 4, 5 32 + 4 2 = 52
2 2×2+1=5 2 × 2(2 + 1) = 12 2 × 2(2 + 1) + 1 = 13 5, 12, 13 52 + 122 = 132
3 2×3+1=7 2 × 3(3 + 1) = 24 2 × 3(3 + 1) + 1 =25 7, 24, 25 72 + 242 = 252
n2 - k 2 n2 + k2
To obtain Pythagorean triplets associated to a number we can devise the formula n, 2k , 2k
where k is factors of n not exceeding n, k<n and k is odd or even according as n is odd or even.
 n is any positive integer.
 k is prime factors of n, their powers or products.
 If the factor is only power of 2, say 2p, maximum value of k is 2p-1.

210 | Mathematics - 9 Geometry


 If the factor is power of other primes like 3p, minimum value of k is 30 = 1 and maximum value
is 3p-1.
 In combination (products), power of 2 rises maximum to 2p-1 and power of other prime rise
maximum to 2p.
Triples associated to n
n2 - k2 n2 + k2 2 2 2 2
n k
2k 2k (n, n - k , n + k )
 2k 2k 
3 1 4 5 3,4,5
4 2 3 5 4,3,5
5 1 12 13 5,12,13
6 2 8 10 6, 8, 10
7 1 24 25 7, 24, 25
2 15 17 8, 15, 17
8
4 6 10 8, 6, 10
1 40 41 9, 40, 41
9
3 12 15 9, 12, 15
Example: To find all the possible triplets associated to 24.
n2 - k2 n2 + k2
n k Associated triplets
2k 2k
2 143 145 24, 143, 145
4 70 74 24, 70, 74
6 45 51 24, 45, 51
24 8 32 40 24, 32, 40
12 18 30 24, 18, 30
16 10 26 24, 10, 26
18 7 25 24, 7, 25
Note: Any multiples of a Pythagorean triplet is also a triplet e.g. if 3, 4, 5 is a triplet then 2(3, 4, 5) i.e. 6,
8, 10 is also triplet similarly 3(3, 4, 5) i.e. 9, 12, 15 is also triplet

Worked Out Examples


Example 1: Find the value of x from the given figure.
Solution: In right angled ABC, A x
AC2 = AB2 + BC2 D
2 2
= (3) + (4) = 9 + 16 = 25
AC = 5 cm
Again, In right angled ACD, B 4cm C
2 2 2 2 2
AD = AC + CD = (5) + (12) = 25 + 144 = 169
AD = x = 13cm.

Geometry Mathematics - 9 |211


Example 2: Find the values of x, y and z from the given figure.
Solution: In the right angled QRS. P
2 2 2
QR = QS + SR
S
= x2 + 25 ... (i)
In the right angled triangle PQS. y
x
2 2 2
PQ = PS + QS
or, y2 = x2 + 16 ... (ii)
Q z R
Again, in the right angled triangle PQR
PR2 = PQ2 + QR2
or, 81 = y2 + QR2 ... (iii) [ PR = PS + RS = 4 + 5 = 9]
Now, setting the relations of (i) and (ii) in (iii)
81 = x2 + 16 + x2 + 25
or, 81 = 2x2 + 41
or, 81 - 41 = 2x2
40
or, x2 = 2 = 20

x = 2 5 cm.
Now, y2 = x2 + 16 = 20 + 16
y2 = 36
 y = 6cm
And QR = z2 = x2 + 25
or, z2 = 20 + 25
or, z2 = 45
 z = 3 5 cm

Example 3: In the adjoining figure, ABC is a right angled triangle where B = 90°. D
is any point on AB. Prove that AB2 + CD2 = AC2 + BD2.
Solution: In the right angled ABC A

BC2 = AC2 - AB2 -------- (i)


Again, in the right angled DBC, D
BC2 = CD2 - BD2 -------- (ii)
From relation (i) and (ii)
B C
 AC2 - AB2 = CD2 - BD2
 AC2 + BD2 = CD2 + AB2 Proved

212 | Mathematics - 9 Geometry


Exercise 5.6
1. (a) Verify and select the Pythagorean triplets from the given.
(i) 3, 4, 5 (ii) 4, 5, 6 (iii) 5, 12, 13
(iv) 9, 12, 15 (v) 8, 15, 17 (vi) 7, 24, 25
(b) (2n + 1), 2n(n + 1) and 2n(n + 1) + 1 are sides of triangle. Substituting the values of
n = 1, 2, 3, 4, 5, verify that all triangles are right angled triangles.
(c) Find the Pythagorean triplets associated with
(i) 7 (ii) 11
(iii) 21 (iv) 15
2. Check whether the following are right angled triangles.
(a) A (b) P

B 13cm C
Q 6cm R

(c) A (d)
G

H 17.5cm I

E 2.1cm F

3. Calculate the unknown length of the sides of following triangles:


(a) A (b) P

x y
S

B 9.6cm C Q R
x
(c) D (d) A
y

A x
x D
B
B C
4cm C

Geometry Mathematics - 9 |213


4. Find the length of all sides of the given figures. E

(a) A F (b)
A D
E
D
B 8cm C B 24cm C

(c) D (d) A
A

E B C
9cm

B 8cm C
D E
7cm
2 2 2 2
5. If the sides of a triangle are a + b , a - b and 2ab, prove that the triangle is a right angled
triangle where a  b.
6. For what value of x, the triangle having sides x, x + 7, and x + 8 is a right angled triangle?
7. For what value of y, the triangle having sides y, y - 1 and y - 18 is a right angled triangle?
E
A

8. In the given figure, ABC is a right angled triangle in which D


AB = 4cm, BC = 3cm. Prove that ABD + BCF = ACE
B C

9. In the given figure semicircles are formed in the sides of the A


right angled ABC. Prove that Area of semicircle on
AC = Area of the circle on AB + Area of semi Circle on 5 cm
4 cm

AB + Area of Semi Circle on BC.

B 3 cm C

A
10. In the adjoining figure, the diagonals of the quadrilateral
intersect at P at right angle.
Prove that:
(a) AB2 - BC2 = AP2 - CP2 P
2 2 2 2 B D
(b) AD + CP = CD + AP

214 | Mathematics - 9 Geometry


11. In the given figure, PR and QS are diagonals of the rhombus. P S
Prove that 4PQ2 = QS2 + PR2
O

Q R

5.6 Construction of Quadrilaterals


A. Construction of square
X
When a side of a square is given
D C
1. Construct a square ABCD in which AB = 6cm.
Steps of construction
(i) Draw a line segment AB = 6cm 6cm

(ii) At A, construct BAX = 90° o


90
(iii) From centre A, cut an arc on AX at D with radius AD = 6cm A 6cm B

(iv) Similarly, from centre D and B, take arcs with radius DC = BC = 6cm which intersect
each other at C.
(v) Join B, C and D, C.
Thus, ABCD is the required square.
When the length of diagonal is given
2. Construct a square ABCD in which diagonal AC = 6.4cm. X

Steps of construction B

(i) Draw the diagonal AC = 6.4cm.


(ii) Draw a perpendicular bisector XY on AC which cuts
AC at P. A P C
(iii) From centre P, take PB = PB = PD = 3.2cm (half of
diagonal AC) along XY.
(iv) Join A, B; B, C; C, D; and A, D. D
Y
Thus, ABCD is the required square.

Geometry Mathematics - 9 |215


B. Construction of rectangle
When the adjacent sides of a rectangle are given
1. Construct a rectangle ABCD in which AB = 7.4cm and BC = X
5.2cm. D C
Steps of construction
(i) Draw a line segment AB = 7.4cm.
(ii) At A, construct BAX = 90°.
(iii) Along AX take AD = 2.5cm. 90
o

A 7.4cm B
(iv) From B and D take arcs with radius BC = 2.5cm and
DC = 7.4cm to intersect at C.
(v) Join B, C and D, C.
Thus, ABCD is the required rectangle.
When the diagonals and angle between them are given
2. Construct a rectangle ABCD in which diagonal AC = 6.8cm and angle between diagonal AC
and BD is 60°.
X
Steps of construction B

(i) Draw AC = 6cm and draw its perpendicular bisector to


find its mid-point P.
(ii) At P, construct APX = 60° and produces XP up to
A P C
Y.
(iii) From P, take PB = 3cm on PX and PD = 3cm on PY
(half of diagonal AC or BD)
D
(iv) Join A, B; A, D; B, C and C, D. Y

Thus, ABCD is the required rectangle.

C. Construction of rhombus
When the length of diagonals are given
1. Construct a rhombus ABCD in which AC = 7.2cm and BD = 5.6cm. X

Steps of construction B

(i) Draw diagonal AC = 7.2cm.


(ii) Draw a perpendicular bisector XY of AC which cuts A P C
AC at P.
(iii) From centre P, take PB and PD both equal to 2.8cm (half
D
of diagonal BD).
Y

216 | Mathematics - 9 Geometry


(iv) Join A, B; B, C; C, D and A, D.
(v) Thus, ABCD is the required rhombus.
When a side and angle made by two adjacent sides are given
2. Construct a rhombus ABCD in which AB = 5.5cm and ABC = 60°
X
Steps of construction D 5.5cm C
(i) Draw line segment AB = 5.5cm.
(ii) At B, construct ABX = 60°.
5.5cm
(iii) Along BX take BC = 5.5cm. 5.5cm
(iv) Centre A and C, take arcs with radius AD = CD =
5.5cm, which intersect each other at D. 60
o

(v) Join A, D and C, D. A 5.5cm B


Thus, ABCD is the required rhombus.

D. Construction of parallelograms
When two adjacent sides and angle between them are given.
1. Construct a parallelogram ABCD in which AB = 6.7cm , BC = 4.8cm and ABC = 45°.
X
Steps of construction D C
(i) Draw a line segment AB = 6.7cm.
4.8cm
(ii) At B, construct ABX = 45°.
(iii) Along BX take BC = 4.8cm.
o
45
(iv) From A and C, take arcs with radius AD = 4.8cm and A B
6.7cm
CD = 6.7cm which intersect at D.
(v) Join A, D and C, D.
Thus, ABCD is the required parallelogram.
When base, diagonal and angle made by the diagonal with base are given
2. Construct a parallelogram ABCD in which AB = 5.1cm, ABD = 30° and BD = 7.4cm.
Steps of construction
X
(i) Draw a line segment AB = 5.1cm. D C

(ii) At B, construct ABX = 30°.


(iii) Along BX, take BD = 7.4cm and join A and D.
(iv) From centre D, cut an arc with radius DC = 5.1cm and
from centre B, cut another arc with centre BC equal to 30o
AD intersecting at C. A 5.1cm B
(v) Join D, C and B, C.
Thus, ABCD is the required parallelogram.

Geometry Mathematics - 9 |217


When the length of two diagonal and base are given
3. Construct a parallelogram ABCD in which AB = 5.7cm, AC = 6.4cm and BD = 8.2cm.
Steps of construction
D
(i) Draw a line segment AB = 5.7cm. C
(ii) From A and B, cut arcs with radius AO = 3.2cm
(half of AC) and with radius BO = 4.1cm O
(half of BD), which are interest at O.
(iii) Produce AO to C making AO = CO and BO to D
making BO = DO.
A 5.7cm B
(iv) Join A, D; B, C and D, C.
Hence, ABCD is the required parallelogram.

E. Construction of trapezium
When two adjacent sides and base angles are given
1. Construct a trapezium ABCD in which AB = 4.7cm, BC = 4.5cm, ABC = 120° and
BAD = 135°.
Steps of construction
(i) Draw a line segment AB = 4.7cm.
(ii) Construct ABX = 120° at B and BAY = 135° at A.
(iii) From B take BC = 4.5cm along BX.
X
Y
D C
(iv) From the point C, draw CD parallel
to AB which cut AY at D
135o 4.5cm
[taking help of alternate angles] 120o
Thus, ABCD is required trapezium.
A 4.7cm B
F. Construction of quadrilaterals
When four sides and a diagonal are given
1. Construct a quadrilateral ABCD in which AB = 5.2cm, BC = 4.7cm, CD = 6.1cm, AD = 4.1cm
and BD = 5.5cm.
Steps of construction
(i) Draw a line segment AB = 5.2cm. 6.1cm C
D
(ii) From the centre A and B cut arcs with radius AD
= 4.1cm and BD = 5.5cm which intersect at D
and join A, D and B, D. 4.1cm
5.5cm 4.7cm
(iii) Similarly, from D and B take arcs with radius DC =
6.1cm and BC = 4.7cm which intersect at C.
(iv) Join D, C and B, C. A 5.2cm B

Thus, ABCD is the required quadrilateral.

218 | Mathematics - 9 Geometry


When four sides and one angle are given
2. Construct a quadrilateral ABCD in which AB = BC = 5.1cm, CD = AD = 4.5cm and
BAD = 60°.
Steps of construction C
X 4.5cm
(i) Draw a line segment AB = 5.1cm. D

(ii) At A construct BAX = 60°.


4.5cm
(iii) From A cut an arc on AX at D with radius 5.1cm
4.5cm.
(iv) From D and B cut arcs with radius DC = 4.5cm 60o
and BC = 5.1cm which intersect each other at C. A 5.1cm B
(v) Join D, C and B, C.
Thus, ABCD is the required quadrilatral.

Exercise 5.7
1. Construct a square PQRS in which,
(a) PQ = 5.4cm (b) QR = 5.7cm
(c) RS = 6.1cm (d) PS = 4.9cm
2. Construct a square ABCD in which,
(a) Diagonal AC = 6.2cm (b) Diagonal BD = 5.8cm
(c) Length of diagonal = 6.4cm (d) length of diagonal = 5.6cm
3. Construct a rectangle ABCD in which,
(a) AB = 7.4cm and BC = 5.1cm
(b) CD = 6.4cm and AD = 5.2cm
(c) Length of diagonal = 7.4cm and diagonals making an angle 45° to each other.
(d) AC = BD = 6.8cm and diagonal making an angle of 30°.
4. Construct a rectangle PQRS in which,
(a) PQ = 5.8cm and diagonal PR = 6.1cm
(b) PQ = 7.2cm and diagonal QS = 8.1cm
(c) PQ = 5.2cm and RPQ = 60°
(d) PQ = 8.1cm and PQS = 30°
5. Construct a rhombus ABCD in which,
(a) AB = 4.8cm, BAD = 75° (b) AB = 5.4cm, BAC = 30°
(c) AB = 5.7cm, ABD = 60° (c) AB = 5.2cm, ABC = 120°
6. Construct a rhombus PQRS in which,
(a) Diagonals PR = 6.6cm and QS = 8.2cm
(b) Diagonals PR = 7.8cm and QS = 5.6cm

Geometry Mathematics - 9 |219


(c) PQ = 5.7cm and PR = 7.6cm
(d) PQ = 6.1cm and QS = 5.6cm
7. Construct a parallelogram WXYZ in which,
(a) WX = 7.1cm, XWY = 30° and XY = 4.9cm
(b) WX = 6.9cm, WXZ = 60° and WZ = 7.2cm
(c) WX = 7.4cm, XWZ = 75° and WZ = 4.7cm
(d) WX = 6.2cm, WXY = 120° and XY = 4.6cm
8. Construct a parallelogram ABCD in which,
(a) AB = 5.3cm, diagonals AC = 6.2cm and BD = 7.8cm
(b) AB = 6.2cm, AC = 8.4cm and BD = 5.6cm
(c) Diagonals AC = 5.6cm, BD = 6.4cm and they make an angle of 30°
(d) AC = 6.6cm, BD = 8.8cm and they make an angle of 45°
9. Construct a trapezium ABCD in which,
(a) AB = 7.2cm, BC = 4.3cm, ABC = 60° and CD = 5.1cm where AB//CD
(b) AB = 7.2cm, ABC = 45°, BAD = 60° and AD = 4.1cm where AB//CD
(c) AB = 5.3cm, ABC = 120°, BAD = 135° and BD = 8.9cm where AB//CD
(d) AB = 6.1cm, BC = 4.2cm, BAC = 30°, AD = 4.9cm where AB//CD.
10. Construct a trapezium PQRS in which,
(a) PQ = 5.7cm, diagonal PR = 6.6cm, QR = 4.3cm, PS = 4.9cm, where PQ//SR
(b) PQ = 7.3cm, PQS = 45°, QS = 5.1cm, SR = 4.2cm where PQ//SR
(c) PQ = 7.6cm, QPS = 60°, PS = 4.4cm, diagonal PR = 6.7cm where PQ//SR
(d) PQ = 4.3cm, QPR = 30°, PQR = 135°, diagonal QS = 6.3cm where PQ//SR
11. Construct a quadrilateral ABCD in which,
(a) AB = 5.3cm, BC = 6.1cm, CD = 4.2cm, AD = 5.9cm and BAD = 75°
(b) AB = BC = 5.6cm, CD = AD = 4.9cm and ABC = 60°
(c) AB = 6.1cm, BC = 5.2cm, CD = 4.5cm, AD = 5.5cm and BD = 4.7cm
(d) AB = 4.6cm, BC = 6.2cm, CD = 5.5cm, AD = 5.1cm and AC = 5.6cm

5.7 Circle
Introduction
Circle and centre of the circle B
In the adjoining figure, ABC is a circle. A circle is a closed plane figure
formed by a curved line, all of whose points are equidistance from a fixed O
point in the same plane. The fixed point is called centre of the circle. In the
given figure, O is the centre of the circle. The circle ABC is represented as A C
ABC.

220 | Mathematics - 9 Geometry


Circumference A
The perimeter of the circle is known as its circumference. In other words,
the total length of the boundary of a circle is known as the circumference of B
the circle. In the adjoining figure, ABCA is the circumference of the circle.
Radius
C
In the adjoining figure, O is the centre of the circle. A, B, C, D and E are the
C
points on the circumference of the circle. So OA is the radius of the circle.
B D
Similarly, OB, OC, OD and OE are also radii (plural form of radius) of the
circle. The distance between the centre and any point on the
O
circumference of the circle is called the radius of the circle. And it is
denoted by r or R. Radii of the same circle are equal. E
A
Diameter
d
In the adjoining figure, O is the centre of the circle. AOB is a diameter. The A A
r O r
length of diameter of a circle is twice its radius. As shown in figure, a line
segment passing through the centre of a circle and having its end points
on circumference of the circle is called the diameter. And it is denoted by
d P
d. i.e. 2r = d or r = 2 . The diameter bisects the circle.

Semi Circle A B
O
In the figure alongside, O is the centre and AOB or AB is a diameter of the
circle. APB or AQB are semi circles. A diameter divides the circle into two
Q
equal parts and each part is called semi circle. Its denoted by .
A B
Chords
M N
In the adjoining figure, AB is a chord. Similarly CD and MN are also chords O
of the circle. The line joining any two points on the circumference of the C
circle is called chord. A diameter is also a chord and it is the longest chord D
of the circle. In the given figure, MN is a diameter which is the longest chord M
of the circle. A B
Arc
In the adjoining figure, AB is a chord. The chord AB divides the
circumference of the circle in two parts which are AMB and ANB. So AMB
N
and ANB are arcs of the circle. An arc is the part of the circumference of a

circle and it is denoted by the symbol . There are two types of arcs,
where is a minor arc and ANB is a major arc.
A
Segment
In the adjoining figure, AB is a chord. and ANB are minor and major M
O N
arcs of the ANBM. There are two regions, one is shaded and other
unshaded. These both regions are called circle segments. The area enclosed
B
Geometry Mathematics - 9 |221
by arc and chord of a circle is called circle segment. In the given figure,
shaded part or AMB is minor circle segment and unshaded part or ANB is
major circle segment.
Sector N

In the adjoining figure, O is the centre and OA and OB are radii of a circle.
O
There are two regions, one is shaded and another unshaded. Both regions are
enclosed by two radii and corresponding arcs. These regions are called B
sectors. In short, the area enclosed between any two radii of a circle and
A M
the corresponding arc is called sector of the circle. In the figure, OAMB is
a minor and OANB major sectors.
H
P
Concentric Circles D A
E Q
In the figure alongside, ABC, DEF, GHI and PQR have same centre G
O
O. These circles have different radii but the same centre. These types of C B
circles are called concentric circles. If two or more circles with different F I
radii and have the same centre, they are called concentric circles.
R
Intersecting Circles: A

In the adjoining figure, AMB and ANB intersect at A and B. If


two circles intersect each other at two points, they are called M X Y N
intersecting circles. The intersecting circles have a common chord.
AB is a common chord as shown in the figure. B

Theorem - 16
The perpendicular drawn from the centre of a circle to the chord bisects the chord.
Given: i. O is the centre of a circle.
ii. PQ is a chord
iii. ORPQ O

To prove: PR = PQ
Construction: Join O, P and O, Q. P R Q
Proof:
S.N Statements S.N Reasons
1. In OPR and OQR 1.
i. ORP = ORQ (R) i. Both of them are 90° (ORPQ) 
ii. OP = OQ (H) ii. Radii of thesame circle.
iii. OR = OR (S) iii. Common side
OPR  OQR By R.H.S.
PR = QR Corresponding sides of the congruent
2. 2.
triangles
Proved

222 | Mathematics - 9 Geometry


Converse of theorem - 16
A line joining the centre of a circle and the mid-point of a chord is perpendicular to the chord.
(chord is not a diameter)
Given: i. O is the centre of a circle.
ii. M is a mid-point of chord PQ.
iii. O and M are joined. O
To prove: OMP = OMQ = 90°
Construction: Join O, P and O, Q. P Q
M
Proof
S.N Statements S.N Reasons
1. In OPM and OQM 1.
i. OP = OQ (S) i. Radii of the same circle.
ii. PM = QM (S) ii. Given (M is mid-point of PQ)
iii. OM = OM (S) iii. Common side
OPM  OQM By S.S.S.
2. OMP = OMQ 2.
Corresponding angles of congruent
triangles.
3. OMPQ 3.
From statement 3 (adjacent angles of the
linear pair equal)
Proved

Theorem - 17
M R
Equal chords of a circle are equidistant from the centre.
Given: i. O is the centre of a circle.
ii. MN and RS are two equal chords. i.e (MN = RS) B
A
iii. OAMN and OBRS
O
To prove: OA = OB
Construction: Join OM and OR. N S
Proof
S.N Statements S.N Reasons
1. (i) MN = RS 1. (i) Given
1 1 Being OAMN and OBRS.
(ii) MA = MN and RB = RS (ii)
2 2
(iii) MA = RB (iii) From statement 1. (i) and (ii)
2. In OAM and OBR 2.
OA and OB are perpendicular to MN
(i) OAM = OBR (R) (i) and RS respectively. (being right
angle)
(ii) OM = OR (H) (ii) Radii of the same circle.
(iii) MA = RB (S) (iii) From statement 1. (iii)
 OAM  OBR By R.H.S
OA = OB Corresponding sides of the congruent
3. 3.
triangles.
Proved

Geometry Mathematics - 9 |223


Converse of theorem - 17
Chords, which are equidistant from the centre of a circle are equal.
Given: i. O is the centre of a circle. A C
ii. AB and CD are two chords.
iii. OXAB, OYCD
X Y
iv. OX = OY
O
To prove: AB = CD
Construction: Join A and C with centre O. B D
Proof
S.N Statements S.N Reasons
1. In OAX and OCY 1.
(i) OXA = OYC (R) (i) Being OXAB and OYCD (given)
(ii) OA = OC (H) (ii) Radii of the same circle
(iii) OX = OY (S) (iii) Given
 OAX  OCY By R.H.S.
2. (i) AX = CY 2. (i) Corresponding sides of the congruent
triangles.
(ii) 2AX = 2CY (ii) Multiplication axiom.
(iii) AB = CD (iii) OXAB so OX bisects AB, at X
similarly, OYCD so OY bisects CD
at Y (2AX = AB, 2CY = CD).
Proved

Theorem 18
Perpendicular bisector of a chord of a circle passes through the centre of the circle.
Given: O is the centre of the circle in which AB is a
chord. M be the mid-point of AB and MP is the
perpendicular bisector of AB.
To prove: MP passes through centre O.
Construction: Join mid-point M and centre O

Proof:
S.N Statements S.N Reasons
1. MP  AB 1. Given
2. MO  AB 2. Line joining the centre of the circle and
the mid point of the chord is the
perpendicular to the chord.
3. OMB = PMB = 90 3. From statements (1) and (2)
4. MO and MP are name line 4. From the given point on the line there
passes only one line perpendicular to
the given line.
5. MP passes through centre 5. 5. From statement (4)
Q.E.D. Proved

224 | Mathematics - 9 Geometry


Worked Out Examples
Example 1: In the adjoining figure, find the length of chord
AB which is at a distance of 2.8cm from the O
centre O of the circle of radius (OA) = 10cm.
Solution: Here, A M B
O is the centre of a circle. OMAB,
OA (r) = 10cm
OM = 2.8cm
Length of AB = ?
In right angled OMA,
AM = (OA)2 - (OM)2
= (10)2 - (2.8)2
= 100 - 7.84
= 92.16 = 9.6cm
 AB = 2AM
= 2 × 9.6 cm = 19.2cm
Example 2: In the given figure, O is the centre of a circle. If
AB//CD, OMAB, ONCD, AB = 16cm, CD = 12cm A M B
and the diameter of the circle is 20cm, find the O
length of MN.
Solution: Here, O is the centre of the circle.
C N D
AB//CD, AB = 16cm, CD = 12cm,
OMAB, ONCD
Join B and D with centre O.
1 1
Now, OB = OD = 2 × 20cm = 10cm [radius = 2 of diameter]

1 1
BM = 2 AB DN = 2 CD

1 1
= 2 × 16cm = 2 × 12cm

= 8cm = 6cm
In the right angled triangle OMB.
MO = (OB)2 - (BM)2
= (10cm)2 - (8cm)2
= 100 - 64 = 36 = 6cm
Again, in the right angled OND
NO = (OD)2 - (DN)2

Geometry Mathematics - 9 |225


= (10cm)2 - (6cm)2
= 100 - 36
= 64 = 8cm
Then,
MN = MO + NO
= 6cm + 8cm = 14cm
Example 3: In the given figure, M and N are the A P C Q B
centres of two intersecting circles. If
1
ACB//MN, prove that MN = AB. M N
2
Solution:
D
Given: i. M and N are the centres of two intersecting circles.
ii. ACB//MN
1
To prove: MN = 2 AB.

Construction: Draw perpendiculars MP and NQ on AB.


Proof
S.N Statements S.N Reasons
1. (i) PMN = 90° 1. (i) Being PQ//MN and MPQ = 90°
(ii) QNM = 90° (ii) Being PQ//MN and NQP = 90°
PMNQ is a rectangle Being all angles right angles.
2. (i) PQ = MN 2. (i) Opposite sides of the rectangle.
1 1
(ii) PC = 2 AC and QC = 2BC (ii) Being MPAC and NQBC.
1
(iii) PC + QC = 2 (AC + BC) (iii) From statement 2. (ii)
1
(iv) PQ = 2 AB (iv) Whole part axiom.
1
(v) MN = 2 AB (v) From statement 2. (i)
Proved
D
Example 4: In the adjoining figure, O is the centre of a
circle. AB and CD are two chords, which A
intersect at P. If OP is angle bisector of
BPD, prove that: (i) AB = CD, (ii) AP = CP. P O
C
Solution:
Given: i. O is the centre of a circle.
ii. AB and CD intersect at P. B
iii. OP is an angle bisector of BPD. i.e OPB = OPD.

226 | Mathematics - 9 Geometry


To prove: i. AB = CD D
ii. AP = CP A
N
Construction: Draw OMAB and ONCD. P
O
C M

B
Proof
S.N Statements S.N Reasons
1. In OPM and OPN 1.
OMP = ONP (A) Being OMAB and ONCD. (by
(i) (i)
construction)
(ii) OPM = OPN (A) (ii) Given OPB = OPD
(iii) OP = OP (S) (iii) Common side
OPM  OPN By A.A.S.
2. (i) OM = ON 2. (i) Corresponding sides of the congruent
triangles.
AB = CD Chords being equidistant from the
(ii) (ii)
centre of a circle.
PM = PN Corresponding sides of the congruent
(iii) (iii)
triangles.
AM = CN 1 1
AM = 2 AB and CN = 2 CD, where
(iv) (iv)
AB = CD.
(v) AM – PM = CN – PN (v) From statements 2. (iii) and (iv)
AP = CP From statement (v) (Remaining
(vi) (vi)
facts)
Proved

Exercise 5.8
Group 'A'

O
1. (a) In the given figure, O is the centre of a circle and OXAB.
If AB = 4cm, OX = 4.8cm, find the length of the diameter.
A X B

P
(b) In the given figure alongside, O is the centre of a circle. PQ
is a chord and OMPQ. If OM = 4.8cm and ON = 6cm, M
find the length of PQ. N O
Q

Geometry Mathematics - 9 |227


C
(c) In the adjoining figure, O is the centre and AB is a diameter
of a circle. Chords AB and CD intersect at E such that
CE = DE = 12cm. If BE = 6cm, find the length of OC. A O B
E

D
P
(d) In the given figure, O is the centre of a circle. OR and PQ
intersect at S at right angle. If PQ = 6cm and SR = 3cm, O R
S
find the length of radius of the circle.
Q

2. (a) In the given figure, O is the centre of a circle. If AB//CD,


AB = 8cm, CD = 6cm and diameter of the circle is 10cm, O
A B
find the distance between AB and CD.
C D

(b) In the adjoining figure, O is the centre of a circle in which P Q


chords PQ and RS are parallel. If PQ = 18cm, RS = 24cm O
and radius of the circle is 15cm, find the distance between
PQ and RS. R S

(c) In the figure alongside, O is the centre of a circle. MN and


RS are equal and parallel chords. If the radius of the circle M N
is 10cm and the distance between two chords is 12cm, find O
the length of the chord MN.
R S

C
(d) In the given figure, O is the centre of a circle. AC is a
O
diameter and AB is a chord. If OMAB, AC = 10cm and
AB = 9.6cm, find the length of OM and BC.
A M B

3. (a) In a circle of radius 2.5cm, AB and CD are two parallel chords of length 2.4cm and 1
cm respectively. Find the distance between the chords, if they lie (i) on the same side of
the centre (ii) on the opposite side of the centre.
(b) PQ and RS are two parallel chords which lie on the opposite sides of the centre of the
circle. If these chords are 14cm apart, PQ = 12cm and RS = 16cm, find the radius of the
circle.
(c) AB and CD are two parallel chords which lie on the same side of the centre of the
circle. If these chord are 1 cm apart, AB = 8cm and CD = 6cm, find the radius of the
circle.

228 | Mathematics - 9 Geometry


(d) MN and RS are two parallel chords of a circle such that MN = 18cm, RS = 24cm. If the
chords are on the opposite sides of the centre and the distance between two chords is
21cm, find the diameter of the circle. P R
Group 'B'
M N
1. In the adjoining figure, PQ and RS are two equal chords of the circle
X
with centre X. If XMPQ and XNRS, show that MX = NX.
Q S

2. In the given figure, P is the centre of the circle. PA and PB are M R


perpendiculars on the chords MN and RS respectively. If PA = P
PB, prove that MN = RS. A B

N S

3. In the adjoining figure, O is the centre of the circle. A chord PQ


intersects two concentric circles at the points P, M, N and Q as
shown in the figure. Prove that PM = QN. O
M N
P Q

D
4. In the given figure, O is the centre of the circle. Two equal chords
A
AB and CD intersect at P. Prove that OP is an angle bisector of
BPD. P O
C

B
5. In the adjoining figure, O is the centre of a circle. Chords MN and M R
RS intersect at P. If OP is an angle bisector of SPN, prove that: P
(i) MN = RS, (ii) MP = RP and (iii) NP = SP
O
S N
A
6. Two equal chords AB and CD meet at an external point E. B
Prove that AE = CE and BE = DE. E
D
C
7. In the figure alongside, two chords PQ and RS of a circle with P
centre O are produced to meet at T. If PT = RT and QT = ST, Q
prove that OT is an angle bisector of PTR. O T
S
R
D
8. In the adjoining figure, O is the centre of a circle. Two equal A N
chords AB and CD intersect at P. If M and N are mid-points of AB P O
and CD respectively, prove that PMN is an isosceles triangle. C M
B
Geometry Mathematics - 9 |229
B
M
9. In the figure alongside, O is the centre of a circle. If PB = PD, A
prove that OM = ON. P O
C
N
D
B

10. In the given figure, O is the centre of circle. If OB is the bisector


of ABC, prove that AB = BC. O
A C

A
M
B
11. In the given circle, O is the centre of a circle. OMAB, ONCD O P
and OM = ON. Prove that AP = CP. D
N
C

A
12. In the adjoining figure, O is the centre of a circle. Two equal
chords intersect at right angle at G. If E and F are mid-points of E O
AB and CD respectively, prove that OEGF is a square. G
C F D
B

13. In the figure alongside, P and Q are centres of two intersecting


circles. If ACB//PQ, prove that 2PQ = ACB. P Q

A C B
14. In the given figure, M and N are centres of two intersecting circles
A
which interest at A and B. Prove that the line joining the centres of
the circle is the perpendicular bisector of the common chord AB.
M N

B
CE
15. In the adjoining figure, two circles with centres A and B are G
intersecting at C and D. If EF//CD, prove that EG = FH and A B
EH = FG. H
DF
A
16. In the given figure, X and Y are centres of two intersecting equal
circles. If XY = AB, prove that AXBY is a square. X Y
P

B

230 | Mathematics - 9 Geometry


Unit Test
Time: 40 minutes F.M.- 24

Group-A A B D
x
1. From the given figure, find the value of x.

2. Write the converse statement of "If two sides of a C


triangle are equal, the angles opposite to them are
equal." A

40°
Group B

3. Find the value of the angles denoted by x and y. 3x y 2x


D C B

A D

4. In the adjoining figure, ABCD is a square and AEC


is an equilateral triangle. Find the size of EAB.
B C

E
P

5. In the given figure, PRQ = QPS. If QR = 9 cm


and QS = 4 cm, find the length of PQ.
Q
Group C S
R
6. Prove that equal chords of a cirlce are equidistance P
A B
from the centure
7. In the given figure, ABCD is a quadrailateral. P, Q, R S
and S are mid poins of the sides AB, BC, CD and AD
D Q
respectively. Prove that PQRS is a parallelogram.
8. Construct a parallelogram ABCD inb which
R
diagonals AC = 6 cm and BD = 8 cm and angle
C
between the diagonals = 60°. Give steps of
construction. D
A
Group D
P
1 1 1
9. In the given figure, AB//DC//PQ. Prove that: PQ = AB + DC.

 B Q C

Geometry Mathematics - 9 |231


Answers ____________________________________________________________
Exercise 5.1
1. (a) 16° (b) 17° (c) 34° (d) 18° (e) 32°, 44°
(f) 44°, 54°, 82° (g) 12°, 120° (h) 22.5°, 45° (i) 48°, 42°, 48°
2. (a) 78° (b) 90° (c) 154° (d) 76°, 52° 52°
3. (a) 127° (b) 36° (c) 120° (d) 12°, 36°
Exercise 5.2
1. (a) 30° (b) 112°, 34° (c) 44° (d) 18°, 54° (e) 30°
(f) 34° (g) 116°, 72° (h) 72°, 36° (i) 64°
2. (a) 34°, 44° (b) 44°, 78° (c) 26°, 52° (d) 22.5°, 22.5°
(e) 45, 25° (f) 22°, 22°
3. (a) 72° (b) 67.5° (c) 83° (d) 50°
Exercise 5.3
A. 1. (a) 108°, 72° (b) 106°, 106° (c) 30°, 120° (d) 45°, 45°
(e) 74°, 106° (f) 45°, 54° (g) 34°, 101° (h) 116°, 52°
(i) 76°, 31° (j) 103°, 122° (k) 38°, 83° (l) 112°
2. (a) 79° (b) 90° (c) 64° (d) 34.5°
3. (a) 15° (b) 68° (c) 120° (d) 107°
Exercise 5.4
1. (a) 49°, 130° (b) 75°, 130° (c) 42°, 32° (d) 76°, 124°
2. (a) 1.7cm, 2.6cm (b) 4.4cm, 5.2cm (c) 3.6cm, 1.7cm (d) 7.4cm, 8.4cm
Exercise 5.5
1. (a) 5cm, 1cm (b) 2.5cm, 3cm (c) 2.25cm, 6cm (d) 2.4cm, 2cm
2. (a) 2cm, 3cm (b) 4.5cm, 6cm (c) 3.6cm, 2.88cm (d) 8cm, 8 3cm
3. (a) 1.25cm, 7cm (b) 6cm (c) 5cm (d) 1.25cm
4. (a) 4cm, 7cm (b) 5cm, 7.5cm (c) 12cm (d) 16cm
5. 7.5cm, 12cm, 7.5cm
Exercise 5.6
1. (a) Show to your teacher (b) Show to your teacher
(c) (i) 7, 24, 25 (ii) 11, 60, 61 (iii) 21, 20, 29; 21,72, 75; 21, 220, 221 (iv) 15, 8, 17; 15, 20, 25; 15, 36, 39; 15, 112, 113
2. Show to your teacher
3. (a) 10.4 cm (b) 5cm, 13cm (c) 5cm, 12cm (d) 12cm, 15cm
4. (a) AC = 10 cm, AD = 5 5 cm, DE = 4.36 cm, AF = 12.65 cm
(b) AC = 25 cm, AD = 26.93 cm, AE = 30.8 cm
(c) AE = CE = 5cm, DE = 12 cm, BE = 6.24 cm
(d) AB = BD = 12 cm, AE = 25 cm
6. 5 cm 7. 25 cm
Exercise 5.7
Show to your Subject Teacher
Exercise 5.8
1. (a) 10.4cm (b) 7.2cm (c) 15cm (d) 3cm
2. (a) 1cm (b) 21cm (c) 16cm (d) 1.4cm, 2.8cm
3. (a) 1.7cm, 3.1cm (b) 10cm (c) 5cm (d) 15cm

232 | Mathematics - 9 Geometry


Chapter

6
Trigonometry
Objectives:
At the end of this chapter, the
students will be able to:
 find the value of trigonometric
ratios of different standard
angles like 0°, 30°, 45°, 60°, 90°
 solve the different identities
related to trigonometric ratios.
 solve the triangle using
trigonometric ratios.

Teaching Materials:
Chart paper, flash card, chart of
trigonometrical formula and value
chart of standard angles.
Historical fact
Traditionally, trigonometric functions were considered with respect to arcs of a circle. Georg Joachim
Rhaeticus (1514 – 1576), a German mathematician was the first to define trigonometric functions as
the ratio of the sides of a right angled triangle. Thus, all six trigonometric functions came into full use
and here trigonometry came of age and started modern trigonometry.
C
6.1 Trigonometric Ratios
Let’s consider a right angled triangle ABC with B = 90°. Among the other two
acute angles, one is considered on reference angle in terms of which the ratios
are defined. The side opposite to the right angle is called hypotenuse (h), the side
opposite to the reference angle is perpendicular (p) and the remaining side is
called base (b). A Base (b) B

Using three sides, six ratios can be formed as C

CB p AC h
(i) AC = h (ii) CB = p
h p
AB b AC h
(iii) AC = h (iv) AB = b
CB p AB b A B
(v) b
AB = b (vi) CB = p
Until this step, we are simply dealing with pure geometry. Further, we relate these ratios with an angle
of the right angled triangle which is an important turn in mathematics from where trigonometry starts.
Now, taking C as reference angle, we define the above six ratios of sides trigonometrically as
CB p AC h
(i) AC = h = sine of A or Sin A and CB = p = Cosecant of A or Cosec A

AB b AC b
(ii) AC = h = Cosine of A or Cos A and AB = h = Secant of A or Sec A
CB p AB b
(iii) AB = b = Tangent of A or Tan A and CB = p = Cotangent of A or Cot A

Relations between the trigonometric ratios


A. Identities related to receiprocal
1. Sin A and Cosec A being reciprocals
p h
Sin A . Cosec A = h × p = 1.

 Sin A . Cosec A = 1
Similarly Cos A. Sec A = 1; Tan A . Cot A = 1

234 | Mathematics - 9 Trigonometry


B. Identities related to quotient
p b
p b tan  b p cot 
1. Sin  = h = h = 2. Cos  = h = h =
sec  cosec 
b p

p
p h SinA CosA
3. Tan A = b = b = CosA 4. Cot A = SinA
h

h h
h p cosec  h b Sec 
5. Sec  = b = b = 6. Cosec  = p = p =
cot  tan 
p b

C. Pythagorus Identities
1. p2 + b 2 = h 2
p 2 b 2
or, h + h = 1
   
or, Sin2 + Cos2 = 1
 Sin2 = 1 – Cos2

 Sin = 1 – Cos2
2. h2 – p2 = b 2
h 2 p 2
or, b + b = 1
   
or, sec2 – tan2 = 1
3. h2 – b2 = p 2
h 2 b 2
or, P + p = 1
   
or, cosec  – cot2 = 1
2

p b
4. Sin A = h and Cos A = h

p2 b2
 Sin2 A = h2 and Cos2 A = h2

Adding these two relations, we get


p2 b2 p2 + b2 h2
Sin2 A + Cos2 A = h2 + h2 = 2 2 2
h2 = h2 = 1 [p + b = h , by Pythagoras theorem]

 Sin2 A + Cos2 A = 1

Trigonometry Mathematics - 9 |235


Similarly, we get
Sec2 A - Tan2 A = 1 and Cosec2 A - Cot2 A = 1
Note:
(i) Reference angles are also represented by Latin alphabets ,, ,  etc.
(ii) Being related to right angled triangle, Pythagoras theorem, h2 = p2 + b2 is very useful in
trigonometry.

Worked Out Examples


Example 1: Write the trigonometric ratios with respect to the given reference angle
of the given right angled triangle.
Solution: Here,
In the right angled triangle ABC,  B = 90° A
Reference angle  ACB = 
 h = AC, p = AB, b = BC
p AB
Now, Sin = h = AC
C 
B
b BC
Cos = h = AC

p AB
Tan = b = BC

h AC
Cosec = p = AB

h AC
Sec = =
b BC
b BC
And Cot = p = AB

P
Example 2: In the right angled triangle PQR,  PQR =
90°, PQ = 8cm, QR = 15cm. Find all the
trigonometric ratios of the angle R = 
Solution: Here,
In right angled triangle PQR, Q R
15cm
 PQR = 90°, PQ = 8cm, QR = 15cm.
 PR = PQ2 + QR2 [By Pythagoras theorem]
= (8cm)2 + (15cm)2
= 64cm2 + 225cm2
= 289cm2 = 17cm.
Now with respect to R = 

236 | Mathematics - 9 Trigonometry


p PQ 8cm 8
Sin = h = PR = 17cm = 17

b QR 15cm 15
Cos = h = PR = 17cm = 17

p PQ 8cm 8
Tan = b = QR = 15cm = 15

h PR 17cm 17
Cosec = p = PQ = 8cm = 8

h PR 17cm 17
Sec = b = QR = 15cm = 15

b QR 15cm 15
And Cot = = = =
p PQ 8cm 8

Example 3: Using the table of values of trigonometric ratios, find the value of the
side x of the given right angled triangle. A
Solution: Here,
35°
In right angled triangle ABC, ABC = 90°,
BAC = 35° and AB = 6.8cm, BC = x = ?
Now, with respect to A = 35°
BC
Tan = AB
C B
x
x
or, Tan 35° = 6.8cm

x
or, 0.70 = 6.8cm [Using table, tan 35° = 0.70]

 x = 4.76cm

Example 4: Using the table of values of trigonometric ratios, find the angle  in the
given right angled triangle.
Solution: Here,
In the rt.  MNO, OM = 51cm, ON = 45cm, MNO = 90°, MON =  = ?
M
Now, with respect to angle 
ON
Cos = OM

45cm 
or, Cos = O N
51cm 45cm
or, Cos = 0.88.
or, Cos = Cos 28°
 = 28°

Trigonometry Mathematics - 9 |237


Example 5: Express Cos in terms tan.
Solution: We know,
1 1
Cos = = [ Sec = 1 + tan2 ]
Sec 1 + tan2
Alternately: A
Let tan = x
1 + x2
 p = x, b = 1
x
h = p2 + b2

= 12 + x 2 = 1 + x2 B 1 C
b
Now, Cos = h

1
=
1 + x2
1
Cos =
1 + tan2

11
Example 6: If Sin = , find tan.
61
Solution: Here,
11
Sin = 61

Now, we have
Sin Sin
Tan = =
Cos 1 - Sin2
11
11
61 61 = 11
Or, tan = =
11
2
60 60
1 - 61
  61
Alternatively:
11
Here, Sin = 61

If P = 11x then, h = 61x


b = h2 - p2 = (61x)2 - (11x)2
= 3721x2 - 121x2
= 3600x2 = 60x
p 11x
Now tan = =
b 60x
11
tan = 60

238 | Mathematics - 9 Trigonometry


Example 7: Prove the given trigonometric identity: (1 + tan2) cos2 = 1.
Solution: Here,
L.H.S = (1 + tan2) cos2
sin2 
= 1 + . cos2
 cos2
cos2 + sin2
= × cos2
cos2
1
= × cos2 [ cos2 + sin2 = 1]
cos2
= 1 R.H.S proved.

Exercise 6.1
1. Write the basic trigonometric ratios with respect to the given angle as the ratios of the sides of
the triangle.
L
(a) A B (b)


N M
C
(c) (d)
D D

B  C B  C
2. In each of the following right angled triangles, find the ratios of the given angle.
(a) A (b)
D 4.8cm E

5cm

B C
F

Trigonometry Mathematics - 9 |239


X
(c) M (d)

 
L 2.4cm N
Y
3. Use the table of values of trigonometric ratios of the given angles and find the lengths of the
unknown sides.
P
(a) A (b)

? ?
10 cm
10 ?

30° R 60° Q
C B 7.2cm
?

(c) X (d) A
D 60°
?
?
45° C B
Y ? Z
?
4. Find the indicated angles in each of the following right angled triangles. (Use table for the
value of the ratios)
(a) A (b) P

6cm

B C
 R
Q 8cm
(c) (d)
P Q
P
D
25.98 cm

30
cm b
R Q R
5. (a) Express tan and cos in terms of sin.
(b) Express sin in terms of tan.
(c) Express tanα in terms of cosα.
(d) Express sin in terms of cos.
6
6. (a) If sin = 10 , find the value of cos.

240 | Mathematics - 9 Trigonometry


8
(b) If tan = 15 , find sin.

BC
(c) If ABC is a right angled triangle with B = 90° and sin (90° - ) = AC , write the ratio
of sin in term of sides of ABC.
7
(d) In ABC, B = 90° and cos (90° - A) = 25 , find sin C. A 5cm B

13cm
7. (a) From the figure given alongside, find the value of tan and cos. 
D 9cm C
A C
(b) From the figure given alongside, find angle cos.
D

B A

8. (a) Write down the value of sine of the smallest angle of the a
given figure.
C B

A
1
(b) In the given ABC, B = 90, prove that sin45 = x
2
45 
C B
x
P

(c) From the given figure, find the trigonometric ratios of sin and
S
tan.


Q 4 cm R

N
(d) From the given figure, find the trigonometric ratios of sin and cos. 
U


R 5 cm A
B


(e) From the given figure, calculate tan.
A

D 4cm C

Trigonometry Mathematics - 9 |241


(f) Find the values of the following from the given figure.
A 24cm C
(i) BC (ii) Cos
(iii)  (iv) DC


B 65cm D
9. Prove the following identities:
sinA tan sin
(a) tanA = cos A (b) =
cos 1- sin2
1 - cos2
(c) = tan2 (d) (cos – sin)2 = 1 – 2cos.sin
1 - sin2
cos
(e) (1 + tan2) cos2 = 1. (f) (sec2 - tan2) = cot
sin
1 + tan cot + 1
(g) =
1 - tan cot - 1
(h) ( 1 + tan2)cos = 1

cos cos 1 + sin cos


(i) + = 2sec (j) + = 2sec
1 + sin 1 - sin cos 1 + sin

6.2 Trigonometric Ratios of Some Standard Angles


Angles which can be drawn by using Euclidean tools (straight edge and compass) are called Standard
angles. e.g. 90°, 60°, 30°, 45°, 75°, 120° etc. We have tables for the values of trigonometric ratios of
any angles. Here, we discuss about the trigonometric ratios of some standard angles which we use
without using tables.
1. Trigonometric ratios of 0° and 90°.
Consider an arc APB of a circle with centre O and radius a.
Let p(x, y) be any point on this arc. Draw PM OX, then we get OM = x, PM = y and OP = a.
Let POM = , then Y
PM y a B
Sin = OP = a , Cosec  = y P(x , y)

OM x a
Cos = OP = a , Sec = x
X
O M A
PM y x
Tan = = , Cot =
OM x y
i. Trigonometric Ratios of 0°.
When  = 0°, P coincides with A so that x = a and y = 0.
0 a
Sin0° = a = 0°, Cosec0° = 0 =  (undefined)

242 | Mathematics - 9 Trigonometry


a a
Cos0° = a = 1, Sec0°= a = 1

0 a
Tan0° = a = 0, Cot0°= 0 =  (undefined)

ii. Trigonometric ratios of 90°.


When  = 90°, P coincides with B so that x = 0 and y = a.
a a
Sin90° = a = 1, Cosec90° = a = 1

0 a
Cos90° = a = 0, Sec90°= 0 =  (undefined)

a 0
Tan90° = 0 =  (undefined), Cot90°= a = 0
A
2. Trigonometric ratios of 30° and 60°.
Consider an equilateral triangle ABC with side 2a (say) and let AD be
perpendicular to BC, then D be mid-point of BC.
BC 2a 60°
BD = DC = 2 = 2 = a
B D C
Since each angle of an equilateral triangle is 60°, ABD = 60° and
AD bisects BAC, BAD = 30°.
From right angled triangle ABD, by Pythagoras theorem, we get,
AD2 = AB2 – BD2
= (2a)2 – a2
= 3a2

AD = 3a

i. Trigonometric ratios of 30°:

Taking BAD = 30° as the reference angle perpendicular (P) = BD = a, base (b) = AD = 3 a
Hypotenuse (h) = AB = 2a
BD 1 2a
Sin 30° = AB = =2; Coses30° = a = 2

AD 3a 3 2a 2
Cos30° = AB = 2a = 2 ; Sec30° = =
3a 3

BD a 1 3a
Tan30° = AD = = ; Cot 30° = a = 3
3a 3

Trigonometry Mathematics - 9 |243


ii. Trigonometric ratios of 60°.

Taking ABD as reference angle, perpendicular (P) = AD = 3 a, base (b) = BD = a and


hypotenuse (h) = AB = 2a.

AD 3a 3 2a 2
Sin60° = AB = 2a = 2 ; Cosec60° = =
3a 3
BD a 1 2a
Cos60° = AB = 2a = 2 ; Sec60° = a = 2

AD 3a a 1
Tan 60° = BD = a = 3 ; Cot60° = =
3a 3
3. Trigonometric ratios of 45°.
Consider an isosceles triangle, right angled at B, with legs AB = BC = a(say),
A
Then ACB = 45°. From right angled triangle ABC, by Pythagoras theorem, we get,
AC2 = AB2 + BC2
= a2 + a2
= 2a2 C 45°
B
 AC = 2 a

AB a 1 2a BC a 1
Sin45° = AC = = , Cosec 45° = a = 2 Cos45° = AC = = ,
2a 2 2a 2

2a AB a a
Sec45° = a = 2 Tan45° = BC = a = 1, Cot45° = a = 1

The values of the ratios of angles 0°, 30°, 45°, 60° and 90° can be summarized as:
Angle
0° 30° 45° 60° 90°
Ratios
1 1 3
Sin 0 1
2 2 2
3 1 1
Cos 1 0
2 2 2
1
Tan 0 1 3 
3
Note:
(i) Values of the ratios Cosec, Sec and Cot are reciprocals of Sin, Cos and Tan respectively.
0 1 2 3 4
(ii) Sines of the angles 0°, 30°, 45°, 60°, 90° are respectively the square roots of 4 , 4 , 4 , 4 , 4 and that of the
Cosines square roots taken in reversed order.
Sin
(iii) Values of tangents can be obtained by taking tan = .
Cos

244 | Mathematics - 9 Trigonometry


Worked Out Examples
Example 1: Find the value of (Cos0° + Sin30° + Sin45°) (Sin 90° + Cos60° - Cos45°)
Solution: (Cos0° + Sin30° + Sin45°) (Sin 90° + Cos60° - Cos45°)
1 1 1 1
= 1 + 2 +  1 + 2 - 
 2  2
3 1 3 1
= 2 +  2 - 
 2  2
2 2
3 1 9 1 7 3
= 2 -   = 4 - 2 = 4 = 14
   2

1 – sin30° 1 – tan30°
Example 2: Prove that: =
1 + sin60° 1 + tan30°
1 – sin30°
Solution: L.H.S = 1 + sin60°

1 1
1–2 2 1
= = =
3 2+ 3 2+ 3
1+ 2 2
2– 3 2– 3
= 2 = 4–3 =2– 3.
(2) – ( 3)
2

1
1–
1 – tan30° 3 3–1
R.H.S = 1 + tan30° = 1 = 3+1
1+
3
3–1 3–1 3+1–2 3 4–2 3
= × = 3–1 = 2 =2– 3
3+1 3–1
L.H.S = R.H.S proved.

Example 3: Solve: 2sin = 1 [0°  90°]


Solution: Here,
2sin
1
or, sin2

or, sin= sin30°  90°]


or, = 30°
Therefore, the required value of is 30°.

Trigonometry Mathematics - 9 |245


Example 4: If sinx = cosx (90°), find the angle x.
Solution: Here,
sinx = cosx
sin x
or, =1
cos x
or, tanx = 1
or, tanx = tan 45° [   x  90°]
 x = 45°

Example 5: In the given figure, ABC is a right angle at B. if BC = 3 units and AC =


2 units AB = y and A = x°, find the value of y using ratio.
Solution: From right angled ABC C

p BC 3
Sinx = h = AC = 2
2
3
or, sinx = sin60°
 x = 60° x°
Now, A y B

b AB
cosx = =
h AC
y
or, cos 60° = 2

1 y 1
or, 2 =2 [cos 60° = 2 ]

 y = 1 unit.

Exercise 6.2
1. Evaluate:
(a) 3cos45° + 2sin30° (b) sin230° + cos245° + sin260°
(c) cos90° + cos245°. sin30°. tan45° (d) cos60° (cos30° + sin60°)
2. Find the value of:
sin245° + cos245° sin30° - sin90° + 2cos0°
(a) tan260° (b) tan30°. tan60°
4 2 2 2 3 2 2
(c) 3 tan 30° + sin 60° - 3cos 60° + 4 tan 60° - 2tan 45°
1 - cos260°
(d) 1-sin230°

246 | Mathematics - 9 Trigonometry


3. Prove that:
(a) cos60° = cos230° - sin230°
(b) 4(sin430° + cos460°) – 3(cos245° - sin290°) = 2
(c) sin30°. cos60° + cos30°. sin60° = sin90°
(d) sin60° = 2sin30. cos30°
4. If A = 30° and B = 60°, verify the following:
tanB - tanA
(a) cosA.cosB – sinA.sinB = cos(A + B) (b) tan(B – A) = 1 + tanB.tanA

B
2tan 2
(c) tanB = B (d) sin(A + B) = sinA.cosB + cosA.sinB
1 - tan2 2

5. Prove that:
1 - tan30° 1 - sin30° sin60°
(a) = (b) = tan30°
1 + tan30° 1 + sin60° 1 + cos60°
(c) 2cos30° - 1 = cos60° (d) (1 + tan260°).cos260° = tan45°
6. Solve for  (0°    90°)
1
(a) 4cos2 = 1 (b) tan = 1
3
(c) 3 sin = sin60° (d) 2sin3 = 3
1
7. (a) If 3sin2 = 24 and  is less than 90°, find the value of .

(b) If 2sin = 1 and  is an acute angle, find the size of .


(c) if 4cos2 = 1 and 0°    90°, find cos2 - sin2.
(d) If  is an acute angle and sin = cos, find the value of  and hence find the value of
2tan2 + sin2 - 1.
8. Find the length of the remaining sides of the following right angled triangles.
(a) A (b) B
A

5cm
60°

C 30° C
B

(c) P (d) A
6cm

Q 45° B
C
48°
R

Trigonometry Mathematics - 9 |247


9. Find the size of the acute angles of the following right angle triangles:
A B
(a) A (b) x


B C
(c) (d) C A
M

C  B

N  O A
10. (a) In ABC, AB = 6cm, AC = 12cm, B = 90° and D
BDAC. Find the trigonometric ratios of sin and
tan.

B C
(b) From the adjoining figure, find the length of BC A
and CD.

30° 60°
D C B

11. Find the acute angles of the right angled triangle by drawing a figure.
(a) The value of Sin is O.5.
(b) The value of Tan is 1
(c) The value of Cos is 0.866.
12. From the given figure, find the value of x.
(a) (b)

248 | Mathematics - 9 Trigonometry


(c) (d)
xm

35 m
30°

5m

(e) (f) A

30  45
D 14.64m C B

Project Work
1. Observe the top of a tree 10 meter away from it’s foot of the tree and find the angle of
elevation using clinometer, then investigate the height of the tree.
2. Prepare a clinometer model.

Unit Test
Time: 40 minutes F.M.- 24
1. Write the six trinometric ratios of angle A in right angled ABC with B = 90° [2]
2. Complete the following: [4]
1
cosec  = ...... sec  × ..... = 1 tan  × cot  = ..... cose2 – ........ = 1

sin  cose  .....


= ....... = ....... ...... + sin2  = 1 cot  = .....
cos  cot 

3. Prove that: (1 + cot2) sin2 = 1 [2]


4. Prepare a table showing the value of trigonometric ratios of standard angles. [2]
A
5. Solve for  (0°    90°): 2 cos = 1 [2]
1
10 cm

6. Prove geometrically: sin 60° = D [4]


2

7. From the given figure, find the value of angle .  [4]


 B C

Trigonometry Mathematics - 9 |249


Answers ____________________________________________________________
Exercise 6.1
1. Show to you subject teacher
7 24 7 5 12 5
2. (a) Sin  = ; Cos  = ; Tan  = (b) Sin  = ; Cos  = ; Tan  =
25 25 24 13 13 12
5 13 5 7 24 7
(c) Sin  = ; Cos  = ; Tan  = (d) Sin  = ; Cos  = ; Tan  =
13 13 12 21 25 24
3. (a) AB = 5 cm, BC = 5 3 cm (b) PQ = 12.47 cm; PR = 14.4 cm
(c) YZ = 14 cm, X = 14 2 cm (d) BD = 21.65 cm, BC = 43.3 cm
4. (a) Q = 30° (b)  = 45° (c)  = 30° (d)  = 30°
5. Show to you subject teacher
4 8 AB 24
6. (a) (b) (c) ,Sin  = (d)
5 17 AC 25
5 3 12
7. (a) Tan  = , Cos  = (b)
12 5 13
3 3 3 5 3 5
8. (a) (c) Sin  = , Tan  = (d) Sin  = ; Cos  = (e)
5 5 4 13 5 12
24
(f) (i) BC = 25 cm (ii) Cos  = (iii)  = 67.4° (iv) 60 cm
25

Exercise 6.2
3 2 1 1 3
1. (a) +1 (b) 1 (c) (d)
2 2 4 2
1 1 25
2. (a) (b) 1 (c) (d) 1
3 2 36
6. (a) 60° (b) 60° (c) 30° (d) 20°
1 3
7. (a) 60° (b) 30° (c) - (d) 45°,
2 2
8. (a) AB = 5 3 cm, BC = 15cm (b) AB = 5 3 cm, AC = 10cm
(c) QR = 8.07cm, PR = 5.4cm (d) AC = 10cm, BC = 14.14cm
9. (a) 30° (b) 60° (c) 45° (d) 23.58°
10. (a) 60° (b) 20m, 60m
11. (a) 30° (b) 45° (c) 30°
12. (a) 40 3 m (b) 56.96 m (c) 10 m
(d) 60m (e) 35.58 m (f) 20 m

250 | Mathematics - 9 Trigonometry


Chapter

7
Statistics
Objectives:
At the end of this chapter, the
students will be able to:
 collect the data, using tally marks,
they can find the frequency table.
 represent the data in bar graph
and circular diagram.
 calculate the average, median,
quartiles of ungrouped data.

Teaching Materials:
Chart paper, data collected for
different purpose, card board paper,
different coloured board markers,
different chart paper related to
graphical representation, etc.
Statistics is the study of collection, presentation, analysis and interpretation of numerical data.
Statistics play very important role to make future plan and policies.
Under presentation, comes graphs and diagrams showing different collected data. The graphs and
diagrams are good visual aids, so here we are going to learn, how to present the collected data in
graphs i.e. histogram, ogive and line graph.

7.1 Graphs and Pie Charts


Line Graph
The rise or fall in prices in different months, rainfall in different years, production in different years,
population in different years etc are shown by a line graph. Thus, the values of a variable at different
time are shown by line graph, which is also known as time series.

Steps to draw line graph


i. Take time along x - axis and values of variable (i.e. temperature, rainfall, rise or fall in price)
along y - axis.
ii. Plot the points on the graph with the help of given information.
iii. Join the points with straight lines.

Worked Out Examples


Example 1: Draw a line graph to show the price rise of potatoes in Rs. for 7 years.
Year 2060 2061 2062 2063 2064 2065 2066
Price/kg(Rs.) 8 15 28 30 35 50 60
Solution: Price of potatoes (2061 BS - 2066 BS)
Y

70
60
50
40
30
20
10
X’ X
0 2060 2061 2062 2063 2064 2065 2066
Y’ Years

252 | Mathematics - 9 Statistics


Pie Chart
A pie chart is a circle divided by radii into sectors. Each sector represents the statistical data. Pie chart
is also known as an angular diagram.
The angle at the centre is 360o. So, the sum of angles representing different items is equal to 360o.

Steps of constructing pie - chart


i. Find the angle at the centre corresponding to each item using the relation,
Value of corresponding item
Angle at the centre = Total value × 360o

ii. Draw a circle with suitable radius and draw the angles representing the angular values of the
items at the centre. The sectors so formed represents the parts of the given items.

Worked Out Examples


Example 1: The monthly expenditure of a family is as below.
Items Food Rent Clothing Education Miscellaneous
Expenditure(Rs.) 6000 4500 2500 10,000 7000

Show it in a pie - chart.


Solution: Here,
Total value = Rs. 30,000
6000
Angle at the centre representing food = × 360o = 72o
30000
4500
Angle at the centre representing rent = 30000 × 360o = 54o
2500
Angle at the centre representing clothing = 30000 × 360o = 30o
10000
Angle at the centre representing education = 30000 × 360o = 120o
7000
Angle at the centre representing miscellaneous = 30000 × 360o = 84o

Miscellaneous
Education Index
Miscellaneous
Education
Clothing
Rent
Clothing
Food Food

Rent

Statistics Mathematics - 9 |253


Example 2: The total number of students enrolled in different streams is 900. It is as
below.
Subject Education Humanities Science Management
No. of students 50 ? 400 300

Find the missing frequency and show it in a pie chart.


Solution: Let, the number of students enrolled in Humanities be x.
 50 + x + 400 + 300 = 900
 x = 150
Then,
50
Central angle representing Education = × 360o = 20o
900
150
Central angle representing Humanities = 900 × 360o = 60o
400
Central angle representing Science = 900 × 360o = 160o
300
Central angle representing Management = 900 × 360o = 120o

Management
Index
Science
Management
Education Education
Science
Humanities

Humanities

Histogram
It is graphical representation of continuous frequency distribution. It is a set of rectangles with class
intervals as bases and their heights as frequencies. There is no gap between two successive rectangles.
Method to draw histogram
i. The data should be in exclusive form. If the data is in inclusive form, it should be converted to
exclusive form.
ii. Class intervals are represented along the x - axis with suitable scale.
iii. The frequencies are represented along the y - axis with suitable scale.
iv. Rectangular bars are constructed with class intervals as bases and frequencies as heights.

254 | Mathematics - 9 Statistics


Worked Out Examples
Example 1: The table shows the ages of people affected by a certain disease during
a year. Represent it in a histogram.
Ages (in years) 5 - 15 15 - 25 25 - 35 35 - 45 45 - 55 55 - 65
No. of cases 6 11 21 23 14 5
Solution:
Y
25

20

15

10

X’ X
0 5 15 25 35 45 55 65
Y’ Ages in years

Example 2: Represent the data given below in a histogram.


C.I 15 - 19 20 - 24 25 - 29 30 - 34 35 - 39 40 - 44 45 - 49
Frequency 3 13 21 15 5 4 2
Solution: The data given above is upper limit inclusive data. It should be converted to upper
limit exclusive data.
Lower limit of exceeding class - Upper limit of preceding class
 Correction factor =
2
20 - 19
= 2 = 0.5

The correction factor is subtracted from lower limit of each class and is added to
upper limit of each class.
C.I Frequency
14.5 - 19.5 3
19.5 - 24.5 13
24.5 - 29.5 21
29.5 - 34.5 15
34.5 - 39.5 5
39.5 - 44.5 4
44.5 - 49.5 2

Statistics Mathematics - 9 |255


Y
25

20

15

10

X’ 0 X
14.5 19.5 24.5 29.5 34.5 39.5 44.5 49.5
Y’ Class interval

The construction near origin along x-axis shows that from 0 to lower limit of the first
bar is not in scale.

Cumulative Frequency Table


The marks obtained by 45 students are as follows,
50, 60, 70, 81, 59, 60, 71, 82, 91, 58, 52, 66, 72, 83, 53, 61, 73, 84, 93, 54, 55, 67, 74, 85, 90, 62, 75,
86, 94, 95, 77, 81, 68, 76, 87, 65, 76, 88, 78, 82, 89, 76, 80, 79, 83.
Let's represent the above data in a cumulative frequency table taking one of the class interval as 50 - 60
Marks Tally bars ‘f’ c.f.
50 - 60 7 7
60 - 70 8 15
70 - 80 12 27
80 - 90 13 40
90 - 100 5 45

In the class, 50 - 60, 50 is the lower limit and 60 is the upper limit. It is the upper limit exclusive form.
That means the upper limit is not included in the class. For example, in the class 50 - 60, the upper
limit 60 is excluded. 60 is included in the class 60 - 70.

Ogive
It is the graphical representation of cumulative frequency distribution of continuous series. Ogive is
also known as cumulative frequency curve. The cumulative frequency means sum of all the
frequencies of the preceding classes and the frequency of the considered class. That means cumulative
frequency is the sum of the frequencies of all classes up to the class considered.
There are two kinds of ogives. They are (i) more than ogive (ii) less than ogive.

256 | Mathematics - 9 Statistics


Worked Out Examples
Example 1: Draw a less than ogive from the following data.
Marks 10-20 20-30 30-40 40-50 50-60 60-70 70-80 80-90 90-100
No. of students 6 5 2 7 2 8 10 3 1
Solution:
Cumulative
Class Frequency Marks
frequency
10 - 20 6 Less than 20 6
20 - 30 5 Less than 30 11
30 - 40 2 Less than 40 13
40 - 50 7 Less than 50 20
50 - 60 2 Less than 60 22
60 - 70 8 Less than 70 30
70 - 80 10 Less than 80 40
80 - 90 3 Less than 90 43
90 - 100 1 Less than 100 44

Example 2: Draw a more than ogive from the following table.


Ages in yrs. 20 - 30 30 - 40 40 - 50 50 - 60 60 - 70
No. of people 5 9 15 6 4

Statistics Mathematics - 9 |257


Solution:
Class F c.f
20 - 30 5 More than 20 39
30 - 40 9 More than 30 34
40 - 50 15 More than 40 25
50 - 60 6 More than 50 10
60 - 70 4 More than 60 4

To draw an ogive
 Prepare less than/greater cumulative frequency table.
 Take classes along x-axis and cumulative frequencies along y-axis.
 Taking lower limit of class (incase of less than ogive) on X coordinate and
corresponding cumulative frequency on y-coordinate plot the points.
 Join the points its order free hand with continuous line.
Finding the median class, lower quartile class and upper quartile class
By using ogives, we can find the class of the quartile, and also estimate the quartiles (Q1, Q2 or
Md, Q3). The curve being down free hand, the quartile values can not be exact.
Example 1: Draw the less than ogive and find the class of median, first quartile
and third quartile from the data.
Marks 0-10 10-20 20-30 30-40 40-50
No. of Students 5 9 12 8 6

258 | Mathematics - 9 Statistics


Solution: Less than cumulative frequency table.
Marks f Marks c.f.
0-10 5 less than 10 5
10-20 9 less than 20 14
20-30 12 less than 30 26
30-4- 8 less than 40 34
40-50 6 less than 50 40
N = 40
Graph:

The graph is the less than ogive, the c.f. value corresponding to the highest point of
the curve is 40.
 N = 40
Nth 40th
Positionof median = 2 item = 2 item = 20th item.

From c.f. value = 20 along y-axis a horizontal line is drawn which cuts the curve at a
point(s). From this point, a vertical line I is drawn that which cuts the x-axis at the
point B lies between 20 and 30.
 Median class = (20-30)
At B, the value of x is 24.
 Estimated median is 24
Nth 40th
Similarly, the first quartile (Q1) = 4 item = 4 item = 10th item. From c.f. value 10
along y-axis horizontal line is drawn which cuts the curve at the point P. From P,
vertical line is drawn which cuts the x-axis at Q. Q lies between 10 and 20.
 Q, class = (10-20)

Statistics Mathematics - 9 |259


At M, value of x is 16. So estimated value of Q1 = 16.
N th 40 th
Similarly, the third quartile Q3 = 3  4  item = 3  4  item = 30th. From c.f. value
   
30 along y-axis, a horizontal line is drawn which cuts the curve at M and from M, a
vertical line is drawn which cuts x-axis at N. The point N lies between (30-40)
 Class of Q3 = (30-40)
At N, value 4x is 35 so, estimated Q3 = 35.
Some important informations from ogives.
i. Difference of the c.f. values of upper limit and lower limit of the class gives the frequency of
the class.
ii. Less than and greater than ogives intersect at a point. The foot of the perpendicular from this
point to x-axis gives the median and foot of perpendicular from the point to y-axis gives N
value.
iii. From ogives we know the value of N and cumulative frequencies of class. Thus, we can reduce
the graph to frequency table.
Note: Lowest marks obtained from above 25% students means Q3 and highest marks obtained from below
25% students means Q1.

Exercise 7.1
1. (a) Draw a line graph of rain fall for 12 months.
Months Bai. Jesh. Ash. Sha. Bha. Asoj. Kart. Man. Poush Magh. Falg. Chai.
Rain fall (mm) 0 5 35 60 50 20 10 5 5 10 5 5
(b) The table below shows the average temperature of different months of Birgunj. Show it
in a line graph.
Months Bai. Jesh. Ash. Sha. Bha. Asoj. Kart. Man. Poush Magh. Falg. Chai.
Temperature
35 42 47 43 40 35 30 25 20 15 20 30
(oC)
(c) The table below shows the population of different years of Jitpur Village. Show it in a
line graph.
Year 2060 2061 2062 2063 2064 2065 2066
Population in thousand 5 6 8 11 15 20 26
(d) Draw a line graph of production of wheat in Chainapur VDC in different years.
Year 2061 2062 2063 2064 2065 2066
Production in
10 15 12 18 20 10
quintals
2. (a) Draw a pie-chart of the expenditure while constructing a house.
Items Labour Bricks Cement Iron Timber
Expenditure (Rs.) 60,000 90,000 1,50,000 2,00,000 1,00,000

260 | Mathematics - 9 Statistics


(b) The monthly expenditure of a family on different items are as follows. Show it in a pie
chart.
Items Food Rent Clothing Education Miscellaneous
Expenditure
7000 6000 5000 12000 18000
(Rs.)
(c) On a survey of certain houses of a municipality about their use of fuel is given as
follows.
Fuel Fire wood Electricity Kerosene Gas
Percentage 10% 15% 30% 45%
Represent the above data in a pie - chart.
(d) In an election, three candidates A, B and C got 60%, 30% and 10% votes. Represent it
in a pie-chart.
3. (a) A family’s budget is given below. Find the missing percentage. Then, represent it in a
pie-chart.
Items Food Rent Clothing Education Saving
Percentage (%) 25 20 ? 30 10
(b) In an election four candidates contested and the percentage of votes they received is
given below.
Candidates A B C D
Percentage (%) 60 15 ? 20
Find the missing percentage and show it in a pie-chart.
(c) The total number of students enrolled in different streams is 1000. It is as follows.
Subjects Education Humanities Science Management
No. of students 100 ? 500 250
Find the missing frequency and show it in a pie-chart.
(d) An alloy 600 gm contains 200gm nickel, 300 gm silver and chromium. Find the
quantity of the chromium. Show it in a pie-chart.
4. (a) The monthly income of 102 working people is given below. Represent it in a histogram.
Monthly 5000- 7000- 9000- 11000- 13000- 15000- 17000-
income (Rs.) 7000 9000 11000 13000 15000 17000 19000
No. of
8 12 15 20 25 17 5
people
(b) The marks obtained by 115 students in mathematics are as follows.
Marks obtained 40 - 50 50 - 60 60 - 70 70 - 80 80 - 90 90 - 100
No. of students 15 10 20 30 25 15
Represent the above data in a histogram.
(c) The daily wages of 82 workers of a factory is given below.
Daily wages
0 - 49 50 - 99 100 - 149 150 - 199 200 - 249 250 - 299
(Rs.)
No. of workers 4 8 15 25 18 12
Represent it in a histogram.

Statistics Mathematics - 9 |261


(d) Show the below data in a histogram.
Class interval 0-9 10 - 19 20 - 2 9 30 - 39 40 - 49
Frequency 22 38 46 35 20
5. (a) Construct a cumulative frequency table of class interval 10.
23 , 5 , 17 , 28 , 39 , 52 , 16 , 21 , 29 , 25 , 41 , 33 , 9, 19 , 34 , 59, 7 , 11 , 51 ,
31 , 2 2 , 41 , 32 , 55 , 18
(b) Construct a cumulative frequency table having one class interval as 40 - 50.
15, 12, 23, 35, 46, 57, 18, 12, 39, 51, 32, 43, 25, 59, 18, 38, 45, 40, 32, 33.
(c) Draw a less than ogive from the following data.
Marks 0 - 10 10 - 20 20 - 30 30 - 40 40 - 50 50 - 60
No. of students 4 6 10 20 6 4
(d) Construct a less than ogive from the following data.
Weight (in kg) 0-4 4-8 8 - 12 12 - 16 16 - 20
No. of children 10 5 7 12 6
6. (a) Draw a more than ogive from the following data.
X 0 - 10 10 - 20 20 - 30 30 - 40 40 - 50
Y 2 3 4 3 1
(b) Construct a more than ogive from the data given below.
Wages in Rs. 15 - 25 25 - 35 35 - 45 45 - 55 55 - 65
No. of workers 4 6 12 5 3

7.2 Measure of Central Tendency


In an arranged data, we find a single value to represent the whole data. It is known as measure of
central tendency. The common measure of central tendency are:
(i) Mean (ii) Median
(iii) Mode (iv) Quartiles.

7.2.1 Mean
Mean is the most used measure of central tendency. It is also called Arithmetic mean. It is defined as
__
the sum of items divided by number of items. It is denoted by X .
sum of items
Arithmetic mean = No. of items

Arithmetic mean of individual series


If x1, x2, x3, ……….., xn represents different items,
__ x1 + x2 + x3 + ……….. + xn x
Then, X = N = N
Where, x = sum of items
N = No. of items
x = item or variable.

262 | Mathematics - 9 Statistics


Worked Out Examples
Example 1: Find the arithmetic mean from the following data 60, 45, 40, 35, 33, 10,
30, 38, 42, 58, 65.
Solution: The sum of items (x) = 60 + 45 + 40 + 35 + 33 + 10 + 30 + 38 + 42 + 58 + 65 = 456.
No. of items (N) = 11
Arithmetic mean (A.M) = ?
We have,
__ x 456
X = N = 11 = 41.4

Hence, the arithmetic mean is 41.4.


Arithmetic mean of discrete series
Let, x1, x2, x3, ……….., xn be the values of a variable and f1, f2, f3, ……… , fn be their corresponding
frequencies. Then,
__ f1x1 + f2x2 ...... + fnxn fx
A.M ( X ) = f + f + .........+ f = N
1 2 n

Example 2: Find the mean from the following data.


Marks obtained 15 20 30 40 50
No. of students 6 12 11 6 5
Solution:
Items (x) Frequency ‘f’ f.x
15 6 90
20 12 240
30 11 330
40 6 240
50 5 250
N = f = 40 fx = 1150
__ fx
 A.M ( X ) = N

1150
= = 28.75
40
Therefore, the arithmetic mean is 28.75.
Example 3: The marks obtained by 40 students are as follows
10, 30, 50, 70, 90, 100, 10, 30, 50, 70, 90, 100, 10, 30, 50, 70, 90,
100, 30, 50, 70, 90, 100, 30, 50, 70, 90, 30, 50, 70, 90, 50, 70, 90,
50, 70, 70, 70, 70, 70.
Represent the above data in a discrete frequency table and calculate the
arithmetic mean.

Statistics Mathematics - 9 |263


Solution: Representing the data in discrete frequency table
Items (x) Frequency ‘f’ f.x
10 3 30
30 6 180
50 8 400
70 12 840
90 7 630
100 4 400
N = f = 40 fx = 2480
fx 2480
A.M = N = 40 = 62

Therefore, the arithmetic mean is 62.


Example 4: If the A.M is 17, find the value of ‘a’ from the following data.
x 5 10 15 20 25 30
f 2 5 a 7 4 2
Solution:
x f fx
5 2 10
10 5 50
15 a 15a
20 7 140
25 4 100
30 2 60
N = f : 20 + a fx = 360 + 15a
We have,
fx
A.M =
N
360 + 15a
or, 17 = 20 + a
or, 340 + 17a = 360 + 15a
or, 2a = 20
 a = 10
Hence, the value of ‘a’ is 10.

264 | Mathematics - 9 Statistics


Exercise 7.2.1
1. (a) Find the A.M of the following data.
100, 125, 120, 140, 150, 130, 160, 145
(b) The weights of 10 students are 35kg, 33kg, 45kg, 48kg, 52kg, 55kg, 72kg, 78kg, 65kg
and 67kg. Find the mean weight.
(c) The weight in kg of 12 people are as follows.
77, 80, 82, 83, 85, 77, 80, 82, 83, 77, 80, 77
Represent it in a discrete frequency distribution and calculate the mean weight.
(d) The marks obtained by 20 students are as follows.
70, 60, 50, 40, 30, 20, 70, 60, 40, 30, 20, 70, 60, 30, 20, 70, 60, 70, 70, 70.
Represent the above data in frequency distribution table and calculate the arithmetic
mean.
2. (a) In a data, the sum of variable is X and sum of frequency is N. Then write down the
–).
formula to calculate mean (x
(b) If X = 50 and N = 10 the. Find mean (x–).
(c) If the sum of 5 number is 45, then find the mean.
(d) If x = 55, –x = 5 then find the valeu of f .
3. (a) –) = 50, then find the number of terms.
In a data. fx = 750, mean (x
(b) –).
In a data (x) = 28 + 4a, N = 7 + a, find mean (x
(c) Find the value of ‘a’ if the mean of 10, 20, a, 40, 50 and 60 is 35.
(d) The mean of 2, a, 6, 8 and 10 is 6. Determine the value of ‘a’.
4. (a) In a data, if mean (x–)= 20, fx = 400 + 20a and F = 18 + 2a then, find the value of a.
(b) In a data, if mean –x = 10, N = 40 + 3a and Fx = 700 + 5a, then find the value of a.
m
(c) In a data the mean –x = 45, fx = 426 and N = 7 + , find the value of m.
15
(d) Average of 9 terms is 60 where average of first five term is 70 and the average of last 5
terms is 55, find the fifth term.
5. (a) Find the mean from the following data.
Marks obtained 10 20 30 40 50 60 70
No. of students 2 5 8 15 19 7 4
(b) Find the mean from the following data.
Wages in Rs. 10 20 30 40 50 60
No. of persons 2 3 9 20 11 5

Statistics Mathematics - 9 |265


6. (a) If the A.M of the following data is 28.75, find the missing frequency.
x 15 20 30 40 50
f 6 ? 11 6 5
(b) If the A.M of the following data is 34, find the value of p.
Marks obtained 15 25 35 45 55
No. of students 7 8 p 7 6
(c) If the mean mark is 58.4, find the missing mark from the following data.
Marks 20 30 50 ? 70 80 90
No. of students 3 4 15 10 8 7 3
(d) If the average mark is 20.5, find the value of b from the following data.
Marks obtained 10 15 b 25 30
No. of students 5 10 18 9 8
(e) If AM of the following observations is 26.25, find the value of m.
Marks 12 20 24 35 42
f 5 m+3 15 8 5

7.2.2 Median
10, 20, 30, 40, 50, 60, 70
Let's observe the adjoining arranged data. 3 terms 3 terms
40 is dividing the arranged data exactly into two equal parts. So, 40 is the median. Thus, median is an
item in arranged data i.e. either in ascending order or descending order dividing the series exactly in
two equal parts. It is the positional average.
In individual series,
N + 1 th
Median =  2  item.
 
In discrete series,
N + 1 th N+1
Median = 
 2  item, the value corresponding to c.f. value 2 or just greater then it.
Where N = No. of items.

Worked Out Examples


Example 1: Find the median of the following data.
5, 25, 35, 15, 45, 55, 65
Solution: Arranging the data in an ascending order,
5, 15, 25, 35, 45, 55, 65
No. of items (N) = 7
N + 1 th
Median =  2  item
 

266 | Mathematics - 9 Statistics


7 + 1 th
=  2  item
 
th
= 4 item
= 35.
Hence, 35 is the median.
Example 2: The marks obtained by 8 students in a unit test are as follows.
12, 10, 2, 14, 8, 16, 18, 6. Find the median mark.
Solution: Arranging the marks in an ascending order,
2, 6, 8, 10, 12, 14, 16, 18
No. of items (N) = 8
N + 1 th
Median =
 2  item
8 + 1 th
=  2  item
 
= 4.5th item.
Since, 4.5th item is the average of 4th item and 5th item.
4th item + 5th item
Median = 2
10 + 12
= 2
22
= 2

= 11
Hence, median mark is 11.
Example 3: Find the median of the following data.
x 5 10 15 20 25 30
f 2 5 10 7 4 2
Solution:
Item ‘x’ Frequency ‘f’ Cumulative frequency c.f
5 2 2
10 5 7
15 10 17
20 7 24
25 4 28
30 2 30
N = f = 30
N + 1 th
Median =  2  item
 
30 + 1 th
=  2  item = 15.5th item.
 

Statistics Mathematics - 9 |267


As there is no c.f. value 15.5 in cumulative frequency column, we have to take next
higher c.f i.e 17.
 Median = 17th item = 15.
Hence, median is 15.
Example 4: The data 3, 6, x + 6, x + 10, 15, 18 are arranged in an ascending order. If
the median is 11, find the value of x.
Solution: The data in the ascending order is:
3, 6, x + 6, x + 10, 15, 18
N + 1 th
Median =  2  item
 
6 + 1 th
=
 2  item
or, 11 = 3.5th item
3rd item + 4th item
or, 11 = 2
x + 6 + x + 10
or, 11 = 2
or, 22 = 2x + 16
or, 6 = 2x
 x =3
Hence, the value of x is 3.

Exercise 7.2.2
1. Find the median of the following data:
(a) 4, 16, 12, 8, 20. (b) 5, 45, 65, 95, 75, 85, 55
(c) 3, 18, 9, 15, 27, 30, 12, 6, 33 (d) 10, 30, 20, 15, 25, 40, 19, 17, 43
2. Find the median of the following data:
(a) 2, 8, 6, 4, 9, 7, 1, 5, 7, 3, 8, 6 (b) 2, 20, 12, 6, 4, 16, 18, 14
(c) 31, 38, 27, 28, 36, 25, 35, 40, 28, 51 (d) 10, 40, 60, 30, 20, 50
3. (a) The data 10, 14, x + 3, x – 3, 26, 29 are in an ascending order. If the median is 20,
calculate the value of x.
(b) The data 5, 10, x + 5, x + 10, 25, 30 are arranged in an ascending order. If the median is
17.5, calculate the value of x.
(c) The median of the data arranged in an ascending order 7, 9, x + 3, 13, 15 is 11.
Calculate the value of x.
(d) The median of the data arranged in an ascending order 5, 10, 15, x + 4, 25, 30, 35 is 20.
Find the value of x.
(e) If the given data is in an ascending order and the median is 70, find the value of x:
3x + 5
50, 60, 2 , 80, 90.

268 | Mathematics - 9 Statistics


4. (a) The marks by 50 students of class IX are given below.
Marks 20 30 40 50 60 70
No. of students 4 5 8 10 6 5
Calculate the median.
(b) Calculate the median from the following data.
Ages in years 25 32 43 54 65
No. of people 35 42 40 30 24
(c) Calculate the median.
x 10 15 20 25 30
f 5 7 9 12 6
(d) Find the median:
x 2 3 5 7 9
f 9 4 6 3 8
7.2.3 Mode
In a distribution, the item with the highest frequency is known as mode. There may exist more than
one mode. The distribution with two modes is known as bimodal. In case of a discrete series, we find
the mode by inspection.

Worked Out Examples


Example 1: Find the mode from the following distribution.
5, 2, 3, 5, 2, 5, 3, 5, 5, 4
Solution: The item 5 is repeated maximum times, so the mode is 5.
Example 2: Find the mode.
Marks 20 40 60 80 100
No. of students 5 15 25 40 10
Solution: By inspection, we come to know that maximum number of students has scored 80
marks. So, 80 is the mode.

Exercise 7.2.3
1. Find the mode
(a) 10, 20, 20, 40, 20, 10, 20.
(b) a, b, c, b, d, b, e, b
2. Find the mode from the given data:
(a) Weight in kg 30 35 40 42 45 50
No. of children 3 5 10 18 20 2

(b) Ages in yrs. 13 14 15 16 17 18


No. of children 5 15 20 10 3 1

Statistics Mathematics - 9 |269


7.2.4 Quartiles
Let’s observe the data.
10, 20, 30, 40, 50, 60, 70, 80, 90, 100, 110
The items 30, 60 and 90 are dividing the arranged data into four equal parts. So, they are called
quartiles 30 is the lowest or first quartile (Q1). 25% items are below 30 and 75% items are above it. 60
is the second quartile (Q2) or median (Md). 50% items are above it and 50% items are below it. 90 is
the third quartile or upper quartile (Q3). 25% items are above it and 75% items are below it.
N + 1 th
1st quartile (Q1) =  4  item
 
N + 1 th
2nd quartile (Q2) = 2
 4  item
N + 1 th
3rd quartile (Q3) = 3 4  item
 

Worked Out Examples


Example 1: Find the lower and upper quartiles from 5, 12, 22, 15, 25, 32, 17, 4, 45, 56,
46.
Solution: Arranging the data in an ascending order 4, 5, 12, 15, 17, 22, 25, 32, 45, 46, 56
No. of items (N) = 11
N + 1 th
Lower quartile (Q1) =  4  item
 
11 + 1 th
= rd
 4  item = 3 item = 12.
N + 1 th
Upper quartile (Q3) = 3 4  item
 
11 + 1 th
= 3 th
 4  item = 9 item = 45.
Example 2: Find the lower quartile and upper quartile from the following data.
4, 16, 8, 24, 20, 12
Solution: Arranging the data in an ascending order,
4, 8, 12, 16, 20, 24
Number of items (N) = 6
N + 1 th
Position of Q1 =  4  item
 
6 + 1 th
=
 4  item
= 1.75th item
Lower quartile (Q1) = 1st item + 0.75 (2nd item – 1st item)

270 | Mathematics - 9 Statistics


= 4 + 0.75(8 – 4)
=4+3
=7
Now,
N + 1 th
Position of Q3 = 3 4  item
 
= 5.25th item
Upper quartile (Q3) = 5th item + 0.25 (6th item – 5th item)
= 20 + 0.25 (24 – 20)
= 21
Example 3: Find the upper quartile and lower quartile from the following data.
Wages in Rs. 30 80 70 40 60 50
No. of workers 4 4 5 2 5 7
Solution: Arranging the data in an ascending order:
Item (x) f c.f.
30 4 4
40 2 6
50 7 13
60 5 18
70 5 23
80 4 27
N = f = 27
We know,
N + 1 th
Position of Q1 =  4  item
 
27 + 1 th
= th
 4  item = 7 item.
c.f. just greater than 7 is 13, so the corresponding value of 13 is 50.
(In c.f. column, there is not c.f. value 7, so we have to take the next higher c.f. i.e. 13th
item.)
So Q1 = 50
N + 1 th 27 + 1 th
Position of Q3 = 3 4  item = 3 4  item = 21st item
   
c.f. just greater than 21 is 23, so the corresponding value of 23 is 70.
 Q3 = 70

Statistics Mathematics - 9 |271


Exercise 7.2.4
1. Find the lower quartile of the data:
(a) 5, 12, 18, 14, 6, 4, 9 (b) 3, 15, 20, 5, 12, 17
(c) 2, 9, 15, 10, 18, 14, 12, 8 (d) 10, 90, 20, 80, 70, 50, 60, 30, 40, 110, 100
2. Find the upper quartile of data.
(a) 3, 9, 7, 4, 6, 8, 5 (b) 0, 16, 12, 4, 8, 20
(c) 5, 20, 30, 40, 25, 45, 55, 35 (d) 5, 25, 45, 95, 15, 35, 55, 65, 75, 85, 105
3. Find the lower quartile of the data.
(a) Weight (kg) 50 48 46 44 42 40 38 36 34
No. of students 2 2 3 4 6 9 8 4 3

(b) Marks 10 20 30 40 50 60 70
No. of students 2 5 8 15 19 7 4
4. Find the upper quartile of the data.
(a) Wages in Rs. 10 60 50 40 20 30
No. of persons 4 4 10 15 5 8

(b) x 15 30 50 20 40
f 7 10 6 11 9
5. (a) If first quartile of the given observations taken in order is 15, find the value of 'P'.
11, P + 5, 18, 24, 30, 35, 40
(b) If lower quartile of the observations taken in order given below is 20, find the value of 'm':
12, 15 + m, 16 + 2m, 28, 34, 38, 45, 48, 52.
(c) If upper quartile of the observation given below taken in order is 39, find 'a':
8, 12, 15, 20, 27, 30 + a, 52 – 2a, 45.
(d) If third quartile of the observations taken in order is 172, find the value of 'k'.
102, 112, 120, 130, 142, 150, 158, 150 + 2k, 188 – k, 182

Project Work
1. Collect the marks obtained by the students of grade IX and X in mathematics, prepare
frequency table and compare average mark.
2. Collect the marks obtained by the students of your class in mathematics in the last two
examinations, find the means and express your views about the progress of the students.
3. Collect the heights of the students of two sections of a class and compare their average heights.

272 | Mathematics - 9 Statistics


Unit Test
Time: 40 minutes F.M.- 24
Group-A (2×1=2)
1. Find the mode of the data:
7. 5, 6, 9, 5, 7, 5, 7, 8, 9, 7, 6
2. Days of the months are 28, 29, 30, 31, 32. What is the average?
Group B (3×2= 6)
3. Find the lower quartile of the data:
5, 6, 13, 16, 18, 23, 26, 33, 46, 47, 57
4. In a data, x = 42 + 24a and N = 7 + 4a, find the numerical value of the mean (X̄).
5. The data 25, x + 5, x + 11, 14, 18 are arranged in order. If the median is 12, find the
value of x.
Group (5 × 4= 12)
6. Show the given data in a pie-chart.
Subject Education Humanities Science Management
No. of students 50 400 50 300
7. If AM of the following observation is 26.25, find the value of m.
Marks 12 20 24 35 42
No. of workers 5 7 m+5 8 5
8. Find the upper quartile of the data:
Wages (Rs.) 300 800 700 400 600 500
No. of workers 4 4 5 2 5 7

Statistics Mathematics - 9 |273


Answers ____________________________________________________________
Exercise 7.1
Show to your teacher
Exercise 7.2.1
1. (a) 133.75 (b) 55 (c) 80.25 (d) 13
X
2. (a) x̄ = (b) 5 (c) 9 (d) 11
N
3. (a) 15 (b) 4 (c) 30 (d) 4
4. (a) 2 (b) 12 (c) 37 (d) 85
5. (a) 43.5 (b) 40
6. (a) 12 (b) 2 (c) 60 (d) 20 (e) 4
Exercise 7.2.2
1. (a) 12 (b) 65 (c) 15 (d) 20
2. (a) 6 (b) 13 (c) 33 (d) 35
3. (a) 20 (b) 10 (c) 8 (d) 16 (e) 45
4. (a) 50 (b) 43 (c) 20 (d) 5
Exercise 7.2.3
1. (a) 20 (b) b
2. (a) 45 (b) 15
Exercise 7.2.4
1. (a) 5 (b) 4.5 (c) 8.25 (d) 30
2. (a) 8 (b) 17 (c) 43.75 (d) 85
3. (a) 38 (b) 40
4. (a) 50 (b) 40
5. (a) 10 (b) 3 (c) 6 (d) 10

274 | Mathematics - 9 Statistics


Chapter

8
Probability
Objectives:
At the end of this chapter, the
students will be able to:
 understand the meaning of
probability and use the probability
practically in their life.
 find the probability of different
events in the experiment.

Teaching Materials:
Chart paper, related to probability,
flash cards, playing cards, dice,
different colour.
8.1 Probability
Introduction
When we observe in the nature and in our daily life, there are two types of facts which are called
deterministic and probabilistic. The fact under which the result can be predicted with certainty is
called the deterministic fact. For example, the day after Monday is always Tuesday.
The fact under which the result cannot be predicted with certainty is called probabilistic fact. For
example, in tossing a coin, we are not sure whether the head or tail of the coin. Even if the sky is
clouded, we are not sure whether it rains.
The two above examples are the cases of uncertainty because nobody can say with certainty whether it
will happen or not. Thus, probability is the chance that something will happen. Probability is a
concept which numerically measures the degree of uncertainty and certainty of the occurrence of
events.

Definitions of basic terms


There are some basic terms which are used in the probability. We shall give certain concept about
these basic terms for the probability.
Experiment
The process of performing the results is called an experiment. In other words, an operation by which
an observation is made is called an experiment. For example, tossing a coin, throwing a die. It is also
called a trial.
Random experiment
An experiment whose result can not be predicated in advance is called a random experiment. In other
word, an experiment whose result is not fixed but may be one of the various possible outcomes is
called the random experiment. For example, if we toss a coin, we may get either head or tail. If we
draw a card from a pack of 52 cards, we may get either face card or non-faced card. So, tossing a coin,
drawing a card are the examples of random experiments.
Outcomes
The results of a random experiment are called outcomes. For example, while tossing a coin, the
occurrence may be a head or a tail. So, the head or the tail is the outcomes of this experiment.
Sample space
The set of all possible outcomes of a random experiment is called the sample space. The sample
space of the random experiment is usually denoted by S. For examples;
i. The possible outcomes in tossing a coin is a head (H) or a tail (T)
The sample space (S) = {H, T}
ii. The possible outcomes while throwing a die is 1 or 2 or 3 or 4 or 5 or 6. Hence the sample
space (S) = {1, 2, 3, 4, 5, 6}

276 | Mathematics - 9 Probability


iii. The possible outcomes while tossing a coin two times simultaneously is H and H or H and T or
T and H or T and T.
The sample space (S) = {HH, HT, TH, TT}
iv. The possible outcomes in a coin is H or T and in a die is 1 or 2 or 3 or 4 or 5 or 6. So, in a
simultaneous toss of a coin and die, the sample space is
S = {H, T} × {1, 2, 3, 4, 5, 6}
= {(H, 1), (H, 2), (H, 3), (H, 4), (H, 5), (H, 6), (T, 1), (T, 2), (T, 3), (T, 4), (T, 5), (T, 6)}
Event
The results of a random experiment are called outcomes. The subset of a sample space is called an
event. The performing of a random experiment is called a trial. The outcome of every trial is called an
event. For example rolling of a die is a trial and turning up one of the faces 1, 2, 3, ……….. , 6 is an
event.
An event containing only one element of S (a single sample point) is called a simple event. It is also
known as elementary event. For example, the sample space in tossing a coin is {H, T}. then {H} and
{T} are the simple event.
Exhaustive cases
The total number of all possible outcomes of a random experiment is called the exhaustive cases. For
example, in case of tossing a coin, the all possible outcomes are {H} and {T}. So the total number of
all possible outcomes is 2. So, the exhaustive cases is 2. Similarly, in case of a die, the total number of
all possible outcomes is 6. Therefore, the exhaustive cases of rolling a die is 6.
Favourable cases
The outcomes of a random experiment which are desirable in an event are called the favourable
cases. For example, when a coin is tossed, the desirable outcomes are head (H) and tail (T). So, the
favourable case of head is 1 and tail is also 1.
When two coins are tossed together, then the sample space is S = {HH, HT, TH, TT}
The number of favourable cases getting both of them tail is 1. The number of favourable cases getting
of at least one tail is 3.
Equally likely cases
Two or more events are said to be equally likely if all of these events have an equal chance of
occurring in the experiment. For example, when a coin is tossed, then the events of the head and the
tail are equally likely cases.
Probability of an event
Let S be a sample space of a random experiment and E be the favourable events of the sample space.
Then n(S) and n(E) are the cardinal number of sample space and favourable events respectively. The
probability of happening of an event (E) is defined by P(E) as:
Number of favourable events n(E)
P(E) = Exhaustive case = n(S)

Probability Mathematics - 9 |277


For example,
What is the probability of getting a head in tossing a coin?
Here, sample space (S) = {H, T} n (S) = 2
Favourable event (E) = {H} n (E) = 1
n(E) 1
Now, P(E) = =
n(S) 2
Probability of non-event

If P(E) is the probability of happening an event E and P(¯¯


E ) is the probability of non-happening an
event E, then the total probability must be unity.

or, P(E) + P(¯¯


E)=1
1
The probability of a head in a coin is P(H) = , then the probability of not getting a head is
2
1 1
P(¯¯
H ) = 1 - P(H) = 1 - 2 = 2 .

Probability scale (probability range)


Let's consider a fair die in which numbers 1 to 6 are written in the faces. Then
1 1
The probability of turning up 1 = P(1) = 6 The probability of turning up 2 = P(2) = 6

1 1
The probability of turning up 3 = P(3) = 6 The probability of turning up 4 = P(4) = 6

1 1
The probability of turning up 5 = P(5) = 6 The probability of turning up 6 = P(6) = 6

0
The probability of turning up 7 = P(7) = 6 = 0

The probability of turning up 7 is 0 because a die consists or the number 1 to 6 only and it contains
only six faces. There is no face 7 in a die.
The probability is 0 means there is no chance of an event happening.
Thus, the probabilities lie between 0 to 1.

Nature of probability
i. P(E)  0 for any event E in the sample space S.
ii. 0  P(E)  1 for any event E in the sample space.
iii. P(E) = 0 for any event E is impossible event.
iv. P(E) = 1 for any event E is sure event.

278 | Mathematics - 9 Probability


Worked Out Examples
Example 1: A card is drawn randomly from a pack of 52 cards. Find the probability
of getting an ace.
Solution: Here, in a pack of cards, there are 52 cards.
n(S) = 52
Out of 52 cards, there are 4 ace cards.
n(E) = 4
n(E) 4 1
Now, P(E) = n(S) = 52 = 13

1
Hence, the probability of getting an ace is 13 .

Example 2: A marble is drawn at random from a box containing 3 white marbles and
7 black marbles. What is the probability that a marble drawn is white?
Solution: Here, total number of marbles in the box = 3 + 7 = 10
n(S) = 10
Number of white marbles n(E) = 3
n(E) 3
Now, P(E) = n(S) = 10

3
Hence, the probability of getting white marbles is 10 .

Example 3: A bag contains 6 red, 4 blue and 3 green balls of the same shape and
size. If a ball is drawn at random, find the probability of not getting a blue
ball.
Solution: Here, there are 6 red, 4 blue and 3 green balls in a bag.
Total number of balls = 6 + 4 + 3 = 13
n(S) = 13
Favourable number of events of getting a blue ball is 4.
n(B) = 4
n(B) 4
Now, P(B) = n(S) = 13

 The probability of not getting the blue ball = 1 – P(B)


4
= 1 – 13

13 - 4 9
= 13 = 13

Example 4: What will be the probability of getting 4 when a dice is rolled once?
Solution: Here, the sample space in a dice (S) = {1, 2, 3, 4, 5, 6}
 n(S) = 6

Probability Mathematics - 9 |279


Favorable event (E) = {4}
 n(E) = 1
n(E) 1
Now, P(E) = n(S) = 6

1
Hence, the probability of getting 4 is
6

Example 5: A bag contains 4 identical balls of red, black, blue and green colours. A
ball is drawn at random from the bag, find the probability of getting a red
ball.
Solution: Here, total number of balls in a bag is 4.
 n(S) = 4
The number of red ball in the bag is 1.
 n(R) = 1
n(R) 1
Now, P(R) = n(S) = 4

1
Hence, the probability of getting a red ball is 4

Example 6: Find the probability of giving birth to a child by a pregnant woman on


Thursday only.
Solution: Here, the sample space of a week (S)
= {Sunday, Monday, Tuesday, Thursday, Wednesday, Friday, Saturday}
 n(S) = 7
Favorable event (E) = {Thursday}
 n(E) = 1
n(E) 1
Now, P(E) = n(S) = 7

1
Hence, the required probability is 7

Example 7: Find the probability of drawing a black king from a well shuffled pack of
52 cards.
Solution: Here, total number of cards = 52
 n(S) = 52
Out of 52 cards, there are 2 black kings.
So, the exhaustive cases of black kings, n(E) = 2
n(E) 2 1
Now, P(E) = n(S) = 52 = 26
1
Hence, the required probability of black king is 26 .

280 | Mathematics - 9 Probability


Example 8: When a die is thrown once, find the probability of getting a number
divisible by 3.
Solution: Here, the sample space in a die (S) = {1, 2, 3, 4, 5, 6}
 n(S) = 6
The favorable events divisible by 3 (E) = {3,6}
 n(E) = 2
n(E) 2 1
Now, P(E) = n(S) = 6 = 3

1
Hence, the probability of getting a number divisible by 3 is 3 .

Example 9: From the number cards numbered from 2 to 20, a card is drawn at random.
Find the probability of getting the card numbered the multiple of 4.
Solution: Here, the sample space of the numbered cards (S) = {2, 3, 4, 5, 6, ... ..., 20}.
 n(S) = 19
The favorable cases which is the multiple of 4 is
E = {4, 8, 12, 16, 20}
 n(E) = 5
n(E) 5
Now, P(E) = n(S) = 19

5
Hence, the probability of getting the card numbered the multiple of 4 is 19 .

Example 10: From the number cards, numbered from 1 to 15, a card is drawn at
random. Find the probability of getting a card having a prime number.
Solution: Here, let S be the sample space. Then
S = {1, 2, 3, 4, ... ..., 15}
 n(S) = 15
Let E be the event of getting a prime number.
Then, E = {2, 3, 5, 7, 11, 13}
 n(E) = 6
n(E) 6 2
Now, P(E) = n(S) = 15 = 5

2
Hence, the probability of getting a card having a prime number is 5 .

Example 11: What is the probability that a leap year selected at random will contain
53 Sundays?
Solution: Here, we know that, a leap year contains 366 days i.e 52 weeks and 2 days. So, there
are 52 Sundays in 52 weeks. For the remaining 2 days, they may be Sunday and
Monday, Monday and Tuesday, Tuesday and Wednesday, Wednesday and Thursday,
Thursday and Friday, Friday and Saturday, Saturday and Sunday.

Probability Mathematics - 9 |281


n(S) = 7
Let E be the event of selecting a leap year containing 53 Sundays. Then
E = {Sunday and Monday, Saturday and Sunday}
 n(E) = 2
n(E) 2
Now, P(E) = n(S) = 7

Example 12: Three unbiased coins are tossed simultaneously. Write down the sample
space. Find the probability of getting:
(i) all tails (ii) one head
(iii) at most one tail (iv) exactly one head or two heads.
Solution: Here, let S be the sample space. Then, the sample space tossing three coins (S)
= {H, T} × {H,T} × {H, T} = {HHH, HHT, HTH, HTT, THH, THT, TTH, TTT}
 n(S) = 8
(i) The favourable cases of all tails (E) = {TTT}
 n(E) = 1
n(E) 1
 n(E) = n(S) = 8

(ii) The favourable cases of one head (E) = {HTT, THT, TTH}
 n(E) = 3
n(E) 3
 n(E) = n(S) = 8

(iii) The favourable cases of at most one tail (E) = {HHH, HHT, HTH, THH}
 n(E) = 4
n(E) 4 1
 n(E) = n(S) = 8 = 2

(iv) The favourable cases of exactly one head or two heads (E) = {HHT, HTH, HTT,
THH, THT, TTH}
 n(E) = 6
n(E) 6 3
 n(E) = n(S) = 8 = 4

Exercise 8.1
1. (a) Define the probability with an example. What is the probability of that event is certain?
(b) What is the probability of an event that is impossible?
(c) If the number of favorable outcomes and possible outcomes are n(E) and n(S)
respectively, find the probability of the event E.

282 | Mathematics - 9 Probability


(d) What do you mean by sample space in probability? Write the sample space of the
following random experiments.
(i) Rolling a die.
(ii) Tossing two coins simultaneously.
(iii) Tossing three coins simultaneously.
(e) Define the favorable and unfavorable events with an example.
(f) What is the scale of probability ?
2. (a) A card is drawn from a well shuffled pack of 52 cards. What is the probability that:
(i) the card drawn will be an ace?
(ii) the card drawn will be a black coloured king?
(iii) the card drawn will be a face card?
(b) A card is drawn randomly from a pack of 52 cards. Find the probability of getting the
card which is an ace to 10.
(c) A card is drawn randomly from a well shuffled pack of 52 cards. What is the probability
that the card so drawn will be a queen?
(d) A card is drawn at random from a pack of 52 cards. What is the probability that the
(i) card so drawn will be a black queen?
(ii) card so drawn will not be a club?
(iii) card so drawn will be a red face card?
3. (a) A bag contains 6 red and 8 black marbles. If a marble is drawn randomly from the bag,
find the probability of getting that the marble is red.
(b) A box contains 3 green, 5 white and 7 blue marbles. If a marble is drawn at random,
find the probability of not getting a white marble.
(c) A bag contains 8 different coloured marbles. A marble is drawn at random from the bag.
What will be the probability of getting a green marble?
(d) A box contains 3 blue and 5 black marbles. If a marble is drawn randomly from the box,
what will be the probability of getting a blue marble?
(e) A marble is drawn from a box containing 15 black, 5 green, 10 red and 10 yellow
marbles. Find the probability of getting a black marble.
4. (a) A bag contains 4 identical balls of red, black, blue and green colours. A ball is drawn
randomly from the bag, find the probability of getting a red ball.
(b) A basket contains 3 red, 4 black and 5 white balls. A ball is drawn at random from the
basket. Find the probability of not getting a black ball.
(c) A box contains 4 red, 5 black and 6 white balls. If a ball is drawn randomly from the
box, find the probability of not getting a white ball.
(d) From a bag containing 5 red and 10 yellow balls, if one ball is drawn at random, what is
the probability that the ball will be yellow?
5. (a) What will be the probability of getting 5 when a die is rolled once?
(b) A dice is rolled, find the probability that the face turned up may be even number only.

Probability Mathematics - 9 |283


(c) Find the probability of not getting 6 when a dice is rolled once.
(d) A die is rolled, find the probability that the face turned up may be a number the which is
multiple of 3.
(e) When a die is rolled, what will be the probability of getting the face turned up may be a
number
(i) divisible by 5? (ii) less than or equal to 1? (iii) greater than 6?
6. (a) Find the probability of giving birth to a child by a pregnant woman on Tuesday only.
(b) What is the probability of giving birth to a child by a pregnant woman on Monday only?
(c) In a class of 51 students, there were 17 boys and the rest were girls. Find the probability
of choosing a girl as the monitor at random.
(d) In a class of 40 students, 3 boys and 5 girls wear spectacles. The principal calls one of
the students in the office. Find the probability that this student is wearing a pair of
spectacles.
7. (a) From the cards numbered 1 to 20, a card is drawn randomly, find the probability of
getting the card numbered the multiple of 3.
(b) A number card numbered from 5 to 32 is drawn randomly. Find the probability of
getting the card having a prime number.
(c) Flash cards numbered from 2 to 21 is drawn at random. Find the probability of getting
that the card is the multiple of 5.
(d) A bag contains 20 tickets marked with numbers 1 to 20. One ticket is drawn at random.
Find the probability that it will be a multiple of 4.
8. (a) Find the probability of touching one letter of the 'probability' by closing the eyes.
(i) touching 'r' (ii) touching 'b' (iii) not touching 'b'
(iv) not touching 'p' (v) touching 'i'
(b) Find the probability of touching the following letters of the word 'MATHEMATICS' by
closing the eyes.
(i) touching 'M' (ii) touching 'T' (iii) not touching 'T'
(iv) not touching 'A' (v) touching 'A'
(c) A leap year is selected at random. What is the probability that it will contain,
(i) 53 Mondays? (ii) 53 Thursday?
(d) A non-leap year is selected at random. What is the probability that it will contain 53
Fridays?
9. (a) Two coins are tossed together simultaneously. Write down the sample space. Also
calculate the probability of getting,
(i) exactly two tails (ii) one head or one tail in any order 2
(iii) at least one head iv) at most one tail
(b) While rotating the needle on the given spinner as in the figure, 1 3
write down the probability of the needle stopping at 1.
4

284 | Mathematics - 9 Probability


(c) While rotating the needle on the given spinner as in the figure,
find the probability of the needle to stop at 6 2
(i) the number 4. 4 8
(ii) the sector of even numbers only. 1 5
(iii) the sector of numbers exactly divisible by 4. 7 3

8.2 Empirical probability


1
If a coin is tossed at random, then the theoretically probability gives the probability of a head as 2.
Thus, if a coin is tossed 30 times, then the theoretically probability gives the probability of the head as
1
2 × 30 times, i.e. 15 times. However, in real experiment it may not happen. In fact, in 30 throws of a
coin, it may not turn up head at all or 2 or 5 heads or any other number of times. The probability of
any event which is calculated on the basis of the number of actual experiment is called an
empirical probability.
If n(S) is the total number of times observed an experiment is repeated at random and n(E) be the
number of observed outcomes, then the empirical probability of getting an event E is given by
Number of observed outcomes n(E)
Empirical probability = P(E) = Total number of times on experiment is repeated = n(S)

Worked Out Examples


Example 1: If a coin is tossed 200 times, then the occurrence of tail is 125 times.
Find the probability of (i) head and (ii) tail.
Solution: Here,
A coin is tossed 200 times.
 n(S) = 200
The occurrence of tail is 125.
 The occurrence of head = 200 - 125 = 75
(i) n(H) = 75
n(H) 75 3
P(H) = n(S) = 200 = 8

(ii) n(T) = 125


n(T) 125 5
P(T) = = =
n(S) 200 8

Example 2: A dice is thrown 1200 times and the record of outcomes is given in the
following table.
Outcomes 1 2 3 4 5 6
Frequency 186 205 211 187 204 207

Probability Mathematics - 9 |285


Calculate the empirical probability that of the given outcomes
(i) less than 4
(ii) greater than 3.
Solution: Here,
The total number of trials = 1200 times
 n(S) = 1200
(i) The outcomes less than 4 are 1, 2 and 3.
 n(E) = 186 + 205 + 211 = 602
n(E) 602 301
P(E) = n(S) = 1200 = 600

(ii) Let E be the event of occurrence of number greater than 3.


 n(E) = 187 + 204 + 207 = 598
n(E) 598 299
P(E) = n(S) = 1200 = 600

Example 3: In a survey of SLC appeared students, the probability of the students


who passed English was found to be 0.85. How many students out of
15000 may pass English?
Solution: Here,
Let E be the events of passed students in English.
Then, P(E) = 0.85, the total number of students = 15000.
 n(S) = 15000
Number of pass students = n(E) = ?
n(E)
Now, P(E) = n(S)

n(E)
or, 0.85 =
15000
or, n(E) = 0.85 × 15000 = 12750
Hence, required number of passed students in English = 12750.

Exercise 8.2
1. (a) When a coin is tossed 75 times, head occurs 45 times. Find the probability of (i) head
and (ii) tail.
(b) The result obtained in tossing a coin 1000 times are given in the following table.
Outcomes Head Tail
Frequency 725 275
Find the probability of occurrence of head.
(c) In a 100 tosses of a coin, 56 tails were observed, what is the empirical probability of
getting a head in the next toss?

286 | Mathematics - 9 Probability


2. (a) A dice is thrown 200 times and the record of outcomes is given in the following table.
Outcomes 1 2 3 4 5 6
Frequency 25 40 60 35 25 15
Calculate the empirical probability of:
(i) 3 (ii) 2 or 5
(iii) the number greater than 4 (iv) 3 and less than 3
(b) A dice is thrown 1200 times. The results are recorded in the following table.
Outcomes 1 2 3 4 5 6
Frequency 207 205 187 211 204 186
Calculate the empirical probability of:
(i) an odd number (ii) less than 3
(iii) greater than 4
(c) The number of tooth-picks in each of 30 boxes were counted. The results are given in
the following table.
No. of tooth-picks 40 45 50 55 60
Frequency 7 10 4 5 4
If one of these boxes is selected at random, what is the probability that
(i) it contains 50 tooth-picks?
(ii) it contains more than 45 tooth-picks?
(iii) it contains less than 50 tooth-picks?
3. (a) Out of 1000 newly born babies, there are 514 boys. What will be the empirical
probability that the newly born baby is (i) a boy? (ii) a girl?
(b) If the probability of germinating a seed of a flower is 0.9, how many seeds out of 1000
will germinate?
(c) Out of 1000 students appeared in SLC examination, 250 students pass securing 1st
division and distinction marks. Find the empirical probability of passing the
examination securing 1st division and distinction marks.
(d) Out of 1000 students admitted in 10+2 of a higher secondary school, the number of
students whose ages are above 16 years has been estimated with the probability 0.25.
Estimate the number of students whose ages are above 16 years.

Probability Mathematics - 9 |287


Unit Test
Time: 40 minutes F.M.- 24

Group-A (10×2=20)
1. Define random experiment.
2. Define sample space.
3. Find the sample space for two throws of a die.
4. Find the sample space for roll of a die and toss of a coin simultaneously.
5. Find the probability of getting an odd number when a dies is rolled.
6. A card is drawn from a well suffled pack of 52 playing cards. Find the probability of getting
a face card.
7. From a bag containing of white and 8 black marbles of same size, a marble is drawn
randomly. What is the probability of getting a white marble?
8. From a pack of number cards numbered from 4 to 32, a card is drawn randomly. Find the
probaiblity of getting a number whicn is multiple of 3 and 5.
9. What is the probability of touching the letter 'B' when a letter is touched randomly in the
word 'PROBABILITY'?
10. In 50 tosses of a coin, head turned up 32 times. Find the empirical probability of getting tail
in the next toss. .

288 | Mathematics - 9 Probability


Answers ____________________________________________________________
Exercise: 8.1
n(E)
1. (c)
n(S)
(d) (i) {1, 2, 3, 4, 5, 6} (ii) {HH, HT, TH, TT} (iii) {HHH, HHT, HTH, HTT, THH, THT, TTH, TTT}
1 1 3 10 1 1 3 3
2. (a) (i) (ii) (iii) (b) (c) (d) (i) (ii) (iv)
13 26 13 13 13 26 4 26
3 2 1 3 3
3. (a) (b) (c) (d) (e)
7 3 8 8 8
1 2 3 2
4. (a) (b) (c) (d)
4 3 5 3
1 1 5 1
5. (a) (b) (c) (d)
6 2 6 3
1 1
(e) (i) (ii) (iii) 0
6 6
1 1 2 1
6. (a) (b) (c) (d)
7 7 3 5
3 9 1 1
7. (a) (b) (c) (d)
10 28 5 4
1 2 9 10 2 2 2 9 9 2
8. (a) (i) (ii) (iii) (iv) (v) (b) (i) (ii) (iii) (iv) (v)
11 11 11 11 11 11 11 11 11 11
2 2 1
(c) (i) (ii) (d)
7 7 7
1 1 3 3 1 1 1 1
9. (a) {HH, HT, TH, TT} (i) (ii) (iii) (iv) (b) (c) (i) (ii) (iii)
4 2 4 4 4 8 2 4

Exercise: 8.2
3 2 29 11
1. (a) (i) (ii) (b) (c)
5 5 40 25
3 13 1 5 299 103 13 2 13 17
2. (a) (i) (ii) (iii) (iv) (b) (i) (ii) (iii) (c) (i) (ii) (iii)
10 40 5 8 600 300 40 15 30 30
257 243 1
3. (a) (i) (ii) (b) 900 (c) (d) 250
500 500 4

Probability Mathematics - 9 |289


Model Question Set
Examination of Grade IX

Proposed Specification Grid for First Terminal Examination

SN Containts Topics K-1 U-2 A-4 HA-5 TQ TM Periods


Introduction
1 Set 1 1 1 1 4 12 8
Operation on sets
Profit & Loss
2 Arithmetic Commission 1 4 3 1 9 26 16
Discount
Factorisation
3 Algebra 2 6 3 1 12 31 18
Indices
Introduction
4 Trigonometry - 2 - - 2 4 6
T. Ratios
Arithmetic mean
5 Statistics 1 2 1 - 4 9 6
Median
Parallel lines
6 Geometry 1 2 2 1 6 18 18
Triangles
Total questions 6 17 10 4
37 100 62
Total Marks 6 34 40 20
K – Knowledge, U – Understanding, A – Application, HA - Higher ability

290 | Mathematics - 9 Model Questions


Model Questions for First Terminal Examination
Attempt All the questions

Group – A [6 x 1 = 6]
1. a. If selling price of an article is x and profit amount is y, what will be the profit percentage?
______
b. Write down the calculating formula of n(AB) .
2. a. For what value of x the term 3 x – 2 has 1?

b. Find the value of 64 –1/ 3.

3. a. Find the mode value of the observations 12, 13, 12, 14, 14, 15, 13, 14, 16.
b. What is the relation of angle
< PRS over the angles < P
P
and < Q?

Q
R S

Group – B [17 x 2 = 34]


4. a. If U = {1, 2, 3, 4, 5, 6, 7, 8, 9}, A = { 3, 4, 5 }, B = { 4, 5, 6, 7, 8 }, find the value of
______
(
n AB . )
b. An article is sold at Rs.800 at a loss of Rs.200, find the loss percentage?

5. a. If Rs 20 is the discount amount on selling an article at Rs 60. Find the discount percentage.
b. The 12 oranges are sold at the cost price of 15 oranges. Find the profit percentage.
c. A sales man makes Rs 2, 00, 000 as the total sale on a month. Find his commission at the rate
of 5% in the month.
2n + 3 – 2n
6. a. Simplify: 2n . 7 .
b. Factorise: x4 + x2 +1
7. a. Solve: 5x + 2 – 5x – 120 = 0.
b. Factorise: 9x 2 – 4y2 + 6x + 1.
3 3 3
c. 2 54 – 3 16 + 486

3 3
8. a. Simplify: 9x4 y –1 . 3x5 y –5
b. Find the mean of the observations 12, 15, 18, 21and 24.
c. If median of the observations: 12, 16, 20, x + 4, x + 8 , 32, 36, & 40 is 26, find the value of ‘x’.

Model Question Set Mathematics - 9 |291


9. a. Find the trigonometric ratios Tanθ and cosecα, from the given diagram

P
Θ

13cm
5cm

α
Q R
° ° °
b. Find the value of : 4Sin30 .Cos60 .Tan45 .

10. a. If PQ bisects <BQR and RP bisects <QRD, find the value of <QPR.

Q
A B

C D
R

b. Find the value of <TPQ from the adjoining diagram where <PTS = 40° and <RST = 120°.
P Q

R
S

Group - C [10 x 4 = 40]


11. In a class of 50 students, 30 like English, 35 like Nepali and 5 don’t like both subjects. Find the
number of students who like both the subjects and like only one subject. Also represent the
information in Venn-diagram.
12. An article is sold at 15% profit. Had it been sold taking Rs.5,400 more, there would have been
25% gain. Find the cost price.

13. Monthly salary of a sales person is Rs 12,000 and earns Rs 20,000 in a month including
commission at the rate of 2%. Find the total sale made by him in the month.

292 | Mathematics - 9 Model Questions


14. A man get 10% loss on selling an article with 20% discount and Rs 180 profit on selling the same
article with out discount. Find the cost price of the article.

xp p2 + pq +q2  xq  q2 + qm +m2 xm m2 + mp +p2


15. Simplify: xq . m .
  x   xp 
16. Factorize: x 4 – 6x2 y2 + 25y 4.

17. Solve: a2x – 3 . b5 – 3x = a3 – x .b11 – 6x

18. If mean marks of given observations is 37, find the value of ‘m’.
Marks 20 30 40 50 60
f 7 m 13 8 2

19. Verify experimentally that the sum of any two angles of a triangle is always greater than third
angle. (Three figures of different measurement are necessary and the triangles should not be right
angled)

20. Prove theoretically that the base angles of an isosceles triangle are equal.

Group - D [4 x 5 = 20]
21. Two Watches are sold at Rs 2000 each where one is sold at 5% loss and the other is at 10% gain.
Find the actual gain or loss percent on selling both the watches.

22. In a survey of students 60% like Science, 70% like Maths, 10% don’t like any one of them and 24
like both of them. Find the number of students took part in the survey. Also find the number of
students who like only one subject by drawing venn diagram.
1 1
23. If x = 33 + 3– 3 , prove that 3x3 – 9x = 10.

24. In the given figure, QB and RA are the perpendiculars drawn from Q and R to the opposite sides
and PQ = PR, prove that QB = RA.

A B

Q R

Model Question Set Mathematics - 9 |293


Proposed Specification Grid for Second Terminal Examination

SN Containts Topics K-1 U-2 A-4 HA-5 TQ TM Periods


1 Set Review - - 1 - 1 4 4
Review
2 Arithmetic Bonus 1 2 2 1 6 18 12
Taxation
Prism
3 Mensuration Area of path 1 4 1 1 7 18 16
Area of four walls
Review
4 Algebra Ratio and Proportion 2 5 2 1 10 25 18
Simultaneous equations
Review
5 Trigonometry - 1 - - 1 2 6
Standard angles
Review
6 Statistics - 2 1 - 3 8 6
Quartiles
Review
Parallelogram
7 Geometry 2 3 3 1 9 25 20
Similarities
Right angled triangle
Total questions 6 17 10 4
37 100 82
Total Marks 6 34 40 20
K – Knowledge, U – Understanding, A – Application, HA - Higher ability

294 | Mathematics - 9 Model Questions


Model Questions for Second Terminal Examination
Attempt All the questions

Group – A [6 x 1 = 6]
1. a. Yearly profit of a company is Rs. 2, 50, 000. Find the bonus amount distributed by it to the
workers of 25% on the profit.
b. Write down the formula of finding LSA and TSA of a prism where A is the base area and P
is the perimeter of base of the prism.

2. a. If (x – 3) is a factor of an algebraic expression x3 – 27, what will be the another factor?


b. Simplify: 3x + 2 ⨉ 34 – x ⨉ 3 – 6 .

3. a. If (2x – 40)° & (10 – x)° are the linear pairs, find the value of ‘x’.

b. What is the relation of side BC over the A


side QR in the given diagram where Q
and R are the mid - points?
Q R

B C

Group – B [17 x 2 = 34]


4. a. A salesman makes total sale in a month Rs. 2,00,000 whose monthly income is Rs. 13, 000
including 2.5% commission and monthly salary. Find his monthly salary.
b. The cost of a machine is Rs. 4000 with 20% discount, find the marked price of the machine.
5. a. If 3m wide path is constructed inside a rectangular field of size 80m x 60m, find the area of
the path.
b. Find the area of four walls having size 20 m x 12 m x 5m where there are one door of Size
2m x 1.5m and two windows of size 3m x 2.5m.
c. Find the LSA of the prism given in diagram where perimeter of base of the triangle is 18 cm.
E

F D
20 cm
B C
3n+2 – 6.3n
6. a. Simplify: 3n . 3
b. Factorise: a4 + 16.

Model Question Set Mathematics - 9 |295


3
7. a Simplify: 4x2 y2 × 2xy3
b. Solve the equations: x + 2y = 7 and 4x – 3y = 6.
c. Solve:- 2x + 1 + 2x – 3 = 0.

8. a. fd = 600, f = 30 and assume mean = 25. Find the value of arithmetic mean.
b. If lower quartile of the observations:12, 2x – 4, 2x + 6, 28, 30, 34, 38 & 40 is 18.5, find the
value of ‘x’.
c. Find the value of:- 3Tan2 45° + 2Cos2 60° + 2Sin2 30°

9 a. If area of an equilateral triangle is 16 3 cm2, find its perimeter.


b. If (x + 20)°, (3x – 10)° and (x + 30)° are supplementary angles, find the value of x.

A F
10. a. Find the value of x, if ˂ ABC = 80°,
˂ CDE = x and CD = CE.

D
B
C

E
b. Find the value of x from the given diagram
where ABCD is a square, AC is a diagonal, A D
<ABP = x0 and <BPC = 800. P

B C

Group – C [10 x 4 = 40]


11. In a class of students 35% like only tea, 40% like only Coffee, 10% don’t like any one and like
both the drinks by 12 students. Find the no. of students who like tea and like Coffee by drawing
venn diagram.

12. A man earns certain amount in a month and he deposits 10% of his income in CIF fund and Paid
Rs. 10,500 as yearly income tax at the rate of 15%. Find the monthly salary of a person where tax
allowance is Rs. 2,00,000.

13. An article is sold with 25% discount. If 20% profit is there the cost price will be Rs. 2500. Find
the marked price of the article.
1 1
14. If x = 33 + 3– 3 , prove that 3x3 – 9x = 10.

296 | Mathematics - 9 Model Questions


12 8 4 24
15. Solve: x + y = 5 and x + y = 7.
16. If mean of the observations is 38, find the value of ‘m’.
Marks 20 30 40 50 60
f 7 m+4 15 10 4

17. A path of 5m wide is run inside of a rectangular field of length 150m and 60m. Find the cost of
paving stones at the rate of Rs.20 per stone of size 80cm x 50cm.

18. Verify experimentally that exterior angles of a triangle is equal to the sum of its opposite interior
angles.

19. Prove that the quadrilateral having opposite angles equal is a parallelogram.

20. Construct a quadrilateral having sides AB = BC = 7 cm, CD = DA = 6 cm and < DAB = 75° .

Group – D [4 x 5 = 20]
21. The cost price of two bicycles is Rs. 8,000. One is sold with 10% loss and the other is at 10% gain
for the same selling price. Find the final gain or loss percentage on both the bicycles.

22. If the cost of carpeting the square room at the rate of Rs 200 per m2 is Rs 51,200. Find the cost of
plastering the 4 walls and ceiling of the room having height 5m at the rate of Rs 18 per m2.

23. If a, b, c and d are in continued proportion, prove that:


(a + b + c)(b + c +d) = ab + bc + cd

24. In a right angled triangle ABC right angled at B, AD is perpendicular to the side BC, prove that
AB2 = BC.BD.

B D C

Model Question Set Mathematics - 9 |297


Proposed Specification Grid for Final Terminal Examination

SN Containts Topics K-1 U-2 A-4 HA-5 TQ TM Periods


1 Set Review - - 1 - 1 4 2
Review
2 Arithmetic 1 4 1 1 7 18 6
Home arithmetic
Review
3 Mensuration 1 2 1 1 5 14 6
Volume
Review
4 Algebra 1 5 3 1 10 28 6
Quadratic equation
5 Trigonometry Review - 1 - - 1 2 2
6 Statistics Review 1 1 1 - 3 7 2
Review
7 Geometry 2 3 3 1 9 25 6
Circle
Introduction
8 Probability - 1 - - 2 4
Probability of an event
Total questions 6 17 10 4
37 100 34
Total Marks 6 34 40 20
K – Knowledge, U – Understanding, A – Application, HA - Higher ability

298 | Mathematics - 9 Model Questions


Model Questions for Final Terminal Examination
Attempt All the questions
Group – A [6 x 1 = 6]
1. a. If cost of an article is Rs 2260, find its selling price to get Rs 240 profit in the article.
b. Write down the formula to find the area of path of width d which surrounds the rectangular
plot of length x m and breadth y m.
2
64 - 3
2. a. Simplify: 125
 
b. Write down the calculating formula of finding arithmetic mean of discrete frequency
distribution.
A
3. a. If P and Q are the mid- point of sides AB
and AC respectively, what will be
the length of side BC where PQ = 5cm?
P Q

B C
b. If OP is the line joining the centre of a circle and mid - point P of a chord AB, write down the
relation of OP over AB.

Group ’B’ [17 x 2 = 34]


4. a. If 20 oranges are sold at the cost price of 25 oranges, what will be the percentage gain?
b. If bonus amount received by a worker is Rs 5,000 among 20 workers at the rate of 15%, find
the net profit earned by the company.

5. a. The cost of a machine is Rs. 15,000 with discount where marked price is Rs. 20,000, find the
discount percentage.
b. If 3m wide cross path is constructed inside a rectangular field of size 80m x 60m, find the
area of the cross path.
c. Find the volume of the prism from the adjoining diagram.

F D

B 10√3
6 cm C

Model Question Set Mathematics - 9 |299


7x+2 – 7x–1 × 43
6. a. Evaluate: 7x+1 + 7x–1
3
b. Simplify: 4x2 y2 × 2 xy3

7. a. If 2x + y : 4x – 3y = 2 : 3, find X : Y.
b. Solve: abx2 – (a + b)c x + c2 = 0
c. Factorise: x2 – 4y 2 – 4y – 1.

8. a. If 5 SinA = 3, find the value of Sec2 A – Tan2 A.


b. Taxi fare is started from Rs. 15 and additional cost per kilometer is Rs. 20. If a customer paid
Rs. 155, what will be the distance travelled by him?
c. If upper quartile of the observations taken in order :12, 16, 20, 28, 32, 2x + 6, 3x – 5 & 44 is
39, find the value of ‘x’.

9. a. A dice is thrown at random. Find the probability of getting prime number in the throw.
b. Find the value of x, from the adjoining diagram, if ˂ ABC
A
= 800, ˂ CDB = x and CD = CB.
C
80°
B
x
D
A P B
10. a. Find the value of ADO where ABCD is a rectangle, 65°
ACD = 35°, POC = 65° in the adjoining diagram. O
35°
D C

A
b. From the adjoining diagram, find the radius of the circle
where OC = 3 cm, AB = 8 cm, O is the centre of the circle and
OC is perpendicular to AB.
C
O

Group - C [10 x 4 = 40]


11. In a survey of people 60 like football team and 80 like cricket team of our country 10 don’t like
any of the teams and 50 like both the teams. Find the total number of people took part in the
survey and who don’t like cricket. Also draw the venn diagram.

300 | Mathematics - 9 Model Questions


12. Cost of an article is marked 40% above the cost price and sold at the discount of 20%, find the
profit percentage.

13. Find the TSA of the given prism.

3cm
5cm
4cm

3cm 20cm
8cm
1 1 1
14. Simplify: 1 + xa-b + xc–b + 1 + xc-a + xb–a + 1 + xa -c + xb–c

15. Solve: 4x – 3.2x–3 + 128 = 0

16. Solve the equations graphically: 3x – 2y = 13 and 5x + 3y = 9.

17. Mean marks of the given observations is 58, find the value of ‘p’.
Marks 45 50 54 60 75
No. of students 4 p p+7 7 6

18. Prove that equal chords of a circle are equidistance from the centre of the circle.

19. Prove that the quadrilateral having opposite sides equal is a parallelogram.

20. Construct a quadrilateral having AB = 7 cm, BC = 6.5 cm, CD = 5 cm, DA= 5.5 cm and diagonal
BD = 8 cm.
Group - D [4 x 5 = 20]
21. 50 caps are bought at the rate of Rs 200 where first 20 are sold at a loss of 10%, other 20 are sold
at a gain of Rs 500. At what rate remaining caps be sold to get the profit of 25% in the outlay?
22. The cost of carpeting a square room at the rate of Rs. 90 per m2 is Rs. 23040 and cost of
plastering the 4 wall at the rate of Rs. 12 per m2 is Rs.3840. Find the height of the room.
x – 2 x +1
23. Solve: x + 1 + x + 7 = 1.
24. If A, B, C and D are the mid points of the sides PQ, PR, RS and QS respectively, prove that
ABCD is a parallelogram.

P A
Q

B D

S R
C
n
Model Question Set Mathematics - 9 |301
Prime questions for more
practice
1. i. In a class of 120 students, 60 like science, 70 like maths and 40 like both
the subject. Find the number of students who like at least one subjects and
who like neither of them by drawing venn diagram. Ans: 90, 10
ii. In a survey of 200 people 130 like tea, 120 like coffee and 15 don't like
any one of them. Find the numbers of people who like only one drink by
drawing venn diagram. Ans: 120
iii. In a hostel of students 35% like only junk food, 40% like only cooked
food and 5% don't like any one of them. If the students who like both of
them is 24, find the numbers of students who like cooked food by using
venn diagram. Ans: 72
iv. In a barrac of 95 solders 45 don't like to play football, 35 don't like to play
cricket and 25 like to play only football. Find the number of soldiers who
like neither of such games by drawing venn diagram. Ans: 10
v. In a class of 60 students, the number of student who like marking system
only is double the students who like only grading system during the
evaluation of examination where 15 students like both type of system and
the students who like at least one system. Find the number of students
who like marking system by drawing venn diagram. Ans: 45

2. i. Yash bought two music systems for Rs. 52,000. He sold them making
10% profit on one and 10% loss on the other. If their selling price is same,
find his total gain or loss percent. Ans: Loss 1%
ii. A sold an article to B at a profit of 10%. B sold the same article to C at a
profit of 15%. If C paid Rs. 3162. 50 to B, how much did A pay for it?
Ans: Rs. 2500
iii. An article when sold at a profit of 15% yields Rs. 380 more than when sold
2
at a loss of 16 3 %. What was the cost price of the article? Ans: Rs.1200
iv. A man bought two shirts for Rs. 1200 each on selling the first shirt, he
had 10% profit and 5% loss on the second shirt. Find his gain or loss
percent or whole. Ans: 2.5%
v. A shopkeeper bought 150 books and sold 100 of them at a profit of 30%
and the rest at 10% loss. If he had sold all the books at once at 25% profit,
he would have received Rs. 300 more. Find the cost price of each book.
Ans: Rs. 24

302 | Mathematics - 9 Practice Questions


3. i. A sales man's monthly salary is Rs. 8000. He gets 8% commission on the
total sales. If his income of a certain month is Rs. 17,000. What is the
total sales of that month? Ans: 1,12,500
ii. A man sells a D.V.D player at a discount of 8% and makes a profit of
Rs.780. If he doesn't allow a discount he would gain a profit of Rs. 1500.
Find the marked price and cost price of the D.V.D. player.
Ans: 9000 and 7500
iii. A trader fixes his goods at 20% above the cost price. He allows 15%
discount. If the marked price is Rs. 14400, find his profit or loss
percentage. Ans: 2%
iv. The monthly income of Rajani is Rs. 31,000. Her yearly tax allowance is
Rs. 2,82,000. After paying income tax, if Rs. 28,100 is her net income per
month, what is the tax rate? Ans: 15%
v. Anmol's monthly salary is Rs. 45,000 where yearly tax allowance is Rs.
3,50,000. He deposit 10% to provided fund and 15% to C.I.F. which are
tax free amounts. What is the yearly tax on the remaining sum at the rate
of 15%? Ans: 8250
4. i. The minimum charge for the 1st 20 units is Rs. 80. The charge for the
units from 21 – 250 is Rs. 7.50 per unit. If a customer paid Rs.1205 in a
month, how many units of electricity was consumed? Ans: 170
ii. The charge for certain number of calls with 10% TSC and 13% VAT is
Rs 149.16. Find the number of calls if the charge per call is Rs. 1. Ans: 120
iii. A rectangular plot of land is 64m long and 50m wide. Two paths, each of
width 2m are parallel to length and other parallel to width of the plot
cross each other inside the plot calculate.
a) The cost of paving the path with bricks of size 10cm × 14cm at Rs. 6
per brick. Ans: Rs. 11904
b) The cost of turfing the remaining space at Rs. 9 per sq.m Ans: 60,000
4 cm
iv. A square lawn is surrounded by a path 9 cm
of uniform width 3m. The cost of 6 cm
paving the path with marble at Rs.
5 cm
200 per square meter is Rs. 36,000.
Find the area of the lawn and cost of 10 cm
watering the lawn at Rs. 3 square
meter. Ans: 432

v. Find the total surface area and volume of the given solid.
Ans: 462 cm2, 540 cm2
5. i. The cost of plastering the four walls of a square room at Rs. 10 per m2 is
Rs. 3240 and the cost of carpeting it's floor at Rs. 25 per m2 is Rs. 8100.
Find the height of the room. Ans: 4.5 m

Prime questions for more practice Mathematics - 9 |303


ii. The cost of plastering the four walls of a room 4 m high and where
breadth is one third of it's length at Rs. 12 per m2 is Rs. 1536. What will
be the cost of panketing the floor at the rate of Rs . 200 per square metre?
Ans: Rs 9600
4 2 2 4
iii. Factorise: x + 6x y + 25y
iv. Factorise: a4 – 6a2 – 7 – 8x – x2
a2 – 1 m a – 1n –m
 b2  b

v. Simplify:
b2 – 1 n b + 1m – n
 a2  a

1 1 1
6. i. Simplify: 1 + ax – y + az –y + 1 + ay –z +ax–z + 1 + az –x +ay–x
2 2
ii. If x2 + 2 = 33 + 3–3 , prove that 3x3 + 9x = 8.
iii. 33x –4 × a5 –2x – 3x +2 × a11 – 4x = 0
1 1
iv. 2x + 2x = 2 2
v. 5 × 4x + 2 – 16x = 64
7. i. The ratio of the present ages of a daughter and her mother is 5:12. If the
ratio of their ages after 6 years will be 1:2, find their present ages.
a3 – b3 – c3 a
ii. If a, b, c and d are in continued proportion, prove that b3 -c3 – d3 = d
p q r
iii. If q = r = s , prove that (p + q +r)(q + r + s) = pq + qr + rs
3 2 4 2 3 9
iv. Solve x – y = 15 and x + y = 10
x + 1 x + 2 2x + 13
v. Solve x – 1 + x – 2 = x + 1
8. i. Solve the quadratic equation: ax2 + bx + c = 0.

ii. Prove that the line segment that joins the vertex and mid – point of the
base of an isosceles triangle is perpendicular to the base and bisects the
vertical angle.
A

iii. In the given figure: ∆ABC and ∆AED are


equilateral triangles. If AC = AE, prove
B x y D
that AX = AY.

C E

304 | Mathematics - 9 Practice Questions


A
iv. In the adjoining figure; A is the mid- D E
point of DE, BAD = CAE and
ADC = AEB. Prove that BCD =
CBE.

B C
A

v. In the adjoining figure, AB = AC; EF // AD


B F G C
and EF = CD. Prove that EG = DG.

D
9. i. Prove that the quadrilateral having opposite angles equal is a
parallelogram. A D
ii. In the adjoining figure; AM and
CN are perpendiculars to diagonal N
BD of the quadrilateral. If AO = o
CO and BM = DN, prove that M
ABCD is a parallelogram. B C
P
A B
iii. In the adjoining figure P, Q, R, S are the
midpoint of AB, BC, CD and AD
respectively. Prove that PQRS is a Q S
parallelogram.

C D
R
iv. In the adjoining figure: ∆ABC is a right angled A
triangle in which B = 90° and BD  AC.
Prove that. D
a) ∆ABC  ∆ABD and AB2 = AC.AD.
C B
v. Construct a rectangle ABCD in which AC = BD = 6.8 cm and diagonal
making an angle of 30°.
10. i. Construct a parallelogram ABCD in which; Diagonals AC = 5.6cm, BD =
6.4 cm and they make an angle of 30°.

Prime questions for more practice Mathematics - 9 |305


ii. Construct a trapezium ABCD in which; AB = 6.1 cm, BC = 4.2 cm,
BAC = 30°, AD = 4.9 cm where AB//CD.
iii. Construct a quadrilateral ABCD in which
a) AB = 5.3 cm, BC = 6.1 cm, CD = 4.2 cm, AD = 5.9 cm and BAD = 75°
b) AB = 6.1 cm, BC = 5.2 cm, CD = 4.5 cm, AD = 5.5 cm and BD = 4.7 cm
iv. Prove that a line joining the centre of a circle and the mid-point of a chord
is perpendicular to the chord.
v. Prove that chords which are equidistant from the centre of a circle are
equal.
11. i. In the adjoining figure; O is the centre of the circle;
A chord PQ intersects two concentric circles; at the
point P, M, N and Q as shown in the figure. Prove O
that PM = QN. M N
P Q

ii. Two equal chords AB and CD meet at an A


external point E. Prove that AE = CE and BE B
= DE. E
D
iii. Find the value of x. C

E
a) b)
E x
50m

A 30°
D
10m

30° 45°
x

O C B
B C 14.64
Ans: 40 3 Ans: 20m

iv. If the A.M. of the following data is 34, find the value of p.
Marks obtained 15 25 35 45 55
No. of students 7 8 p 7 6
v. The data 5, 10, x+5, x+10, 25, 30 are arranged in an ascending order. If
the median is 17.5. Calculate the value of x. Ans = 10
vi. If third quartile of the observation taken in order is 172. Find the value of k.
102, 112, 120, 130, 142, 150, 158, 150+2k, 188–k, 182 Ans: 10

306 | Mathematics - 9 Practice Questions

You might also like